You are on page 1of 752

SAT ®

Manual
Version 6.0
© 2018 by TPR Education IP Holdings, LLC. All Rights Reserved.

No part of this manual may be reproduced for distribution to a third party in any form or by any means,
electronic or mechanical, including photocopying, recording, or any information retrieval system,
without the prior consent of the publisher, The Princeton Review.

Permission to reprint this material does not constitute review or endorsement by the Educational Testing
Service or the College Board of this publication as a whole or of any other sample questions or testing
information it may contain.

This manual is for the exclusive use of The Princeton Review course students and is not legal for resale.

SAT is a registered trademark of the College Board.

The Princeton Review is not affiliated with Princeton University or the Educational Testing Service.

1-866-TPR-PREP

PrincetonReview.com
Contributors
Very special thanks to the primary instructional authors of this book: Aaron Lindh,
Amy Minster, Elizabeth Owens, and Alice Swan.

Special thanks to Deborah A. Weber for her work in producing this edition.

Special thanks also to Susan Swinford.

Thank you also to the following individuals who contributed to this edition:
Chris Aylward, Anne Bader, Grace Cannon, Alex Charles, Chris Chimera, Jenine
Faulkner, Anne Goldberg-Baldwin, Brian Hong, Ted Klein, Anthony Krupp,
Sara Kuperstein, Spencer Ledoux, Gianna Marciarille, David McGillicuddy, Scott
O’Neal, Stephen Ronkowski, Rodi Steinig, Christina Torturo, Cynthia Ward,
Andrew Williamson.

—Jonathan Chiu
National ACT & SAT
Content Director
Contents
SAT: What You Need to Know............................................................. 1

Reading................................................................................................. 9
Reading Introduction....................................................................................... 11
Basic Approach............................................................................................... 19
Basic Approach Practice................................................................................. 37
Mastering POE................................................................................................ 53
Mastering POE Practice.................................................................................. 67
Identifying Question Types............................................................................. 83
Identifying Question Types Practice............................................................... 101

Writing and Language................................................................ 117


Introduction......................................................................................... 119
Words Part I........................................................................................ 129
Words Part I Practice...................................................................................... 143
Questions........................................................................................................ 169
Questions Practice.......................................................................................... 183
Punctuation..................................................................................................... 223
Punctuation Practice....................................................................................... 239
Words Part II................................................................................................... 255
Words Part II Practice..................................................................................... 263
Grammar 101................................................................................................... 283
Full-Length Practice Passages........................................................................ 295

Math.......................................................................................... 359
Global Techniques................................................................................ 361
Plugging In........................................................................................... 373
Plugging In Practice........................................................................................ 383
Plugging In the Answers................................................................................. 395
Plugging In the Answers Practice................................................................... 403
Translating and Meaning in Context.............................................................. 409
Translating and Meaning in Context Practice................................................ 421
Solving 101.......................................................................................... 427
Solving (Linear).................................................................................... 435
Solving (Linear) Practice........................................................................ 449
Solving 201.......................................................................................... 457
Solving (Nonlinear)............................................................................... 471
Solving (Nonlinear) Practice.................................................................. 485
Advanced Solving................................................................................ 493
Advanced Solving Practice............................................................................. 503
Charts and Data 101....................................................................................... 507
Charts and Data.............................................................................................. 517
Charts and Data Practice................................................................................ 531
Word Problems 101......................................................................................... 543
Word Problems............................................................................................... 553
Word Problems Practice................................................................................. 573
Functions......................................................................................................... 585
Functions Practice........................................................................................... 599
Coordinate Geometry 101............................................................................... 609
Coordinate Geometry...................................................................................... 619
Coordinate Geometry Practice....................................................................... 635
Advanced Coordinate Geometry.................................................................... 645
Advanced Coordinate Geometry Practice...................................................... 657
Plane Geometry 101........................................................................................ 665
Plane Geometry.............................................................................................. 683
Plane Geometry Practice................................................................................ 705

Answer Key............................................................................... 723

vi | Contents
SAT:
WHAT YOU
NEED TO
KNOW

1
SAT MANUAL

INTRODUCTION: THE SAT AND HOW IMPORTANT


IS IT, REALLY?

What DOES it all mean, anyway?

S ______________________

A ______________________

T ______________________

How important is it?

Per
so
SAT

nal
Personal SAT
GPA GPA

Small Liberal Large State


Arts Colleges Universities

What’s on the test?


Evidence-based Reading and Writing:________________________________

Math:_________________________________________________________

“Optional” Essay: ________________________________________________

2  |  © TPR Education IP Holdings, LLC


WHAT YOU NEED TO KNOW

The SAT and YOU!


How beatable is the SAT? ________________________________________

When is the SAT administered? ____________________________________

How many times can you take the SAT? _____________________________

When will YOU take the SAT? ____________________________________

Knowing the structure and scoring of the SAT will help you
use the techniques and strategies you will learn in this course.

STRUCTURE OF THE SAT

Evidence-based Reading and Writing


Passages Questions Time
Reading
Writing & Language

Math
Multiple- Grid-In Time
Choice
No Calculator (Section 3)
Calculator (Section 4)

Essay
Topic: _______________________________ Time: ______________________

© TPR Education IP Holdings, LLC | 3


SAT MANUAL

Notes:

• Total time: 3 hours 50 minutes (with Essay).


• Multiple-choice questions have four answer choices.
• There is no penalty for wrong answers, so don’t leave any questions
blank.
• Questions are Easy, Medium, or Hard and are each worth 1 point.
• Questions in Math are in a rough order of difficulty.
• Questions in Reading and Writing are NOT in order of difficulty.

How is the SAT scored?

Total Score: ___________________________________________________

Two Area Scores: _______________________________________________

Three Test Scores: ______________________________________________

Two Cross-Test scores: ___________________________________________

Seven Subscores: _______________________________________________

What’s a good score?

A good score gets you admitted to the school of your dreams.


A GREAT score gets the admissions people calling you, begging
YOU to come to THEIR school!

4 | © TPR Education IP Holdings, LLC


WHAT YOU NEED TO KNOW

SAT STRATEGIES
We’re going to teach you how to nail every part of the SAT. Each section will have
its own approaches and techniques to master. However, there are some strategies
that apply across the SAT.
ersonal
P _________________________ P _________________________
rocess

O ________________________
order O _________________________
f

O ________________________
f E _________________________
limination

D ________________________
ifficulty

Pacing

11 12 1
10 2

9 3

8 4
7 6 5

Guessing

?
Why is it better to
use LOTD instead of
randomizing guesses?
etter
L _____________
f
O _____________

he
T _____________

D _____________
ay

© TPR Education IP Holdings, LLC | 5


SAT MANUAL

HOW TO USE THIS COURSE TO NAIL THE SAT


Please keep some things in mind as we go through this course:

• This isn’t school. We don’t give you a grade, but we do expect you to
realize that YOUR score improvement reflects the amount of work
YOU put into the course.
• The homework assigned is, therefore, for YOUR benefit.
• Try to do at least a half an hour of SAT each day. Like playing an
instrument or a sport, preparing for the SAT will be easier (and you
will improve faster) if you do at least some work each day. Cramming
three hours of SAT practice the night before class won’t give you the
improvement you can enjoy. Consistent daily practice is the best way
to ensure success.
• You must try the techniques. Many will seem strange at first. You will
get used to them, but only if you practice them.
• Use a pencil! And not only on the bubble sheet: write all over the test.
Seriously. Think on the page, not just in your head.

Setting Goals
Of course, everyone wants to increase his or her score. A big difference between
those who succeed and those who don’t is whether they set realistic goals and pur-
sue them systematically. Also keep in mind that learning doesn’t always happen
in a linear fashion or overnight; sometimes it takes a step backwards to take two
steps forwards. Don’t be discouraged! Keep working in consult with your instruc-
tor, and you will enjoy success in the long run.

If you are serious about increasing your scores significantly, then


you MUST:
1. Come to ALL the classes.
2. Complete ALL the homework.
3. Come to ALL the diagnostic exams.
4. Use the techniques in class, on the homework, on the diag-
nostic exams, and on the real SAT.

6 | © TPR Education IP Holdings, LLC


WHAT YOU NEED TO KNOW

Your present score:

Reading _________ W & L _________ Math _________ Essay _________

Your target score:

Reading _________ W & L _________ Math _________ Essay _________

What are my biggest strengths from the first practice test?

_____________________________________________________________

What are my biggest areas of improvement from the first practice test?

_____________________________________________________________

By the end of this course, I will have accomplished the following (feel free to add
more!):

1. ______________________________

2. ______________________________

3. ______________________________

© TPR Education IP Holdings, LLC  |  7


READING

9
INTRODUCTION

While we read a novel, we are insane—


bonkers. We believe in the existence of people
who aren’t there, we hear their voices... Sanity
returns (in most cases) when the book is closed.
—Ursula Le Guin

11
SAT MANUAL
READING

GOALS REVIEW
At the conclusion of this chapter, you will know:

• The structure of the SAT Reading Test


• Global strategies to earn a higher score
WRITING AND
LANGUAGE

What to Expect
You will have 65 minutes to complete ______ questions spread out over five pas-
sages.

Each passage will have ____________ questions.


MATH

The passages will cover topics including

____________________________________________________________

____________________________________________________________

____________________________________________________________

One of the five passages will be a ________________ set.

Two of the five passages will contain _____________________________.

12  |  © TPR Education IP Holdings, LLC


INTRODUCTION

READING
Why SAT Reading is Challenging
Who caused the deaths of Romeo and Juliet? Friar Lawrence? Lord Capulet and
Lord Montague? Mercutio? In an English class, any of those answers could be
right, given the right support and a solid argument. On the SAT, however, none
of them are right. On the SAT, Romeo and Juliet caused the deaths of Romeo and
Juliet. Case closed.

WRITING AND
LANGUAGE
Let’s take a look at the following statements to consider the differences and chal-
lenges between SAT Reading and your English class.

SAT Reading English Class


Analysis, interpretation,
and critical thinking are
rewarded

MATH
Prior knowledge of a
topic helps
You can explain your
thinking in short answer
or essay format
Knowledge of the life
and times of the author
helps on the test
Nothing matters except
what’s actually written in
the passage

On the SAT, you only get points for _____________________, not for
__________________. So, let’s maximize those points!

© TPR Education IP Holdings, LLC  |  13


SAT MANUAL
READING

What Can Help?


1. _________________________: Some questions will tell you exactly
where to go in the passage to find the information you need to answer
those questions. Use those lines to maximize your efficiency.
WRITING AND

2. _________________________: Questions are generally arranged in


LANGUAGE

chronological order, so use that to your advantage. Working through


the questions in order as much as possible means working through
the passage in order.

3. __________________________: Even if a question does not have a


line reference, it might have words or phrases that help you find what
you’re looking for in the passage.
MATH

4. ____________________________: Approximately two questions in


every passage will come with a second question that gives you exact
lines in which to find supporting evidence. Use those to be as focused
and efficient as possible.

Personal Order of Difficulty


Use your POOD to play to your strengths.

For example:

If your target Reading score is a 30, that means you need a raw score
of approximately 38.

Do all five passages and get about 75% of the questions right? That’s
a raw score of about 38.

Drop an entire passage and get nearly all of the questions right on
the remaining four passages? That’s a raw score of about 38.

SAME SCORE, DIFFERENT APPROACH.

Not all passages are created equal, and based on your POOD, you might opt to
do them in a different order than that in which they are presented. Consider the
following:

Type of passage ______________________________________________________

Type of questions ______________________________________________________

14 | © TPR Education IP Holdings, LLC


INTRODUCTION

READING
Your goal is to make a quick decision about which of the passages would be the
best place for you to start. Find that passage and do it first. As you move through
the Reading Test, save the most challenging passages for last, regardless of where
they come in the test.

WRITING AND
LANGUAGE
Scoring and Pacing
Your Reading score makes up half of your Evidence-Based Reading and Writing
score:

  W&L Scale Score


10 15 20 25 30 35 40
10 200 250 300 350 400 450 500

MATH
15 250 300 350 400 450 500 550
Reading 20 300 350 400 450 500 550 600
Scale 25 350 400 450 500 550 600 650
Score 30 400 450 500 550 600 650 700
35 450 500 550 600 650 700 750
40 500 550 600 650 700 750 800

For a Reading Test You need about


Score of: this many Correct
Answers:
10 <3
12 5
14 7
16 10
18 14
20 18
22 21
24 26
26 29
28 33
30 37
32 41
34 44
36 47
38 50
40 52

© TPR Education IP Holdings, LLC  |  15


SAT MANUAL
READING

Food for Thought:

Reading Score from first test: _________

Number of additional questions needed for a 2-point improvement: _________


WRITING AND

Each two-point improvement in your Reading Test Score could be worth 20 com-
LANGUAGE

posite points.

Process of Elimination
MATH

Multiple-choice Test = Open-book Test

All the right answers are there in front of you, just packed in tight with lots of
wrong answers.

Rather than trying to find the one ________________________, try to eliminate


answers that are clearly ____________________________ based on the text of the
passage.

A) A) A) A)
B) B) B) B)
C) C) C) C)
D) D) D) D)

16  |  © TPR Education IP Holdings, LLC


INTRODUCTION

READING
Keep Your Pencil Moving!
When you’re working on a math problem, you write out your steps, avoid doing
work in your head, and track your thinking on the page. You must do the same
thing on the Reading test! Mark your windows, underline or bracket relevant lines,
and physically mark out wrong answers. If your pencil stops moving, you’re prob-
ably stuck. Use your LOTD and move on.

WRITING AND
LANGUAGE
How a Rockstar Student’s Passage Should Look

MATH

© TPR Education IP Holdings, LLC  |  17


SAT MANUAL

Summary
• The topics of the passages will include:

________________________________________

________________________________________

________________________________________

• What three things can help you find the location of


the relevant text for a question?

1. ______________________________________

2. ______________________________________

3. ______________________________________

• Make sure your pencil is

________________________________________

• I have accomplished _________ of the 2 goals


stated in the beginning of this chapter.

18 | © TPR Education IP Holdings, LLC


BASIC
APPROACH

If you have enough book space, I don’t want to


talk to you.
— Terry Pratchett

19
SAT MANUAL
READING

GOALS REVIEW
At the conclusion of this chapter, you will have mastered the following:

• Know the five steps of the SAT Reading Basic Approach


• Apply each of the five steps in order to effectively and accurately
answer questions
WRITING AND

• Understand how to use the text to effectively work through answers


LANGUAGE

SAT READING: BASIC APPROACH


With only 65 minutes to answer 52 questions spread out over five passages, you
have to maximize the use of your time. The questions are where you get points, so
MATH

the questions are where you should focus.

Once you have established the order in which you’re doing the passages, use the
following approach for each passage.

1. Read the Blurb


2. Select and Understand a Question
3. Read What You Need
4. Predict the Correct Answer
5. POE

20 | © TPR Education IP Holdings, LLC


BASIC APPROACH

READING
Step 1: Read the Blurb
The bibliographic information before each passage may not contain a lot of details,
but it will give you a frame of reference for the content of the passage.

Read the following blurb:

WRITING AND
This passage is excerpted from Clarence Darrow, Crime: Its Cause and Treatment. © 1922.

LANGUAGE
In the passage, Darrow discusses the difficulties involved in defining the words “crime”
and “criminal,” and the ways in which these words can be interpreted.

Is this passage from US/World Literature, History/Social Studies, or Science? What


is the general topic of the passage?

MATH
social studies
general topics: inerpretation of the words crime and criminal

© TPR Education IP Holdings, LLC  |  21


SAT MANUAL

Here is the text of the passage, but we’re not going to read it yet. Move on to Step 2: Select and Understand a
READING

Question.

There can be no sane discussion of “crime” and satisfied that a thing is intrinsically wrong because it
“criminals” without an investigation of the meaning is forbidden by a legislative body.
of the words. A large majority of men, even among Other more or less popular opinions of the way
WRITING AND
LANGUAGE

Line the educated, speak of a “criminal” as if the word to determine right or wrong are found to be no
5 had a clearly defined meaning and as if men were 50 more satisfactory. Many believe that the question of
divided by a plain and distinct line into the criminal whether an act is right or wrong is to be settled by a
and the virtuous. As a matter of fact, there is no such religious doctrine; but the difficulties are still greater
division, and from the nature of things, there never in this direction. First of all, this involves a thorough
can be such a line. and judicial inquiry into the merits of many, if not
10 Strictly speaking, a crime is an act forbidden by 55 all, forms of religion, an investigation which has
the law of the land, and one which is considered never been made, and from the nature of things
MATH

sufficiently serious to warrant providing penalties cannot be made. The fact is, that one’s religious
for its commission. It does not necessarily follow opinions are settled long before he begins to
that this act is either good or bad; the punishment investigate and quite by other processes than reason.
15 follows for the violation of the law and not 60 Then, too, all religious precepts rest on interpretation,
necessarily for any moral transgression. No doubt and even the things that seem the plainest have ever
most of the things forbidden by the penal code are been subject to manifold and sometimes conflicting
such as are injurious to the organized society of the construction. Few if any religious commands
time and place, and are usually of such a character can be, or ever were, implicitly relied on without
20 as for a long period of time, and in most countries, 65 interpretation. The command, “Thou shalt not kill,”
have been classed as criminal. But even then it does seems plain, but does even this furnish an infallible
not always follow that the violator of the law is not rule of conduct?
a person of higher type than the majority who are Of course this commandment could not be
directly and indirectly responsible for the law. meant to forbid killing animals. Yet there are many
25 It is apparent that a thing is not necessarily bad 70 people who believe that it does, or at least should.
because it is forbidden by the law. Legislators are No Christian state makes it apply to men killing in
forever repealing and abolishing criminal statutes, war. Neither can it be held to apply to accidental
and organized society is constantly ignoring laws, killings, or killings in self-defense, or in defense of
until they fall into disuse and die. The laws against property or family. Laws, too, provide all grades of
30 witchcraft, the long line of “blue laws,” the laws 75 punishment for different kinds of killing, from very
Q4 affecting religious beliefs and many social customs, light penalties up to death. Manifestly, then, the
are well-known examples of legal and innocent commandment must be interpreted, “Thou shalt
acts which legislatures and courts have once made not kill when it is wrong to kill,” and therefore it
criminal. Not only are criminal statutes always dying furnishes no guide to conduct. As well say: “Thou
35 by repeal or repeated violation, but every time a 80 shalt do nothing that is wrong.”
legislature meets, it changes penalties for existing
crimes and makes criminal certain acts that were not
forbidden before.
Judging from the kind of men sent to the State
40 legislatures and to Congress, the fact that certain
things are forbidden does not mean that these
things are necessarily evil; but rather, that politicians
believe there is a demand for such legislation from
the class of society that is most powerful in political
45 action. No one who examines the question can be

22  |  © TPR Education IP Holdings, LLC


BASIC APPROACH

READING
Step 2: Select and Understand a Question
Start with the specific questions first, and save the general questions for later, regardless of which question type is first
in the set. The specific questions will be arranged generally in chronological order.

WRITING AND
LANGUAGE
1. The position that Darrow takes is primarily that of 6. The author’s attitude toward the views of the “many”
mentioned in line 50 can be described as
general question, last

MATH
2. In the passage, Darrow draws a distinction between 7. As used in line 60, “rest” most nearly means

general question, later

3. Which choice provides the best evidence for the 8. Darrow mentions the command “Thou shalt not
answer to the previous question? kill” (line 65) primarily in order to

general

4. In lines 29–34, Darrow mentions the “laws against 9. Which choice provides the best evidence for the
witchcraft” and the “blue laws” primarily in order answer to the previous question?
to
specific question

5. Lines 39–45 suggest that laws are often created 10. As used in line 66, “furnish” most nearly means

Though Question 1 is the first question in the set, when will you do it? Why?

© TPR Education IP Holdings, LLC  |  23


SAT MANUAL
READING

Once you’ve chosen a question, take the time to understand what it is asking.
Make sure you understand the question by turning it back into a question—that
is, back into a sentence that actually ends with a question mark.
WRITING AND
LANGUAGE

4. In lines 29–34, Darrow mentions the “laws against witchcraft” and the
“blue laws” primarily in order to

How can you rephrase the open-ended statement in Q4 into a “what” or “why”
question?
MATH

Why did Darrow mention the "laws against witchcraft" and the blue laws?

Step 3: Read Only What You Need


• Use line references, lead words, or chronology to find the location of
the question in the text.

• Read a window of about 10–12 lines in order to get context and find
an answer to the question.

• Read with the question in mind! Remember, this test doesn’t require
you to interpret the text. It requires you to apply what’s actually in
the text to the questions and answer choices. Try to find a particular
phrase, sentence, or set of lines that answers the question.

• Save the general questions until after you’ve worked the specific ques-
tions. You’ll have a much better idea of what is and isn’t in the text.

Read a window of about 5 lines above and


5 lines below your line reference to get the
context for the question.

24 | © TPR Education IP Holdings, LLC


BASIC APPROACH

4. In lines 29–34, Darrow mentions the “laws against witchcraft” and the

READING
“blue laws” primarily in order to

Where in the text will you find the answer to this question?

WRITING AND
LANGUAGE
somewhere 27-36

MATH
Step 4: Predict the Correct Answer
Know what you’re looking for before you look at the answer choices, and you’ll be
less likely to fall for a trap answer. Before you even glance at the answer choices,
take the time to underline the specific, stated information in your window that
supplies the answer to the question.

4. In lines 29–34, Darrow mentions the “laws against witchcraft” and the
“blue laws” primarily in order to

Using information from the text, explain why the author mentions the “laws
against witchcraft” and the “blue laws.”

are well-known examples of legal...

© TPR Education IP Holdings, LLC  |  25


SAT MANUAL
READING

Step 5: Use Process of Elimination


Each question has Once you’ve found something in the text that answers the question, eliminate any
three incorrect of the answer choices that don’t match your prediction.
answers and only
one correct answer,
so it’s easier to
find an incorrect Eliminate answers that don’t match
WRITING AND
LANGUAGE

answer than it is your prediction.


to find a correct
answer.

Avoid the temptation to reconsider your prediction based on an answer choice.


Answer choices that don’t match the text are likely wrong no matter how good
MATH

they sound.

4. In lines 29–34, Darrow mentions the “laws against witchcraft” and the
“blue laws” primarily in order to
A) assert that penalties for certain actions
Yes/No/Maybe
should not be altered.
B) provide illustrations of an earlier
Yes/No/Maybe
statement.
C) hint at the value of laws long since
Yes/No/Maybe
repealed.
D) qualify a position by conceding
Yes/No/Maybe
exceptions to a rule.

Once you have eliminated any answer choices that don’t match your prediction,
you may still be left with more than one choice. Before you select an answer, ask
yourself if you might be falling for one of the most common—and avoidable!—
traps:

• Mostly Right/Slightly Wrong

• Could Be True (But Isn’t Supported by the Text)

• Deceptive Language

26 | © TPR Education IP Holdings, LLC


BASIC APPROACH

READING
THE IMPORTANCE OF THE TEXT
Remember, this is not a test in your English class at school. As you go through the
answer choices, you should be more focused on whether or not each answer choice
is consistent with your prediction rather than whether or not you could explain
that answer to your English teacher in 500 words or less.

WRITING AND
LANGUAGE
Make sure that when you predict the answer,
you are using the actual words in the text
instead of your own interpretation.

MATH
5. Lines 39–45 suggest that laws are often created

What is this question asking?

why are laws often created

Where in the text will you find the answer?

right or wrong is to be settled by a


religious doctrine

What exactly does the text say about the creation of laws?

Which answer choices can you eliminate?

A) to prevent wicked actions.


B) to satisfy the demands of crime victims.
C) by men who are morally weak.
D) to appease prevailing political powers.

© TPR Education IP Holdings, LLC | 27


SAT MANUAL
READING

The text always matters, even on questions that might seem to be asking for your
opinion. Consider the following question.

6. The author’s attitude toward the views of the “many” mentioned in line 50 can
best be described as
WRITING AND
LANGUAGE

In an English class, a question about an author’s “attitude” or “tone” might be a


great place for you to use some solid critical thinking skills. On the SAT, however,
it’s still a straight reading comprehension question.

Who are the “many” described in the lines?


MATH

the citizens

What do they think?

Does Darrow agree or disagree? How do you know?

dis.... since

What answer choices can you eliminate?


A) apathetic.
B) curious.
C) puzzled.
D) skeptical.

28  |  © TPR Education IP Holdings, LLC


BASIC APPROACH

READING
Another way your comprehension of context will be tested is with Vocabulary in
Context questions. Although the SAT no longer tests your knowledge of obscure
four-syllable words, College Board still cares that you can figure out what words
mean, based on the contexts in which they are used.

WRITING AND
LANGUAGE
7. As it is used in line 60, “rest” most nearly means

Where is the word “rest”? What do you think of

MATH
when you hear the word
“rest”? Pillows? Your
couch? Peace and quiet?

Mark out the word. Based on the context, what other word or short phrase could
you put into the sentence?

Which answer choices can you eliminate, based on your prediction of the correct
answer?

A) recline.
B) depend.
C) pause.
D) conceal.

Try this one.

10. As it is used in line 66, “furnish” most nearly means


A) clothe.
B) decorate.
C) provide.
D) complicate.

© TPR Education IP Holdings, LLC  |  29


SAT MANUAL
READING

Paired Questions
Most reading passages have two sets of paired questions. The second question asks
you to match lines from the text to the answer to the first question, which could
be general or specific. There are a few different ways to approach these sets.
WRITING AND
LANGUAGE

Specific
8. Darrow mentions the command “Thou shalt not kill” (line 65) primarily in
order to

Why did Darrow mentioned ....


MATH

What is this question asking?

Where in the text will you find the answer?

What exactly does the text say about why Darrow mentions the command?

Few commnads can be impli...

What answer choices can you eliminate?


A) argue that even accidental killing or killing in self-defense should be
made punishable by law.
B) demonstrate that even some seemingly straightforward laws may be
understood in multiple ways.
C) provide evidence to show that laws do not punish all types of killings
with equal severity.
D) indicate the importance of investigating the religious opinions of
those conducting judicial inquiries.

Once you have determined your answer, the next question asks for the textual
evidence you used to reach that answer. Consider what it was in the text that an-
swered your “Why?” question.

9. Which choice provides the best evidence for the answer to the previous
The Basic Approach makes question?
some paired sets a Buy-One-
A) Lines 34–38 (“Not only . . . before”)
Get-One-Free for points!
B) Lines 53–59 (“First of . . . reason”)
C) Lines 60–65 (“Then, too . . . interpretation”)
D) Lines 74–76 (“Laws, too . . . death”)

30  |  © TPR Education IP Holdings, LLC


BASIC APPROACH

READING
Parallel POE
Sometimes the question will be a general question, or the location of the question If you were looking for
might be tough to find. In those cases, it’s helpful to consider the question and the treasure on an island,
textual evidence at the same time. Remember, the citation must support the cor- would it be easier with
rect answer, so if there are no clear connections between the answer for the ques- or without landmarks
tion and the line given in the next question, both answers can be eliminated. and a map?

WRITING AND
LANGUAGE
2. In the passage, Darrow draws a distinction between
A) actions that are illegal and actions that are morally wrong.
B) the opinions of legislators and the rituals of organized society.
C) criminals and individuals who are inherently virtuous.
D) personal responsibility and responsibility imposed by religious doctrine.

MATH
3. Which choice provides the best evidence for the answer to the previous
question?
A) Lines 3–7 (“A large . . . virtuous”)
B) Lines 26–29 (“Legislators are . . . die”)
C) Lines 39–42 (“Judging from . . . evil”)
D) Lines 60–63 (“Then, too . . . construction”)

© TPR Education IP Holdings, LLC  |  31


SAT MANUAL
READING

General Questions
The general and main idea questions that may seem a bit overwhelming at the
beginning of a passage become much more approachable once you’ve completed
the specific questions. Even if you haven’t read every word of the passage, you’ve
spent time with the parts the test writers think are important.
WRITING AND
LANGUAGE

1. The position that Darrow takes is primarily that of


A) an authority arguing a particular point of view.
B) a critic discounting the opinion of an uninformed adversary.
C) a jurist advocating the right of citizens to defend themselves.
D) an intellectual presenting an historical overview.

What position does Darrow take?


MATH

32  |  © TPR Education IP Holdings, LLC


BASIC APPROACH

READING
BASIC APPROACH DRILL
(13 minutes)

The following passage is excerpted from I Know Why the 40 She said she was going to give me some books
Caged Bird Sings by Maya Angelou. © 1969 and renewed and that I not only must read them, I must read
1997 by Maya Angelou. Used by permission of Random them aloud.

WRITING AND
LANGUAGE
House, an imprint and division of Penguin Random “I’ll accept no excuse if you return a book to
House LLC. All rights reserved. This passage from the me that has been badly handled.” My imagination
autobiographical novel describes an incident from the 45 boggled at the punishment I would deserve if in fact
author’s youth. I did abuse a book of Mrs. Flowers’s. Death would be
too kind and brief.
One summer afternoon, sweet-milk fresh in my
The odors in the house surprised me. Somehow
memory, Mrs. Flowers stopped at the Store to buy
I had never connected Mrs. Flowers with food or
provisions. Another Negro woman of her health and
50 eating or any other common experience of common

MATH
Line age would have been expected to carry the paper
people. There must have been an outhouse, too, but
5 sacks home in one hand, but Momma said, “Sister
my mind never recorded it.
Flowers, I’ll send Bailey up to your house with these
The sweet scent of vanilla had met us as she
things.”
opened the door.
She smiled that slow dragging smile. “Thank
55 “I made tea cookies this morning. You see, I had
you, Mrs. Henderson. I’d prefer Marguerite, though.”
planned to invite you for cookies and lemonade so
10 They gave each other age-group looks.
we could have this little chat.”
Momma said, “Well, that’s all right then. Sister, go
They were flat round wafers, slightly browned on
and change your dress. You going to Sister Flowers’s.”
the edges and butter-yellow in the center. With the
There was a little path beside the rocky road, and
60 cold lemonade they were sufficient for childhood’s
Mrs. Flowers walked in front swinging her arms and
lifelong diet. Remembering my manners, I took nice
15 picking her way over the stones.
little lady-like bites off the edges. She said she had
She said, without turning her head, to me, “I hear
made them expressly for me. So I jammed one whole
you’re doing very good school work, Marguerite,
cake in my mouth and the rough crumbs scratched
but that it’s all written. The teachers report that they
65 the insides of my jaws, and if I hadn’t had to swallow,
have trouble getting you to talk in class.” We passed
it would have been a dream come true.
20 the triangular farm on our left and the path widened
As I ate she began the first of what we later called
to allow us to walk together. I hung back in the
“my lessons in living.” She said that I must always
separate unasked and unanswerable questions.
be intolerant of ignorance but understanding of
“Come and walk along with me, Marguerite.” I
70 illiteracy. That some people, unable to go to school,
couldn’t have refused even if I wanted to. She
were more educated and even more intelligent than
25 pronounced my name so nicely. Or more correctly,
college professors. She encouraged me to listen
she spoke each word with such clarity that I was
carefully to what country people called mother wit.
certain a foreigner who didn’t understand English
When I finished the cookies she brushed off
could have understood her.
75 the table and brought a thick, small book from
“Now no one is going to make you talk—possibly
the bookcase. I had read A Tale of Two Cities and
30 no one can. But bear in mind, language is man’s
found it up to my standards as a romantic novel. She
way of communicating with his fellow man and it is
opened the first page and I heard poetry for the first
language alone which separates him from the lower
time in my life.
animals.” That was a totally new idea to me, and I
80 “It was the best of times and the worst of times . . .”
would need time to think about it.
Her voice slid in and curved down through and
35 “Your grandmother says you read a lot. Every
over the words. She was nearly singing. I wanted to
chance you get. That’s good, but not good enough.
look at the pages. Were they the same that I had read?
Words mean more than what is set down on paper.
Or were there notes, music, lined on the pages, as in a
It takes the human voice to infuse them with the
85 hymn book?
shades of deeper meaning.”

© TPR Education IP Holdings, LLC  |  33


SAT MANUAL

“How do you like that?” 4


READING

It occurred to me that she expected a response. In the context of the passage, Marguerite’s
The sweet vanilla flavor was still on my tongue statement “My imagination boggled at the
and her reading was a wonder in my ears. I had to punishment I would deserve if in fact I did abuse a
90 speak. book of Mrs. Flowers’s” (lines 44–46) is primarily
I said, “Yes ma’am.” It was the least I could do, but meant to convey the idea that
it was the most also.
A) Mrs. Flowers is known for her strict and
WRITING AND

On that first day, I ran down the hill and into the
LANGUAGE

unforgiving nature.
road (few cars ever came along it). I was liked, and
95 what a difference it made. I was respected not as Mrs. B) Mrs. Flowers is overly concerned with the
Henderson’s grandchild or Bailey’s sister but for just importance of books.
being Marguerite Johnson. C) Marguerite would fear for her life if she harmed
one of Mrs. Flowers’s books.

1 D) Marguerite is unlikely to mistreat one of Mrs.


Flowers’s books.
MATH

The narrative point of view of the passage is that of


A) a woman explaining the importance of reading.
B) a child presenting her opinions on a particular 5
novel. According to Mrs. Flowers, which of the following
C) an adult recounting a memorable childhood is a “lesson in living”?
experience. A) Intelligence is not dependent on formal
D) a writer describing why she chose to write. education.

general question => save for later B) Intellectuals are not as clever as many people
suppose.
2 C) Well-educated people lack common sense.
In the context of the passage, lines 23–28 D) Impoverished people are deserving of
(“I couldn’t . . . her) are primarily meant to compassion.
A) recount an anecdote.
B) describe a theory. what is the
purpose of 6
C) present an example. line 23 -28?
Which choice provides the best evidence for the
D) note an impression.
answer to the previous question?
specific
A) Lines 40–42 (“She said . . . aloud”)
3 B) Lines 61–62 (“Remembering my . . . edges”)
As used in line 39, “shades” most nearly means C) Lines 68–70 (“She said . . . illiteracy”)
A) shadows. D) Lines 70–72 (“That some . . . professors”)
B) reflections.
C) levels.
7
D) insights.
Marguerite’s statement in lines 76–77 (“I had . . .
novel”) suggests that she initially viewed A Tale of
Two Cities as
A) original.
B) sentimental.
C) satisfactory.
D) stunning.

34  |  © TPR Education IP Holdings, LLC


BASIC APPROACH

8 10

READING
In the context of the passage, Marguerite’s question Mrs. Flowers’s main objective in inviting
in lines 83–85 (“Were they . . . book”) primarily Marguerite to her house was to
serves to A) help Marguerite to appreciate the importance of
A) imply that Marguerite was bewildered by Mrs. the spoken word.
Flowers’s unusual speech patterns. B) urge Marguerite to spend less time reading and
B) show the religious fervor that Mrs. Flowers

WRITING AND
more time living.

LANGUAGE
brought to her reading. C) expose Marguerite to a wide variety of literary
C) indicate that Mrs. Flowers had set the words of influences.
the book to music. D) convince Marguerite to put more effort into her
D) convey Marguerite’s admiration for the schoolwork.
eloquence of Mrs. Flowers’s reading.

11

MATH
9
Which choice provides the best evidence for the
Marguerite’s attitude toward Mrs. Flowers in lines answer to the previous question?
87–92 (“It occurred . . . also”) is best described as A) Lines 16–19 (“I hear . . . class”)
one of
B) Lines 35–39 (“Your grandmother . . . meaning”)
A) respectful awe.
C) Lines 62–66 (“She said . . . true”)
B) grudging acceptance.
D) Lines 93–95 (“On that . . . made”)
C) relaxed affection.
D) guarded fear.

© TPR Education IP Holdings, LLC  |  35


SAT MANUAL

Summary
• The five steps of the Reading Basic Approach
are:

1. _______________________________________

2. _______________________________________

3. _______________________________________

4. _______________________________________

5. _______________________________________

• How much of the passage do you need to read


to find the answer to a specific question?

________________________________________

________________________________________

• What’s generally true about the order of the


questions?

________________________________________

• How can correct answers on the SAT Reading


Test differ from correct answers in an English
class?

________________________________________

________________________________________

• What is the most important thing to remember


in the Reading Test?

________________________________________

________________________________________

• I have accomplished _________ of the 3 goals


stated in the beginning of this chapter.

36 | © TPR Education IP Holdings, LLC


BASIC
APPROACH
PRACTICE

Work through the following passages using the steps of


the Basic Approach. Don’t worry about timing yourself yet.
This is all about accuracy and focusing on the text.
• Read the Blurb
• Select and Understand the Question
• Read What You Need
• Predict the Answer (Using the Text)
• POE

37
SAT MANUAL
READING

PRACTICE PASSAGE 1: LITERATURE

Questions 1–10 are based on the following 40 addressed to Mrs. Zeena-Frome, and usually bearing
passage. conspicuously in the upper left-hand corner the
address of some manufacturer of patent medicine.
WRITING AND

This passage is adapted from Edith Wharton, Ethan Frome.


LANGUAGE

Everyone in Starkfield knew him and gave him


© 1911 by Scribner’s. The novel tells the story of a man
a greeting tempered to his own grave bearing; but
whose dreams have been defeated by an unexpected turn
45 his taciturnity was respected and it was only on rare
of events.
occasions that one of the older men of the place
I had the story, bit by bit, from various people, detained him for a word. When this happened he
and, as generally happens in such cases, each time it would listen quietly, his blue eyes on the speaker’s
was a different story. face, and answer in so low a tone that his words
Line If you know Starkfield, Massachusetts, you know 50 never reached me; then he would climb stiffly into
MATH

5 the post-office, and you must have seen Ethan Frome his buggy, gather up the reins in his left hand and
drive up to it, drop the reins on his hollow-backed drive slowly away toward his farm.
bay and drag himself across the brick pavement, and “It was a pretty bad smash-up?” I questioned
you must have asked who he was. Harmon, looking after Frome’s retreating figure, and
It was there that, several years ago, I saw him for 55 thinking how gallantly his lean brown head, with
10 the first time; and the sight pulled me up sharp. Even its shock of light hair, must have sat on his strong
then he was the most striking figure in Starkfield, shoulders before they were bent out of shape.
though he was but the ruin of a man. It was not “The worst kind,” my informant assented. “More
so much his great height that marked him, for than enough to kill most men. But the Fromes are
the “natives” were easily singled out by their lank 60 tough. Ethan’ll likely touch a hundred.”
15 longitude from the stockier foreign breed: it was the “Good God!” I exclaimed. At the moment Ethan
careless powerful look he had, in spite of a lameness Frome, after climbing to his seat, had leaned over
checking each step like the jerk of a chain. There to assure himself of the security of a wooden box,
was something bleak and unapproachable in his face, which he had placed in the back of the buggy, and I
and he was so stiffened and grizzled that I took him 65 saw his face as it probably looked when he thought
20 for an old man and was surprised to hear that he was himself alone. “That man touch a hundred? He looks
not more than fifty-two. as if he was dead and in hell now!”
“He’s looked that way ever since he had his “Guess he’s been in Starkfield too many winters.
smash-up; and that’s twenty-four years ago come next Most of the smart ones get away,” said Harmon.
February,” said Harmon Gow, who had driven the 70 “Why didn’t he?”
25 stage from Bettsbridge to Starkfield in pre-trolley days “Somebody had to stay and care for the folks.
and knew the chronicle of all the families on his line. There weren’t ever anybody but Ethan. First his
The “smash-up,” I gathered, had, besides drawing father—then his mother—then his wife.”
the red gash across Ethan Frome’s forehead, so “And then the smash-up?”
shortened and warped his right side that it cost 75 Harmon chuckled sardonically. “That’s so. He
30 him a visible effort to take the few steps from his had to stay then.”
buggy to the post-office window. He used to drive in “I see. And since then they’ve had to care for him?”
from his farm every day at about noon, and I often Harmon thoughtfully passed his tobacco to the
passed him in the porch or stood beside him while other cheek. “Oh, it’s always Ethan done the caring.”
we waited on the motions of the distributing hand 80 Though Harmon Gow developed the tale as far
35 behind the grating. I noticed that, though he came as his mental and moral reach permitted there were
so punctually, he seldom received anything but a perceptible gaps between his facts, and I sensed that
copy of the Bettsbridge Eagle, which he put without a the deeper meaning of the story was in the gaps.
glance into his sagging pocket. At intervals, however, But one phrase nevertheless stuck in my memory:
the post-master would hand him an envelope 85 “Guess he’s been in Starkfield too many winters.”
CONTINUE
38  |  © TPR Education IP Holdings, LLC
BASIC APPROACH PRACTICE

Before my own time there was up I would come 5

READING
to know what that meant. Harmon’s statement in line 79 (“Oh, it’s . . . caring”)
primarily serves to
1 A) speculate on a probable outcome.
The passage is primarily concerned with B) contradict a presumption.
A) describing life in a rural community. C) qualify a primary argument.

WRITING AND
LANGUAGE
B) comparing possible causes of a tragedy. D) justify a point of view.
C) examining an unusual figure.
D) criticizing one man’s habits.
6
Based on the passage, which of the following most
2 accurately describes Ethan Frome?
As used in line 17, “checking” most nearly means A) Rigid and reticent

MATH
A) studying. B) Rude and imposing
B) halting. C) Decrepit and ignorant
C) limping. D) Generous and friendly
D) verifying.

7
3 Which choice provides the best evidence for the
As revealed in the third paragraph (lines 9–21), answer to the previous question?
the narrator’s attitude toward Frome is best A) Lines 4–12 (“If you . . . man”)
characterized as B) Lines 35–52 (“I noticed . . . farm”)
A) startled and intrigued. C) Lines 61–69 (“Good god . . . Harmon”)
B) dismayed and disapproving. D) Lines 71–79 (“Somebody had . . . caring”)
C) bewildered and fearful.
D) saddened and protective.
8
The passage suggests which of the following about
4 Harmon Gow?
As used in line 56, “shock” most nearly means A) He is originally from Bettsbridge.
A) surprise. B) He does not stay in Starkfield for the winters.
B) lack. C) He knows more about Ethan’s story than he
feels it is his place to say.
C) clash.
D) He works at the Post Office in Starkfield.
D) mass.

CONTINUE
© TPR Education IP Holdings, LLC  |  39
SAT MANUAL

9 10
READING

Which choice provides the best evidence for the Which of the following can best be inferred from
answer to the previous question? the narrator’s mention of Ethan’s face in line 65?
A) Lines 22–26 (“He’s looked . . . line”) A) The narrator believes that Ethan would be angry
B) Lines 53–60 (“It was . . . hundred”) if he knew anyone was watching him.

C) Lines 68–69 (“Guess he’s . . . Harmon”) B) The narrator feels guilty for spying on Ethan.
WRITING AND

C) The narrator presumes that Ethan wears a


LANGUAGE

D) Lines 80–85 (“Though Harmon . . . winters”)


different face in public than in private.
D) The narrator believes that Ethan is older than
Harmon Gow thinks he is.
MATH

CONTINUE
40  |  © TPR Education IP Holdings, LLC
BASIC APPROACH PRACTICE

READING
PRACTICE PASSAGE 2: SOCIAL STUDIES

Questions 11–21 are based on the following of Bavaria could more easily collect a tax on the salt
passage. being transported from the city of Salzburg (German
for “Salt Castle”).

WRITING AND
This passage is adapted from The Monuments Men: Allied

LANGUAGE
40 And throughout the centuries, as cities and
Heroes, Nazi Thieves, and the Greatest Treasure Hunt in
empires rose and fell, the Steinberg mine in the
History by Robert M. Edsel with Bret Witter. Copyright
Sandling Mountain of Austria, just above the
© 2009 by Robert M. Edsel. Reprinted with the permission
village and lake known as Altaussee, continued to
of Center Street, a division of Hachette Book Group USA
produce salt. The salt was not mined with picks
Inc. The passage describes the setting of a salt mine in the
45 and shovels, but dissolved by the flow of water
Austrian Alps. litterature - historic through special pipes and sluices. The water came
From there, the road begins a climb so steep that from the mountain above, especially during the

MATH
the Pötschen Pass seems a mild slope by comparison. spring snowmelt, and descended by gravitational
Along the road runs a clear, crashing alpine stream, force through the mine. There it was inundated
Line and beyond are the immense and breathtaking 50 with rock salt, then sent down the mountain to Bad
5 mountains. They are limestone deposits, formed Ischl, more than seventeen miles away, where the
in the depths of an ancient sea, and even on the brine was evaporated to form pure crystal salt. It
sunniest day they are pale gray beneath their caps of was left to 125 miners to maintain the pipes and
snow. A bleak stone building, perched precariously sluices, shore up the catacombs against the pressure
above a thousand-foot precipice, marks the 55 of the mountain, and make sure the vast labyrinth
10 beginning of the end. Beyond is only a low irregular of rooms and tunnels didn’t merge together and
building and a wall of rock, the steep side of the destabilize the entire structure.
Sandling Mountain. Bored into the mountain is a Since the 1300s, this job had been performed
small tunnel, the main entrance to an ancient salt by members of a small group of families, all living
mine. Local legend holds that salt had been mined 60 in the hills near the mine. Over the centuries
15 here for three thousand years—before the founding humans grew larger, but the miners stayed the same
of Rome, at the height of the ancient Egyptian size, until they eventually seemed dwarfed by the
empire. Local written records, however, only date demands of the mine and their time underground
back to the 1100s. (diet and inbreeding were more likely causes). Even
In those days, at the turn of the first millennium, 65 in the early twentieth century, this small isolated
20 salt was one of the foundations of civilization. community spoke a dialect last popular in the
Without it, food couldn’t be preserved or transported, Middle Ages. They explored their tunnels with
so whole societies survived because of salt. Roman acetylene torches, and wore the white linen suits and
legionnaires were sometimes paid in salt (the basis peaked caps of medieval miners.
of the English word “salary”), and merchants trod 70 But in the winter of 1943-1944, the salt mine at
25 the salt roads in large caravans, linking the Western Altaussee was assaulted by the modern world. First
world of Europe with the Eastern world of Asia came the tracked vehicles necessary for maneuvering
and Arabia. In Tibet, Marco Polo noticed that salt over the roads in the winter, when the five meters of
was pressed into wafers, imprinted with the image snow were almost level with the treetops. They were
of the Grand Khan, and used as money. Timbuktu, 75 followed by supply jeeps, and eventually a seemingly
30 the great lost civilization of Africa, valued salt as endless line of Nazi officers descended on the mine
highly as gold. The early Germans, whose Visigoth as guards. Workers arrived, expanding catacombs
ancestors sacked Rome and threw civilization into and building wooden floors, walls, and ceilings in
darkness, were economically dependent on their salt dozens of salt chambers. Giant wooden racks were
mines, and especially the taxes for their salt trading 80 assembled in workrooms deep within the mountain
35 routes. The city of Munich, an early base of power and hammered into position, in some places three
for the Nazi Party, was founded in 1158 so the ruler stories high. Experts and clerks moved in; a shop
CONTINUE
© TPR Education IP Holdings, LLC  |  41
SAT MANUAL

was built deep inside the mine where technicians 15


READING

could work and even live for days at a time. And it The author most likely includes the description
85 was all done for art. of the importance of salt through history in the
second paragraph in order to general

11 A) share some interesting but ultimately irrelevant


facts.
Which of the following best expresses the main idea
B) suggest that the Nazi party would not exist
WRITING AND

of the passage?
LANGUAGE

general
without the salt tax.
A) The salt mines in Altaussee have a long and
varied history. C) track the fluctuating value of salt in many cultures.
B) Salt has been an essential component of D) provide historical context for the output of the
civilization since the Roman Empire. Steinberg mine.
C) Mining techniques in Altaussee were not
modernized until the Nazis arrived. 16
MATH

D) The presence of the salt mines made Germany The passage most strongly suggests which of the
into a modern world power. following about the Steinberg Mine?
A) It was owned by the same family for many
centuries until the Nazis came.
12
B) It used a unique system to mine salt with water
The passage indicates that Sandling Mountain is instead of picks and shovels.
A) a salt mine first established by Egyptians. C) It was inaccessible during the winter before the
B) a milder slope than Pötschen Pass. Nazis brought their tracked vehicles.
C) the remnant of an ancient sea bed. D) It was mostly unaffected by political and social
D) a bleak and boring place. upheaval through the ages.
What does
general
17
13
Which choice provides the best evidence for the
The author uses the word “precariously” (line 8) in answer to the previous question? specific
order to
A) Lines 40–44 (“And throughout . . . salt”)
A) suggest that the stone building is in danger of
B) Lines 44–46 (“The salt . . . sluices”)
falling down the cliff.
Why did the C) Lines 58–60 (“Since the, . . . mine”)
author used B) indicate the rugged and remote terrain around
the word " the salt mine. D) Lines 71–74 (“First came . . . treetops”)
precaiously" C) imply that the buildings associated with the salt
? mine were poorly constructed. 18
D) indicate that the people who built the stone
As used in line 54, “shore up” most nearly means
building were skilled mountaineers.
A) disembark.
specific B) catch.
14 C) strengthen.
As used in line 16, “height” most nearly means D) build.
A) pinnacle. the most successful point; the culmination.
B) epitome.
C) elevation.
D) stature.
CONTINUE
42  |  © TPR Education IP Holdings, LLC
BASIC APPROACH PRACTICE

19 21

READING
The author of the passage most likely mentions the In the context of the passage, the author’s use of the
“acetylene torches” (line 68) and “white linen suits phrase “And it was all done for art” (lines 84–85) is
and peaked caps” (lines 68–69) in order to primarily meant to convey the idea that
A) show how eager the miners were to modernize A) the Nazis carved sculptures into the salt mines
the mines. in Altaussee.
B) indicate the miners’ reluctance to adopt new B) modern mining techniques were seen by the

WRITING AND
LANGUAGE
traditions. Nazis as an art form.
C) prove that the miners of Altaussee were unique C) the Nazis took over the salt mines for a purpose
among all other salt miners. other than mining.
D) provide imagery to create a clear idea of the D) the salt mines were turned into a mining
miners. museum by the Nazis.

MATH
20
Which choice provides the best evidence for the
answer to the previous question?
A) Lines 64–67 (“Even in . . . Ages”)
B) Lines 70–71 (“But in . . . world”)
C) Lines 74–77 (“They were . . . guards”)
D) Lines 79–82 (“Giant wooden . . . high”)

CONTINUE
© TPR Education IP Holdings, LLC  |  43
SAT MANUAL
READING

PRACTICE PASSAGE 3: SCIENCE

Questions 22–31 are based on the following A male’s system is designed to recognize the
passage. songs of other males and copy his father’s. If he
doesn’t learn, perfect and memorize his father’s
WRITING AND

This passage is adapted from JoAnna Klein, “Some


LANGUAGE

song within the first 90 days of life, when his brain


Songbirds Have Brains Specifically Designed to Find Mates
45 is especially malleable, he never will. He still sings,
for Life.” From New York Times, February 13, 2018 © 2018
but “he sings a disaster,” said Dr. Woolley. “And the
If Cupid wanted to make two songbirds fall in females want nothing to do with him.”
love, he’d have better luck aiming at their brains. When a female’s brain is young and malleable, she
That’s because songbirds, which form lifelong tunes into her father’s song, memorizes it and then
Line mating pairs, have brain systems perfectly tuned to 50 stores it as a template for evaluating a mate’s song later.
5 fit together. While you sort through the messages This example reminds her that she didn’t die, and her
MATH

of admirers, deciding who to make your Valentine, father helped ensure that. Perhaps something similar
consider finches. will work for her offspring. Females tend to prefer
Young males in this family of feathered crooners elaborate songs with more syllables.
learn the song of their father, perfect it and perform 55 How well the birds learn depends on a genetic
10 it as adults to attract a lifelong mate. It’s loud, predisposition to tune into sounds specific to their
elaborate and precise. With their songs they say species. But experience is important too. Because
“chirp, chirp—my brain is healthy, and my body is social relationships are so powerful, a baby bird
strong. That’s something you’re into, right?” reared by the wrong species, Dr. Woolley has
A female finch also learns the songs of her father 60 found, can learn the wrong species’ song even if its
15 from a young age, but she doesn’t perform. She’s biological father’s song is audible.
the critic. She analyzes every detail of a potential “The magic of the songbird is that vocal learning
mate’s song, compares it to her father’s example and is incredibly rare to find in animals,” said Dr. Woolley.
decides if this performer is one she’d like to keep “No ape can do it (except the human), no monkey can
around. If she detects a song is too simple or off in 65 do it, and no rodent can do it.” And she believes that
20 any way, she’ll have nothing to do with its performer. by understanding more about how songbirds use their
She’s very picky, as she should be, because the mate brains to make sense of sound, she can learn more
she chooses will help raise their young—till death do about how humans use theirs to develop a spoken
they part. language early to communicate later in life.
Over the past decade, researchers looking into the 70 For songbirds that form bonds with members of
25 chickpea-sized brains of finches have discovered that the same sex for life, songs, though still important
each sex uses what’s called its sound control system to message bearers, may be less important for finding
convert sound waves to social messages and then use a match. And although some humans may be less
them to find mates, kind of how humans use vocal interested in words than other aspects like looks,
sounds to communicate. And while these systems 75 scent, youth, money, power or whatever we find
30 are well-developed and finely tuned in both sexes of attractive in a partner, birdsongs remind us that
songbirds, the wiring is different. good communication, in any pair, makes love
“The biggest difference between male and female possible. “The way that people fall in love, is talking
brains of the same species is found in songbirds,” to each other,” Dr. Woolley said.
said Sarah Woolley, a neuroscientist who studies
35 finches at Columbia University’s Zuckerman
Institute. Dr. Woolley’s lab has been looking into
the acoustic systems of zebra, bengalese and long-
tailed finches to see how their brains take in and
process sounds—learning, performing and analyzing
40 different parts of them to make sense of songs.
CONTINUE
44  |  © TPR Education IP Holdings, LLC
BASIC APPROACH PRACTICE

22 26

READING
The main purpose of the passage is to What is the most likely reason the author includes
A) describe the sound of the mating songs of the quantity of 90 days in line 44?
finches that makes them unique. A) To demonstrate how long it takes most birds to
B) explain the role songbird brains play in their learn one song
mating rituals. B) To present accurate data on the length of the
mating process for finches

WRITING AND
C) discuss the reasons songbirds have for

LANGUAGE
establishing and maintaining lifelong mating C) To show at what age finches find their mates
pairs. D) To provide the length of time male finches have
D) illustrate how songbird brains process sound before they must know their song exactly
compared to other species.
general
27
23

MATH
The author includes the quotation “he sings a
What does the author imply about the mating songs disaster” (line 46) to
of male and female finches? A) explain how zebra, bengalese, and long-tailed
A) Finches all learn the same mating song when finches’ songs sound.
they are young. B) emphasize the importance of female finches
B) Males and females learn the songs the same way learning their songs at a young age.
but use them differently. C) suggest that male finches do not sing well.
C) Male finches can learn every song that they hear. D) indicate what happens when a male finch does
D) Male and female finches prefer different songs. not learn his song.

24 28
Which choice provides the best evidence for the As used in line 50, “stores” most nearly means
answer to the previous question? A) remembers.
A) Lines 14–16 (“A female . . . critic”) B) stockpiles.
B) Lines 29–31 (“And while . . . different”) C) conceals.
C) Lines 32–36 (“The biggest . . . Institute”) D) guards.
D) Lines 57–61 (“Because social . . . audible”) Why Wooller used finches for
his study
29
25
What can reasonably be inferred about the reason
The author uses the phrase “chickpea-sized brains” Woolley used finches as the subjects of her study?
in line 25 most likely to A) Male and female finches make beautiful music
A) suggest that a certain size of brain is best for to hear.
learning birdsong. B) Finches are very good at a particular skill that is
B) diminish the cognitive capabilities of the birds. uncommon for most animals.
C) describe the subject of research. C) Woolley knew finches would be easy to study
D) indicate the finches’ powerful mental capacity because of their small brains.
for thought. D) Finches and humans share a number of
important similarities that Woolley hopes to
explain.

CONTINUE
© TPR Education IP Holdings, LLC  |  45
SAT MANUAL

30 31
READING

Which choice provides the best evidence for the What is a main idea of the ninth paragraph
answer to the previous question? (lines 70–79)?
A) Lines 24–29 (“Over the . . . communicate”) A) Only finches are capable of such exceptional
B) Lines 55–57 (“How well . . . species”) vocal learning.

C) Lines 62–65 (“The magic . . . it”) B) Observations of one species can inform
understanding of other species.
WRITING AND
LANGUAGE

D) Lines 65–69 (“And she . . . life”)


C) Different types of animals use vocal learning in
unique ways.
D) Songbirds are capable of communication using
a spoken language as humans do.
MATH

CONTINUE
46  |  © TPR Education IP Holdings, LLC
BASIC APPROACH PRACTICE

READING
PRACTICE PASSAGE 4: DUAL HISTORY

Questions 32–41 are based on the following 35 goals. The case can be made that it was not the
passage. American Revolution that secured independence
from Britain; it was not the Civil War that freed the

WRITING AND
The first passage is excerpted from Nonviolence: 25 Lessons

LANGUAGE
slaves; and World War II did not save the Jews. But
from the History of a Dangerous Idea by Mark Kurlansky.
this possibility has rarely been considered, because
© 2006 by Mark Kurlansky. Used by permission of Modern
40 the Caesars and Napoleons of history have always
Library, an imprint of Random House, a division of Penguin
used their power to muffle the voices of those who
Random House LLC. All rights reserved. The second passage
would challenge the necessity of war—and it is these
is excerpted from Mahatma Gandhi, Freedom Battle, © 1922.
Caesars, as Napoleon observed, who get to write
The non-cooperation movement aimed to resist British rule
history. And so the ones who have killed become the
in India through nonviolent means.
ones who are revered.

MATH
45
Passage 1
The first clue, lesson number one from human Passage 2
history on the subject of nonviolence, is that there If people do not respond to the movement of
is no word for it. The concept has been praised by non-co-operation, it would be a pity, but that can
Line every major religion. Throughout history there have be no reason for a reformer not to try. It would be
5 been practitioners of nonviolence. Yet, while every to me a demonstration that the present position of
major language has a word for violence, there is no 50 hopefulness is not dependent on any inward strength
word to express the idea of nonviolence except that it or knowledge, but it is hope born of ignorance and
is not another idea, it is not violence. In Sanskrit, the superstition.
word for violence is himsa, harm, and the negation If non-co-operation is taken up in earnest, it
10 of himsa, just as nonviolence is the negation of must bring about a cessation of all other activities
violence, is ahimsa—not doing harm. But if ahimsa 55 including the Reforms, but I decline to draw
is “not doing harm,” what is it doing? therefore the corollary that it will set back the
The only possible explanation for the absence of clock of progress. On the contrary, I consider
a proactive word to express nonviolence is that not non-co-operation to be such a powerful and pure
15 only the political establishments but the cultural instrument, that if it is enforced in an earnest spirit,
and intellectual establishments of all societies have 60 it will be like seeking first the Kingdom of God
viewed nonviolence as a marginal point of view, a and everything else following as a matter of course.
fanciful rejection of one of society’s key components, People will have then realised their true power.
a repudiation of something important but not a They would have learnt the value of discipline, self-
20 serious force in itself. It is not an authentic concept control, joint action, non-violence, organisation and
but simply the abnegation of something else. It 65 everything else that goes to make a nation great and
has been marginalized because it is one of the rare good, and not merely great.
truly revolutionary ideas, an idea that seeks to I find their cause to be just and pure. Although
completely change the nature of society, a threat to therefore their viewpoint is different from mine I do
25 the established order. And it has always been treated not hesitate to associate with them and invite them
as something profoundly dangerous. 70 to give my method a trial, for, I believe that the use
Advocates of nonviolence—dangerous people— of a pure weapon even from a mistaken motive does
have been there throughout history, questioning the not fail to produce some good, even as the telling of
greatness of Caesar and Napoleon and the Founding truth if only because for the time being it is the best
30 Fathers and Roosevelt and Churchill. For every policy, is at least so much to the good.
Crusade and Revolution and Civil War there have
always been those who argued, with great clarity,
that violence not only was immoral but that it was
even a less effective means of achieving laudable
CONTINUE
© TPR Education IP Holdings, LLC  |  47
SAT MANUAL

32 36
READING

In Passage 1, Kurlansky most likely mentions the As used in line 67, “just” most nearly means
word “ahimsa” (line 11) in order to A) simple.
A) offer another definition for a confusing word. B) only.
B) illustrate a point made earlier in the paragraph. C) approximate.
C) make a distinction between two different D) honorable.
WRITING AND

languages.
LANGUAGE

D) provide evidence to weaken another’s argument.


37
In Passage 2, Gandhi most strongly suggests that a
33
strength of non-cooperation is that
As used in line 20, “force” most nearly means A) it allows those who use it to demonstrate their
A) influence. inward strength and knowledge.
MATH

B) draft. B) it can be effective even when used by those with


C) strain. a variety of motivations.

D) division. C) it is the only way people can learn the value of


discipline and self-control.
D) it is the key to making a nation merely great.
34
Kurlansky implies that non-violence has been
38
considered “profoundly dangerous” (line 26)
because Which choice provides the best evidence for the
A) it is a much larger movement than it initially answer to the previous question?
appears to be. A) Lines 48–51 (“It would . . . knowledge”)
B) it can have a much larger impact on society than B) Lines 51–52 (“it is . . . superstition”)
some might think. C) Lines 63–66 (“They would . . . great”)
C) the word that describes it simply conveys the D) Lines 67–72 (“Although therefore . . . good”)
absence of another word.
D) many influential world leaders would not have
been successful without non-violence. 39
Kurlansky in Passage 1 would most likely
characterize the position taken by Gandhi in
35
lines 57–61 (“On the . . . course”) as
Which choice provides the best evidence for the A) too idealistic to be an actionable plan for change.
answer to the previous question?
B) dangerous because of the power it gives to those
A) Lines 5–8 (“Yet, while . . . violence”) who lead.
B) Lines 13–17 (“The only . . . view”) C) a strong and reasonable assessment of a social
C) Lines 21–25 (“It has . . . order”) movement.
D) Lines 27–30 (“Advocates of . . . Churchill”) D) too heavily influenced by religion to be effective.

CONTINUE
48  |  © TPR Education IP Holdings, LLC
BASIC APPROACH PRACTICE

40 41

READING
Which choice best describes the authors’ views of Based on Passage 1, Kurlansky would most likely
powerful social change? respond to Gandhi’s claim that non-cooperation
A) Kurlansky believes that the idea of nonviolence will not “set back the clock of progress”in lines
is so powerful that it can be frightening, while (lines 56–57) with
Gandhi believes that non-cooperation is a A) confusion.
worthy idea that should be tried.

WRITING AND
B) disapproval.

LANGUAGE
B) Kurlansky believes that nonviolence is C) agreement.
ineffective as a means for social change, while
Gandhi believes non-cooperation is the most D) delight.
effective means.
C) Kurlansky believes that leaders fear nonviolence
so much that they have not allowed it to be
named, while Gandhi believes non-cooperation

MATH
should be referred to as the Kingdom of God.
D) Kurlansky believes Caesar and Napoleon were
successful due to non-violence, while Gandhi
believes success comes from the use of pure
weapons.

CONTINUE
© TPR Education IP Holdings, LLC  |  49
SAT MANUAL
READING

PRACTICE PASSAGE 5: SCIENCE WITH GRAPH

Questions 42–52 are based on the following “The size of the discrepancy is huge—20 to 2,000
passage and supplementary material. times more than the range of estimates of floating
40 debris. That is pretty shocking, especially when you
WRITING AND

This passage is adapted from Laura Parker, “Eight Million


LANGUAGE

consider that the amount going into the ocean in a


Tons of Plastic Dumped in Ocean Every Year.” © 2018
single year and what we’re counting in the oceans
National Geographic.
has been going in for 50 years.”
Scientists have come up with a new way to To make the figure eight million tons
measure ocean trash—and the numbers are 45 comprehensible, Jenna Jambeck, the University of
even worse than thought. In 2010, eight million Georgia environmental engineer who led the study,
Line tons of plastic trash ended up in the ocean from likens it to lining up five grocery bags of trash on
5 coastal countries—far more than the total that every foot of coastline around the globe. “And by
MATH

has been measured floating on the surface in the 2025, those five grocery bags of plastic are going to
ocean’s “garbage patches.” 50 be ten bags,” she says. That would be 155 million
That’s the bad news. The even worse news is that tons a year if present garbage management practices
the tonnage is on target to increase tenfold in the remain the same.
10 next decade unless the world finds a way to improve Jambeck and her team combined population and
how garbage is collected and managed. economic data from 192 coastal countries bordering
The findings are part of a groundbreaking study 55 the Atlantic, Pacific, and Indian Oceans in addition
published Thursday in Science that for the first time to the Black and Mediterranean Seas. They found
quantifies how much garbage flows into the world’s that these countries created 275 million tons of
15 oceans every year. garbage annually, of which 4.8 to 12.7 million tons
Until now, most efforts to measure ocean debris of plastic flowed into the oceans. That’s only 2 to 5
have involved sample counts of plastic floating 60 percent of the total waste created in those countries.
on the surface in large garbage patches in each of The use of plastics for consumer products has
the world’s oceans. A study last year, for example, become increasingly dominant, and production
20 estimated the amount of floating trash to be 245,000 has steadily increased since the material was first
tons at most. put into wide use a half century ago. In 2012,
The new study also identifies the major sources 65 for example, 288 million tons of plastic were
of plastic debris and names the top 20 countries manufactured globally. Ocean plastic has turned up
generating the greatest amount of ocean-bound literally everywhere. It has been found in the deep
25 trash. China is first. The United States is 20th. The sea and buried in Arctic ice. It has been ingested
rest of the list includes 11 other Asian countries, with dire consequences by some 700 species of
Turkey, five African countries, and Brazil. 70 marine wildlife.
Even though the United States has a highly The pioneering study also creates a new mystery.
developed garbage collection system, it nevertheless Because the gap between what is found floating and
30 made the top 20 for two reasons: It has a large, dense what flows into the ocean is so large, scientists now
coastal population and, as a wealthy nation, is a large have to figure out where else it collects and in what
consumer of products. “What we have done is look 75 amounts. “What we need to do now is close the gap,”
at the other side of the equation—what’s coming says Richard Thompson, a marine biologist at the
out of the faucet, rather than what’s already in the U.K.’s Plymouth University.
35 bathtub,” says Kara Lavender Law, an oceanographer
at the Sea Education Association in Woods Hole,
Massachusetts, and a co-author of the report.

CONTINUE
50  |  © TPR Education IP Holdings, LLC
BASIC APPROACH PRACTICE

44

READING
Top 10 sources of ocean's plastic waste

Which choice provides the best evidence for the


answer to the previous question?
China
PACIFIC
OCEAN
A) Lines 3–7 (“In 2010 . . . patches)
ATLANTIC
OCEAN
Egypt
Thailand
Vietnam
B) Lines 8–11 (“The even . . . managed”)
Bangladesh
Phillippines
Nigeria
Malaysia C) Lines 22–25 (“The new . . . trash”)

WRITING AND
Ocean-bound Sri Lanka

LANGUAGE
Indonesia
plastic waste
2010, in million tons INDIAN D) Lines 28–32 (“Even though . . . products”)
10 OCEAN

45
NG STAFF; J. L. WANG. SOURCE: SCIENCE As used in line 30, “dense” most nearly means
A) impenetrable.
B) dimwitted.

MATH
42
C) complicated.
The main purpose of the passage is to
D) crowded.
A) discuss the sources and measurement of ocean
trash in the present day.
B) show how scientist’s efforts influence the 46
amount of trash flowing into oceans.
The author most likely includes the quotation in
C) challenge the conventional view that trash is lines 38–43 to
currently well-managed by most countries
A) cite expert opinion.
around the world.
B) present contradictory information.
D) analyze the scientific developments that have
affected the production, management, and C) criticize previous actions.
measurement of plastic waste. D) suggest viable solutions.

43 47
The passage suggests that which of the following is As used in line 62, “dominant” most nearly means
the most pressing challenge countries around the
A) commanding.
world face?
B) prevalent.
A) They must change the way they measure ocean
debris. C) superior.
B) They must commit to a reduction in the D) central.
consumption of plastic products.
C) They must rethink the methods used to process
trash at various stages.
D) They must discover who is responsible for ocean
trash.

CONTINUE
© TPR Education IP Holdings, LLC | 51
SAT MANUAL

48 51
READING

The author indicates that scientists are coming to Which statement is best supported by information
believe that studies of ocean debris should presented in the figure?
A) remove personal judgments from present waste A) In 2010, the amount of ocean-bound plastic
management practices. waste from China was more than the other nine
B) consider input from authorities in the countries producers combined.
that are most affected. B) In 2010, the amount of ocean-bound plastic
WRITING AND
LANGUAGE

C) expand to investigate undiscovered sites of waste from China was approximately double
accumulated refuse in the ocean. that of Indonesia.

D) include more information about the process of C) In 2010, the amount of ocean-bound plastic
trash measurement. waste coming from Egypt, Thailand, and
Vietnam changed very little.
D) In 2010, the amount of ocean-bound plastic
49 waste produced was greatest in China, but
MATH

the amount produced was smallest in the


Which choice provides the best evidence for the Philippines.
answer to the previous question?
A) Lines 50–52 (“That would . . . same”)
B) Lines 56–59 (“They found . . . oceans”) 52
C) Lines 67–70 (“It has . . . wildlife”) The figure best serves as evidence of
D) Lines 72–75 (“Because the . . . amounts”) A) “the amount of floating trash” (line 20).
B) “countries generating the greatest amount of
ocean-bound trash” (lines 23–25).
50 C) “present garbage management practices”
Based on the figure, in which country was the most (line 51).
ocean-bound plastic waste produced? D) “[t]he use of plastics for consumer products”
A) Bangladesh (line 61).
B) Sri Lanka
C) Indonesia
D) China

Answers can be found on page 724.

52 | © TPR Education IP Holdings, LLC


MASTERING
POE

“There is more treasure in books than in all the


pirate’s loot on Treasure Island.”
— Walt Disney

53
SAT MANUAL
READING

GOALS REVIEW
By the end of this chapter, you will be able to:

• Recognize the trap answers College Board often gives on Reading


questions
• Use Process of Elimination to effectively eliminate wrong answers
WRITING AND
LANGUAGE

MASTER THE POE


By eliminating wrong answers, you leave yourself with the answer that must be
right, even if you aren’t completely in love with it.
MATH

Look for wrong answers instead


of right answers.

The best SAT answers:

• Are supported by the text of the passage


• Agree with the main idea of the passage
• Answer the question that’s asked

You must read the answer choices carefully. College Board is very good at creating
answers that look good with a cursory skim, so make sure you’re reading for con-
tent. Don’t simply match words from the text.

Mark every answer choice as you work through them. Give it a  if you like it,
cross it off if you know it’s wrong, or give it a ~ if you aren’t sure. Marking them
allows you to focus on what’s relevant.

A) A) A) A)
B) B) B) B)
C) C) C) C)
D) D) D) D)

It’s quite common for you to get down to two answers and then feel stuck. Or feel
like you always pick the wrong answer. Often, both answers will seem like they
could be correct. College Board is very good at creating attractive wrong answers.
Learn the traps and they’ll be easier to avoid!

54 | © TPR Education IP Holdings, LLC


MASTERING POE

READING
Common Trap Answers

Mostly Right/Slightly Wrong


This trap answer mostly looks perfect, but it doesn’t quite say what it needs to.
Even if 90% of the answer choice is exactly what you’re looking for, if there’s one

WRITING AND
wrong word, you can eliminate that answer choice. The entire answer must be

LANGUAGE
supported by the text. Be careful with answer choices that use extreme words like
only, never, or must.

Could Be True
This trap answer is usually one of the two you’ll narrow the answers down to. It
might make sense based on the text, and you could justify it if you had a chance

MATH
to explain your answer, but the concrete support isn’t actually there in the text.
Put your pencil on actual words in the text that support keeping or eliminating an
answer, and you’ll be more accurate. If you’re justifying your answer in your head
without using the text, you’re likely talking yourself into a trap.

Deceptive Language
This trap answer looks very attractive if you’re just matching individual words
from the text. If you aren’t reading thoroughly (question, answers, and text), and
instead are simply matching words, you may find your accuracy negatively affected.
College Board will give you answer choices that look really familiar but don’t
always say what you want them to say or answer the question that was asked. This
could also include answers that are too literal.

© TPR Education IP Holdings, LLC  |  55


SAT MANUAL

The following passage is adapted from a speech given nothing to be argued. What point in the anti-slavery
READING

on July 5, 1852 by Frederick Douglass, fugitive slave and creed would you have me argue? On what branch of
Abolitionist. 50 the subject do the people of this country need light?
Must I undertake to prove that the slave is
Pardon me, and allow me to ask, why am I
a man? That point is conceded already. Nobody
called to speak here today? What have I or those I
doubts it. Would you have me argue that man is
represent to do with your national independence?
entitled to liberty? That he is the rightful owner of
Line Are the great principles of political freedom and
WRITING AND

55 his own body? You have already declared it. Must I


LANGUAGE

5 natural justice, embodied in that Declaration of


argue the wrongfulness of slavery? Is that a question
Independence, extended to us? And am I, therefore,
for republicans? How should I look today in the
called upon to bring our humble offering to the
presence of Americans to show that men have a
national altar, and to confess the benefits, and
natural right to freedom, speaking of it relatively
express devout gratitude for the blessings resulting
60 and positively, negatively and affirmatively? To do so
10 from your independence to us?
makes me ridiculous and would offer an insult your
I say it with a sad sense of disparity between
understanding.
us. Your high independence only reveals the
MATH

Am I to argue that it is wrong to make men


immeasurable distance between us. The rich
brutes, to rob them of their liberty, to work them
inheritance of justice, liberty, prosperity, and
65 without wages, to keep them ignorant of their
15 independence bequeathed by your fathers is shared
relations to their fellow men, to beat them with
by you, not by me. The sunlight that brought life
sticks, to flay their flesh with the last, to load their
and healing to you has brought stripes and death to
limbs with irons, to hunt them with dogs, to sell
me. This Fourth of July is yours, not mine. You may
them at auction, to sunder their families, to knock
rejoice, I must mourn. Do you mean, citizens, to
70 out their teeth, to burn their flesh, to starve them
20 mock me, by asking me to speak today?
into obedience and submission to their masters?
Fellow citizens, above your national, tumultuous
Must I argue that a system thus marked with blood
joy, I hear the mournful wail of millions, whose
and stained with pollution is wrong? No; I will not.
chains, heavy and grievous yesterday, are today
I have better employment for my time and strength
rendered more intolerable by the jubilant shouts
75 than such arguments would imply.
25 that reach them. If I do forget, if I do not remember
At a time like this, scorching irony, not
those bleeding children of sorrow this day, “may my
convincing argument, is needed. What, to the
right hand forget her cunning, and may my tongue
American slave, is your Fourth of July? I answer: a
cleave to the roof of my mouth!” To forget them,
day that reveals to him more than all other days of
to pass lightly over their wrongs, and to chime in
80 the year, the gross injustice and cruelty to which he
30 with the popular theme, would be treason most
is the constant victim. To him your celebration is a
scandalous and shocking, and would make me a
sham; your denunciation of tyrants, brass-fronted
reproach before God and the world.
impudence; your shouts of liberty and equality,
My subject, then, fellow citizens, is American
hollow mockery; your prayers and hymns are to him
Slavery. I shall see this day and its popular
85 mere bombast, deception, impiety, and hypocrisy’s
35 characteristics from the slave’s point of view. In the
thin veil to cover up crimes which would disgrace a
name of humanity, which is outraged, in the name
nation of savages. There is not a nation of the earth
of liberty, which is fettered, in the name of the
guilty of practices more shocking and bloody than
Constitution and the Bible, which are disregarded
are the people of these United States at this very
and trampled upon, I will dare to denounce
90 hour.
40 everything that serves to perpetuate slavery.
But I fancy I hear some of my audience say it is
just in this circumstance that you and your brother
Abolitionists fail to make a favorable impression
on the public mind. Would you argue more and
45 denounce less, would you persuade more and
rebuke less, your cause would be much more likely
to succeed. But, I submit, where all is plain there is

56  |  © TPR Education IP Holdings, LLC


MASTERING POE

12. The principle rhetorical effect of the second Could Be True

READING
paragraph (lines 11–20) is to
A) Which answer is the trap answer?
B)
Why?
C) demonstrate how upset Douglass is about the
speech he is about to give.
Which answer is the right answer?

WRITING AND
D) highlight the irony of asking a slave to speak

LANGUAGE
about a holiday whose theme is liberty.
Why?

13. The primary purpose of lines 44–47 as they relate to Deceptive Language
the rest of the passage as a whole is to

MATH
A) discuss criticisms of Abolitionism which Which answer is the trap answer?
Douglass finds legitimate.
B) Why?
C) introduce a perspective which Douglass later
Which answer is the right answer?
refutes.
D) Why?

14. As used in line 50, “light” most nearly means Could Be True
A) brilliance.
Which answer is the trap answer?
B)
C) enlightenment. Why?
D)
Which answer is the right answer?

Why?

15. Douglass most likely uses the word “republicans” in Could Be True
line 57 to refer to
A) those who are socially conservative. Which answer is the trap answer?
B) those living in a democracy.
C) Why?
D)
Which answer is the right answer?

Why?

© TPR Education IP Holdings, LLC  |  57


SAT MANUAL

16. Douglass asks his audience, “Am I to argue…” Deceptive Language


READING

(lines 63–71) in order to


A) draw attention to the absurdity of the arguments Which answer is the trap answer?
in question.
B) ask his audience for assistance in determining Why?
the right subject for the speech.
C) Which answer is the right answer?
WRITING AND
LANGUAGE

D)
Why?

17. Based on lines 76–90, Douglass’ opinion of the Deceptive Language


Fourth of July celebration at the time of his speech
was one of Which answer is the trap answer?
MATH

A) excitement.
B) disgust. Why?
C)
Which answer is the right answer?
D)
Why?
What was Douglas' opnion...

Why does.. draw an distinction

18. In lines 81–87, Douglass draws a distinction Mostly Right/Slightly Wrong


between
A) the practices of Americans and savages. Which answer is the trap answer?
B)
Why?
C)
D) the ideals and practices of Americans. Which answer is the right answer?

Why?

19. Douglass’ primary rhetorical strategy in this passage Could Be True


is to
A) Which answer is the trap answer?
B) mention a series of arguments and then refuse
to make them. Why?
C)
Which answer is the right answer?
D) offer personal anecdotes as proof of slavery’s
evils.
Why?

58  |  © TPR Education IP Holdings, LLC


MASTERING POE

READING
WRITING AND
LANGUAGE
MATH
No Material On This Page

© TPR Education IP Holdings, LLC  |  59


SAT MANUAL

The passage that follows is adapted from a 1910 short story 45 But how was he to find him? On this the great
READING

that follows the actions of Aristide Valentin, head of the Valentin’s ideas were still in process of settlement.
Paris police, as he tracks the world’s most famous criminal, There was one thing which Flambeau, with all
a clever crook named Flambeau who is a master of disguise. his dexterity of disguise, could not cover, and that
was his singular height. If Valentin’s quick eye had
Flambeau was in England. Probably he would
50 caught a tall apple-woman, a tall grenadier, or even
travel as some minor clerk or secretary connected
a tolerably tall duchess, he might have arrested them
with it; but, of course, Valentin could not be certain.
WRITING AND

on the spot. But all along his train there was nobody
LANGUAGE

Line Nobody could be certain about Flambeau.


that could be a disguised Flambeau, any more than
5 It is many years now since this colossus of
a cat could be a disguised giraffe. About the people
crime suddenly ceased keeping the world in a
55 on the boat he had already satisfied himself; and
turmoil; and when he ceased, as they said after the
the people picked up at Harwich or on the journey
death of Roland, there was a great quiet upon the
limited themselves with certainty to six. There was
earth. But in his best days (I mean, of course, his
a short railway official travelling up to the terminus,
10 worst) Flambeau was a figure as statuesque and
three fairly short market gardeners picked up two
international as the Kaiser. Almost every morning
MATH

60 stations afterwards, one very short widow lady going


the daily paper announced that he had escaped
up from a small Essex town, and a very short Roman
the consequences of one extraordinary crime by
Catholic priest going up from a small Essex village.
committing another. He was a Gascon of gigantic
When it came to the last case, Valentin gave it up and
15 stature and bodily daring; and the wildest tales were
almost laughed. The little priest was so much the
told of his outbursts of athletic humour; how he
65 essence of those Eastern flats; he had a face as round
turned the juge d’instruction upside down and stood
and dull as a Norfolk dumpling; he had eyes as
him on his head, “to clear his mind”; how he ran
empty as the North Sea; he had several brown paper
down the rue de Rivoli with a policeman under each
parcels, which he was quite incapable of collecting.
20 arm. Each of his thefts was almost a new sin, and
The Eucharistic Congress had doubtless sucked out
would make a story by itself. It was he who ran the
70 of their local stagnation many such creatures, blind
great Tyrolean Dairy Company in London, with no
and helpless, like moles disinterred. Valentin was a
dairies, no cows, no carts, no milk, but with some
sceptic in the severe style of France, and could have
thousand subscribers. These he served by the simple
no love for priests. But he could have pity for them,
25 operation of moving the little milk cans outside
and this one might have provoked pity in anybody.
people’s doors to the doors of his own customers.
75 He had a large, shabby umbrella, which constantly
It was he who had kept up an unaccountable
fell on the floor. He did not seem to know which the
and close correspondence with a young lady
right end of his return ticket was. He explained with
whose whole letter-bag was intercepted, by the
a moon-calf simplicity to everybody in the carriage
30 extraordinary trick of photographing his messages
that he had to be careful, because he had something
infinitesimally small upon the slides of a microscope.
80 made of real silver “with blue stones” in one of his
A sweeping simplicity, however, marked many of his
brown-paper parcels. His quaint blending of Essex
experiments. It is said that he once repainted all the
flatness with saintly simplicity continuously amused
numbers in a street in the dead of night merely to
the Frenchman till the priest arrived (somehow) at
35 divert one traveler into a trap. It is quite certain that
Tottenham with all his parcels, and came back for his
he invented a portable mailbox, which he put up at
85 umbrella. When he did the last, Valentin even had
corners in quiet suburbs on the chance of strangers
the good nature to warn him not to take care of the
dropping postal orders into it. Lastly, he was known
silver by telling everybody about it. But to whomever
to be a startling acrobat. Despite his huge figure, he
he talked, Valentin kept his eye open for someone
40 could leap like a grasshopper and melt into the tree-
else.
tops like a monkey. Hence the great Valentin, when
he set out to find Flambeau, was perfectly aware that
his adventures would not end when he had found
him.

60  |  © TPR Education IP Holdings, LLC


MASTERING POE

Let’s try it without some answers! Use the line references, lead words, chronology, and best evidence answers to find

READING
your windows. Read carefully and answer the questions on your own, using support from the text.

1. The information in line 4 suggests primarily that 6. Which choice provides the best evidence for the
Flambeau was answer to the previous question?
A) Lines 32–33 (“A sweeping . . . experiments”)

WRITING AND
LANGUAGE
B) Lines 38–39 (“Lastly, he . . . acrobat”)
C) Lines 47–49 (“There was . . . height”)
D) Lines 52–54 (“But all . . . giraffe”)

2. As used in line 40, “melt” most nearly means 7. As used in line 49, “singular” most nearly means

MATH
3. The information in lines 14–41 primarily serves to 8. The statement in lines 63–64 (“When it . . .
laughed”) serves primarily to emphasize that
Valentin

4. As described in lines 45–46, Valentin could best be 9. According to the information in the passage, the
described as Roman Catholic priest can best be described as

5. Based on the information in the passage, it can be 10. Which choice provides the best evidence for the
inferred that Valentin believes that he is most likely answer to the previous question?
to recognize Flambeau through his A) Lines 57–64 (“There was . . . laughed”)
B) Lines 71–74 (“Valentin was . . . anybody”)
C) Lines 75–81 (“He had . . . parcels”)
D) Lines 85–89 (“When he . . . else”)

© TPR Education IP Holdings, LLC  |  61


SAT MANUAL

Armed with actual words from the text, let’s tackle the 6. Which choice provides the best evidence for the
READING

answer choices now! answer to the previous question?


A) Lines 32–33 (“A sweeping . . . experiments”)
1. The information in line 4 suggests primarily that B) Lines 38–39 (“Lastly, he . . . acrobat”)
Flambeau was C) Lines 47–49 (“There was . . . height”)
A) never in the same region as Valentin. D) Lines 52–54 (“But all . . . giraffe”)
WRITING AND
LANGUAGE

B) able to take on different roles to disguise himself.


C) not ready to give up a life of crime. 7. As used in line 49, “singular” most nearly means
D) likely to change his career at any given moment. A) original.
B) sole.
2. As used in line 40, “melt” most nearly means C) single.
A) thaw. D) exceptional.
B) vanish.
MATH

C) flow. 8. The statement in lines 63–64 (“When he . . .


laughed”) serves primarily to emphasize that
D) disintegrate.
Valentin
A) is frustrated by the number of short passengers
3. The information in lines 14–41 primarily serves to
on the train.
A) illustrate the ingenious and adventuresome
B) does not believe it likely that the priest is
nature of a criminal.
actually Flambeau.
B) outline the type of corruption common in large
C) is amused at the prospect of conversing with the
cities.
priest.
C) reveal the motivations behind one man’s
D) has decided to quit his detective work.
mischievous pranks.
D) evaluate the morality of a crook’s actions in light
of circumstances. 9. According to the information in the passage, the
Roman Catholic priest can best be described as
A) burdened and exhausted.
4. As described in lines 45–46, Valentin could best be
described as B) confused and naïve.
A) determined. C) pious and unlovable.
B) amused. D) good-natured and street-smart.
C) uncertain.
D) merciless. 10. Which choice provides the best evidence for the
answer to the previous question?
A) Lines 57–64 (“There was . . . laughed”)
5. Based on the information in the passage, it can be
inferred that Valentin believes that he is most likely B) Lines 71–74 (“Valentin was . . . anybody”)
to recognize Flambeau through his C) Lines 75–81 (“He had . . . parcels”)
A) athleticism. D) Lines 85–89 (“When he . . . else”)
B) stature.
C) simplicity.
D) gender.

62  |  © TPR Education IP Holdings, LLC


MASTERING POE

READING
MASTERING THE POE DRILL
(13 minutes)

The following passage is from Shannon Hall, “Volcanoes, As such, the study provides proof that the
Then an Asteroid, Wiped Out the Dinosaurs.” © 2018 volcanoes destabilized the climate, priming the
National Geographic. world for catastrophe, argues co-author Paul Wignall,

WRITING AND
a paleontologist at the University of Leeds.

LANGUAGE
What killed the dinosaurs? Few questions in
50 “Our paper swings the pendulum back a little
science have been more mysterious—and more
more toward volcanism,” he says.
contentious. Today, most textbooks and teachers
A hotter climate certainly helped send the non-
Line tell us that non-avian dinosaurs, along with three-
avian dinosaurs to their early grave, says Paul Renne,
5 fourths of all species on Earth, disappeared when
a geochronologist at the University of California,
a massive asteroid hit the planet near the Yucatán
55 Berkeley, who was not involved in the study. That’s
Peninsula some 66 million years ago.
because the uptick in temperature was immediately

MATH
But a new study published in the journal Geology
followed by a cold snap—a drastic change that likely
shows that an episode of intense volcanism in
set the stage for planet-wide disaster.
10 present-day India wiped out several species before
Imagine that some life managed to adapt to those
that impact occurred.
60 warmer conditions by moving closer toward the
The result adds to arguments that eruptions plus
poles, Renne says. “If you follow that with a major
the asteroid caused a one-two punch. The volcanism
cooling event, it’s more difficult to adapt, especially if
provided the first strike, weakening the climate so
it’s really rapid,” he says.
15 much that a meteor—the more deafening blow—was
In this scenario, volcanism likely sent the world
able to spell disaster for Tyrannosaurs rex and its late
65 into chaos, driving many extinctions alone and
Cretaceous kin.
increasing temperatures so drastically that most of
Whereas most geological evidence about
Earth’s remaining species couldn’t protect themselves
the mass extinction has been gathered from the
from that second punch when the asteroid hit.
20 North American continent, Laiming Zhang at
“The dinosaurs were extremely unlucky,” Wignall
China University of Geosciences in Beijing and
70 says.
his colleagues looked toward northern China. The
But it will be hard to convince Sean Gulick, a
region, they argue, is far enough from both the
geophysicist at the University of Texas at Austin,
impact crater in Mexico and the ancient lava flows
who co-led recent efforts to drill into the heart
25 known as the Deccan Traps in India, that it’s an
of the impact crater in Mexico. He points toward
ideal location to test for far-ranging climatic signals
75 several studies that have suggested that ecosystems
related to both events.
remained largely intact until the time of the impact.
The team analyzed ancient sediments at the
While he admits that the new research does
bottom of a lake basin and found that the lake’s
suggest there was some early ecological distress
30 temperature started to rise hundreds of thousands
that coincided with the Deccan Traps volcanism, he
of years before the impact. That warming coincides
80 would like to see evidence of this trend in places
with eruptions at the Deccan Traps in India, which
other than China.
likely spewed immense amounts of carbon dioxide
“There isn’t anything about that single basin that
into the atmosphere well before the impact. (Recent
equals a global phenomenon,” he says.
35 research also suggests that the dino-killing asteroid
Additionally, a forthcoming paper might make
hit just the right place on Earth to cause the mass
85 an even stronger case that the impact drove the
extinction.)
extinction alone, notes Jay Melosh, a geophysicist at
Although that jives with previous data showing
Purdue University who has worked on early results
pre-impact temperature swings across the world, the
from the drilling project. It looks as though the
40 team also made a more alarming find. Zhang and
divisive debate will continue with nearly as much
his colleagues discovered that many fossils within
90 ferocity as the events that rocked our world 66
the ancient sediments disappeared at the time of this
million years ago.
warming. Two-thirds of the extinctions within the
region actually occurred after the onset of volcanism
45 and before the impact.
© TPR Education IP Holdings, LLC  |  63
SAT MANUAL

22 25
READING

The primary purpose of this passage is to According to the passage, Laiming Zhang and his
A) provide conclusive evidence for the cause of the colleagues went to northern China to test their
extinction of the dinosaurs. theory because
B) criticize the work of Laiming Zhang as too A) it was close to the impact crater in Mexico.
narrow. B) it was at the base of the volcano in the Deccan
Traps.
WRITING AND

C) describe the findings and limitations of a recent


LANGUAGE

geological study. C) it was sufficiently far from both the impact


D) contrast the geological effects of volcanic crater and the volcano.
eruptions with those of a meteor impact. D) it was on the exact opposite side of the planet
from the impact crater.

23
26
MATH

What question was Zhang’s geological study


primarily intended to answer? Which choice provides the best evidence for the
A) Was a lack of adaptability a larger factor in the answer to the previous question?
extinction of dinosaurs than were geological A) Lines 22–27 (“The region . . . events”)
factors?
B) Lines 28–31 (“The team . . . impact”)
B) Were there additional circumstances beyond
C) Lines 31–34 (“That warming . . . impact”)
the meteor strike that led to extinctions?
D) Lines 38–40 (“Although that . . . find”)
C) How did the meteor strike and volcanic
eruptions affect the weather patterns of the
Cretaceous period?
27
D) Which critical event occurred first?
As used in line 38, the phrase “jives with” most
nearly means
24 A) corresponds to.
Which choice provides the best evidence for the B) proves.
answer to the previous question? C) undermines.
A) Lines 12–17 (“The result . . . kin”) D) highlights.
B) Lines 18–22 (“Whereas most . . . China”)
C) Lines 34–37 (“Recent research . . . extinction”)
28
D) Lines 61–63 (“If you . . . says”)
Which choice best expresses a reason that volcanic
eruptions might amplify the effect of a subsequent
meteor impact?
A) Lines 52–53 (“A hotter . . . grave”)
B) Lines 59–61 (“Imagine that . . . poles”)
C) Lines 64–68 (“In this . . . hit”)
D) Lines 69–70 (“The dinosaurs . . . says”)

64  |  © TPR Education IP Holdings, LLC


MASTERING POE

29 31

READING
As used in line 57, “snap” most nearly means Lines 88–91 (“It looks . . . ago”) primarily serve to
A) clip. A) show that Zhang’s research did a great deal to
B) ease. advance the consensus on dinosaur extinction.

C) crack. B) argue that the debate on dinosaurs will continue


for another 66 million years.
D) period.

WRITING AND
C) prove that researchers are closer to a definitive

LANGUAGE
answer on extinction than they were before
Zhang and Melosh’s studies.
30
D) acknowledge the strong research supporting
Based on lines 77–81, Sean Gulick would most both sides of an issue.
likely agree that
A) Zhang’s research caused him to re-evaluate his
own research.

MATH
B) the data from Zhang’s research provides some
new and useful information.
C) Zhang’s research expanded the geographical
range of previous studies.
D) previous studies had failed to prove ecological
distress at the Deccan Traps.

© TPR Education IP Holdings, LLC  |  65


SAT MANUAL

Summary
• What three markings should you use as you
work through POE?

________________________________________

• As you work through the answer choices, you


should be looking for ______________________.
wrong answers

• With Reading, it’s important to remember that


the correct answers are always
in the text
________________________________________.

• What are three common trap answers you’ll see


on the SAT?
deceptive languauge
________________________________________

________________________________________
mostly/slightlu wrong
coulb be true
________________________________________

• I have accomplished _________ of the 2 goals


stated at the beginning of this chapter.

66 | © TPR Education IP Holdings, LLC


MASTERING
POE
PRACTICE

As you work through the following passages, remember


to eliminate what you know is wrong, and then compare
your remaining answers.
Common Wrong Answers:
• Mostly Right/ Slightly Wrong
• Could Be True (But Isn’t Supported)
• Deceptive Language
It all comes down to the text!

67
SAT MANUAL
READING

PRACTICE PASSAGE 1: LITERATURE

Questions 1–10 are based on the following 35 even remember what happens in the story, but I do
passage. remember the faces of the dogs with their gigantic
eyes; it must have been a children’s picture book or
WRITING AND

This passage is adapted from Mieko Kawakami, “Ms Ice


LANGUAGE

something… Anyway, Ms Ice Sandwich has eyes just


Sandwich.” Translated by Louise Heal Kawai. © 2017 by
like those dogs do in that story, which has a soldier
Pushkin Press.
40 in it, and a castle, and there’s a princess—that story.
Around the train station, there’s only the The dogs with the giant eyes run around like crazy
chemist’s and the level crossing and the supermarket everywhere. Where was it they came from? And
that are lit up at night. But to be honest there’s not then someone got married to someone else, or they
Line much there in the daytime either—this town is really didn’t get married, I forget what the story was about.
5 just made up of houses, and the top floor of that two- 45 The day I first saw Ms Ice Sandwich, I was with
MATH

storey supermarket is full of laundry detergent and Mum, but when I said out loud in surprise, Look
buckets and dishes and toilet paper, all those things at her eyes!, Mum pretended not to hear me and
that’s not food, and the meat and the vegetables and started talking about something totally different, and
yogurt and fish and stuff is all on the ground floor, it wasn’t until we’d paid for our shopping and got
10 and everyone in the town comes here nearly every 50 completely outside the supermarket that she started
day to buy what they need. I watch Ms Ice Sandwich in on me. You have to stop that! You cannot say things
from the only door in and out of the supermarket; like that out loud, she can hear you, it’s rude. Mum’s
she’s always standing behind a big round glass face is awesome whenever she gets annoyed, if there
case, just to the left and a little bit behind the cash was an animal that didn’t know what being annoyed
15 registers, with that look on her face that’s like a 55 meant, then just one look at my mum’s face and
mixture of surprise and boredom, as she’s selling they’d get the idea. You could make a rubber stamp
sandwiches and salads and bread and things like that. of Mum’s face as a demo. I say, Why can’t I talk out
“Ms Ice Sandwich” is a name I made up; of loud about her eyes? They’re huge, they’re amazing!
course, I thought of it the minute I first saw her. Mum says, It doesn’t matter what they are, it’s not
20 Ms Ice Sandwich’s eyelids are always painted with 60 proper to talk about other people’s faces. Me: Why?
a thick layer of a kind of electric blue, exactly the Her: Because! All the way home I keep asking Mum
same colour as those hard ice lollies that have been why, but now she’s busy playing with her mobile
sitting in our freezer since last summer. There’s one phone and just keeps nodding and saying yeah every
more awesome thing about her—if you watch when so often. Well, I’m kind of getting used to her being
25 she looks down, there’s a sharp dark line above 65 like that these days, not paying attention to me, but
her eyes, as if when she closed her eyes, someone the more we walk the more it bugs me, so I stop
started to draw on two extra eyes with a felt-tip and say, If video games make you stupid, then what
pen but stopped halfway. It’s the coolest thing. And do mobile phones do to you? (This is me being real
then when she looks straight at me, she has these extreme to her.) She answers, What?, not stopping,
30 enormous eyes which are so big I feel like I get 70 I’m not playing a game, I’m updating something. It’s
swallowed up in them. They look exactly like the work. It’s hot, can we walk faster? And of course she
great big eyes of the dogs that I read about in a hasn’t taken her eyes off the screen for a second,
storybook long ago… What is the title of that book? madly pressing buttons, keeps on walking.
Well, it’s not only the title that I’ve forgotten, I can’t

CONTINUE
68  |  © TPR Education IP Holdings, LLC
MASTERING POE PRACTICE

1 5

READING
Over the course of the passage, the main focus It can reasonably be inferred from the passage that
shifts from the main reason the narrator gave Ms Ice Sandwich
A) a synopsis of a trip to the supermarket to an that name is because
examination of the eyes of two women. A) the narrator gave everyone in the store names
B) a description of the setting to an account of like that.
encounters with two women in the narrator’s life. B) Ms Ice Sandwich had sold the narrator the ice

WRITING AND
LANGUAGE
C) a summary of the narrator’s relationship with her cream stored in her freezer.
mother to a specific interaction between them. C) a physical feature reminded the narrator of a
D) a character sketch of a woman to a description type of ice cream treat.
of a confrontation between that woman and the D) Ms Ice Sandwich was always staring at her
narrator’s mother. phone.

MATH
2 6
The main purpose of lines 1–17 (“Around the . . . Which choice provides the best evidence for the
that”) is to answer to the previous question?
A) introduce the setting and title character of the A) Lines 11–17 (“I watch . . . that”)
story. B) Lines 18–23 (“Ms Ice . . . summer”)
B) describe the layout of the town and train station. C) Lines 41–44 (“The dogs . . . about”)
C) convey the narrator’s impressions of Ms Ice D) Lines 57–58 (“I say . . . amazing!”)
Sandwich before seeing her eyes.
D) illustrate the narrator’s annoyance with her
mother. 7
Which statement best characterizes the relationship
between the narrator and Ms Ice Sandwich?
3
A) They both share an unspoken understanding
As used in line 24, “awesome” most nearly means based on their boredom with small-town life.
A) artificial. B) The narrator doesn’t like how Ms Ice Sandwich
B) overwhelming. ignores her.
C) large. C) The narrator thinks Ms Ice Sandwich is the
D) enjoyable. most interesting person she knows.
D) The narrator enjoys looking at Ms Ice
Sandwich’s face.
4
The author mentions the storybook in line 33 in
order to 8

A) use something from her memory to help The narrator indicates that her mother is most likely
describe Ms Ice Sandwich’s eyes. looking at her phone because
B) describe the way dogs understand her mother’s A) her mother is looking up answers to the
expression. narrator’s questions.
C) illustrate how her encounter with Ms Ice B) walking too fast irritates her mother.
Sandwich felt like a fairytale. C) her mother is using the phone for her job.
D) express her regret at having forgotten the book’s D) her mother is playing a video game.
plot.
CONTINUE
© TPR Education IP Holdings, LLC  |  69
SAT MANUAL

9 10
READING

Based on the passage, the narrator would most Which choice provides the best evidence for the
likely agree with which of the following statements answer to the previous question?
about her mother? A) Lines 28–31 (“And then . . . them”)
A) She did not think video games are a good B) Lines 47–52 (“Mum pretended . . . rude”)
influence.
C) Lines 52–57 (“Mum’s face . . . demo”)
WRITING AND

B) She was always working too much.


LANGUAGE

D) Lines 64–70 (“Well I’m . . . something”)


C) She disliked Ms Ice Sandwich.
D) She was annoyed less often when the narrator
was younger.
MATH

CONTINUE
70  |  © TPR Education IP Holdings, LLC
MASTERING POE PRACTICE

READING
PRACTICE PASSAGE 2: HISTORY WITH GRAPH

Questions 11–21 are based on the following It was we, the people; not we, the white male
passage. citizens; nor we, the male citizens; but we, the whole
people, who formed the Union. And we formed

WRITING AND
This passage was adapted from a speech given by Susan

LANGUAGE
it, not to give the blessings of liberty, but to secure
B. Anthony in 1872 after she was arrested for voting in the
45 them; not to the half of ourselves and the half of our
Presidential Election.
posterity, but to the whole people—women as well
Friends and fellow citizens; I stand before you as men. And it is a downright mockery to talk to
tonight under indictment for the alleged crime of women of their enjoyment of the blessings of liberty
having voted at the last presidential election, without while they are denied the use of the only means
Line having a lawful right to vote. It shall be my work this 50 of securing them provided by this democratic-
evening to prove to you that in thus doing, I not only republican government—the ballot.

MATH
5
committed no crime, but, instead, simply exercised Charles Sumner, in his brave protests against the
my citizen’s rights, guaranteed to me and all United Fourteenth and Fifteenth Amendments,* insisted
States citizens by the Constitution, beyond the power that as soon as by the Thirteenth Amendment the
of any State to deny. 55 slaves became free men, the original powers of the
10 Our democratic-republican government is based United States Constitution guaranteed to them equal
on the idea of the natural right of every individual rights—the right to vote and to be voted for. When,
member thereof to a voice and a vote in making in 1871, I asked that senator to declare the power of
and executing the laws. We assert the province the United States Constitution to protect women in
of government to be to secure the people in the 60 their right to vote—as he had done for black men—
15 enjoyment of their inalienable right. We throw to he handed me a copy of all his speeches during that
the winds the old dogma that government can give reconstruction period, and said:
rights. No one denies that before governments were Put “sex” where I have “race” or “color,” and you
organized each individual possessed the right to have here the best and strongest argument I can
protect his own life, liberty and property. When 100 65 make for woman. There is not a doubt but women
20 to 1,000,000 people enter into a free government, have the constitutional right to vote, and I will never
they do not barter away their natural rights; they vote for a Sixteenth Amendment to guarantee it to
simply pledge themselves to protect each other in them. I voted for both the Fourteenth and Fifteenth
the enjoyment of them through prescribed judicial under protest; would have insisted that the power
and legislative tribunals. They agree to abandon the 70 of the original Constitution to protect all citizens in
25 methods of brute force in the adjustment of their the equal enjoyment of their rights should have been
differences and adopt those of civilization. The vindicated through the courts.
Declaration of Independence, the United States But, friends, when in accordance with Senator
Constitution, the constitutions of the several states Sumner’s counsel I went to the ballot-box last
and the organic laws of the territories, all alike 75 November, and exercised my citizen’s right to vote,
30 propose to protect the people in the exercise of their the courts did not wait for me to appeal to them—
God-given rights. Not one of them pretends to they appealed to me, and indicted me on the charge
bestow rights. of having voted illegally.
The preamble of the Constitution says: Though the words “persons,” “people,”
We, the people of the United States, in order to 80 “inhabitants,” “electors,” “citizens,” are all used
35 form a more perfect union, establish justice, insure indiscriminately in the national and state
domestic tranquility, provide for the common constitutions, there was always a conflict of
defense, promote the general welfare, and secure the opinion, prior to the war, as to whether they were
blessings of liberty to ourselves and our posterity, do synonymous terms, but whatever room there was
ordain and establish this Constitution for the United 85 for doubt, under the old regime, the adoption of the
40 States of America.
CONTINUE
© TPR Education IP Holdings, LLC  |  71
SAT MANUAL

Fourteenth Amendment settled that question forever 12


READING

in its first sentence:


As used in line 15, “enjoyment” most nearly means
All persons born or naturalized in the United
States, and subject to the jurisdiction thereof, are A) use.
90 citizens of the United States, and of the state wherein B) delight.
they reside. C) satisfaction.
The only question left to be settled now is: Are
D) justice.
WRITING AND

women persons? And I hardly believe any of our


LANGUAGE

opponents will have the hardihood to say they are


95 not. Being persons, then, women are citizens; and
no state has a right to make any law, or to enforce 13
any old law, that shall abridge their privileges or Based on the passage, which of the following best
immunities. describes the relationship between laws and natural
*The Fourteenth and Fifteenth Amendments granted citizenship to persons rights?
born in the United States and prohibited denying anyone the right to vote A) Allowing elected officials to govern a country
MATH

based on race, respectively.


requires the citizens of that country to give up
some of their natural rights.
B) A government makes laws in order to create
natural rights for its citizens.
C) Judicial and legislative tribunals decide on and
confer the natural rights of citizens.
D) It is through laws that a government protects
citizens’ preexistent natural rights.

14
Which choice provides the best evidence for the
answer to the previous question?
A) Lines 4–9 (“It shall . . . deny”)
B) Lines 27–32 (“The Declaration . . . rights”)
C) Lines 41–43 (“It was . . . Union”)
D) Lines 88–91 (“All persons . . . reside”)
This graphic, created by Sandra Opdycke in 2000 for
The Rutledge Historical Atlas of Women in America, depicts
women’s suffrage in the United States before the passage
of the Nineteenth Amendment in 1920, which guaranteed
15
women’s right to vote. The author mentions the preamble of the
Constitution (lines 34–40) primarily in order to
11 A) provide support for her argument for gender
equality.
What is the primary purpose of this passage?
B) illustrate the degree to which the Constitution
A) To argue for a new amendment granting women
does not grant women certain liberties.
the right to vote
C) prove that the Constitution should be revised to
B) To provide a legal defense for the author’s
include women.
upcoming trial
D) explain the bill for which she was voting when
C) To demonstrate that women already have the
arrested.
right to vote
D) To encourage voters to elect supporters of
women’s suffrage CONTINUE
72  |  © TPR Education IP Holdings, LLC
MASTERING POE PRACTICE

16 20

READING
Which choice provides the best evidence for the The principal rhetorical effect of the question in
answer to the previous question? lines 92–93 is to
A) Lines 17–19 (“No one . . . property”) A) highlight the obviousness of an answer in the
B) Lines 24–26 (“They agree . . . civilization”) affirmative.

C) Lines 41–43 (“It was . . . Union”) B) begin a genuine dialogue on the legal definition
of “person.”

WRITING AND
LANGUAGE
D) Lines 47–51 (“And it . . . ballot”)
C) demonstrate the absolute necessity of revising
voting rights laws.
17 D) suggest a topic for a future political debate.

Based on the information in the passage, Sumner


protested the Fifteenth Amendment because
21
A) he did not believe that former slaves deserved

MATH
the right to vote. Which claim about women’s suffrage is supported
by the graphic?
B) he thought it was redundant with the
Constitution. A) After the passage of the Thirteenth Amendment,
American women in all states had no barriers to
C) he was primarily focused on the Thirteenth and voting.
Fourteenth Amendments.
B) Anthony could have avoided jail had she
D) he had been counseled by Anthony to do so. traveled to Colorado (CO) to vote in 1872.
C) Until 1920, the state governments of Alabama
(AL) and Georgia (GA) didn’t believe women
18
were people.
As used in line 86, “settled” most nearly means D) Some states allowed women to vote even before
A) rested. the 19th Amendment guaranteed this right.
B) established.
C) silenced.
D) decided.

19
The primary purpose of lines 88–91 as they relate to
the rest of the passage is to
A) provide evidence for an earlier claim.
B) offer a position that the author then refutes.
C) illustrate the difficulties of interpreting
constitutional law.
D) justify the author’s personal interpretation of
the law.

CONTINUE
© TPR Education IP Holdings, LLC  |  73
SAT MANUAL
READING

PRACTICE PASSAGE 3: SCIENCE

Questions 22–31 are based on the following you only care if the workers are going on strike.
passage. 40 That’s what you get from reading the newspaper.
Your conscious mind is that newspaper. Your
WRITING AND

This passage is an excerpt from David Eagleman, Incognito:


LANGUAGE

brain buzzes with activity around the clock, and, just


The Secret Lives of the Brain. © 2011 by David Eagleman.
like the nation, almost everything transpires locally:
Used by permission of Pantheon Books, an imprint of the
small groups are constantly making decisions and
Knopf Doubleday Publishing Group, a division of Penguin
45 sending out messages to other groups. Out of these
Random House LLC. All rights reserved.
local interactions emerge larger coalitions. By the
The brain is a complex system, but that doesn’t time you read a mental headline, the important
mean it’s incomprehensible. Our neural circuits were action has already transpired, the deals are done.
carved by natural selection to solve problems that You have surprisingly little access to what happened
MATH

Line our ancestors faced during our species’ evolutionary 50 behind the scenes. Entire political movements
5 history. Your brain has been molded by evolutionary gain ground-up support and become unstoppable
pressures just as your spleen and eyes have been. before you ever catch wind of them as a feeling or an
And so has your consciousness. Consciousness intuition or a thought that strikes you. You’re the last
developed because it was advantageous, but one to hear the information.
advantageous only in limited amounts. 55 However, you’re an odd kind of newspaper
10 Consider the activity that characterizes a nation reader, reading the headline and taking credit for the
at any moment. Factories churn, telecommunication idea as though you thought of it first. You gleefully
lines buzz with activity, businesses ship products. say, “I just thought of something!”, when in fact
People eat constantly. Sewer lines direct waste. All your brain performed an enormous amount of work
across the great stretches of land, police chase 60 before your moment of genius struck. When an idea
15 criminals. Handshakes secure deals. Lovers is served up from behind the scenes, your neural
rendezvous. Secretaries field calls, teachers profess, circuitry has been working on it for hours or days or
athletes compete, doctors operate, bus drivers years, consolidating information and trying out new
navigate. You may wish to know what’s happening combinations. But you take credit without further
at any moment in your great nation, but you can’t 65 wonderment at the vast, hidden machinery behind
20 possibly take in all the information at once. Nor the scenes.
would it be useful, even if you could. You want a And who can blame you for thinking you deserve
summary. So you pick up a newspaper—not a dense the credit? The brain works its machinations in
paper like the New York Times but lighter fare such secret, conjuring ideas like tremendous magic. It
as USA Today. You won’t be surprised that none 70 does not allow its colossal operating system to be
25 of the details of the activity are listed in the paper; probed by conscious cognition. The brain runs its
after all, you want to know the bottom line. You show incognito.
want to know that Congress just signed a new tax
law that affects your family, but the detailed origin
of the idea—involving lawyers and corporations and
30 filibusters— isn’t especially important to that new
bottom line. And you certainly wouldn’t want to
know all the details of the food supply of the nation—
how the cows are eating and how many are being
eaten—you only want to be alerted if there’s a spike
35 of mad cow disease. You don’t care how the garbage
is produced and packed away; you only care if it’s
going to end up in your backyard. You don’t care
about the wiring and infrastructure of the factories;
CONTINUE
74  |  © TPR Education IP Holdings, LLC
MASTERING POE PRACTICE

22 26

READING
Based on the passage, the author’s intended Which choice provides the best evidence for the
audience is most likely answer to the previous question?
A) medical students to whom he teaches clinical A) Lines 20–21 (“Nor would . . . could”)
psychology. B) Line 40 (“That’s what . . . newspaper”)
B) fellow scientists who work in evolutionary C) Line 41 (“Your conscious . . . newspaper”)
psychology.

WRITING AND
LANGUAGE
D) Lines 60–64 (“When an . . . combinations”)
C) enthusiasts who are interested in how the mind
functions.
D) newspaper readers seeking to understand their 27
own biases.
As used in line 49, “surprisingly” most nearly means
A) unexpectedly.
23 B) frighteningly.

MATH
As used in line 5, “molded” most nearly means C) surreptitiously.
A) curved. D) abnormally.
B) decayed.
C) formed.
28
D) pressed.
The author mentions “odd kind . . . reader”
(lines 55–56) primarily in order to
24 A) criticize the audience’s limited comprehension
skills.
The primary purpose of the second paragraph
(lines 10–40) is to B) support the thesis that the brain is a newspaper.
A) prove that a neurological framework is alert to C) introduce the final aspect of the metaphor.
all dangers. D) contradict a prior assumption.
B) summarize every detail covered in a newspaper.
C) demonstrate how one’s neural circuitry
29
constantly processes ideas.
D) provide a metaphor that develops one of the Based on the passage, the “credit” mentioned in
author’s main points. line 64 would most likely be claimed by
A) the brain, for performing tremendous
machinations.
25 B) the machinery, for its work behind the scenes.
The principal rhetorical purpose of the phrases in C) the consciousness, for coming up with an idea.
lines 26–39 (“You want . . . strike”) is to D) the news industry, for filtering massive amounts
A) support the argument in the previous paragraph. of information.
B) resolve a contradiction in neurological versus
sociological theory.
C) offer proof that society needs different elements
in order to work effectively.
D) illustrate that one’s conscious mind has limited
access to sensory information.

CONTINUE
© TPR Education IP Holdings, LLC  |  75
SAT MANUAL

30 31
READING

The passage suggests that the brain Which choice provides the best evidence for the
A) functions most effectively when it’s divided into answer to the previous question?
separate parts. A) Lines 24–26 (“You won’t . . . line”)
B) operates on a different level than the B) Lines 45–46 (“Out of . . . coalitions”)
consciousness. C) Lines 53–54 (“You’re the . . . information”)
WRITING AND

C) prevents the conscious mind from reaching its


LANGUAGE

D) Lines 69–71 (“It does . . . cognition”)


full potential.
D) masks discoveries that are upsetting or
unsettling.
MATH

CONTINUE
76  |  © TPR Education IP Holdings, LLC
MASTERING POE PRACTICE

READING
PRACTICE PASSAGE 4: DUAL HISTORY

Questions 32-41 are based on the following that any nation that is in the wrong and waits nine
passages. 40 months before it goes to war never will go to war.

WRITING AND
Passage 1 is adapted from a speech by Woodrow Wilson

LANGUAGE
at the Coliseum, St. Louis, MO. September 1919. Passage 2
Passage 2
is from a speech by Henry Cabot Lodge, August 12, 1919,
I am as anxious as any human being can be to
before the US Senate.
have the United States render every possible service
to the civilization and the peace of mankind, but I
Passage 1 am certain we can do it best by not subjecting our
I have heard some men say with an amazing 45 sovereignty to other nations. The independence
ignorance that the covenant of the League of of the United States is not only more precious

MATH
Nations was an arrangement for war. Very well, to ourselves but to the world than any single
Line then, the other arrangement—what would it be? An possession. We have made mistakes in the past. We
5 arrangement for peace? I cannot bring my credulity shall make mistakes in the future and fall short of
up to that point. I wonder if some of the gentlemen 50 our own best hopes. But nonetheless is there any
who are commenting upon this treaty ever read it! country today on the face of the earth which can
If anybody will tell me which of them has not, I compare with this in ordered liberty, in peace, and
will send him a copy. It is written in two languages. in the largest freedom? But it is well to remember
10 On this side is the English and on that side is the that we are dealing with nations every one of which
French, and since it is evident that some men do 55 has a direct individual interest to serve, and there is
not understand English, I hope that they understand grave danger in an unshared idealism. Ask yourself
French. There are excellent French dictionaries by whether the situation of the United States is not
which they can dig out the meaning if they cannot the best to be found. I will go as far as anyone in
15 understand English. It is the plainest English that world service, but the first step to world service is
you should desire, particularly the covenant of the 60 the maintenance of the United States. You may call
League of Nations. There is not a phrase of doubtful me selfish if you will, conservative or reactionary,
meaning in the whole document. but an American I was born, an American I have
And what is the meaning? It is that the covenant remained all my life. I can never be anything else
20 of the League of Nations is a covenant of arbitration but an American, and I must think of the United
and discussion. I dare say that everybody you have 65 States first, I am thinking of what is best for the
heard talk about this discusses Article X. Well, there world, for if the United States fails the best hopes
are 25 other articles in it, and all of them are about of mankind fail with it. I have never had but one
something else. They discuss how soon and how allegiance—I cannot divide it now. Internationalism
25 quick we can get out of it. The essential matter, my is to me repulsive. National I must remain, and in
fellow-citizens, is this: all the fighting nations of the 70 that way I, like all other Americans, can render the
world are in it, and what do they promise? This is amplest service to the world. The United States is
the center of the document. They promise that they the world’s best hope, but if you fetter her in the
never will go to war without first either submitting interests and quarrels of other nations, you will
30 the question at issue to arbitration and absolutely destroy her power for good and endanger her very
abiding by the decision of the arbitrators, or, if they 75 existence. Leave her to march freely through the
are not willing to submit it to arbitration, they will centuries to come. Strong, generous and confident,
submit it to discussion by the Council of the League, she has nobly served mankind. Beware how you
that they will give the Council of the League six trifle with your marvelous inheritance, for if
35 months in which to consider it, and that if they do we stumble and fall, freedom and civilization
not like the opinion of the Council they will wait 80 everywhere will go down in ruin.
three months after the opinion is rendered before
going to war. And I tell you, my fellow-citizens,
CONTINUE
© TPR Education IP Holdings, LLC  |  77
SAT MANUAL

32 36
READING

In the first paragraph of Passage 1, Wilson Based on Passage 2, Lodge would be most likely
primarily discusses the languages in which the to agree with which claim about the United States
treaty is written in order to regarding its foreign relations?
A) imply that the meaning of the document should A) It will serve others best if it prioritizes its own
be clear. welfare before the welfare of other nations.
B) suggest that members of the League of Nations B) It should attempt to improve its foreign
WRITING AND
LANGUAGE

do not understand the document. relations by providing the aid that struggling
C) indicate that England and France are the nations have demanded.
nations that wrote the document. C) It has suffered unfairly due to past political and
D) introduce the argument that the document economic conflicts between other nations.
promotes only peace. D) It can only succeed as a nation if it remains
completely isolated from other countries.
MATH

33
37
What does Passage 1 suggest about the League of
Nation’s provisions for conflict, as framed in its Which choice provides the best evidence for the
covenant? answer to the previous question?
A) It only includes one article that relates to the A) Lines 53–56 (“But it . . . idealism”)
topic of conflict. B) Lines 63–67 (“I can . . . it”)
B) It requires disagreements to be mediated C) Lines 67–69 (“I have . . . repulsive”)
through talks before military conflict is
considered. D) Lines 75–77 (“Leave her . . . mankind”)

C) It promotes going to war without trying to


resolve conflict any other way.
38
D) It works best if every nation agrees to it.
As used in line 75, “march” most nearly means
A) conquer.
34 B) assemble.
Which choice provides the best evidence for the C) proceed.
answer to the previous question? D) parade.
A) Lines 19–21 (“It is . . . discussion”)
B) Lines 21–22 (“I dare . . . X”)
C) Lines 24–25 (“They discuss . . . it”)
D) Lines 38–40(“And I . . . war”)

35
As used in line 52, “ordered” most nearly means
A) dictated.
B) regulated.
C) purchased.
D) categorized.

CONTINUE
78  |  © TPR Education IP Holdings, LLC
MASTERING POE PRACTICE

39 40

READING
Which choice identifies a central tension between Both passages discuss the issue of United States
the two passages? foreign policy in relationship to
A) Wilson expresses uncertainty about the A) the avoidance of war.
necessity of maintaining peace, but Lodge B) global trade deals.
insists that doing so is both a national and
international priority. C) patriotic sentiment.

WRITING AND
LANGUAGE
B) Wilson claims that an international alliance D) potential obligations between countries.
would limit military conflicts, but Lodge fears
that international agreements would force the
U.S. into more of them. 41
C) Wilson faults people for misunderstanding U.S. In the context of each passage as a whole,
foreign policy, but Lodge praises nationalists for the questions in lines 25–27 (“The essential . . .
their patriotism. promise?”) of Passage 1 and lines 50–53

MATH
D) Wilson favors the economic benefits of (“But nevertheless . . . freedom?”) of Passage 2
globalism, but Lodge prefers the cultural primarily function to help each speaker
stability that isolationism affords. A) stress the importance of cooperation.
B) refute a contrary idea.
C) emphasize the main point of their argument.
D) propose a route to peace.

CONTINUE
© TPR Education IP Holdings, LLC  |  79
SAT MANUAL
READING

PRACTICE PASSAGE 5: SCIENCE

Questions 43–52 are based on the following control, and the ventral striatum, which is involved
passages. in the reward system. The jokes “sent the brain into
overdrive.”
WRITING AND

The following passage is adapted from Virginia Hughes,


LANGUAGE

But the most interesting data concerns a little-


“What Makes a Pun Funny.” © 2011 from The Last Word on
45 known brain region called the inferior frontal gyrus
Nothing.
(IFG), a ridge on the bottom half of the frontal
My name is Ginny and I’m an adult pun-lover. lobe. The left IFG lights up more during funny puns
When I hear a good one—Photons have mass? I than regular jokes, and shows more activity during
didn’t even know they were Catholic!—I don’t roll my funny puns than unfunny puns, the study found. So
Line eyes or smirk. I double over laughing, like a 7-year- 50 this tiny little area, it seems, is the part of our brain
5 old. What is it exactly that makes a pun funny (at that distinguishes between funny and unfunny, at
MATH

least to those of us who humbly accept the power of least in these participants and for these kinds of
the pun)? jokes.
That’s the underlying question of a brain imaging Of course, pinpointing the regions of the brain
study I came across last week. Its pretty pictures 55 that process humor doesn’t tell us why we experience
10 don’t answer the question, really, but they’re jokes as funny, as the researchers readily admit. But
interesting all the same. And provocative: the it could be useful for something far more important,
data could have way-down-the-road relevance for in my opinion.
communicating with people in vegetative states. The reason I found the study, which was
The researchers, led by Adrian Owen at the 60 published back in June, is because I’ve been thinking
15 University of Western Ontario, focused on three a lot about what brain imaging can and cannot
types of jokes: tell us. The lead investigator, Adrian Owen, made
Regular joke: Why did Cleopatra bathe in milk? headlines several years ago for demonstrating an
Because she couldn’t find a cow tall enough for a incredible application of fMRI: using brain activity
shower. 65 to communicate with people whose active minds are
20 Funny pun: Why were the teacher’s eyes crossed? trapped inside of inactive bodies.
Because she couldn’t control her pupils. In 2006, Owen reported in Science that when
Unfunny pun: What was the problem with the a woman in a vegetative state was put in a brain
other coat? It was difficult to put on with the paint- scanner and asked to think about playing tennis
roller. 70 or walking around her house, these thoughts lit
25 Reading each joke produces a distinct cognitive up predictable areas of her brain. Then last year, in
experience, at least for me. So I’d guess that my brain the New England Journal of Medicine, Owen showed
is doing a different kind of processing for each. The that a seemingly unconscious man could answer yes
study investigated this using a functional magnetic or no questions by mentally acting out the tennis
resonance imaging (fMRI) machine, which measures 75 scene for ‘yes’ or the house-walking for ‘no’.
30 blood flow (and therefore, indirectly, neuron In news stories about this research, the question
activity) in the brain. A dozen adult volunteers that came up over and over again was whether
rolled on their backs into the scanner and listened a doctor could use this method to ask patients
to recordings of jokes and sentences read out loud as whether they wanted to be taken off life support. An
the machine recorded their brain activity. 80 ethical minefield, right?
35 Humor is complicated, apparently. Many, many In order for that to be permissible, you’d have to
areas of the brain light up more during jokes (regular show (among many, many other things, I’d hope)
or puns) than non-jokes, the study found. These that the person can not only answer relatively simple,
include the amygdala, which processes emotions, objective questions, but can grapple with much more
the hypothalamus, which is responsible for a lot of 85 complex thinking, such as the consequences of their
40 automatic processes, like temperature and hormone decision and their feelings about it. That’s where the
CONTINUE
80  |  © TPR Education IP Holdings, LLC
MASTERING POE PRACTICE

HW: PRACTICE 1,2 UNDER MASTERING POE

jokes come in: humor is not PRACTIC


simple. If 3researchers
(P44) (P68) (P71)
46P47)

READING
could figure out a way to tap into various humor Which choice provides the best evidence for the
circuits with jokes, could they gauge the emotional answer to the previous question?
90 capacity of an unconscious patient?
“That’s actually the sole reason for doing this A) Lines 27–31 (The study . . . brain)
study,” Owen told a reporter in July. “We need a way B) Lines 35–37 (Many, many . . . found)
to try to access emotions in comatose and vegetative C) Lines 37–42 (These include . . . system)

WRITING AND
states.”

LANGUAGE
D) Lines 47–49 (The left . . . found)

47
43
The main purpose of the seventh paragraph
Which choice best summarizes the passage?
(lines 54–58) is to
A) A researcher describes her affinity for puns and
A) introduce the potential applications of brain

MATH
conducts experiments as to why they are funny.
activity during jokes.
B) An author shares research findings of a study
B) discount the findings of the study because it is
and their potential implications.
unclear why some jokes are considered funny.
C) A biologist gives a detailed description of
C) challenge the researchers to design a more
brain imagery on an fMRI scan to record brain
widely-applicable study.
activity in the amygdala.
D) note a concession to the researchers’ findings
D) An individual evaluates the emotional capacity
that are troublesome to further study.
of unconscious patients by using fMRI scans.

48
44
The author mentions the woman in a vegetative
In lines 17–24, the comparison of types of jokes
state in lines 67–71 primarily in order to
mainly serves to
A) illustrate the cognitive benefits of sharing jokes
A) convey the differences in cognitive processes
with vegetative individuals.
found in the study.
B) show a different use for a technology applied in
B) distinguish between jokes and puns.
an earlier study.
C) suggest the study done at the University of
C) explain an occurring phenomenon that
Western Ontario was subjective.
discredits the previous study.
D) make a distinction among different focuses of
D) present a situation in which non-humor is more
research.
predictable than humor.

45
The passage indicates that a funny joke
A) is pinpointed to regions in the brain that
process humor.
B) is processed by the left inferior frontal gyrus.
C) is directly related to blood flow to the
hypothalamus.
D) could produce more brain activity than does a
fact.

CONTINUE
© TPR Education IP Holdings, LLC  |  81
SAT MANUAL

49 51
READING

Based on information in the passage, it can As used in line 90, “capacity” most nearly means
reasonably be inferred that critics of Owen’s studies A) amplitude.
A) are concerned with the moral issues B) magnitude.
surrounding communication with individuals
who are in vegetative states. C) faculty.

B) believe that communicating with individuals in D) extent.


WRITING AND
LANGUAGE

vegetative states is inherently unethical.


C) argue that scientifically measuring responses to
52
humor is inappropriate.
D) refuse to see the connection between The purpose of the last paragraph is to
predictable responses to non-humor and A) inspire hope that patients in vegetative or
unpredictable responses to humor. unconscious states can heal.
B) explain why researchers chose to use jokes for
MATH

brain imaging research.


50 C) illustrate the limitations of simple brain
Which choice provides the best evidence for the functions in patients in vegetative states.
answer to the previous question? D) show the ethical dangers of telling jokes to
A) Lines 54–56 (“Of course . . . admit”) unresponsive patients.
B) Lines 59–62 (“The reason . . . us”)
C) Lines 81–86 (“In order . . . it”)
D) Lines 91–94 (“That’s actually . . . states”)

Answers can be found on pages 724–725.

82 | © TPR Education IP Holdings, LLC


IDENTIFYING
QUESTION
TYPES

I have always imagined that Paradise will be a


kind of library.
— Jorge Luis Borges

83
SAT MANUAL
READING

GOALS REVIEW
By the end of this chapter you will be able to

• Identify questions from each category


• Understand specific approaches for each question type
WRITING AND
LANGUAGE

QUESTION TYPES
All the Reading questions on the SAT fit into one of three categories. Each cat-
egory requires different tasks, ranging from retrieving details explicitly stated in
the text to analyzing an author’s point of view to synthesizing information from
the text with information in a chart or graph. Recognizing the question types and
MATH

knowing how to approach each one will help you move through the test more
accurately and more efficiently.

QUESTION CATEGORIES
1. Information and Ideas
2. Rhetoric
3. Synthesis

Information and Ideas


The Information and Ideas questions focus on the informational content of the
text. Information and Ideas questions will ask you to:

• Determine meanings of words and phrases based on context


_____________

• main idea
Determine the _________________________________ of a passage

• Summarize a passage or its key elements

• Trace _______________
relationships (cause/effect, compare/contrast)

As you work through these questions, make sure you can physically put your pen-
cil on evidence in the text that supports keeping or eliminating certain answers.

84 | © TPR Education IP Holdings, LLC


IDENTIFYING QUESTION TYPES

READING
Rhetoric
The Rhetoric questions focus on analyzing the rhetorical content of the text.
Rather than asking what did the author say?, these questions will focus on
__________________________.
why did the author say it This content may include:

• word choice

WRITING AND
• text structure

LANGUAGE
• point of view
• purpose
• arguments

MATH
Synthesis
Synthesis questions assess your ability to make connections ___________________
between mutiple sources
________________.

• analyze two different but related passages


• analyze quantitative information in an accompanying graph, table,
chart, or other graphic

© TPR Education IP Holdings, LLC  |  85


SAT MANUAL

This passage is adapted from Robert M. Yerkes, Ph.D., The Since 1903 I have had under observation
READING

Dancing Mouse: A Study in Animal Behavior. © 1907 by constantly from two to one hundred dancing mice.
Macmillan. 50 The original pair was presented to the Harvard
Psychological Laboratory by Doctor A.G. Cleghorn
The variety of mouse which is known as the
of Cambridge. I have obtained specimens, all
Japanese dancing or waltzing mouse has been
strikingly alike in markings, size, and general
of special interest to biologists and to lovers of
behavior, from animal dealers in Washington,
Line pets because of its curious movements. Haacke,
WRITING AND

55 Philadelphia, and Boston. Almost all of the dancers


LANGUAGE

5 in Brehm’s “Life of Animals,” writes as follows


which I have had, and they now number about
concerning certain mice which were brought to
four hundred, were white with patches, streaks, or
Europe from China and Japan: “From time to time
spots of black. The black markings occurred most
a Hamburg dealer in animals sends me two breeds
frequently on the neck, ears, face, thighs, hind legs,
of common mice, which he calls Chinese climbing
60 about the root of the tail, and occasionally on the
10 mice and Japanese dancing mice. It is true that the
tail itself. In only one instance were the ears white,
first are distinguished only by their different colors,
and that in the case of one of the offspring of a male
for their climbing accomplishments are not greater
MATH

which was distinguished from most of his fellows


than those of other mice. The color, however, is
by the possession of one white ear. I have had a few
subject to many variations. Besides individuals of
65 individuals whose markings were white and gray
15 uniform gray, light yellow, and white color, I have
instead of white and black.
had specimens mottled with gray and white, and
All of my dancers had black eyes and were
blue and white. Tricolored mice seem to be very
smaller as well as weaker than the albino mouse
rare. It is a known fact that we also have white,
and the gray house mouse. The weakness indicated
black, and yellow mice and occasionally pied ones,
70 by their inability to hold up their own weight or
20 and the Chinese have profited by these variations
to cling to an object curiously enough does not
of the common mouse also, to satisfy their fancy
manifest itself in their dancing; in this they are
in breeding animals. The Japanese, however, who
indefatigable. Frequently they run in circles or whirl
are no less enthusiastic on this point, know how
about with astonishing rapidity for several minutes
to transform the common mouse into a really
75 at a time. Zoth, who measured the strength of the
25 admirable animal. The Japanese dancing mice,
dancer in comparison with that of the common
which perfectly justify their appellation, also occur
mouse, found that it can hold up only about 2.8
in all the described colors. But what distinguishes
times its own weight, whereas the common white
them most is their innate habit of running around,
mouse can hold up 4.4 times its weight. No other
describing greater or smaller circles or more
80 accurate measurements of the strength, endurance,
30 frequently whirling around on the same spot with
or hardiness of the dancer are available. They are
incredible rapidity. Sometimes two or, more rarely,
usually supposed to be weak and delicate, but my
three mice join in such a dance, which usually
own observations cause me to regard them as
begins at dusk and is at intervals resumed during
exceptionally strong in certain respects and weak in
the night, but it is usually executed by a single
85 others.
35 individual.”
As a rule the dancing mouse is considerably
smaller than the common mouse, and observers
agree that there are also certain characteristic
peculiarities in the shape of the head. One of the
40 earliest accounts of the animal which I have found,
that of Landois, states, however, that the peculiarities
of external form are not remarkable. Landois further
remarks, with reason, that the name dancing mouse
is ill chosen, since the human dance movement is
45 rather a rhythmic hopping motion than regular
movement in a circle. As he suggests, they might
more appropriately be called “circus course mice”.

86  |  © TPR Education IP Holdings, LLC


IDENTIFYING QUESTION TYPES

READING
Information and Ideas
These questions usually ask what the author or passage says. Find specific evidence in the text to support your answer.

4. The passage most strongly suggests that a “pied” Where in the passage will you find this answer?
(line 19) mouse would be most easily identified

WRITING AND
LANGUAGE
by its
A) color.
B) size.
C) speed.
What does the text actually say?
D) head shape.

MATH
8. According to the passage, the author mainly Where in the passage will you find this answer?
gets his dancing mice from
A) university laboratories.
B) Dr. Cleghorn of Cambridge.
C) various dealers in different locations.
D) circus dealers. What does the text actually say?

Are there any trap answers?

© TPR Education IP Holdings, LLC  |  87


SAT MANUAL

12. The main idea of the final paragraph (lines 67–85) Where in the passage will you find this answer?
READING

is that
A) the gray house mouse makes a better pet
than does a dancing mouse.
B) dancing mice have less strength but greater
endurance than other types of mice. What does the text actually say?
WRITING AND

C) although dancing mice appear weaker, they


LANGUAGE

can actually lift twice as much as common


white mice.
D) dancers have to be strong in order to dance
as long as they do.
Are there any trap answers?
MATH

Vocab-in-Context
These questions are a common subset of Information and Ideas questions. You have at least one on every Reading
passage. Remember that these questions are not testing whether or not you know the definition of a word, but rather
that you can establish the meaning of the word based on the context.

1. As used in line 4, “curious” most nearly means Where in the passage will you find this answer?
A) prying.
B) active.
C) unusual.
D) interested.
What word or short phrase would you put in the
blank?

Are there any trap answers?

88  |  © TPR Education IP Holdings, LLC


IDENTIFYING QUESTION TYPES

5. As used in line 29, “describing” most nearly means Where in the passage will you find this answer?

READING
A) explaining.
B) twisting.
C) classifying.
D) outlining.
What word or short phrase would you put in the

WRITING AND
blank?

LANGUAGE
Are there any trap answers?

MATH
13. As used in line 78, “common” most nearly means Where in the passage will you find this answer?
A) daily.
B) mediocre.
C) communal.
D) ordinary.
What word or short phrase would you put in the
blank?

© TPR Education IP Holdings, LLC  |  89


SAT MANUAL
READING

Paired Questions
Another question type in the Information & Ideas category is the Best Evidence
question. Remember, these most often come in pairs. There are several different
ways you can work through them.

• Can you answer the first question on its own, and then identify the
WRITING AND

lines you used?


LANGUAGE

• Can you find any connections between the lines in the best evidence
question and the answers in the first question?
• Can you eliminate any of the lines in the best evidence answers
because they simply do not address the question asked in the first
question?
What characteristcs are the Chinese climbing mice remarkable for?
MATH

2. The passage most strongly suggests that Chinese climbing mice are most
remarkable for which of the following characteristics?
A) Their agility
B) Their lack of strength
C) Their rhythmic movements
D) Their diverse colors

3. Which of the following line references provides the best support for the
answer to the previous question?
A) Lines 7–10 (“From time . . . mice”)
B) Lines 10–13 (“It is . . . mice”)
C) Lines 27–31 (“But what . . . rapidity”)
D) Lines 75–79 (“Zoth, who . . . weight”)

90  |  © TPR Education IP Holdings, LLC


IDENTIFYING QUESTION TYPES

11. The passage suggests that which of the following common assumptions about

READING
Japanese dancing mice may be incorrect?
A) The “dance” which the mice perform involves regular motion in a circle.
B) Dancing mice are generally weak and feeble.
C) Dancing mice are similar in markings, size, and general behavior.
D) Dancing mice occasionally perform in small groups, but they more often

WRITING AND
LANGUAGE
dance alone.

12. Which choice provides the best evidence for the answer to the previous
question?
A) Lines 31–35 (“Sometimes two . . . individual”)
B) Lines 42–46 (“Landois further . . . circle”)

MATH
C) Lines 52–55 (“I have . . . Boston”)
D) Lines 81–85 (“They are . . . others”)

Be sure to pay attention to the question! Although the best evidence questions
usually come in pairs, occasionally there will be a stand-alone question with
best evidence answers. Simply eliminate any of the lines that do not answer the
question.

10. Which choice provides the best evidence for the claim that a mouse’s physical
strength is not necessarily related directly to its endurance?
A) Lines 67–69 (“All of . . . mouse”)
B) Lines 69–73 (“The weakness . . . indefatigable”)
C) Lines 73–75 (“Frequently they . . . time”)
D) Lines 79–81 (“No other . . . available”)

© TPR Education IP Holdings, LLC  |  91


SAT MANUAL
READING

RHETORIC
These questions ask why an author makes certain choices. Be careful of trap answers that only address the what
response!
WRITING AND
LANGUAGE

6. In lines 39–42, the author initially mentions Rhetoric-


Landois primarily in order to
First, what does the author say about Landois?
A) present an authority who disagreed with
the prevailing view.
B) argue that Japanese dancing mice have no
remarkable characteristics.
MATH

C) demonstrate the differences between two


different types of rodents. Why does the author mention him?
D) illustrate the process by which one creature
received its name.

See the phrase “in order to”? Remember to


answer the why question instead of the
what question!

7. According to the information in the passage, the Is the attitude positive, neutral, or negative?
author’s attitude toward Landois’ claim in
lines 43–44 can best be described as one of
A) skepticism.
B) approval.
C) bewilderment. What can you eliminate first?
D) indifference.

What evidence points you to the best answer?

92 | © TPR Education IP Holdings, LLC


IDENTIFYING QUESTION TYPES

9. The author mentions the albino mouse and the gray What does the author say about the albino mouse and

READING
house mouse (lines 67–69) primarily in order to the gray house mouse?
A) provide examples of rodents that share the
dancing mouse’s unusual habits.
B) indicate that most types of rodents possess
relatively little physical strength.

WRITING AND
C) illustrate ways in which endurance proves Why does the author mention them?

LANGUAGE
to be an advantage to some species.
D) contrast the dancing mouse’s strength with
that of other types of mice.

MATH

© TPR Education IP Holdings, LLC  |  93


SAT MANUAL
READING

SYNTHESIS
Synthesis questions ask you to combine information from different sources. In
some passages, these questions ask about the passage and a graphic. In others,
these questions will be about dual passages. Let’s start with some questions about
the graphic for the Dancing Mice passage.
WRITING AND
LANGUAGE

Number of Twirls Per Five-Minute Interval


Twirls to the right Twirls to the left
140
120
100
80
MATH

60
40
20
0
.

.
.

.
.m

p.m

p.m

p.m
m

p.m
a.

0a

00

00

00
00
00

:0

3:

5:

7:
1:
9:

11

Adapted from data collected by Robert M. Yerkes, Ph.D. Yerkes collected data for ten mice
at various intervals on a given day. He counted the number of twirls to the left and twirls
to the right that the mice made during five minute intervals, and then averaged the
number of twirls in each direction. The results are graphed above.

14. Which claim about the dancing mice studied by Yerkes is supported by the
graph?
A) During each interval, the dancing mice twirled more often to the right
than to the left.
B) At 3 p.m., the mice twirled an equal amount to the right and the left on
average.
C) At 9 a.m., the mice twirled more often to the left on average than to the
right.
D) As the day progressed, the average number of twirls to the right that the
mice made always increased.

94  |  © TPR Education IP Holdings, LLC


IDENTIFYING QUESTION TYPES

15. Which claim could best be supported by evidence from the passage and the

READING
figure?
A) Dancing mice are generally more active at dusk and early evening.
B) Dancing mice dance together in the morning and alone in the afternoon.
C) Dancing mice dance less in the morning because they are fatigued.
D) Dancing mice have a predictable pattern to their twirls.

WRITING AND
LANGUAGE
DUAL PASSAGES
One of the History/Social Studies or Science Reading passages will be dual pas-

MATH
sages, with two shorter passages about one topic. When you come to the Dual
Passages, focus on each passage individually before you worry about the passages
together. Before you Select a Question, label all the questions: label with a “1” if
the question is about the first passage, a “2” if it’s about the second passage, or a
“1/2” if it’s about both passages.

1. Do all the questions that deal with Passage 1 first.


2. Do all the questions that deal with Passage 2 second.
3. Finally, do the remaining questions that ask about both
passages together.

© TPR Education IP Holdings, LLC | 95


SAT MANUAL
READING

IDENTIFYING QUESTION TYPES DRILL


(13 minutes)

Questions 22–31 are based on the following something the size of Mars. Finding a new planet
passages. will really mean something.”
Owen Gingerich is an astronomer and historian
WRITING AND
LANGUAGE

Passage 1 is from Mason Inman, “Pluto Not a Planet,


at Harvard University in Cambridge, Massachusetts,
Astronomers Rule.” © August 2006 by National Geographic
40 and head of the IAU committee proposing the
News. Passage 2 is from Sara Chodosh, “Pluto might be a
definition. He favored a special distinction for Pluto.
planet again. Let’s talk about why this matters.” Used with
Gingerich supported a proposal to call the big eight
permission of Popular Science Copyright© 2018. All rights
planets classical planets—as opposed to just plain
reserved.
“planets”—and Pluto and the others dwarf planets, so
Passage 1 45 there would be two classes of planets.
The IAU members overwhelmingly rejected this
MATH

Pluto has been voted off the island. The distant,


ice-covered world is no longer a true planet, idea.
according to a new definition of the term voted on “I think they voted primarily on scientific
Line by scientists today. grounds and were not sensitive to the historical and
5 In a move that’s already generating controversy 50 cultural role that Pluto has played,” Gingerich said.
and will force textbooks to be rewritten, Pluto will
now be dubbed a dwarf planet. A clear majority Passage 2
of researchers voted for the new definition at a Prepare yourself—the Pluto debate has returned,
meeting of the International Astronomical Union and people are not going to be able to shut up about
10 (IAU) in Prague, in the Czech Republic. The IAU it. Pluto might be about to regain its planethood.
decides the official names of all celestial bodies. The It might feel like scientists are jerking you around.
tough decision comes after a multiyear search for a 55 A decade ago they all decided that Pluto wasn’t a
scientific definition of the word “planet.” The term planet—it was actually a dwarf planet—and now
never had an official meaning before. all of a sudden they want to change it back? Maybe
15 According to the new definition, a full-fledged you even think that this just goes to show how
planet is an object that orbits the sun and is large meaningless it all was to begin with. Planet, dwarf
enough to have become round due to the force 60 planet—it’s all a made-up system determined by
of its own gravity. In addition, a planet has to some esoteric group anyway.
dominate the neighborhood around its orbit. Pluto But categories do matter, and so do the
20 has been demoted because it does not dominate definitions we use to arrive at those categories. The
its neighborhood. Charon, its large “moon,” is fact that people (even experts like the scientists at
only about half the size of Pluto, while all the true 65 NASA) go back and forth on what definitions we
planets are far larger than their moons. In addition, should use doesn’t make them less meaningful. It
bodies that dominate their neighborhoods, “sweep just means that we’re still learning. That’s what
25 up” asteroids, comets, and other debris, clearing a science is all about: we have to be able to adjust our
path along their orbits. By contrast, Pluto’s orbit is definitions to fit our understanding. And this whole
somewhat untidy. 70 Pluto business is a perfect example.
The new definition also establishes a third When the International Astronomical Union
class of objects that orbit the sun—”solar system (IAU) defined “planet” back in 2006, they landed on
30 bodies,” which would apply to many asteroids, this: a celestial body orbiting our Sun with enough
comets, and moons. The new definition of “planet” mass to make it round in shape and to clear its own
retains the sense that a true planet is something 75 orbit of other objects. That means that any newly
special. “It’s going to be hard to find a new planet,” discovered “planet” outside of our solar system isn’t
[astronomer Mike Brown, of the California Institute technically speaking a planet, but an exoplanet. The
35 of Technology in Pasadena] said. “You’d have to find New Horizons scientists take issue with that. They

96  |  © TPR Education IP Holdings, LLC


IDENTIFYING QUESTION TYPES

also think that requiring a planet to clear its orbit is 25

READING
80 unreasonable, because it requires planets with wide In the first paragraph of Passage 2 (lines 51–53), the
orbits to be very large. If Earth were in Pluto’s orbit, author establishes a tone that is primarily
it wouldn’t be able to clear all the objects out of its
orbital path either. Plus, even the orbits that are A) critical.
“clear” are often cluttered with transient small objects, B) informal.
85 so you could argue that no “planets” actually meet C) educational.

WRITING AND
this measure of success.

LANGUAGE
D) troubled.
All of this is part of their larger argument: a
planet isn’t a planet because it’s in a particular orbit
or because it has a particular size. A planet is a
90 planet because of its physical properties. We study 26
planets because they have ice volcanoes and flowing The author of Passage 2 indicates which of the
lakes of methane and roiling magnetic fields—not following about shifting scientific definitions?
because they sweep cosmic debris out of their way A) They can be applied to anything in the natural

MATH
as they circle the sun. The categories we use to make world.
95 sense of our world are only useful if they describe
B) They cause controversy when they are discussed.
things in a meaningful way.
C) They allow science to change as needed.
D) They cannot be relied upon.
22
As used in line 17, “force” most nearly means
A) strength. 27
B) fury. Which choice provides the best evidence for the
C) strain. answer to the previous question?
A) Lines 59–61 (“Planet, dwarf . . . anyway”)
D) ability.
B) Lines 63–66 (“The fact . . . meaningful”)
C) Lines 67–69 (“That’s what . . . understanding”)
23 D) Lines 71–75 (“When the . . . objects”)
The author of Passage 1 indicates that Pluto’s orbit is
A) irregularly-shaped.
28
B) round due to gravity.
C) twice the size of Charon. It can most reasonably be inferred from Passage 2
that one issue the New Horizons scientists take with
D) littered with solar system bodies. the new definition of “planet” is that
A) it is too narrowly-focused.
24 B) it no longer defines Earth as a “planet.”

The author of Passage 1 refers to Mars (line 36) to C) it sets an impossible standard for a celestial
suggest that any new planets object to be a planet.
A) will be easier to find than Mars. D) it should only be focused on orbits and sizes.
B) will be more challenging to find now that the
definition of “planet” has changed.
C) will not affect any of the current planets in our
solar system.
D) will only be identified as “planets” if they are the
same size as Mars.

© TPR Education IP Holdings, LLC  |  97


SAT MANUAL

29 31
READING

The main purpose of each passage is to The New Horizons scientists (lines 77–78) would
A) determine the clearest definition for a certain most likely respond to Owen Gingerich’s comment
celestial body. about the IAU’s decision (lines 48–50) with
B) establish a clear argument against changing A) derision, because they agree with the decision.
Pluto’s current status. B) agreement, but for different reasons.
WRITING AND

C) criticize the decision made by the International C) criticism, because Gengerich is too sentimental.
LANGUAGE

Astronomical Union. D) excitement, because he’s head of the IAU


D) discuss a change in classification and consider committee.
ramifications of that change.

30
MATH

Which choice best describes the relationship


between the two passages?
A) Passage 2 presents a counterargument to a
proposal put forth in Passage 1.
B) Passage 2 considers the larger context of a
decision made in Passage 1.
C) Passage 2 provides a conversational look at a
more scientific assessment offered in Passage 1.
D) Passage 2 uses expert testimony to refute the
conclusion made in Passage 1.

98  |  © TPR Education IP Holdings, LLC


IDENTIFYING QUESTION TYPES

Summary
• Information & Ideas Questions often ask
about the __________________ in the passage.

• Rhetoric questions ask about the


___________________ rather than the
____________________.

• Synthesis questions ask you to consider infor-


mation from ____________________________.

• When faced with Dual Passages, approach the


passage as you normally would (read the blurb,
select and understand a question, etc.) but go
through the passages and questions in a more
specific order:

________________________________________

________________________________________

________________________________________

• I have accomplished _________ of the 2 goals


stated at the beginning of this chapter.

© TPR Education IP Holdings, LLC | 99


IDENTIFYING
QUESTION
TYPES
PRACTICE

101
SAT MANUAL
READING

VOCABULARY-IN-CONTEXT
• Mark out the word.
• Read for context.
• Write in your own word or short phrase.
• POE
WRITING AND
LANGUAGE

Pat tried to ignore the shoe salesman in the 1


next booth, who was telling his children more than
Line anyone wanted to know about the importance of As used in line 12, “buckle” most nearly means
10 job security. She flipped through the newspaper, A) belt.
stopping at the astrology section. Her horoscope B) bend.
read: “As a Libra, you do not easily buckle under
pressure.” She sighed. Clearly, the horoscope knew C) bolt.
MATH

nothing of her sudden flight from work earlier that D) fasten.


15 day.

The police officer standing at my front door 2


Line quotes directly from a thick book: “Whoever
As used in line 25, “harbors” most nearly means
25 harbors or conceals any person who he knows, or
has reasonable grounds to believe or suspect, has A) ships.
committed, or is about to commit, an offense under B) begrudges.
sections 793 or 794 of this title, shall be fined under C) doubts.
this title or imprisoned not more than ten years, or
30 both.” D) shelters.
Yesterday, a man dressed like a sailor knocked,
politely asked if he could sleep the night in my barn,
and chopped some wood for me in return. He kept
the axe, but I didn’t find that strange at the time.

Line As I listened to members of the South Beach 3


55 Literary Diet Group calmly discuss their plans for
As used in line 64, “novel” most nearly means
the coming year, I wondered how such diverse
readers could want to regularly meet together to A) unique.
discuss texts as different as bleak anti-stories by B) depressing.
Samuel Beckett, young readers’ science fiction tales C) fictional.
60 by Madeleine L’Engle, A Day in the Life of Marlon
Bundo, and The Story of My Teeth. This was clearly D) bookish.
no traditional book-of-the-month club. The fact
that meetings always begin with interpretive dance
moves struck me as another novel feature of this
65 group.

CONTINUE
102  |  © TPR Education IP Holdings, LLC
IDENTIFYING QUESTION TYPES PRACTICE

“What’s a lady like you doing in a place like this?” 4

READING
Sophie sipped from her neon-blue cocktail As used in line 6, “current” most nearly means
before replying to the two-bit detective. “And what
Line makes you think I am a lady?” Everything about A) electric.
5 her was painfully up-to-date. The sparkly red shoes, B) contemporary.
reflecting current taste in this realm, complemented C) drifting.
the emerald socks and hyper-modern tan pant-suit.
D) watery.

WRITING AND
His left eye tearing slightly, he replied, “You have

LANGUAGE
this ... je-ne-sais-quoi.”
10 Her eyebrows rose; the air crackled with tension.
She sipped once more, flung the cherry from the
drink, and said as she stood to leave, “I don’t know
what that means.”

MATH
The wind howling through the cracks of the door, 5
Toby clung to his mug of hot cider, shrinking back
As used in line 20, “season” most nearly means
Line further toward the feeble hearth.
20 “If you plan to season yourself to this climate, A) spice.
you must embrace the discomfort,” said the kindly B) infuse.
innkeeper. Toby wondered what had ever possessed
C) accustom.
him to agree to this expedition—pure folly, it was. 
D) direct.

Line The first mate glowered at the sailors. 6


30 “Wherefore ye be a playin’ at balls when there be As used in line 35, “racket” most nearly means
work to do?”
A) court.
The parson added: “And gambling, melads, does
not befit good folk, as the good book sayeth.” B) scam.
The captain flung open his cabin door, C) ruckus.
35 exclaiming: “And what be the reason for this racket?”
D) ship.
The first mate looked left, the parson looked right,
and the sailors all sprung into the sea for shame.

CONTINUE
© TPR Education IP Holdings, LLC  |  103
SAT MANUAL
READING

PAIRED SETS
• Can you answer the first question on its own, and then identify the lines you used?
• Can you find any connections between the lines in the best evidence question and the answers in the
first question?
• Can you eliminate any of the lines in the best evidence answers because they simply do not address the
question asked in the first question?
WRITING AND
LANGUAGE

PRACTICE PASSAGE 1: PAIRED SETS


Questions 42–52 are based on the following It is tempting to see the priming fracas as an
passage. 35 isolated case in an area of science—psychology—
MATH

easily marginalized as soft and wayward. But


This passage is excerpted from “Trouble at the Lab.” © 2013
irreproducibility is much more widespread. A
by The Economist Newspaper Limited, London October 19th.
few years ago scientists at Amgen, an American
“I see a train wreck looming,” warned Daniel drug company, tried to replicate 53 studies that
Kahneman, an eminent psychologist, in an open 40 they considered landmarks in the basic science of
letter last year. The premonition concerned research cancer, often co-operating closely with the original
Line on a phenomenon known as “priming”. Priming researchers to ensure that their experimental
5 studies suggest that decisions can be influenced by technique matched the one used first time round.
apparently irrelevant actions or events that took According to a piece they wrote last year in Nature,
place just before the cusp of choice. They have been 45 a leading scientific journal, they were able to
a boom area in psychology over the past decade, and reproduce the original results in just six.
some of their insights have already made it out of Academic scientists readily acknowledge that
10 the lab and into the toolkits of policy wonks keen on they often get things wrong. But they also hold
“nudging” the populace. fast to the idea that these errors get corrected
Dr. Kahneman and a growing number of his 50 over time as other scientists try to take the work
colleagues fear that a lot of this priming research further. Evidence that many more dodgy results
is poorly founded. Over the past few years various are published than are subsequently corrected or
15 researchers have made systematic attempts to withdrawn calls that much-vaunted capacity for
replicate some of the more widely cited priming self-correction into question. There are errors in
experiments. Many of these replications have failed. 55 a lot more of the scientific papers being published,
In April, for instance, a paper in PLoS ONE, a written about and acted on than anyone would
journal, reported that nine separate experiments had normally suppose, or like to think.
20 not managed to reproduce the results of a famous Various factors contribute to the problem.
study from 1998 purporting to show that thinking Statistical mistakes are widespread. The peer
about a professor before taking an intelligence test 60 reviewers who evaluate papers before journals
leads to a higher score than imagining a football commit to publishing them are much worse at
hooligan. spotting mistakes than they or others appreciate.
25 The idea that the same experiments always Professional pressure, competition and ambition
get the same results, no matter who performs push scientists to publish more quickly than would
them, is one of the cornerstones of science’s 65 be wise. A career structure which lays great stress
claim to objective truth. If a systematic campaign on publishing copious papers exacerbates all these
of replication does not lead to the same results, problems. “There is no cost to getting things wrong,”
30 then either the original research is flawed (as the says Brian Nosek, a psychologist at the University of
replicators claim) or the replications are (as many of Virginia who has taken an interest in his discipline’s
the original researchers on priming contend). Either 70 persistent errors. “The cost is not getting them
way, something is awry. published.”
CONTINUE
104  |  © TPR Education IP Holdings, LLC
IDENTIFYING QUESTION TYPES PRACTICE

In testimony before Congress on March 5th

READING
Bruce Alberts, then the editor of Science, outlined
what needs to be done to bolster the credibility of
75 the scientific enterprise. Journals must do more 44
to enforce standards. Checklists such as the one
In discussing the phenomenon known as “priming,”
introduced by Nature should be adopted widely,
the author of the passage suggests that
to help guard against the most common research
A) it is a theory proven by systematic evidence.

WRITING AND
errors. Budding scientists must be taught technical

LANGUAGE
80 skills, including statistics, and must be imbued with B) it affects how people perform on intelligence
scepticism towards their own results and those tests.
of others. Researchers ought to be judged on the
C) it is not soundly based on repeatable
basis of the quality, not the quantity, of their work.
experiments.
Funding agencies should encourage replications and
85 lower the barriers to reporting serious efforts which D) it is only influenced by irrelevant actions.
failed to reproduce a published result. Information

MATH
about such failures ought to be attached to the
original publications. 45
And scientists themselves, Dr. Alberts insisted, Which choice provides the best evidence for the
90 “need to develop a value system where simply answer to the previous question?
moving on from one’s mistakes without publicly
acknowledging them severely damages, rather than A) Lines 4–7 (“Priming studies . . . choice”)
protects, a scientific reputation.” This will not be B) Lines 7–11 (“They have . . . populace”)
easy. But if science is to stay on its tracks, and be C) Lines 12–14 (“Dr. Kahnman . . . founded”)
95 worthy of the trust so widely invested in it, it may be
necessary. D) Line 17 (“Many of . . . failed”)

42 46
One of the primary claims the author of this The author’s reference to the “cornerstones of
passage makes is that science’s claim to objective truth” in lines 27–28
A) problems with irreproducibility can weaken a primarily serves to
defining quality of science. A) debunk a flawed claim.
B) psychology provides efficient techniques for B) acknowledge a potential consequence.
swaying voters during a campaign. C) underscore a tenet of scientific thought.
C) scientists should not be concerned about their D) correct an earlier assumption.
careers and only concerned about their results.
D) budding scientists should be responsible for
setting parameters to effectively replicate prior 47
research.
Which choice provides the best evidence for the
answer to the previous question?
43 A) Lines 18–24 (“In April . . . hooligan”)
Which choice provides the best evidence for the B) Lines 28–32 (“If a . . . contend”)
answer to the previous question? C) Lines 34–36 (“It is . . . wayward”)
A) Lines 7–11 (“They have . . . populace”) D) Lines 51–54 (“Evidence that . . . question”)
B) Lines 25–28 (“The idea . . . truth”)
C) Lines 47–48 (“Academic scientists . . . wrong”)
D) Lines 63–65 (“Professional pressure . . . wise”)
CONTINUE
© TPR Education IP Holdings, LLC  |  105
SAT MANUAL

48 51
READING

According to the passage, academic scientists The author of this passage most likely believes one
believe any errors in their research usually get way to publish more accurate research findings is to
corrected A) encourage new scientists to consider their own
A) gradually, as other scientists build on their work. research with a more critical eye.
B) immediately, in the peer review phase of B) require congressional review of scientific
WRITING AND

publication. research.
LANGUAGE

C) over time, as the general public reads and C) cut funding to publications that fail to
rereads their reports. reproduce results.
D) before the research is ever reviewed or D) instigate a process of peer review before a paper
published. is published.

49 52
MATH

Which choice provides the best evidence for the Which choice provides the best evidence for the
answer to the previous question? answer to the previous question?
A) Lines 44–46 (“According to . . . six”) A) Lines 14–17 (“Over the . . . experiments”)
B) Lines 48–51 (“But they . . . further”) B) Lines 36–37 (“But irreproducibility . . .
widespread”)
C) Lines 54–57 (“There are . . . think”)
C) Lines 72–75 (“In testimony . . . enterprise”)
D) Lines 59–62 (“The peer . . . appreciate”)
D) Lines 79–82 (“Budding scientists . . . others”)

50
Which of the following best supports the claim that
scientists feel more pressure to publish quantity
than they do to publish quality?
A) Lines 37–43 (“A few . . . round”)
B) Line 59 (“Statistical mistakes . . . widespread”)
C) Lines 67–71 (“There is . . . published”)
D) Lines 76–79 (“Checklists such . . . errors”)

CONTINUE
106  |  © TPR Education IP Holdings, LLC
IDENTIFYING QUESTION TYPES PRACTICE

READING
PRACTICE PASSAGE 2: LITERATURE

Questions 1–10 are based on the following I don’t know why I could talk to her; maybe for
passage. the same reason she could talk to me. The first thing
40 I knew I was telling her about Mickey Mouse, Soda’s

WRITING AND
This passage is excerpted from S.E. Hinton, The Outsiders.

LANGUAGE
horse. I had never told anyone about Soda’s horse. It
Copyright © 1967 by S.E. Hinton, © renewed 1995 by S.E.
was personal.
Hinton. Used by permission of Viking Children’s Books,
Soda had this buckskin horse, only it wasn’t his.
an imprint of Penguin Young Readers Group, a division
It belonged to a guy who kept it at the stables where
of Penguin Random House LLC. All rights reserved. The
45 Soda used to work. Mickey Mouse was Soda’s horse,
narrator, Ponyboy, is a member of the Greasers, a social
though. The first day Soda saw him he said, “There’s
group whose rival is the Socs. Sodapop (sometimes called
my horse,” and I never doubted it. I was about ten
Soda) is Ponyboy’s brother.
then. Sodapop is horsecrazy. I mean it. He’s always

MATH
After the movie was over … we walked to Two- hanging around stables and rodeos, hopping on
Bit’s house to pick up the car. It seemed funny to me 50 a horse every time he gets a chance. When I was
that Socs—if these girls were any example—were ten I thought that Mickey Mouse and Soda looked
Line just like us. They liked the Beatles and thought Elvis alike and were alike. Mickey Mouse was a dark-gold
5 Presley was out, and we thought the Beatles were buckskin, sassy and ornery, not much more than a
rank and that Elvis was tuff, but that seemed the only colt. He’d come when Soda called him. He wouldn’t
difference to me. Of course greasy girls would have 55 come for anyone else. That horse loved Soda. He’d
acted a lot tougher, but there was a basic sameness. I stand there and chew on Soda’s sleeve or collar. Gosh,
thought maybe it was money that separated us. but Sodapop was crazy about that horse. He went
10 “No,” Cherry said slowly when I said this. “It’s down to see him every day. Mickey Mouse was a
not just money. Part of it is, but not all. You greasers mean horse: He kicked other horses and was always
have a different set of values. You’re more emotional. 60 getting into trouble. “I’ve got me a ornery pony,”
We’re sophisticated—cool to the point of not feeling Soda’d tell him, rubbing his neck. “How come you’re
anything. Nothing is real with us.” so mean, Mickey Mouse?” Mickey Mouse would just
15 That was the truth. Socs were always behind a chew on his sleeve and sometimes nip him. But not
wall of aloofness, careful not to let their real selves hard. He may have belonged to another guy, but he
show through. I had seen a social-club rumble once. 65 was Soda’s horse.
The Socs even fought coldly and practically and “Does Soda still have him?” Cherry asked.
impersonally. “He got sold,” I said. “They came and got him one
20 “That’s why we’re separated,” I said. “It’s not day and took him off. He was a real valuable horse.
money, it’s feeling—you don’t feel anything and we Pure quarter.”
feel too violently.” 70 She didn’t say anything else and I was glad. I
“And”—she was trying to hide a smile—“that’s couldn’t tell her that Soda had bawled all night long
probably why we take turns getting our names in the after they came and got Mickey Mouse. I had cried,
25 paper.” too, if you want to know the truth, because Soda
Two-Bit and Marcia weren’t even listening to us. never really wanted anything except a horse, and
They were engaged in some wild conversation that 75 he’d lost his. Soda had been twelve then, going-on-
made no sense to anyone but themselves. thirteen. He never let on to Mom and Dad how he
I have quite a rep for being quiet, almost as felt, though, because we never had enough money
30 quiet as Johnny. Two-Bit always said he wondered and usually we had a hard time making ends meet.
why Johnny and I were such good buddies. “You When you’re thirteen in our neighborhood you
must make such interestin’ conversation,” he’d say, 80 know the score. I kept saving my money for a year,
cocking one eyebrow, “you keepin’ your mouth shut thinking that someday I could buy Mickey Mouse
and Johnny not sayin’ anything.” But Johnny and back for Soda. You’re not so smart at ten.
35 I understood each other without saying anything.
Nobody but Soda could really get me talking. Till I CONTINUE
met Cherry Valance.
© TPR Education IP Holdings, LLC  |  107
SAT MANUAL

1 5
READING

Over the course of the passage, the primary focus Two-Bit’s reference to “interestin’ conversation”
shifts from (line 32) is most likely meant to be
A) a character’s conversation with a girl to his A) complimentary.
reflection about his brother. B) ironic.
B) a description of a conversation to a description C) insulting.
WRITING AND

of a family.
LANGUAGE

D) confusing.
C) an encounter on a street to an encounter on a
farm.
D) the perspective of one brother to the perspective 6
of another.
Which choice provides the best evidence for the
answer to the previous question?
2 A) Line 26 (“Two-Bit . . . us”)
MATH

As used in line 6, “rank” most nearly means B) Lines 27–28 (“They were . . . themselves”)
A) unlikeable. C) Lines 30–31 (“Two-Bit . . . buddies”)
B) pungent. D) Lines 33–34 (“you keepin’ . . . anything”)
C) luxuriant.
D) ordered. 7
The relationship between Mickey Mouse and
Sodapop could best be described as
3
A) confrontational.
Based on the passage, Cherry and Ponyboy would
most likely agree with which claim about Socs and B) affectionate.
Greasers? C) perplexing.
A) The greatest difference between the two groups D) convenient.
is their financial status.
B) Both groups thrive on publicity.
C) Each group operates by a different set of 8
principles. What does the narrator most likely mean by his
D) Both groups enjoy the same types of music. statement in lines 64–65 (“He may . . . horse”)?
A) Sodapop loved Mickey Mouse so much he was
willing to steal him.
4 B) Sodapop was working hard at the barn to earn
Which choice provides the best evidence for the enough money to buy Mickey Mouse.
answer to the previous question? C) Sodapop and Mickey Mouse loved each other as
A) Lines 4–7 (“They liked . . . me”) if Sodapop owned Mickey Mouse.
B) Lines 11–14 (“You Greasers . . . us”) D) Sodapop was the only one who could ride
Mickey Mouse safely.
C) Lines 38–39 (“I don’t . . . me”)
D) Lines 70–72 (“I couldn’t . . . Mouse”)

CONTINUE
108  |  © TPR Education IP Holdings, LLC
IDENTIFYING QUESTION TYPES PRACTICE

9 10

READING
In the final paragraph, the narrator draws a As used in line 80, “score” most nearly means
distinction between A) amount.
A) upper class and lower class. B) accomplishment.
B) honesty and deception. C) reality.
C) thriftiness and wastefulness. D) expense.

WRITING AND
LANGUAGE
D) experience and innocence.

MATH

CONTINUE
© TPR Education IP Holdings, LLC  |  109
SAT MANUAL
READING

PRACTICE PASSAGE 3: SCIENCE

Questions 22–31 are based on the following tsunami will result. Once the seismic waves from an
passage. earthquake have reached enough seismometers, the
distinction becomes clear. But near any given quake
WRITING AND

The following passage is excerpted from “Finding More


LANGUAGE

there are rarely enough seismometers around.


Time to Detect a Tsunami.” © Feb 2018. The Economist
45 Except, as the panelists pointed out, there are.
Group Limited, London February 22.
America’s satellite-based Global Positioning System
Tsunamis are terrible things. And part of their and subsequent similar efforts from other countries
terror lies in their unpredictability. Even when (known collectively as GNSS, the Global Navigation
a submarine earthquake that may cause one Satellite System) have permitted the creation in
Line is detected, the information that is needed to 50 many places of networks of sensors that measure,
5 determine whether a giant wave has actually been within millimetres, local distortions of Earth’s crust.
MATH

created takes time to gather. That is time unavailable The main reason for doing this is to understand the
for the evacuation of coastlines at risk. Contrariwise, build-up of earthquake-causing strain in the crust,
issuing a warning when no subsequent wave arrives so such monitors are most abundant where tremors
provokes cynicism and a tendency to ignore future 55 are commonest. And, if a tremor does happen,
10 evacuation calls. monitors nearby will be shaken by it.
Such tsunami-warning systems as do exist rely on There are, by the panelists’ estimates, about
seismometers to detect earthquakes, and tide gauges 17,000 such monitoring devices around the world.
and special buoys to track a wave’s passage. That is Of those, around 2,300 make their data available
reliable, but can often be too late to get people away 60 instantly. If these instant monitors’ signals could
15 from threatened coastlines. What these warning all be gathered together and run through suitable
systems cannot do reliably is predict immediately software, the true nature of a big submarine
whether a given earthquake will cause a tsunami. earthquake would be apparent almost at once, and
And that, in the view of some seismologists, is a appropriate warnings could be issued.
scandal. For, as the annual meeting of the American 65 At the moment, two regional projects are
20 Association for the Advancement of Science learned testing this idea. One, READI, on the Pacific coast
from Gerald Bawden of NASA, Paul Huang of of America, is under the aegis of NASA. The other,
America’s National Tsunami Warning Centre, Tim GEONET, in Japan, is organised by that country’s
Melbourne of Central Washington University, land-mapping agency. The hope is that, if these local
and Meghan Miller of UNAVCO, a geoscience 70 ventures work, other countries will join in and a
25 research consortium, the tools for accurate tsunami global network can be created over the next decade.
prediction already exist. All that needs to happen is Really clever use of the GNSS, moreover, might
to connect them up. be able to do even better than this, by tracking
The nub of the problem is that it is hard to a tsunami as it travels. Though the most visible
distinguish immediately whether a submarine 75 consequence of a tsunami is a wave in the ocean,
30 earthquake is powerful enough to cause a tsunami. it also creates one in the atmosphere. This affects
Big quakes (those above about magnitude 7.3) the arrival time of GNSS radio waves in a way that,
involve slippage along many kilometres of a fault. with enough ground-based detectors, would permit
That means their energy is not radiating from a the passage of the wave to be followed. And these
point. A single seismometer therefore has difficulty 80 detectors, too, will soon be commonplace. For many
35 distinguishing between a quake of magnitude 7.3, years, smartphones have contained GNSS receivers,
8.3 or even 9.3 (about as large as they get). The so a phone’s apps can use location information. The
logarithmic nature of the earthquake-magnitude latest phones have equipment so sensitive that it
scale, though, means the third of these is 1,000 times could, in principle, detect a passing tsunami in the
more powerful than the first. And the more powerful 85 atmosphere. All this would require is for someone to
40 the shock, the more likely it is that a dangerous write a suitable app, and for enough phone users to
download it. CONTINUE
110  |  © TPR Education IP Holdings, LLC
IDENTIFYING QUESTION TYPES PRACTICE

22 25

READING
The main purpose of the passage is to What does the author imply about programs in
A) illustrate how seismometers sense and measure America and in other countries?
submarine quakes. A) The Global Positioning System is developing
B) explain the research of GEONET and READI monitoring devices 1,000 times more powerful
and future changes that will be put in place. than previous generations of devices.

WRITING AND
B) Programs in America and other countries in

LANGUAGE
C) describe the current seismometer systems and
how improvements can be made. earthquake-prone areas hope to inspire more
collaborative connections.
D) discuss the consequences of the out-of-date
system of detecting earthquakes around the C) Tremors will shake nearby monitors in these
world. systems, but they do not yet have the capability
to predict tsunamis.
D) Competition between countries has hindered
the global system for monitoring tsunamis in

MATH
23
real time.
The author uses the phrase “And that, . . . scandal”
in lines 18–19 most likely to
A) express seismologists’ dismay with the use of 26
current technologies.
Which choice provides the best evidence for the
B) stress the importance of tide gauges and special answer to the previous question?
buoys in the current warning system.
A) Lines 36–39 (“The logarithmic . . . first”)
C) explain that improvements to the current
warning system already exist. B) Lines 43–44 (“But near . . . around”)

D) describe how disastrous the current system has C) Lines 55–56 (“And, if . . . it”)
proven in the past for alerting civilians. D) Lines 69–71 (“The hope . . . decade”)

24 27
What is the main idea of the third paragraph As used in line 51, “distortions” most nearly means
(lines 28–44)? A) falsehoods.
A) Different seismometers along fault lines can B) disturbances.
detect slippage many kilometers away.
C) sections.
B) Strategic location and placement of
seismometers helps to distinguish between 7.3, D) features.
8.3 and 9.3 magnitude earthquakes.
C) It is difficult to detect whether an earthquake
will cause a tsunami given the current
placement of seismometers.
D) The size of the earthquake is irrelevant to the
formation of tsunamis.

CONTINUE
© TPR Education IP Holdings, LLC  |  111
SAT MANUAL

28 30
READING

What can be reasonably inferred about why the The author includes the phrase, “There are, by the
panelists believe there are enough seismometers panelists’ estimates, about 17,000 such monitoring
available to measure earthquakes around the world? devices around the world.” (lines 57–58) to
A) Monitoring devices already in the world A) suggest that materials available are
could be used to work together with proper underutilized and wasted.
technology to issue warnings.
WRITING AND

B) indicate that more resources are needed to


LANGUAGE

B) Suitable software is only possible through the make significant improvements to the warning
creation of an app for phone users to download. systems.
C) The amount of monitoring devices is C) stress how complacent scientists have been in
proportional to the amount of smartphones in implementing improvements.
various locations. D) emphasize how many devices are already in
D) No new monitoring devices need to be added. existence as resources for improvements.
MATH

29 31
Which choice provides the best evidence for answer What is the most likely reason the author includes
to the previous question? the last sentence in lines 85–87?
A) Lines 57–60 (“There are . . . instantly”) A) To illustrate the simplicity of the current
warning systems
B) Lines 60–64 (“If these . . . issued”)
B) To present data results that suggest
C) Lines 69–71 (“The hope . . . decade”)
improvements to warning systems are miniscule
D) Lines 85–87 (“All this . . . it”)
C) To question why improvements to warning
systems have not been done already
D) To show how improvements to warning systems
are achievable

CONTINUE
112  |  © TPR Education IP Holdings, LLC
IDENTIFYING QUESTION TYPES PRACTICE

READING
PRACTICE PASSAGE 4: SOCIAL STUDIES WITH GRAPH

Questions 12–22 are based on the following than a hundred other soldiers had come down with
passage and supplemental material. a similar condition, and more would fall ill over the
following weeks. In April more American troops

WRITING AND
The following passage is adapted from Toby Saul’s “Inside

LANGUAGE
arrived in Europe and brought the virus with them.
the Swift, Deadly History of the Spanish Flu Pandemic”
45 The first wave of the pandemic had arrived.
© 2018 National Geographic.
The drama of the war also served to obscure the
Scientist Johan Hultin traveled to Brevig unusually high mortality rates of the new virus. At
Mission, Alaska, a town of a few hundred souls this early stage, the illness was not well understood
[possible VIC] in the summer of 1997. He was and deaths were often attributed to pneumonia.
Line searching for buried bodies, and Alaska’s frozen 50 Strict wartime censorship meant that the European
ground was the perfect place to find them. Digging and North American press were unable to report

MATH
5
through the permafrost—with permission from outbreaks. Only in neutral Spain could the press
the town’s authorities—he eventually uncovered a speak freely about what was happening, and it was
woman who died almost 80 years previously and from this media coverage that the disease took its
was in a state of excellent preservation. Hultin 55 nickname.
10 then extracted samples of the woman’s lung before The overcrowded trenches and encampments
reinterring her. He intended to use this to decode of the First World War became the perfect hosts
the genetic sequence of the virus that had killed this for the disease. As troops moved, so the infection
Inuit woman along with 90 percent of the town’s traveled with them. The wave that had first appeared
population. 60 in Kansas abated after a few weeks, but this was
15 Brevig Mission was just one place that was part only a temporary reprieve. By September 1918 the
of a global tragedy, one of the worst ever to befall epidemic was ready to enter its most lethal phase.
humanity: the influenza pandemic of 1918-19. The As the crisis reached its zenith, the medical
outbreak of this influenza virus, also known as services began to be overwhelmed. Morticians and
Spanish flu, spread with astonishing speed around 65 gravediggers struggled, and conducting individual
20 the world, overwhelming India, and reaching funerals became impossible. Many of the dead ended
Australia and the remote Pacific islands. In just 18 up in mass graves. The end of 1918 brought a hiatus
months at least a third of the world’s population was in the spread of the illness and January 1919 saw the
infected. Estimates on the exact number of fatalities beginning of the third and final phase. By then the
vary wildly, from 20 million to 50 million to 100 70 disease was a much diminished force. The ferocity of
25 million deaths. If the upper end of that estimate is the autumn and winter of the previous year was not
accurate, the 1918 pandemic killed more people repeated and mortality rates fell.
than both World Wars put together. The pandemic left almost no part of the world
By March 1918 the United States had been at untouched. In Great Britain 228,000 people died.
war with Germany and the Central Powers for 11 75 The United States lost as many as 675,000 people,
30 months. During that time America’s small, prewar Japan some 400,000. The south Pacific island of
army had grown into a vast fighting force that would Western Samoa (modern-day Samoa) lost one-fifth
eventually send more than two million men to of its population. Researchers estimate that in India
Europe. alone, fatalities totaled between 12 and 17 million.
American forts experienced a massive expansion 80 Exact data in the number of deaths is elusive, but
35 as the entire nation mobilized for war. One of global mortality figures are estimated to have been
these was Fort Riley, Kansas, where a new training between 10 and 20 percent of those who were
facility, Camp Funston, was built to house some of infected.
the 50,000 men who would be inducted into the
Army. It was here on March 4 that a feverish soldier
40 reported to the infirmary. Within a few hours more
CONTINUE
© TPR Education IP Holdings, LLC  |  113
SAT MANUAL

In 1997 the samples taken by Johan Hultin from 13


READING

85 the woman found in the frozen mass grave in Brevig


The author most likely included the estimates of
Mission added to scientists’ knowledge as to how
fatalities in lines 21–27 (“In just . . . together”) in
flu viruses mutate and spread. Drugs and improved
order to
public hygiene—in conjunction with international
institutions such as the World Health Organization A) provide supporting data.
90 and national bodies such as the Centers for Disease B) perform some calculations.
WRITING AND

Control and Prevention in the United States—put


LANGUAGE

C) disprove a statement.
the international community in a much better
position to meet the challenges of a new outbreak. D) propose further research.
However, scientists know a lethal mutation could
95 occur at any time, and a century on from the
mother of all pandemics, its effects on a crowded, 14
interconnected world would be devastating. As used in line 24, “wildly” most nearly means
A) manically.
MATH

U.S. Regular Military troop strength in


selected years before and during WWI B) excitedly.
4,000,000 C) violently.
3,500,000
3,000,000 D) broadly.
# of soldiers

2,500,000
2,000,000
1,500,000
1,000,000
15
500,000 The primary purpose of lines 35–39 (“One of . . .
0
1902 1907 1911 1917 1918 Army”) is to
A) quantify an outbreak.
B) contextualize an event.
C) describe a place.
12 D) give an example.
The main purpose of the passage is to
A) provide an overview of the progression and
impact of an influenza outbreak. 16

B) discuss why the Spanish flu spread so quickly As used in line 45, “wave” most nearly means
and how to prevent similar pandemics in the A) phase.
future. B) flood.
C) question the scientific conclusions drawn by C) greeting.
Johan Hultin and other scientists.
D) ripple.
D) show how military conflict can make a virus
worse than it would have been otherwise.

CONTINUE
114  |  © TPR Education IP Holdings, LLC
IDENTIFYING QUESTION TYPES PRACTICE

17 21

READING
According to the passage, which of the following is Based on the table, which period saw the greatest
true about the Spanish Flu? percent increase in the number of troops?
A) It spread faster than any other virus like it. A) 1902–1907
B) Its name was the result of uncensored reporting B) 1911–1917
in Spain. C) 1917–1918

WRITING AND
LANGUAGE
C) Its genetic code can only be found in Alaska and D) 1918–1920
Spain.
D) It was cured by the end of 1919.
22
The data in the graph best serve as evidence of
18
A) “America’s small, prewar army” (lines 30–31).
What choice provides the best evidence for the

MATH
answer to the previous question? B) “some of the 50,000 men” (lines 37–38).
A) Lines 9–14 (“Hultin then . . . population”) C) “the unusually high mortality rates”
(lines 46–47).
B) Lines 39–43 (“It was . . . weeks”)
D) “as many as 675,000 people” (line 75).
C) Lines 52–55 (“Only in . . . nickname”)
D) Lines 56–62 (“The overcrowded . . . phase”)

19
The author indicates that scientists have used
information about the Spanish flu to
A) determine the virus’s actual mortality rates.
B) identify an Inuit town as the virus’s point of
origin.
C) be better prepared for similar outbreaks.
D) find a cure for the virus.

20
What choice provides the best evidence for the
answer to the previous question?
A) Lines 1–9 (“Scientist Johan . . . preservation”)
B) Lines 15–21 (“Brevig Mission . . . islands”)
C) Lines 73–83 (“The pandemic . . . infected”)
D) Lines 84–93 (“In 1997 . . . outbreak”)

Answers can be found on pages 725–726.

© TPR Education IP Holdings, LLC | 115


WRITING AND
LANGUAGE

117
INTRODUCTION

Grammar is like your overarching compulsion.


It’s math with words.
—Thomm Quackenbush

119
SAT MANUAL
READING

GOALS REVIEW
At the conclusion of this chapter you will be able to accomplish the following:

• Know the three steps of the SAT Writing and Language Basic Approach
• Identify what questions without questions are testing by looking at
the answer choices
WRITING AND

• Use POOD on the Writing and Language test


LANGUAGE

FUN FACTS ABOUT THE WRITING AND LANGUAGE


TEST
The Writing and Language Test gives you _____ minutes to answer _____
MATH

questions.

• There are _____ passages with _____ questions each.


• The questions cover a limited range of topics on ________________,
________________, and ________________.
• There is no ____________________________ in the placement of the
passages or of the questions.
• One or more of the passages will feature a ________________.

WRITING AND LANGUAGE: A PROOFREADER AND


AN EDITOR
Most of the questions you see on the Writing and Language Test will fall into
one of two categories: Standard English Conventions and Expression of Ideas.
Essentially, you have two responsibilities: you need to be a proofreader and an
editor.

A proofreader looks for errors. On the SAT, a proofreader


should look for mistakes in grammar, punctuation, and English
usage.
An editor reviews and improves the writing. On the SAT, an
editor should look for ways to improve the continuity and preci-
sion of the writing.

120 | © TPR Education IP Holdings, LLC


INTRODUCTION

READING
THE ANSWERS HOLD THE ANSWER
The majority of questions that you see on the SAT will be proofreader questions.
These are questions that give you a list of answer choices but no actual question.

Writing a book is only the first step 1. A) NO CHANGE


in the process of publication. Once a B) complete, it

WRITING AND
LANGUAGE
Changes in the answer
manuscript is 1 complete; it must be C) complete, choices tell you what the
submitted to a publisher. D) complete—it question is.

What’s changing in the answer choices? ____________________________________

MATH
What is this question testing? _____________________________________________

Ask yourself “What’s changing in the answer choices?”


every time you’re faced with a list of answer choices
but no question.

© TPR Education IP Holdings, LLC | 121


SAT MANUAL
READING

Let’s try a few more. There’s no passage for these, just the answer choices. What is
each one testing?

i. A) NO CHANGE
B) to run, to jump, and standing still
WRITING AND
LANGUAGE

C) running, having jumped, and standing there still


D) running, jumping, and standing still

What’s changing in the answer choices? ____________________________________

What is this question testing? _____________________________________________


MATH

ii. A) NO CHANGE
B) could of
C) could have
D) would of

What’s changing in the answer choices? ____________________________________

What is this question testing? _____________________________________________

iii. A) NO CHANGE
B) books on a variety of subjects including
C) books, on a variety, of subjects, including
D) books, on a variety of subjects, including

What’s changing in the answer choices? ____________________________________

What is this question testing? _____________________________________________

iv. A) NO CHANGE
B) valuable and having lots of worth
C) valuable for being worth so much
D) valuable

What’s changing in the answer choices? ____________________________________

What is this question testing? _____________________________________________

122  |  © TPR Education IP Holdings, LLC


INTRODUCTION

READING
YOUR EARS CAN’T HEAR IT ALL
You don’t need to be a grammarian to hear that some things are incorrect, but
trusting your ear exclusively won’t get you all the points that you want.

2. A) NO CHANGE Is there a difference


The publisher will have 2 their own in the way (C) and (D)
standards for how to lay out a book, but B) they’re own standards sound?

WRITING AND
LANGUAGE
there are still many decisions to be made. C) its own standards
D) it’s own standards

What’s changing in the answer choices? ____________________________________

MATH
What is this question testing? _____________________________________________

Written and spoken English are similar, but they’re not


the same! The SAT tests your knowledge of written
English.

CONTEXT IS EVERYTHING
3. A) NO CHANGE What clues does the
It has often been said that you can’t
non-underlined portion
judge a book by its cover, but cover design B) will result give you about what
matters. The history of publishing is full of C) resulted verb tense to use?

examples of how a change in cover design D) is resulting


3 results in dramatically increased sales.

What’s changing in the answer choices? ____________________________________

What is this question testing? _____________________________________________

© TPR Education IP Holdings, LLC | 123


SAT MANUAL
READING

POE SAVES THE DAY


Many Writing and Language questions have more than one thing changing in the
answer choices. Deal with the changes one at a time.

It doesn’t matter which It may seem that a beautiful illustration 4. A) NO CHANGE


change you start
on the cover would do the 4 trick. B) trick. However, that’s a
WRITING AND

with; eliminate all the


LANGUAGE

answers that make the However, that’s a complex issue than it may more complex issue
same error! C) trick, however, there’s
seem.
a more complex issue
D) trick. However, there’s
a complex issue
MATH

What’s changing in the answer choices? ____________________________________

What is this question testing? _____________________________________________

THE MORE THINGS CHANGE…


Don’t get so focused on The text on a cover is also 5. A) NO CHANGE
the list of answers that
5 important: if consumers can’t read B) important, it being
you forget to consider
how the sentence is the title, they aren’t likely to buy the book. the case that when
written! C) important, although
if
D) important; it is factual
that if

What’s changing in the answer choices? ____________________________________

What is this question testing? _____________________________________________

NO CHANGE is correct about 25% of the time it


appears. Don’t force yourself to find errors where
there aren’t any!

124 | © TPR Education IP Holdings, LLC


INTRODUCTION

WRITING AND LANGUAGE BASIC APPROACH

READING
Every time you tackle a Writing and Language question that doesn’t have a ques-
tion, follow the same approach:

1. Read to the end of a sentence with an underlined portion.

WRITING AND
2. Look at the answer choices to determine what’s being tested.

LANGUAGE
3. Use POE.

PACING

MATH
Keep Moving
Answer all 44 questions, but don’t get stuck! If a question seems too time-
consuming, guess and move on.

Make the Obvious POOD Choice


On the Writing and Language test, more words usually means more work. The
long questions are easy to spot: they have a lot of words and therefore take up
more space on the page. If you feel pressed for time, skip these questions and move
on to the ones you can do more quickly.

Scoring and Pacing


Your Writing and Language score makes up half of your Evidence-Based Reading
and Writing score:

  Writing and Language Scale Score


10 15 20 25 30 35 40
10 200 250 300 350 400 450 500
15 250 300 350 400 450 500 550
Reading 20 300 350 400 450 500 550 600
Scale 25 350 400 450 500 550 600 650
Score 30 400 450 500 550 600 650 700
35 450 500 550 600 650 700 750
40 500 550 600 650 700 750 800

© TPR Education IP Holdings, LLC  |  125


SAT MANUAL
READING

For a Writing and You need about Answer about this


Language Test Score this many Correct many questions
of: Answers: per passage:
10 3 1
12 5 1
14 8 2
WRITING AND
LANGUAGE

16 10 3
18 13 3
20 16 4
22 19 5
24 22 5
26 25 6
28 28 7
MATH

30 31 8
32 34 8
34 37 9
36 40 10
38 42 11
40 44 11

Food for Thought:

Writing and Language Score from first test: _________

Number of additional questions needed for a 2-point improvement: _________

Each two-point improvement in your Writing and Language Test Score is worth
20 composite points.

Whatever your scoring goals, finish the Writing and Language


Test. You may not work every question, but you should answer
all of them.

126 | © TPR Education IP Holdings, LLC


INTRODUCTION

Summary
• The two tasks you have in Writing and
Language are:

________________________________________

________________________________________

• The Basic Approach for questions without


questions is:

________________________________________

________________________________________

________________________________________

• When more than one thing is changing in the


answer choices,

________________________________________

• NO CHANGE is correct about ______


percent of the time.

• I have accomplished _________ of the 3 goals


stated at the beginning of this chapter.

© TPR Education IP Holdings, LLC | 127


WORDS
PART I

The difference between the almost-right word and


the right word is really a large matter—’tis the differ-
ence between the lightning-bug and the lightning.
—Mark Twain

129
SAT MANUAL
READING

GOALS REVIEW
At the conclusion of this chapter, you will be able to accomplish the following:

• Apply the Basic Approach to questions dealing with words


• Choose transitions, verbs, and pronouns that are consistent with their
sentences
WRITING AND

• Choose pronouns and other words that make the meaning of a


LANGUAGE

sentence precise
• Recognize and select the most concise answer

CONSISTENCY, PRECISION, AND CONCISION


MATH

There are hundreds, if not thousands, of grammar rules for English, but most SAT
questions focus on just a handful of rules. Regardless of the rules, keep the follow-
ing in mind:

• The correct choice is _______________ with the rest of the sentence


and passage.
• The correct choice features words and phrases that are as
_______________ as possible.
• The correct choice, free of any errors, will be the most
_______________.

In this chapter, we’ll focus on how these three rules apply to three particular types
of words: transitions, verbs, and pronouns. Along with these rules, remember to
apply the Basic Approach for Writing and Language:

1. ___________________________________________________________

2. ___________________________________________________________

3. ___________________________________________________________

130  |  © TPR Education IP Holdings, LLC


WORDS PART I

READING
TRANSITIONS
Many students of English find the 1. A) NO CHANGE
language frustrating because there are B) By contrast, Do any of these words
so many rules. 1 However, most of C) In other words, have the same meaning?
the rules have exceptions, which makes D) Moreover,

WRITING AND
LANGUAGE
them even harder to learn.

What’s changing in the answer choices? ____________________________________

Which word is consistent with the ideas it is connecting? ______________________

MATH
Ideas that agree require a same-direction transition, and ideas that contrast require
an opposite-direction transition.

Same-Direction Transitions
Accordingly Likewise*
Consequently Moreover *Starred transition
words are the ones
For example* Therefore*
that appear most
For instance Thus* frequently on
the SAT!
Furthermore Similarly*
In addition* Subsequently*
In effect

© TPR Education IP Holdings, LLC | 131


SAT MANUAL
READING

Opposite-Direction Transitions
Besides Nevertheless*
Conversely Nonetheless
Despite this On the other hand
WRITING AND
LANGUAGE

However* Regardless
Instead
MATH

Still confused about On top of the endless grammar 2. A) NO CHANGE


transitions? Check out rules in English, spelling can be very B) Therefore,
pages 284–287 for
more information! difficult to figure out. 2 For example, C) In effect,
why do we say that we caught a cold but D) Furthermore,
that we bought cold medicine?

What’s changing in the answer choices? ____________________________________

Do the ideas being connected agree or disagree with each other? _______________

Are there any wrong direction transitions you can eliminate? __________________

How do the ideas being connected agree with each other? Is one idea an example
of the other? Is there a cause-effect relationship?

_______________________________________________________________________

132 | © TPR Education IP Holdings, LLC


WORDS PART I

Once you’ve eliminated

READING
3 Obviously, the number of 3. A) NO CHANGE transitions that go the
different letter combinations that can B) Indeed, wrong direction, use
C) In effect, POE: some transitions
make the same sound can make one’s
will work better in a
head spin. D) At this time, certain context than
others.

WRITING AND
LANGUAGE
What’s changing in the answer choices? ____________________________________

Which transitions don’t work in this context? _______________________________

MATH
When you see transitions changing in the answers:
1. Check direction: look at whether the ideas being
connected agree or disagree. Eliminate answers that
indicate the wrong direction.
2. Use POE: choose the transition word that’s most
consistent with the ideas being connected.

© TPR Education IP Holdings, LLC | 133


SAT MANUAL
READING

VERBS
Beware of The reason English spelling is 4. A) NO CHANGE
prepositional phrases so complicated has to do with the B) is divided
between the subject
and the verb! Not sure history of the language. Historians of C) divide
what a prepositional English 4 divides the development D) are divided
WRITING AND
LANGUAGE

phrase is? Go to
page 288 for a review. of the language into three periods: Old
English (450–1100), Middle English
(1100–1500), and modern English
(1500–present). HW: WORD PT STARTING ON
PAGE 144 => 146, (Q1 => 12) ;
=> 136 (1=>13), + DO
PASSAGE READING 1
MATH

What’s changing in the answer choices? ____________________________________

What is the subject of the verb? ___________________________________________

The key with verbs is consistency. Verbs must be consistent with their subjects:
singular subjects take singular verbs, and plural subjects take plural verbs.

Still confused about Today, Old and Middle English are 5. A) NO CHANGE
verbs? Look at essentially dead languages, and most B) will have read
pages 287–289 and
people 5 had read the great texts C) read
293–294 for a review!
of those eras, such as Beowulf or The D) reads
Canterbury Tales, in modern English
translations.

What’s changing in the answer choices? ____________________________________

What tense is the sentence in? _____________________________________________

When you see verbs changing in the answers:


1. Find the subject: choose the verb that is consistent with
its subject.
2. Check the tense: choose the answer that is consistent in
tense with the other verbs in the sentence, unless a time
indicator word tells you to change tense.

134 | © TPR Education IP Holdings, LLC


WORDS PART I

READING
PRONOUNS
Although most people don’t realize 6. A) NO CHANGE
it, by speaking English, 6 he or she is B) we are
giving voice to 1,500 years of history. C) you are
D) they are

WRITING AND
LANGUAGE
What’s changing in the answer choices? ____________________________________

Which word does the pronoun refer back to? ________________________________

MATH
Like a verb with its subject, a pronoun must be consistent with the noun it refers
back to and with other pronouns in the sentence.

7. A) NO CHANGE There’s more


When Julius Caesar’s armies
information about
landed on British shores in 55 BCE, B) their pronouns and
neither 7 there language nor that of C) it’s apostrophes in the
Punctuation chapter.
the local inhabitants bore any relation to D) its
what would eventually become English.

What’s changing in the answer choices? ____________________________________

Which word does the pronoun refer back to? Is it singular or plural?

_______________________________________________________________________

Which form of their/there is most appropriate here? __________________________

© TPR Education IP Holdings, LLC  |  135


SAT MANUAL
READING

Precision is Better Than Like, Whatever


Still confused Caesar’s armies spoke Latin, 8. A) NO CHANGE
about pronouns? Go to and the British natives spoke Celtic; B) some of them
pages 290–293 for
Old English did not arrive in Britain C) those
a review!
until Germanic tribes invaded in the D) Germanic languages
WRITING AND
LANGUAGE

5th century and 8 they became


dominant.

What’s changing in the answer choices? ____________________________________


MATH

What does the pronoun refer back to? ______________________________________

Pronouns can make writing more concise, but using a noun instead of a pronoun
usually makes a sentence more precise.

When you see pronouns changing in the answers:


1. Find the original: choose the pronoun that is most
consistent with the noun or other pronoun it refers to.
2. Make it precise : if it’s unclear what a pronoun refers to,
use a noun instead.

136 | © TPR Education IP Holdings, LLC


WORDS PART I

READING
WHAT’S A SYNONYM FOR PRECISE?
According to one study, 9. A) NO CHANGE If you run into a word
B) power. you don’t know the
approximately 26% of modern English
meaning of, rely heavily
words come from this Germanic C) shock. on POE with the other
9 impact. D) influence. answer choices.

WRITING AND
LANGUAGE
What’s changing in the answer choices? ____________________________________

Which word gives the most precise meaning in this context? ___________________

MATH
When you see vocabulary changing in the answers:
• Use context: read the non-underlined portion to get a
sense of the necessary meaning.
• Choose the most precise word: watch out for
nuances of meaning between words that might appear
to be synonyms.

© TPR Education IP Holdings, LLC | 137


SAT MANUAL
READING

SHORT AND SWEET


Old English is essentially a foreign 10. A) NO CHANGE
language, but Middle English bears B) very much closer
a 10 closer resemblance to modern C) altogether closer and
nearer
English.
WRITING AND

D) much more close


LANGUAGE

What’s changing in the answer choices? ____________________________________

Do the extra words make the sentence more precise? __________________________


MATH

On the SAT, you should only add words if they clarify the meaning of a sentence.

Sometimes, removing Modern English may be 26% 11. A) NO CHANGE


words from a sentence percent Germanic, but a much larger B) influence of the
can make it incomplete. C) impact on the language of
See the Words Part II
portion of it comes from the 11 highly
the
chapter for more influential Normans, who invaded
information! D) DELETE the underlined
England in the 11th century.
portion.

What’s changing in the answer choices? ____________________________________

Do the extra words make the sentence more precise? _________________________

When “DELETE” appears in the answer choices, try it first!


It’s not always right, but if the sentence is otherwise consistent
and precise, choose it.

138 | © TPR Education IP Holdings, LLC


WORDS PART I

READING
WORDS PART I DRILL
Time: 8 minutes

Questions 1–11 are based on the following passage.

WRITING AND
LANGUAGE
The Story of English 1
The Norman Conquest took place in 1066 and A) NO CHANGE
1 is led by William the Conqueror of Normandy, B) is leading
which was part of what is now France. With the Norman C) was led
Conquest came a new language: French. At first, D) were led

MATH
Norman French was only used by the royal court and
the king’s Norman supporters, but by 1362 2 they had 2
been established as the language of the law, as it was the A) NO CHANGE
language used to conduct parliamentary debates. This B) it
was not the only major change in language around this C) he or she
time: only fifteen years earlier, English had 3 replaced D) those
Latin in schools as the primary language of instruction.
By the end of the 14th century, Geoffrey Chaucer had
3
begun to write his famous collection of stories, The
A) NO CHANGE
Canterbury Tales, which 4 are still recognized as
B) fixed
the gold standard of Middle English literature. The
C) repaired
Canterbury Tales might be difficult for modern readers
D) changed
to understand, 5 and Middle English is not the
illegible dialect that Old English is.
4
A) NO CHANGE
B) are still noticed
C) is still recognized
D) was still recognized

5
A) NO CHANGE
B) but
C) so
D) for

© TPR Education IP Holdings, LLC  |  139


SAT MANUAL

Although English was spoken all over the British 6


READING

Isles, Middle English never 6 will become a fully A) NO CHANGE


standardized written language. Such standardization B) becomes
came after a technological innovation, the printing press, C) became
made it possible for readers far and wide to read the D) had been becoming
same texts. The fact that readers from all over England
WRITING AND
LANGUAGE

could have access to a text meant that writers had to


7
standardize 7 there language. Since London was
A) NO CHANGE
the center of population and intellectual life, its dialect
B) its
became the closest thing that English had to an “official”
C) one’s
language.
D) their
MATH

140  |  © TPR Education IP Holdings, LLC


WORDS PART I

Attempts to standardize English spelling and 8

READING
grammar 8 hardened during the 17th and 18th A) NO CHANGE
centuries, culminating in three major dictionaries in the B) intended
18th and 19th centuries, which were published in 1755, C) desensitized
1828, and 1884. However authoritative those books D) intensified
may have been, English continued to evolve. American

WRITING AND
LANGUAGE
English, 9 by contrast, has been influenced by its
9
contact with Spanish colonists and Native Americans.
A) NO CHANGE
British English has been influenced by its proximity to
B) for instance,
Europe and by its history as a major colonial empire.
C) nevertheless,
Establishing English footholds in South Africa and
D) moreover,
10 setting them up in India may have forced English

MATH
on the colonized peoples, but the English language was
also influenced by those peoples. 10
A) NO CHANGE
English is a living language, and its history is a
B) India
living history, regardless of any attempts to halt
C) also establishing them in India
11 such constant transformation.
D) India, too

11
A) NO CHANGE
B) it.
C) them.
D) that.

© TPR Education IP Holdings, LLC  |  141


SAT MANUAL

Summary
• The three most important things to look for
in a Writing and Language answer are that
it is ______________________ with rest of
the sentence and passage, it uses the most
______________________ words, and it is as
______________________ as possible.

• Ideas that agree with each other require a


______________________ transition.

• Ideas that disagree with each other require an


______________________ transition.

• In general, a transition should be


______________________ with the ideas it is
connecting.

• A verb should be ______________________ with


its subject and with other verbs in the sentence.

• A pronoun should be ______________________


with the word it refers back to.

• If you can’t identify only one possible


thing a pronoun could refer to, it is not
______________________ and you should
______________________.

• When you see words that mean different things


in the answer choices, pick the one that gives
the most ______________________ meaning in
context.

• When all grammatical errors are eliminated,


choose the ______________________ answer.

• I have accomplished ___________ of the 4 goals


stated at the beginning of this chapter.

142 | © TPR Education IP Holdings, LLC


WORDS
PART I
PRACTICE

143
SAT MANUAL
READING

TRANSITIONS

Public defenders help uphold an important legal 1


right for American citizens. 1 Nevertheless, they save A) NO CHANGE
the legal system and the American public a great deal of B) Besides,
WRITING AND
LANGUAGE

time and money. C) Specifically,


D) Moreover,

There have been many advances in the tools that 2


MATH

nations can use to communicate with one another, A) NO CHANGE


2 but technology has not yet been able to make B) and
simultaneous translation into multiple languages an C) for
automated task. D) so

When a child feels safe at home, he may 3


communicate with ease. But put him in a stressful A) NO CHANGE
situation like a classroom, and suddenly his ability to B) Consequently,
articulate his thoughts is gone. 3 Subsequently, when C) By contrast,
working with a patient, a speech pathologist must first D) Meanwhile,
try to make him feel as comfortable as possible.

Those who believe that the U.S. embargo on Cuba is 4


an effective method of influencing the Cuban government A) NO CHANGE
to change its policies cite several rationales. Primarily, they B) Regardless,
believe that the embargo pressures the Cuban government C) In effect,
to improve its human rights record. 4 More specifically, D) Furthermore,
they argue that since the Cuban government controls
the economic power of the individual in Cuba, lifting
restrictions on trade would benefit only the Cuban
government, not the people of Cuba.

CONTINUE
144  |  © TPR Education IP Holdings, LLC
WORDS PART I PRACTICE

After three years of marriage and one daughter, 5

READING
Henry VIII had Anne Boleyn beheaded for adultery A) NO CHANGE
after he had the marriage annulled. 5 Similarly, there B) Moreover,
is something odd about this sequence of events: if the C) However,
marriage was annulled—and therefore was ruled to have D) In fact,
never actually existed—how could Anne have betrayed

WRITING AND
LANGUAGE
it?

Negative reciprocity makes rational sense: of course 6


people would want to get as much as they can for as A) NO CHANGE
little as they can give. Then there is balanced reciprocity, B) For example,

MATH
which appears irrational. 6 By definition, balanced C) Still,
reciprocity is balanced, which means both parties D) Therefore,
essentially give and receive equally.

While endorphins themselves are not well 7


understood, the effect they have is. Once their release A) NO CHANGE
is stimulated by exercise, they lock into special receptor B) As a rule,
cells called opioid receptors. 7 Regardless, endorphins C) Then,
are able to block the transmission of pain and D) However,
simultaneously produce feelings of euphoria.

In early human existence, time was strictly limited 8


by the natural pattern of day and night. Once the art A) NO CHANGE
of fire-making was established, humans were able to B) Foremost,
stay active during the night. The progression from fire C) Initially,
to more developed sources of artificial light was slow, D) Nevertheless,
with little change until the advent of gas illumination in
the early 1800s. 8 Furthermore, the biggest change
in night activity was in the workplace: artificial light
enabled mills and factories to employ their workforces
for longer hours. Following the increase of nighttime
labor was an increase in nighttime entertainment.

CONTINUE
© TPR Education IP Holdings, LLC  |  145
SAT MANUAL

On the one hand, a drop in the fertility rate can be 9


READING

good for a country’s economy. Children are quite costly A) NO CHANGE


to raise in developed countries. Fewer children means B) On the other hand,
that people can invest their time and productivity in C) In fact,
growing their personal wealth. 9 Therefore, many D) Likewise,
argue that more children are likely to serve as a benefit
WRITING AND
LANGUAGE

to the economy rather than as a drain.

The chemicals that leach into the public water 10


system over time in locations where fracking occurs A) NO CHANGE
MATH

have demonstrated harmful and long-term effects on the B) In conclusion,


local populations. 10 However, because of the strong C) For example,
support fracking has received based on its economic D) Besides,
value, it’s unlikely to end any time soon.

A couple of decades ago, there was a belief that by 11


now there would be solar panels everywhere: on the A) NO CHANGE
lawns of business parks, covering the roofs of private B) To these ends,
homes, and filling the empty space of deserts. 11 Thus, C) Indeed,
this wish has not yet materialized. D) Unfortunately,

May of 2012 marked the 372nd consecutive month 12


in which the average temperature of the entire globe A) NO CHANGE
exceeded average temperatures of the twentieth century. B) Furthermore,
12 Conversely, scientists have concluded that in order C) Nevertheless,
to keep global warming in check and prevent massive D) On the other hand,
changes to our environment, the temperature cannot
rise by another 2°C.

CONTINUE
146  |  © TPR Education IP Holdings, LLC
WORDS PART I PRACTICE

Because some in society think it is immoral to 13

READING
defend criminals, they assume that public defenders A) NO CHANGE
lack personal and professional morals as well. These B) therefore,
criticisms, 13 however, are misguided. C) indeed,
D) surprisingly,

WRITING AND
LANGUAGE
Most dieters never consult a dietitian—a 14
professional who advises people on diet choices for A) NO CHANGE
healthy living. Dietitians differ from nutritionists B) regardless,
in that nutritionists are not all regulated through C) on the other hand,
registered licenses. Dietitians, 14 for instance, must D) similarly,

MATH
earn a bachelor’s degree from an accredited college or
university and complete a professional internship before
sitting for licensing exams.

When speaking on matters of great international 15


importance, diplomats are particularly attentive to A) NO CHANGE
the nuances of their words, which makes the job of B) Furthermore,
translation difficult. 15 However, all languages have C) To these ends,
their own specific colloquialisms and idiomatic phrases, D) Therefore,
which can further complicate a translator’s task.

Having experience as a classroom teacher is an 16


important qualification for a school superintendent. A) NO CHANGE
16 Besides, it isn’t practical for a superintendent to B) Despite this,
implement the best plans for students and teachers C) After all,
without knowing much about their experiences. D) Instead,

CONTINUE
© TPR Education IP Holdings, LLC  |  147
SAT MANUAL

The initial goal of the Cuban embargo was 17


READING

to influence the newly established communist A) NO CHANGE


government in Cuba towards greater democratization. B) Nevertheless,
17 In addition, there was great fear of the spread of C) Meanwhile,
communism and its threat to capitalist and democratic D) At that time,
nations.
WRITING AND
LANGUAGE

In early human existence, time was limited by the 18


natural pattern of day and night. Due to the lack of A) NO CHANGE
artificial light, humans were diurnal. Once the art of fire- B) finally,
MATH

making was established, 18 however, humans were able C) subsequently,


to stay active during the night. D) likewise,

Over the past several decades, the roles of service 19


animals have been expanding. 19 These days, service A) NO CHANGE
animals were trained to help with the physical needs of B) Soon,
disabled individuals: they acted as guides for the blind, C) Next,
provided alerts for the deaf, and served as physical aides D) Historically,
for those with limited mobility. 20 Previously, the
tasks of service animals have expanded considerably.
20
21 Increasingly, service animals are no longer used
A) NO CHANGE
solely to assist with physical needs: they are now used
B) Formerly,
to help individuals with a wide variety of medical and
C) Despite this,
emotional needs.
D) Since then,

21
A) NO CHANGE
B) In any case,
C) By contrast,
D) However,

CONTINUE
148  |  © TPR Education IP Holdings, LLC
WORDS PART I PRACTICE

Algae has many advantages over other plant-based 22

READING
biofuels. 22 However, it produces between ten and a A) NO CHANGE
hundred times more oil per acre than any other fuel crop. B) Although,
This is due to the fact that fifty percent of the weight of C) First,
algae is the lipid oil that is used as biofuel. 23 In sum, D) Instead,
algae needs fewer natural resources than other plants

WRITING AND
LANGUAGE
for its cultivation. It can be grown using wastewater
23
rather than clean water, and it can be grown in only
A) NO CHANGE
wastewater—it does not require land. The amount of
B) Additionally,
land that would be required for facilities to produce
C) Accordingly,
enough algae to replace all fossil fuel use in the United
D) In other words,
States is less than one-seventh of that required for corn.

MATH
24 Consequently, many do not believe that algae as a
biofuel is as promising as it first seems. 24
A) NO CHANGE
B) Finally,
C) Nevertheless,
D) As a rule,

VERBS
Becoming a professional pianist takes years of 1
practice and 1 requires a great deal of sacrifice. A) NO CHANGE
B) require
C) will require
D) has required

The United Nations 2 have more than tripled in 2


size since it was established in 1945, and currently has A) NO CHANGE
193 member countries. B) will have
C) has
D) was

CONTINUE
© TPR Education IP Holdings, LLC  |  149
SAT MANUAL

Among many noteworthy ways in which 3


READING

biomedical engineers have improved our general health, A) NO CHANGE


a group of Portuguese engineers 3 are responsible for B) is
developing angiography, which uses X-rays to examine C) was
blood vessels and thus helps in the diagnosis and D) were
treatment of heart disease.
WRITING AND
LANGUAGE

Due to the lack of interest officials in the Cuban 4


government 4 has shown towards negotiations with A) NO CHANGE
America, to lift the embargo would be to sanction their B) have
MATH

behavior. C) have been


D) might have

Disagreement between the Soviet Union and the 5


other three Allied powers over how Germany should be A) NO CHANGE
governed 5 was one of the contributing factors of the B) were
Cold War. C) have been
D) having been

The sentiments reflected in the Preamble of the 6


Constitution—justice, liberty, and prosperity for all— A) NO CHANGE
6 was highly honored by our country’s Founding B) were
Fathers. C) was to be
D) had been

While endorphins themselves are not well 7


understood, the effect they 7 have are widely A) NO CHANGE
recognized. B) has is
C) have is
D) has are

CONTINUE
150  |  © TPR Education IP Holdings, LLC
WORDS PART I PRACTICE

Under current federal law, the use of service 8

READING
animals 8 are protected. A) NO CHANGE
B) have been
C) has been
D) is

WRITING AND
LANGUAGE
The high rate of infection in the most recent Ebola 9
outbreaks 9 are due to both the long incubation A) NO CHANGE
period and the types of symptoms that present. B) is
C) were

MATH
D) have been

The conclusion of the scientists 10 were that 10


dietary profile was directly related to children’s A) NO CHANGE
socioeconomic status. B) was
C) having been
D) had been

Permits are issued at the state level, and the 11


requirements for receiving one 11 varies from state to A) NO CHANGE
state. B) has varied
C) will vary
D) vary

Since 1959, human activity 12 have been emitting 12


at least 350 billion tons of carbon dioxide into the A) NO CHANGE
atmosphere. B) emit
C) has emitted
D) emits

CONTINUE
© TPR Education IP Holdings, LLC  |  151
SAT MANUAL

13 As the amount of fossil fuels are finite, the 13


READING

eventual depletion of global supply will create job losses, A) NO CHANGE


tax and tariff deficits, and economic decline. B) was
C) were
D) is
WRITING AND
LANGUAGE

While it is true that every profession experiences its 14


share of corrupt behavior, there is no data whatsoever to A) NO CHANGE
support the idea that there 14 has been a concentration B) is
of unethical behavior in the field of public defense. C) have been
D) will be
MATH

As Jon steps out onto the stage, he 15 had 15


remembered not to stare up directly into the lights. A) NO CHANGE
B) remembered
C) remembers
D) was remembering

As stated in the 1992 Cuban Democracy Act, the 16


United States 16 refuses to lift the embargo until the A) NO CHANGE
Cuban government demonstrates movement towards B) refused
“democratization and greater respect for human rights.” C) has refused
D) refuse

On the same day that Bell filed his application, 17


another inventor, Elisha Gray, 17 was filing a caveat for A) NO CHANGE
the same device. B) filed
C) had filed
D) had been filing

CONTINUE
152  |  © TPR Education IP Holdings, LLC
WORDS PART I PRACTICE

This violent act infuriated the nation, and within 18

READING
two weeks a judge 18 was ordering that the prisoners A) NO CHANGE
be released and cleared of all charges. B) had been ordered
C) has ordered
D) ordered

WRITING AND
LANGUAGE
Both the freedom of the West and emptiness of pre- 19
dawn streets 19 offer feelings of relief and autonomy A) NO CHANGE
for those who wished to be released from the restrictions B) offers
of mainstream society. C) offering

MATH
D) offered

The older generation will be supported by the 20


younger generation through systems of pensions and A) NO CHANGE
health care. Members of a smaller generation 20 earns B) will earn
fewer wages and thus will pay fewer taxes than their C) will have been earning
parents’ generation did. D) had earned

Changes to the section of the Americans with 21


Disabilities Act regarding the use of service animals A) NO CHANGE
21 are consisting of revisions to the definition of B) consisting
a service animal and extended the rights of service C) consisted
animals and their owners in publically- and privately- D) consist
owned places.

To clearly establish what the difference is between 22


creationism and evolution, biologist Kenneth Weiss A) NO CHANGE
22 introduces what he called the “scale of causal effects.” B) is introducing
C) had introduced
D) introduced

CONTINUE
© TPR Education IP Holdings, LLC  |  153
SAT MANUAL

As the 21st century progresses, we 23 faces many 23


READING

important environmental concerns. A) NO CHANGE


B) will have faced
C) faced
D) will face
WRITING AND
LANGUAGE

A good ear, a strong work ethic, and a natural 24


ability 24 is not enough on its own. A) NO CHANGE
B) is not going to suffice.
C) is not sufficient.
MATH

D) are not enough.

PRONOUNS
There are 70 million farms in India. For every farm 1
to have a two-animal till-and-plow draft team, and thus A) NO CHANGE
provide India with 1 their agricultural needs, 140 B) her
million animals would be required. C) its
D) the

In most economies, the older generation is 2


supported by the younger generation. The older A) NO CHANGE
generation earned this support when 2 they worked B) they worked to support its
to support their elders. C) it worked to support their
D) it worked to support its

For a single household to install enough solar 3


panels to meet the majority of 3 their energy needs A) NO CHANGE
would cost $10,000 or more. B) energy
C) its energy needs
D) the energy

CONTINUE
154  |  © TPR Education IP Holdings, LLC
WORDS PART I PRACTICE

Despite being popular with the French public 4

READING
on the occasion of her marriage to Louis XVI, Marie A) NO CHANGE
Antoinette quickly lost favor due to her lavish lifestyle B) the French poor
and expenditures. Her excessive spending on fashion, C) everyone
palace beautification, and gambling while 4 they D) she
starved was a likely motivator for anti-aristocratic

WRITING AND
LANGUAGE
propaganda.

When the board of Coca-Cola first decided to enter 5


the Chinese marketplace, 5 it was unaware that A) NO CHANGE

MATH
6 it would be read as “Ke-kou-ke-la,” which means B) they were
“female horse stuffed with wax.” C) they had been
D) he was

6
A) NO CHANGE
B) they
C) the name
D) the print

The movement for women’s suffrage began in 7


earnest in the 1840s. Although no official war was fought A) NO CHANGE
for 7 it, an intense cultural battle took place. B) them,
C) her,
D) the women’s vote,

When the telephone was first invented, consumers 8


marveled at the new technology but worried 8 they A) NO CHANGE
might cause electrocution. B) he or she
C) one
D) it

CONTINUE
© TPR Education IP Holdings, LLC  |  155
SAT MANUAL

The House and Senate also had the power to form 9


READING

a national army and navy, which were tasked with A) NO CHANGE


protecting the nation. 9 Their very existence served as B) Its
a deterrent to attacks from other countries. C) The military organizations’
D) Congress’s
WRITING AND
LANGUAGE

If we are able to 3D print organs, can 10 you 3D 10


print a human being? A) NO CHANGE
B) they
C) we
MATH

D) one

When I was young and would get angry at my 11


mother and grandmother, 11 she would tell me to go A) NO CHANGE
walk it off until I felt better. B) my mother
C) someone
D) one

He brought a wild chimpanzee to be raised 12


alongside his son Donald, 12 who was ten months old. A) NO CHANGE
B) whom
C) which
D) he

One of the fascinating differences the study 13


demonstrated was that, despite the fact that apes have A) NO CHANGE
the same social and physical capabilities that humans B) species
have, 13 we did not evolve to the same extent. C) they
D) apes

CONTINUE
156  |  © TPR Education IP Holdings, LLC
WORDS PART I PRACTICE

In India, cows provide an important fuel source for 14

READING
homes through 14 their dung. A) NO CHANGE
B) its
C) his or her
D) one’s

WRITING AND
LANGUAGE
People who live in warm climates wear sandals 15
more often than 15 these who live in colder climates A) NO CHANGE
do. B) that
C) those

MATH
D) DELETE the underlined portion.

If you give a friend a gift worth ten dollars for her 16


birthday, 16 you are likely to give you a present worth A) NO CHANGE
roughly the same amount. B) they are
C) one is
D) she is

The first people to notice that a child is in need of 17


speech therapy usually report 17 one’s concerns to a A) NO CHANGE
doctor or school counselor. B) their
C) his or her
D) our

Givers do not expect a return on the value of 18


18 our gifts. A) NO CHANGE
B) his or her
C) their
D) those

CONTINUE
© TPR Education IP Holdings, LLC  |  157
SAT MANUAL

The world has changed drastically over the last 19


READING

several decades, and cultural norms have changed with A) NO CHANGE


19 it. B) them.
C) him or her.
D) us.
WRITING AND
LANGUAGE

Globalization has had an enormous impact 20


on many aspects of modern life, but no change has A) NO CHANGE
been more drastic than 20 these in international B) that
MATH

marketplaces. C) this
D) DELETE the underlined portion.

The diet and weight-loss industry in the United 21


States alone is worth twenty billion dollars, with 108 A) NO CHANGE
million Americans on some kind of diet. Most of B) these
21 them will attempt four or five different diets in one C) it
year. D) DELETE the underlined portion.

Dr. John Snow recognized that a shared water 22


pump was the source of a cholera epidemic in two A) NO CHANGE
London neighborhoods, so he sanitized the water source B) their impact.
and removed the handle of the pump, thereby greatly C) the impact of the epidemic.
reducing 22 its impact. D) that one’s impact.

CONTINUE
158  |  © TPR Education IP Holdings, LLC
WORDS PART I PRACTICE

There is growing support in America to end the 23

READING
Cuban embargo. Those who are against the embargo A) NO CHANGE
argue that, since the embargo has been in place for B) it serves
decades without accomplishing its goal, 23 they serve C) we serve
no positive purpose. D) that one serves

WRITING AND
LANGUAGE
The greater a country’s proclivity towards 24
individualism is, the faster 24 it’s pace of life is. A) NO CHANGE
B) their
C) its

MATH
D) there

PRECISION
Public defenders are a vital part of our legal system, 1
but they are often treated with 1 contempt. A) NO CHANGE
B) corruption.
C) contemplation.
D) slander.

Epidemiologists study the spread of disease to help 2


2 know the risk factors for certain diseases and health A) NO CHANGE
conditions and to help establish preventive healthcare B) identify
policy. C) improve
D) summarize

It has become clear that non-aggressive physical 3


contact after an aggressive incident is 3 considerable A) NO CHANGE
to forming a positive bond between individuals. B) vital
C) meaningful
D) worthy

CONTINUE
© TPR Education IP Holdings, LLC  |  159
SAT MANUAL

Biomedical engineering is an emerging field. It 4


READING

merges two historically 4 distinguished disciplines: A) NO CHANGE


medicine and engineering. B) obvious
C) distinct
D) explicit
WRITING AND
LANGUAGE

The ability to speak is generally taken for granted, 5


despite its 5 ubiquity. After all, there is no other A) NO CHANGE
species that can communicate through speech. B) independence.
C) uniqueness.
D) creativity.
MATH

One point of 6 convention among historians is 6


the number of wives Henry VIII had. A) NO CHANGE
B) resolution
C) contest
D) contention

In 1874, Meucci neglected to send in the ten-dollar 7


fee to renew his caveat, and so his claim 7 passed A) NO CHANGE
away. B) elapsed.
C) stalled.
D) expired.

It has been less than 100 years since all Americans 8


were 8 crowned the right to vote. A) NO CHANGE
B) granted
C) endowed
D) authorized

CONTINUE
160  |  © TPR Education IP Holdings, LLC
WORDS PART I PRACTICE

The 9 conduct with which the Founding Fathers 9

READING
protected the wealth of the elite is demonstrated in A) NO CHANGE
Article 1, Section 8 of the Constitution. B) behavior with
C) means by
D) etiquette with

WRITING AND
LANGUAGE
It may be that people donate money because they 10
have 10 a deep interest in what they are donating to. A) NO CHANGE
B) a distant
C) an immersed

MATH
D) an intricate

Two animals 11 institute a till-and-plow draft 11


team. A) NO CHANGE
B) constitute
C) embody
D) represent

The ADA defines a service animal as “any dog that 12


is individually trained to do work or perform tasks for A) NO CHANGE
the benefit of an individual with a disability, including B) inclusive
a physical, sensory, psychiatric, intellectual, or other C) exclusive
mental disability.” This definition is significantly more D) certain
12 conclusive than the previous one, due to the
expanding number of uses service animals are being
trained for.

The Tragedy of the Commons is an economic 13


theory dating to the early 19th century and A) NO CHANGE
13 polarized in the 1960s. B) subsidized
C) popularized
D) habituated

CONTINUE
© TPR Education IP Holdings, LLC  |  161
SAT MANUAL

Historically, a child’s growth has been considered a 14


READING

14 conclusive indicator of the quality of a child’s A) NO CHANGE


environment. B) winning
C) final
D) mandatory
WRITING AND
LANGUAGE

The scientists described the results as a harbinger 15


of what can come from neglecting the health of our A) NO CHANGE
own children and of the need to 15 administer change B) indicate
through political and social action. C) implement
D) perform
MATH

3D printing opens up a world of 16 probabilities 16


to us. A) NO CHANGE
B) forecasts
C) chances
D) possibilities

There is nothing like a good “I told you so”: 17


everyone loves the triumphant feeling that comes A) NO CHANGE
from being proved correct in the face of others’ doubt. B) drain
But that victorious feeling may 17 wane if what C) compress
you had correctly predicted was something with dire D) ripen
consequences for humanity.

Although many people think of a polar vortex as 18


a singular cold weather event that affects an area from A) NO CHANGE
time to time, it is not. There are two 18 persistent polar B) insistent
vortices, one at the North Pole and one at the South Pole. C) repeated
D) firm

CONTINUE
162  |  © TPR Education IP Holdings, LLC
WORDS PART I PRACTICE

Solar energy has the 19 skill to do a great deal 19

READING
more than it does now if humanity can better harness A) NO CHANGE
the energy the sun provides. B) qualification
C) potential
D) aptitude

WRITING AND
LANGUAGE
Historically found primarily in Asian and Central 20
and South American dishes, cilantro is now a popular A) NO CHANGE
flavoring 20 sum around the world. B) addition
C) extension

MATH
D) attachment

Fracking is usually done by large oil and gas 21


companies that lease land and mineral rights from A) NO CHANGE
private owners in areas that are known or thought to B) payments
have large 21 entries of these resources. C) investments
D) deposits

To establish the 22 collision that economic 22


situation has on child growth, scientists collected data on A) NO CHANGE
the height, weight, and triceps skin-fold measurements B) impact
of children residing in eastern Kentucky. C) shock
D) contact

Once a person is infected with Ebola, it takes 23


anywhere from 2-21 days for symptoms to 23 present. A) NO CHANGE
B) gift.
C) divulge.
D) offer.

CONTINUE
© TPR Education IP Holdings, LLC  |  163
SAT MANUAL

Although every case is different, children who are 24


READING

24 pinpointed with autism may have a tendency to A) NO CHANGE


wander away from safe areas. B) determined
C) diagnosed
D) recognized

CONCISION
WRITING AND
LANGUAGE

Due to the recent increase in global trade 1 in 1


the last few decades, companies are consulting with A) NO CHANGE
cultural anthropologists on how the companies can best B) in the recently past few decades,
market their products to foreign populations. C) in recent years,
D) DELETE the underlined portion.
MATH

2 Advising dieticians are dieticians who provide 2


counsel on matters of nutrition on a large scale. A) NO CHANGE
B) Advising dieticians
C) It is advising dieticians who
D) Those who are advising dieticians

UN translators (other than Chinese-English 3


ones) are required to know at 3 least three or more A) NO CHANGE
languages: their native tongue plus two additional B) the very least three
official languages. C) minimum three or more
D) least three

Since this 4 triumphant and victorious success 4


of disease control, the importance of studying health and A) NO CHANGE
disease patterns has grown. B) triumph that was successful in disease
C) successful overcoming of
D) victory of disease

CONTINUE
164  |  © TPR Education IP Holdings, LLC
WORDS PART I PRACTICE

Without 5 physical connection and bodily 5

READING
contact the relationship is never fully repaired. A) NO CHANGE
B) physical contact
C) physical, bodily connection
D) connection and contact

WRITING AND
LANGUAGE
Cameron’s career as a concert pianist began when 6
he was 6 no older than three years of age. A) NO CHANGE
B) not any younger than three years old.
C) three years old.
D) around that time of three years aged.

MATH
Medical imaging is part of the 7 diagnostic 7
aspect and detection of ailments in medicine. A) NO CHANGE
B) diagnostic area
C) detecting and diagnosing of ailments
D) detection of diagnostics

The process of speech is an 8 amazingly intricate 8


one. A) NO CHANGE
B) amazing, intricate, and complicated
C) amazing, in that it’s complicated,
D) amazingly intricate and complicated

Even the most well documented events and eras 9


leave a great deal open to 9 inference and conjecture. A) NO CHANGE
B) conjecture and guesswork.
C) inferring and supposition.
D) conjecture.

CONTINUE
© TPR Education IP Holdings, LLC  |  165
SAT MANUAL

Although the fighting had ended, the war would 10


READING

not officially be over until all parties involved reached A) NO CHANGE


peace 10 settlements. B) settlements, ending the war completely.
C) settlements, concluding the conflict forever.
D) settlements, bringing a final end to the conflict.
WRITING AND
LANGUAGE

11 Generalized reciprocity and negative 11


reciprocity are generally considered to come with their A) NO CHANGE
own types of rationality. B) Generalized and negative reciprocity
C) Reciprocities that are both general and negative
D) Both generalized reciprocity and negative
MATH

reciprocity

After electricity came radio and television 12


broadcasting, which had an immense effect on how A) NO CHANGE
people used nighttime hours when 12 radio and B) radio and television shows began being
television broadcasting expanded to a 24-hour cycle. broadcast on
C) radio and television expanded to
D) broadcasting expanded to

When examined more closely, there appears to be 13


an end goal in the 13 selective nature of evolution. A) NO CHANGE
B) selective, discriminating
C) selective and discriminating
D) selectively discriminating

Elected officials are often 14 torn and cannot 14


decide between short-term fixes and more costly but A) NO CHANGE
ultimately more beneficial long-term plans. B) torn and indecisive
C) torn, unable to decide
D) torn

CONTINUE
166  |  © TPR Education IP Holdings, LLC
WORDS PART I PRACTICE

Our dependence on fossil fuels for the 15 vast 15

READING
majority and the bulk of our transportation needs is a A) NO CHANGE
major environmental concern. B) majority
C) vast majority of the bulk
D) majority, and also the bulk,

WRITING AND
LANGUAGE
One of the most exciting but also contentious 16
modern inventions is 16 that of 3D printing. A) NO CHANGE
B) 3D printing.
C) that we can 3D print.

MATH
D) the invention of 3D printing.

Everyone loves the triumphant feeling that comes 17


from being proved correct in the face of 17 the doubt A) NO CHANGE
and disbelieving skepticism of others. B) the disbelieving skepticism of others.
C) doubt and skepticism.
D) skepticism.

In order to communicate more easily with his 18


wife 18 while she was confined upstairs and he A) NO CHANGE
worked mostly downstairs, Meucci began work on an B) when they were on different floors,
electromagnetic sound device. C) from the lower floor where he was to the upper
floor where she was,
D) from the basement of the house where he
worked,

Representative democracy is based on a system of 19


19 at least two or more political parties that offer the A) NO CHANGE
electorate distinct ideological alternatives. B) at the very least two
C) two
D) no less than two

CONTINUE
© TPR Education IP Holdings, LLC  |  167
SAT MANUAL

Increased concern about domestic issues is 20


READING

demonstrated in both party affiliation 20 as well as A) NO CHANGE


voting trends. B) along with
C) and in
D) and
WRITING AND
LANGUAGE

As the value of a population’s productivity increases, 21


the value 21 is increasing of that population’s time. A) NO CHANGE
B) increases time as well.
C) of that population’s time increases.
MATH

D) for that population, of time, increases.

Based on the assumption that there might be a 22


genetic link to cilantro aversion, a study was conducted A) NO CHANGE
on 22 identical sets of twins and others who were B) identical sets of twins and fraternal sets of twins.
fraternal. C) identical fraternal twins.
D) identical and fraternal sets of twins.

The reason for the strong aversion to cilantro 23


has been a puzzle for some time and 23 is only now A) NO CHANGE
becoming more and more clear. B) has only very recently begun to become much
more clear.
C) has now been made clearer and more
understandable.
D) is only now becoming clearer.

The funding distribution measurement 24


demonstrated a sharp difference depending on which A) NO CHANGE
funding system— 24 either the progressive system or B) the progressive system versus the regressive
system—
the regressive system—was used.
C) either progressive or regressive systems—
D) progressive or regressive—

Answers can be found on pages 726–727.

168 | © TPR Education IP Holdings, LLC


QUESTIONS

169
SAT MANUAL
READING

GOALS REVIEW
At the conclusion of this chapter, you will be able to accomplish the following:

• Be familiar with the types of questions asked on the Writing and


Language test
• Find answers that are consistent with other parts of the passage
WRITING AND

• Find answers that make the meaning of part of the passage more
LANGUAGE

precise

QUESTIONS
In the Words chapter, all the questions we looked at were proofreader questions:
MATH

the ones that have a list of answer choices with no actual questions. In this chapter,
we’ll look at the other type of questions: editor questions.

The most important thing about questions on


Writing and Language is to notice them, so be
sure you’re reading carefully!

Many editor questions ask for different things than proofreader questions do,
but you still want to look for answers that are _______________________
consistent and
_______________________.
precise

170 | © TPR Education IP Holdings, LLC


QUESTIONS

READING
PURPOSE
The most common questions on the SAT Writing and Language Test will ask
you to choose an answer that best fulfills a particular purpose. Read the question
carefully and look for an answer that is consistent with the stated purpose.

Though most people are familiar 1. Which choice best establishes What should you read

WRITING AND
LANGUAGE
the main idea of the before answering this
with the Nobel Prizes, 1 they
paragraph? question?
can’t always list all five categories in
A) NO CHANGE
which the prizes are awarded. Alfred
B) most people haven’t
was born in 1833 into a Swedish attended one of the awards
manufacturing family. His father had ceremonies in Oslo.
a background in construction but had C) they are less familiar with

MATH
Alfred Nobel, the man who
started manufacturing equipment for
established the prizes.
the Russian army when Alfred was
D) many of the winners of
young. 2 Alfred had three brothers. the prizes never become
By the time he was seventeen, he could famous.
speak five languages, and he excelled
in chemistry and engineering. After 2. Which choice most effectively
sets up the examples in the
travelling abroad and working in a next sentence?
Parisian chemistry lab for several years, A) NO CHANGE
he returned to St. Petersburg in 1852 to B) The family moved to
help with the family business. Russia when Alfred was a
small child.
C) Alfred was a strong
student.
D) Alfred’s father did not
approve of his son’s interest
in poetry.

Sometimes purpose questions ask about main


ideas, and sometimes they ask about details.
Either way, use POE heavily!

© TPR Education IP Holdings, LLC | 171


SAT MANUAL
READING

WORDS IN DISGUISE
Some editor questions test the same thing that some proofreader questions test,
transitions and precision in particular.

If you need a review of 3 Even though it was dangerous, 3. Which choice provides the best
WRITING AND
LANGUAGE

transitions, go transition from the previous


Alfred had been studying a new but
back and look at paragraph to this one?
pages 131–133! highly volatile explosive, nitroglycerine.
A) NO CHANGE
He recognized the potential of the
B) During his time in Paris,
powerful explosive but wanted to
C) Although he was not the
make it safer for commercial uses. By original inventor,
1863, the family business in Russia had
MATH

D) Because of the Crimean


shut down, and Alfred moved back to War,
Stockholm with his father and one of
See page 137 to review his brothers, where he 4 focused on 4. Which choice most effectively
what the proofreader suggests that Alfred Nobel
making nitroglycerine safer. A tragic was driven by a particular
version of this question
looks like. accident at his factory in 1864, in which objective?
several men were killed (including one A) NO CHANGE
of Alfred’s brothers), only 5 doubled B) thought about
down his desire to tame nitroglycerine. C) forgot about
D) had intentions of

5. Which choice best maintains


When you’re asked to
the style and tone of the
maintain style and/or
tone, avoid slangy or passage?
very strong words. A) NO CHANGE
B) beefed up
C) reinforced
D) jacked up

172  |  © TPR Education IP Holdings, LLC


QUESTIONS

READING
ADDING AND DELETING
When choosing a two-part answer, both parts need to make sense!

One of the ways in which Alfred 6. At this point, the writer The best reasons for

WRITING AND
LANGUAGE
is considering adding the adding a sentence or
succeeded in making explosives safer
following sentence. not will refer to the ideas
was by inventing the detonator and the of consistency and
blasting cap. These devices provide a Nobel was a prolific inventor: precision.
by the time he died, he held
small initial explosion that then sets 355 different patents.
off a larger, secondary explosion.
Should the writer make this
6 He also experimented with mixing addition here?

MATH
nitroglycerine with different substances A) Yes, because it adds an
to improve its stability. important biographical
detail about Alfred Nobel.
B) Yes, because it clarifies
the role that Alfred Nobel
played in the development
of modern explosives.
C) No, because it does
not provide enough
explanation of which of
the patents were most
important.
D) No, because it distracts
from the paragraph’s main
focus on how Alfred Nobel
worked to improve the
safety of explosives.

Is the new sentence consistent with the topic of the paragraph? _________________

D, no not consistent

© TPR Education IP Holdings, LLC  |  173


SAT MANUAL
READING

7 He eventually was able to turn the 7. The writer is considering


explosive into a paste that could be deleting the underlined
sentence. Should the sentence
shaped into sticks, and thus dynamite
be kept or deleted?
was born. Dynamite was far more A) Kept, because the
stable than liquid nitroglycerine, it was invention of dynamite was
WRITING AND

what Alfred Nobel was


LANGUAGE

far more powerful than gunpowder,


most notorious for.
and it became widely used for large
B) Kept, because it provides a
construction projects such as building logical transition between
tunnels and canals. ideas in the paragraph.
C) Deleted, because it is
only loosely related to
the paragraph’s focus
MATH

on blasting caps and


detonators.
D) Deleted, because most
readers already know from
Road Runner cartoons that
dynamite comes in sticks.

yes
Is the sentence consistent with the topic of the paragraph? _____________________
yes
Does it make the meaning of the paragraph more precise? _____________________

174  |  © TPR Education IP Holdings, LLC


QUESTIONS

READING
ORDER
When reordering sentences in a paragraph, look for transitions and pronouns that
can help you place a sentence in the appropriate context. A sentence should be
consistent with the sentences before and after it.

[1] In 1888, Alfred’s brother Ludvig 8. To make this paragraph most

WRITING AND
LANGUAGE
died, and many newspapers mistakenly logical, sentence 4 should be
placed
ran obituaries of Alfred instead. [2] One
A) where it is now.
French newspaper referred to Alfred in
B) after sentence 1.
its erroneous obituary as “the merchant
C) after sentence 2.
of death,” because he owned more than
D) after sentence 5.
90 armaments factories. [3] He was

MATH
horrified at the way the newspaper had
memorialized him. [4] Despite this
business focus, Alfred was a committed
pacifist. [5] He decided that he needed
to do something to change the way
people would remember him. 8

COMBINING SENTENCES
Concision should not be a concern on most editor questions, but when combining
sentences, that’s the first thing you should look for.

Alfred rewrote his 9 will. His 9. Which choice most effectively


combines the sentences at the
new will left the majority of his fortune
underlined portion?
to establish the Nobel Prize.
A) will; by rewriting it he left
B) will: in the new will he left
C) will, because he decided to
leave
D) will in order to leave

D
Which choice is most concise? ____________________________________________

Is it also correctly punctuated and precise? _________________________________


yes

© TPR Education IP Holdings, LLC  |  175


SAT MANUAL
READING

CHARTS AND GRAPHS


As you do in the Math sections, read labels and keys for graphs carefully. Choose
answers that are consistent both with the data shown and with the focus of the
passage.

The Nobel Peace Prize is perhaps 10. Which choice provides an


WRITING AND

accurate interpretation of the


LANGUAGE

the most famous of the five prizes


chart?
that Alfred’s will established. Alfred’s
A) NO CHANGE
intention was for the prize to be
B) It was not until the end of
international: his will stated, “it is the twentieth century that
my express wish that in awarding the the prize become truly
international.
prizes no consideration be given to the
MATH

C) In the first quarter


nationality of the candidates, but that
of the twentieth
the most worthy shall receive the prize, century, International
whether he be Scandinavian or not.” It Organizations received far
more Peace Prizes than
took some time, however, for the Peace people from any particular
Prize to address struggles for peace in geographic region.
all corners of the world. 10 The prize D) There were no Latin
had winners from all over the world American winners of the
Peace Prize until the last
throughout the twentieth century. quarter of the twentieth
century.
Geographical Distribution of
Peace Prize Laureates, 1901–2000
% %

100 100
5% 4%
11%
15% 9%
4%
80 32% 80
29%
26%

60 60
11%
74%
6%
40 45% 35% 40
6%

17%

20 20

22% 20%
18%
11%
0 0
1901–1925 1926–1950 1951–1975 1976–2000

Africa Asia Latin North Eastern Western International


America America Europe Europe Organizations

176  |  © TPR Education IP Holdings, LLC


QUESTIONS

The expanding scope of the Peace Prize is 11. The writer is considering Need more work on

READING
adding the following sentence reading charts and
due in part to globalization, and in part
based on information from the graphs? Go to
to the changing nature of war through the chart. pages 507–512
twentieth century. 11 for a review!
The percentage of International
Organizations that have been
awarded the Nobel Peace

WRITING AND
LANGUAGE
Prize stayed mostly stable over
the course of the twentieth
century.

Should the writer make this


addition here?
A) Yes, because it supports
the idea that the

MATH
changing nature of war is
responsible for increased
diversity of prize winners.
B) Yes, because it shows that
there was some diversity
in prize winners early in
the twentieth century, even
when most winners were
from Western Europe.
C) No, because it contradicts
the argument that Alfred
Nobel wanted the prize to
be international.
D) No, because it includes
details that are not related
to the paragraph’s focus
on how the prize winners
changed over the course of
the twentieth century.

Is the sentence consistent with the graph? __________________________________

Is it consistent with the passage? __________________________________________

© TPR Education IP Holdings, LLC  |  177


SAT MANUAL
READING

QUESTIONS DRILL
Time: 10 minutes

Questions 1–11 are based on the following passage.


WRITING AND
LANGUAGE

The Socratic Problem to the Socratic Method 1


1 Teacher of Crito and Alcibiades, Socrates Which choice most effectively introduces the essay
by contrasting Socrates’s intellectual legacy with the
left few clues behind for modern historians.
other topics discussed in this paragraph?
The “Socratic problem,” as it is known, has
A) NO CHANGE
2 interested historians and philosophers for many
B) An Athenian born amid historical
centuries. Socrates himself wrote nothing that survives,
MATH

transformation,
and his biographical details are 3 sparse. All that we C) Despite his central role in modern thought,
D) Although his name is often mispronounced,

2
Which choice best conveys the idea that Socrates
remains a mystery?
A) NO CHANGE
B) battled
C) annoyed
D) perplexed

3
The writer is considering revising the underlined
portion to the following:
sparse, even though the deeds of
his prominent contemporaries were
recorded.
Should the writer make this revision here?
A) Yes, because it implies that details about
Socrates are hard to come by because he was not
well known when he was alive.
B) Yes, because it supports the point made earlier
that Socrates is a particularly difficult subject of
research.
C) No, because it unnecessarily repeats
information given earlier in the paragraph.
D) No, because it interrupts discussion of Socrates’s
contributions to philosophy with an irrelevant
detail.

178  |  © TPR Education IP Holdings, LLC


QUESTIONS

4
know about Socrates comes from three 4 sources.

READING
Which choice most effectively combines the
These three sources are the philosophical writings of
sentences at the underlined portion?
Plato and Xenophon and the plays of Aristophanes.
A) sources:
[1] It is generally agreed that Socrates was born in
B) sources: these include
Athens in 470 or 469 BCE, 5 approximately 400 years
C) sources, including

WRITING AND
before the birth of the Roman emperor Caesar Augustus.

LANGUAGE
D) sources; the sources are
[2] His father was a sculptor, and his mother was a
midwife. [3] Socrates eventually married Xanthippe,
with whom he had three sons. [4] Socrates’s father cut 5

stone for the Parthenon, and Socrates, having learned his Which choice offers an accurate interpretation of
the chart?
father’s trade, initially earned his living as a stonecutter.
A) NO CHANGE

MATH
[5] After his retirement from stonecutting, Socrates
B) after Aristotle’s great philosophical discoveries.
began his life’s great work: discussing philosophy.
C) amid the era of Greek domination.
[6] Regardless of whether he was paid, this teaching,
D) around the time the Jewish Book of Enoch was
and its supposed “corruption of the young,” ultimately written.
contributed to the philosopher’s death sentence by an
Athenian court. [7] The plays of Aristophanes suggest
6
that Socrates was paid for his philosophical teaching
To make the paragraph most logical, sentence 7
of the young, though Plato’s account of Socrates’s final
should be placed
words disagrees with such an idea. 6
A) where it is now.
B) before sentence 3.
500 BCE 400 BCE 300 BCE 200 BCE 100 BCE C) before sentence 5.
ALEXANDER SCIPIO (Elder)
JULIUS CAESAR
D) before sentence 6.
THE GREAT
356–323 236–184 100–44
PLATO
427–327 AUGUSTUS
CAESAR
SOCRATES ARISTOTLE
470–399 384–322 63BC–14 AD
ZENO CICERO
335–263 106–43
THE NATION OF ISRAEL
PERSIAN DOMINATION GREEK DOMINATION MACCABEAN RULE ROME
EZRA
458–390 Book of Enoch PHILO
Book of Daniel 20BC–50AD
I Maccabees

© TPR Education IP Holdings, LLC  |  179


SAT MANUAL

Socrates makes a few appearances in the military 7


READING

and political history of his era. 7 He was a soldier At this point, the writer is considering adding the
in the Athenian army during three campaigns, and he following sentence.
eventually played a role in government, most notably in Thucydides’s classic work, History of
the Peloponnesian War, is an excellent
the defense of Athenian generals who abandoned their source on this era’s military history.
dead in the water in order to continue to pursue the
WRITING AND

Should the writer make the addition here?


LANGUAGE

Spartan navy. Political life at this time was in transition. A) Yes, because it provides an additional source for
8 Governments frequently go through such historical background on the topic.
transitions, though they are not always so intolerant of B) Yes, because it names one of Socrates’s greatest
dissent. Socrates clashed with Athenian authorities over works.

how the defeated province should govern itself. C) No, because it strays from the paragraph’s focus
on the life of Socrates.
MATH

D) No, because it cites a text that is too old to be


considered historically reliable.

8
Which choice most effectively supports the idea in
the previous sentence?
A) NO CHANGE
B) That doesn’t excuse the fact that the Athenians
executed such a great man.
C) The powerful Athenians had recently been
defeated by the Spartans in the Peloponnesian
War.
D) Athens is known fondly as the “school of
Greece,” because it produced so many great
minds.

180  |  © TPR Education IP Holdings, LLC


QUESTIONS

9 For most of his life, Socrates was an average, 9

READING
if at times controversial, citizen. His most important Which choice provides the best transition from the
contributions came late in life as a philosopher. His great previous paragraph to this one?
contribution to the future of philosophy and instruction A) NO CHANGE

was the Socratic method. 10 The method is used today B) His political views rarely won the day, and they
sometimes got him in trouble.
by teachers worldwide, in classes that range from English

WRITING AND
C) Socrates has a wonderful cameo in the movie

LANGUAGE
to molecular biology. Those discussing the topic would Bill and Ted’s Excellent Adventure.
question it tirelessly, discarding hypotheses that could D) No one is really sure whether The Republic is
not stand up to reasonable questioning. It was from filled with Plato’s ideas or Socrates’s ideas.
this method that Plato and Xenophon built their own
philosophies. Beyond Plato and Xenophon, the Socratic
10
method has provided the foundation for much of

MATH
The writer wants to add a supporting detail to
Western academics, 11 particularly in law school. further explain how the Socratic method works.
Which choice best accomplishes this goal?
A) NO CHANGE
B) In Raphael’s famous painting The School of
Athens, Socrates is shown engaging in Socratic
dialogue with a group of students.
C) The method broke any problems down into a
series of questions, or testable hypotheses.
D) The word Plato used to describe Socrates’s
method was maieuticos, which means “skilled in
midwifery.”

11
Which choice most effectively concludes the
sentence and the paragraph?
A) NO CHANGE
B) though it requires a level of mutual respect,
both between teacher and students and among
the students, to be most effective.
C) and it is also the basis of effective
psychotherapy—that’s why therapists are always
asking, “how did that make you feel?”
D) from the basics of the scientific method to the
thesis, antithesis, synthesis model of essay-
writing.

© TPR Education IP Holdings, LLC  |  181


SAT MANUAL

Summary
• The most important thing about questions on
the Writing and Language Test is
to notice them
_________________________________________.

• On a purpose question, choose the answer that


is most ______________________
consistent with the ques-
tion. You r do r do not need to worry about
grammar.

• When adding information to a passage, the


information should be ______________________
consistent
with the passage and should make the passage
as ______________________
concise as possible.

• When combining sentences, make the new sen-


tence as ______________________ as possible.

• When you see charts and graphs


on Writing and Language, be sure to
______________________. Choose answers that
are ______________________ with the data and
with the passage.

• I have accomplished _________ of the 3 goals


stated at the beginning of this chapter.

182 | © TPR Education IP Holdings, LLC


QUESTIONS
PRACTICE

183
SAT MANUAL
READING

PURPOSE
Most dieters never consult a dietitian, despite 1
the fact that dietitians are crucial members of the Which choice adds the most relevant supporting
information to the paragraph?
nutrition and weight loss industry. A dietitian is a
A) NO CHANGE
professional who advises people on the role nutrition
WRITING AND
LANGUAGE

B) often major in science.


plays in attaining medical goals and maintaining a
C) are good at working with people.
healthy lifestyle. Dietitians differ from nutritionists in
D) must pass a registration exam in order to
that dietitians are regulated through registered licenses.
practice.
Unlike nutritionists, dietitians must have a bachelor’s
degree from an accredited college or university and
1 take other clearly specified steps.
MATH

Second of all, public defenders ensure that those 2


accused of crimes can exercise 2 their guaranteed Which choice most effectively emphasizes the
right to counsel. importance of the guaranteed right to counsel?
A) NO CHANGE
B) a right so fundamental that it was included in
the Bill of Rights.
C) a right that most people hope never to have to
think about.
D) their right in order to avoid amassing a great
deal of debt.

When I began, I certainly did not realize all that 3


it would take to become a concert pianist. Being a Which choice best supports the main point of the
professional musician isn’t just a job—it’s a calling that paragraph?

dictates how you live. A good ear, a strong work ethic, A) NO CHANGE

and a natural ability are necessary but not sufficient to B) mean most people will recognize your name.

achieve your goals. Years and years of daily practice are C) always make a person as happy as she’d
anticipated she’d be.
required, which in turn means that many other activities
D) necessarily make things easier, as practice
must be sacrificed. Even then, because demand for schedules and travel dominate your life.
professional pianists is limited, only a lucky few are able
to completely support themselves through such work.
Reaching the pinnacle of the field doesn’t 3 guarantee
great financial rewards either.

CONTINUE
184  |  © TPR Education IP Holdings, LLC
QUESTIONS PRACTICE

Other talents are needed as well. Had I known as an 4

READING
undergraduate what I know now, I would have signed up Which choice results in a sentence that best
for a few marketing classes rather than focusing solely supports the point developed in this paragraph?
on music. Breaking into the music world, as a soloist or A) NO CHANGE
an ensemble musician, is very difficult. As in many jobs, B) talents that are not usually developed by years of
solitary practice.
you have to be able to market yourself and maintain a

WRITING AND
C) unless you are so gifted that nothing else

LANGUAGE
network of connections, 4 skills that come naturally matters.
to outgoing types. With so many talented musicians D) and some people are better than others at that.
around, how you present yourself might determine
whether you earn a coveted audition spot.

MATH
5 A superintendent works in a school district. A 5
district’s school board, composed of elected members Which choice best establishes the argument that
of the community, hires a superintendent to oversee follows?
A) NO CHANGE
all aspects of the school district. The superintendent is
B) It can be very difficult to run
in charge of 6 all decisions relating to curriculum,
C) A superintendent functions like the Chief
staffing, distribution and expenditure of funds, school
Executive Officer of
safety, current and future facility needs, and more.
D) If there is a superintendent, then there must be

6
Which choice provides the most comprehensive
description of a superintendent’s responsibilities?
A) NO CHANGE
B) school principals and their staffs as well as
teachers.
C) ensuring the district’s students meet established
benchmarks.
D) keeping the many different people within the
system happy.

CONTINUE
© TPR Education IP Holdings, LLC  |  185
SAT MANUAL

Biomedical engineers don’t just design and 7


READING

construct items to help the functioning of our bodies: Which choice adds the most relevant supporting
they also design and produce actual body parts. This information to the sentence?
aspect of biomedical engineering is called tissue A) NO CHANGE
engineering and involves the creation of human tissue. B) if they produce more noteworthy advances.
The engineers work with stem cells and manually guide C) by growing non-human tissues as well.
WRITING AND
LANGUAGE

the growth of human tissue and organs. This work has D) by prompting an influx of funding opportunities
in the field.
already successfully grown full jawbones, tracheas, and
urinary bladders. Further advances promise to save
money, time, and lives 7 by drastically reducing the
need for organ donations.
MATH

It can take anywhere from months to years to 8


build the skills and confidence necessary to overcome a Which choice adds the most relevant supporting
communication or swallowing disorder, 8 and some information to the paragraph?
patients achieve better results than others. Knowing just A) NO CHANGE

how much time and effort patients have devoted to the B) as there are many disorders that might be
treated with therapy.
process makes being a part of their eventual success all
C) and sometimes this is frustrating.
the more rewarding.
D) and some patients will work with therapists
through their entire academic careers.

From an ethical perspective, critics argue that it 9


is hypocritical of the United States to limit trade with Which choice most effectively concludes the
Cuba based on ideological differences and human rights sentence?
concerns 9 when some would argue that our country A) NO CHANGE
could have a better human rights record. B) even though we trade with Russia.
C) but to trade freely with other nations with bad
human rights records.
D) when Cuba has shown signs of improvement.

CONTINUE
186  |  © TPR Education IP Holdings, LLC
QUESTIONS PRACTICE

It took a war to secure the right to vote at all 10

READING
in America. The war that came to be known as the Which choice most effectively supports the
American Revolution was fought to escape an oppressive preceding statement?
monarchy and establish a free democracy in which A) NO CHANGE
individuals were given a say in matters of government. B) many people consider this fact ironic.
Yet, after so much struggle and sacrifice, only some C) it’s possible some people should have fought

WRITING AND
harder.

LANGUAGE
individuals initially gained the hard-won right: 10 in
D) equality was definitely not fully achieved.
the immediate postwar years, only white, landowning
men were entitled to vote in the newly-created nation.
It took another 100 years—and a great deal more
violence—to secure the vote for African-American
males. The 15th amendment was ratified in 1870 as the

MATH
nation still reeled in the aftermath of the Civil War.

Emotional states are closely tied to the physiology 11


of the body. When we are nervous, our hearts beat Which choice is most consistent with the previous
examples in the sentence?
faster, our glands produce sweat, and 11 our minds
A) NO CHANGE
trigger other reactions. These sensations are usually
B) just by looking at us, people can see how we feel.
only temporary, the results of a release of the hormone
C) our voices tremble.
cortisol in response to a triggering stimulus. Similarly,
D) we exhibit other telltale signs.
the release of other hormones—endorphins—into
our systems promotes a sense of well-being. Because
it is taxing for the body to maintain elevated levels 12
of hormones for long periods, 12 both anxiety and Which choice most effectively concludes the
happiness tend to be short-lived emotional states. sentence and paragraph?
A) NO CHANGE
B) people sometimes have trouble controlling their
emotions.
C) some people turn to opiates to help them.
D) people have a hard time predicting how they
will feel in a given situation.

CONTINUE
© TPR Education IP Holdings, LLC  |  187
SAT MANUAL

As a school superintendent, Dr. Goren works 13


READING

with teachers, coaches, students, parents, and other Which choice best supports the main point of the
administrators to accomplish the district’s goals. He is paragraph?
responsible for finding a way to address the needs of A) NO CHANGE
each group as fully as possible without overextending B) the bells that signal the passage of time in
the system’s resources. To be an effective leader, he C) the possibility of violence stalking the halls of
WRITING AND
LANGUAGE

sometimes must make difficult decisions—and then D) interactions with the many people who have a
stake in
explain them well enough that everyone accepts and
moves forward with what must be done. The word
“administrator” might summon the image of someone
hidden away in an office dealing with paperwork, but
in reality, his days are shaped by 13 a schedule the
MATH

calendar committee establishes for all of the district’s


schools.

Early human existence was shaped by the natural 14


pattern of day and night. Lacking a means to artificially Which choice most effectively provides examples
illuminate the darkness, humans were diurnal, getting of the “do almost anything at any hour” trend
described at the end of the sentence?
up with the sun and going to sleep when it set. Once the
A) NO CHANGE
art of fire-making was established, humans were able to
B) the large number of people who suffer from
be more active during the night. Gaining control of fire insomnia,
did eventually lead us to 14 the bright glare of neon C) the huge electricity bills that some of us must
signs, but the innovations that so easily enable us now to pay,
do almost anything at any hour were relatively slow to D) night shifts, midnight sale extravaganzas, 24-
hour pharmacies, and all-night diners,
develop. 15 Ironically, we now have so much artificial
light, scientists warn of the dangers of “light pollution.”
15
Which choice most effectively supports the claim
made in the preceding sentence?
A) NO CHANGE
B) It was only with the advent of coal-gas
illumination in the early 1800s that the pace of
technological change really accelerated.
C) It’s hard to even imagine what the next big
innovation could be.
D) Invention can take a very long time due to the
limitations of nature and restricted working
hours.
CONTINUE
188  |  © TPR Education IP Holdings, LLC
QUESTIONS PRACTICE

One nutrition study was conducted in eastern 16

READING
Kentucky in 1994. At the time the data were collected, Which choice adds the most relevant supporting
the community was experiencing significant economic information to the paragraph?
depression, 16  with over 25% of households living on A) NO CHANGE
an income at or below the poverty line. To measure the B) at a time when the national poverty threshold
for a family of four was $16,450.
impact of this economic situation on children’s growth

WRITING AND
C) though much of the nation was more negatively

LANGUAGE
and nutrition, the researchers collected data on the impacted in 2008 when the housing bubble
height, weight, arm circumference, and triceps skin-fold burst.
measurements of students attending the community D) although not every family suffered equally.
elementary school. This information was then analyzed
to see what percentages of boys and girls living in the
community exhibited physical signs of either stunted

MATH
growth or obesity. 

Fracking has become a key method of gas 17


extraction in the United States, enabling the extraction Which choice most effectively sets up the
of some reserves that would otherwise have remained information that follows?
A) NO CHANGE
inaccessible. Along with horizontal drilling, fracking
B) Water is an increasingly scarce resource in
has been primarily responsible for increases in the
much of the world, and
U.S. production of natural gas since 2005—and those
C) Vast quantities of water are needed:
increases have meant that now the United States
D) Water is a resource humans too often take for
produces nearly all of the natural gas it uses. As a result, granted;
the United States is much less dependent on other
countries to fill its energy needs.
Despite this obvious benefit, there are compelling
arguments against the practice. One major concern is
fracking’s environmental impact. 17 Water is part of
the process: the average water consumption has surged
from 5 million to almost 25 million gallons per well.
Once the water has been injected into a well, only about
half of the now-contaminated resource is recovered. The
rest is lost and can filter into local water sources. This
potential for contamination drives further concerns
about public health. The chemicals that leach into the
public water system in the aftermath of fracking have
demonstrated harmful and long-term effects on the local
populations.
CONTINUE
© TPR Education IP Holdings, LLC  |  189
SAT MANUAL

The presence of aldehydes in cilantro also accounts 18


READING

for the differing tastes reported by those who dislike Which choice most effectively sets up the
it. 18 As in many issues of taste, there are at least information that follows?
two sides to this debate. Both camps can point to the A) NO CHANGE
presence of aldehydes to account for the similarity B) The most common complaint is that it tastes
like soap, though some people insist instead that
they perceive: soap contains fat molecules that form it tastes like bugs.
WRITING AND
LANGUAGE

aldehydes, while many bugs carry aldehyde chemicals on C) Some people are reminded of soap, some of
their skin. In fact, aldehyde is the chemical responsible bugs, and still others can’t quite name what it
for the dreaded stink that gives the stink bug its name, reminds them of.

which suggests that perhaps the only reason more people D) People who like the taste of cilantro tend not to
think of either soap or bugs.
think of soap instead of bugs is that more people have
eaten soap than have eaten bugs.
MATH

19 A polar vortex can have a significant effect on 19


other areas of the globe when it breaks into two or more Which choice best establishes the main idea of the
vortices. A break occurs when a shift in the jet stream paragraph?
A) NO CHANGE
causes a piece of the vortex to be broken off from the
B) Most people are more familiar with jet streams
main cyclone and diverted along the path of the polar jet
and less knowledgeable about polar vortices.
stream. Such a break in a polar vortex can have serious
C) Polar vortices are a fascinating weather
effects on the regions it travels over. In 1985, the United phenomenon, about which we need to learn
States suffered one of its worst weather events due to a more.
break in the arctic polar vortex. A wave of extreme cold D) Sometimes faraway weather events can affect us
too.
swept through the United States, primarily along the
East Coast and Midwest regions. In all, 129 deaths were
attributed to the storm, over $2.3 billion in agricultural 20
losses were reported, and 20 many people were Which choice is most consistent with the previous
reluctant to go outside. examples in the sentence?
A) NO CHANGE
B) it was very, very cold.
C) hundreds of homes were lost to wind damage.
D) some schools had to employ “inclement
weather” days.

CONTINUE
190  |  © TPR Education IP Holdings, LLC
QUESTIONS PRACTICE

A recent example of the tragedy of the commons 21

READING
is the appropriation and use of fossil fuels. Fossil fuels Which of the following choices best clarifies
are a limited resource found in relatively few regions of information presented in the first part of the
sentence?
the globe. These resources have been seized by a limited
A) NO CHANGE
few, resulting in a great disparity of wealth and power
B) erupt, including global job losses, tax and tariff
between individuals with equal regional claim to the deficits, and economic market collapse.

WRITING AND
LANGUAGE
resource. A compounding factor in an already troubling C) erupt, though it’s impossible to predict just how
story is the inevitability of even further suffering widespread they’ll be.
related to fossil fuel use. As the amount of fossil fuels is D) erupt in ways that we might not be able to
finite, their stores will eventually be depleted. Without imagine yet.

a stronger plan to compensate for that eventuality,


significant and widespread problems could 21 erupt,

MATH
which will surely be detrimental to the global population.

22 It can be hard to find a job in epidemiology 22


with only a bachelor’s degree. Currently, very few Which choice most effectively sets up the
universities and colleges worldwide offer undergraduate information that follows?

classes in epidemiology, let alone a bachelor’s degree. A) NO CHANGE

Although a master’s in epidemiology is becoming more B) The paths to becoming a professional


epidemiologist can be as varied as the
common, there are still very few doctoral programs that populations and diseases studied.
offer a degree in the field. Most epidemiologists come C) At the moment, the education system is in
to the profession via a background in clinical medicine, shambles.
public health, or both. A fair number, though, come D) Not very many people enter college planning to
from neither background and have instead been drawn study epidemiology.
to the field through another focus altogether.

CONTINUE
© TPR Education IP Holdings, LLC  |  191
SAT MANUAL

While a drop in the fertility rate may positively 23


READING

affect a population’s spending in the short term, research Which choice most effectively sets up the
has shown it to be detrimental in the long term. In information that follows?
most economies, the younger generations support A) NO CHANGE
older generations 23 —for example, through systems B) in ways that some people will never have
considered.
of pensions and health care. Members of a smaller
WRITING AND

C) —despite the current trends of young adults


LANGUAGE

generation will earn fewer wages and pay fewer taxes moving back in with their parents.
than their parents’ generation did. As a result, a smaller D) as best they can.
population will not pay enough into a pension system
to offset the payouts that retired members of the older
generation are receiving. Likewise, without at least an
equal number of healthy young people paying into an
MATH

insurance program, the cost of older people’s health care


will be too expensive for the labor market to support. 

Ebola’s long incubation period and the types of 24


symptoms the virus causes have contributed to the high Which choice most effectively completes the
number of people affected in the most recent outbreaks. explanation of factors affecting recent Ebola
outbreaks?
Because Ebola symptoms can take so long to emerge,
A) NO CHANGE
many infected people do not know they are sick and
B) Clearly, much more education is needed to
therefore pose a risk to others around them. When lessen Ebola’s threat.
people do begin to feel ill, the symptoms they experience C) An obvious challenge, then, is to encourage
can mimic those of the common cold, leading them more medical professionals to work in countries
to delay seeking medical treatment. And because that lack the necessary facilities.

Ebola outbreaks have always originated in countries D) Thus, as is the case with many diseases, the
nature of Ebola outbreaks is shaped both by the
whose medical infrastructure remains underdeveloped, virus’s characteristics and by the particulars of
adequate medical treatment has not necessarily been the communities in which it emerges.
available when patients finally seek help. 24  The
situation is a powerful example of the adage, “an ounce
of prevention is worth a pound of cure.”

CONTINUE
192  |  © TPR Education IP Holdings, LLC
QUESTIONS PRACTICE

READING
WORDS IN DISGUISE
It took a war to secure the right to vote at all 1
in America. The American Revolution was fought to Which choice most effectively conveys an attitude
escape 1 an optimistic monarchy and establish a of dislike towards the monarchy?
free democracy in which individuals were given a say A) NO CHANGE

WRITING AND
LANGUAGE
in matters of government. Yet, after so much struggle B) a benevolent

and sacrifice, only some individuals gained the hard- C) an oppressive

won right: in the immediate postwar years, only white, D) a disinterested

landowning men were entitled to vote in the newly-


created nation. It took another 100 years—and a great 2
deal more violence—to secure the vote for African- Which choice best connects the sentence with the

MATH
American males. The fifteenth amendment was ratified previous paragraph?
in 1870 as the nation still reeled in the aftermath of the A) NO CHANGE
Civil War. B) Prior to both the Civil War and the fifteenth
amendment,
2 Despite the fact that the women’s suffrage
C) Because the women’s suffrage movement was
movement had been firmly established by the 1840s,
established before the Civil War,
both black and white women had to wait much longer
D) Reconstruction created lots of tension, so
to gain the vote. Although no official war was fought, an
intense cultural battle took place. The issue came to a
head in 1917 when members of the National Women’s
Party, led by Alice Paul, picketed outside the White
House to urge President Wilson and Congress to pass
a constitutional amendment that would guarantee their
voting rights. This inaugural White House protest was
conducted in an orderly and peaceful fashion.

CONTINUE
© TPR Education IP Holdings, LLC  |  193
SAT MANUAL

The year is 1789, and the revolutionary spirit has 3


READING

taken hold of France’s starving poor. Devastating food Which choice provides the best transition from the
shortages have prompted riots. When asked what the previous paragraph to this one?
starving people should do when they have no bread to A) NO CHANGE
eat, the spoiled and frivolous Queen of France replies, B) What could have led her to be so uncaring?
“Let them eat cake.” With those words she ensures her C) Of course, people who can’t afford bread would
WRITING AND

be even less likely to be able to afford cake.


LANGUAGE

place in history as an extreme example of the callous


D) Even worse, cake contains a lot of sugar and fat,
contempt those at the top of the social hierarchy showed making it an unhealthy option for poor people
to those at the bottom. even if they could have afforded it.
3 This story is memorable but almost certainly
not true. The phrase commonly attributed to the
queen was already in popular circulation by the time
MATH

she was born in 1755. In 1765, Jean-Jacques Rousseau,


a philosopher and political advisor, wrote in his
Confessions that he had heard the phrase uttered “by
a great princess.” Though he could not have been
referring to Marie Antoinette, the myth was likely
inspired by his story. One reason the misattribution is
so pervasive may be that the phrase “let them eat cake”
succinctly epitomized the deep divide between poorer
French people and the aristocracy. Claiming that Marie
Antoinette blithely suggested that the starving peasants
should eat cake served to underscore the lack of empathy
the French elites demonstrated toward the working class
in the years before the Revolution.

The persistence of the quotation’s misattribution 4


to Marie Antoinette in particular is understandable. Which choice provides the most effective transition
4 Being quite popular with the French public on the between ideas in the paragraph?
occasion of her marriage to Louis XVI, she quickly lost A) NO CHANGE
favor due to the extravagance of her life at court. Her B) Despite being
lavish spending on fashion, palace beautification, and C) Fortunate to be quite
gambling came to seem particularly excessive while so D) Since she was
many suffered. And because she died on the guillotine,
she had no opportunity to alter her reputation after the
Revolution.

CONTINUE
194  |  © TPR Education IP Holdings, LLC
QUESTIONS PRACTICE

Service animals are useful in addressing more than 5

READING
just physical health issues; they can also be helpful when Which choice most effectively completes the
working with individuals diagnosed with intellectual sentence?
disorders. An autistic child, for instance, may have a A) NO CHANGE
tendency to wander into danger. Such behavior requires B) boring.
constant vigilance on the part of a caregiver, which can C) draining.

WRITING AND
LANGUAGE
be 5 exhilarating. When a service animal is properly D) heartening.

trained and leashed to the belt of a child, it can act as an


ever-present anchor that keeps the child out of harm’s
way. Further, preliminary research suggests that some
autistic children find the constant presence of a service
animal calming and focusing.

MATH
6 Translators play a vital role in diplomacy. Clear 6
communication is essential for diplomatic meetings to Which choice is the best introduction to the
proceed smoothly. The more languages that are involved paragraph?
in a meeting, the more challenging it is to ensure A) NO CHANGE

that clear communication occurs. The task becomes B) Few people truly understand the complexities of
international diplomacy.
even more pressure-filled when the success of a peace
C) To do a good job, a translator needs to aspire to
agreement or international settlement is at stake. For a be a diplomat.
UN translator, though, such considerations are all in a D) Effective translators spend a great deal of time
day’s work. perfecting their skills.

CONTINUE
© TPR Education IP Holdings, LLC  |  195
SAT MANUAL

Fracking has become a key method of gas 7


READING

extraction in the United States, enabling the extraction Which choice provides the most effective transition
of some reserves that would otherwise have remained from the previous paragraph?
inaccessible. Combined with horizontal drilling, A) NO CHANGE
fracking has been primarily responsible for increases B) Despite this obvious benefit,
in the U.S. production of natural gas since 2005—and C) On the other hand,
WRITING AND
LANGUAGE

those increases have meant that now the United States D) Because life is often very complicated,

produces nearly all of the natural gas it uses. As a result,


the United States is much less dependent on other 8
countries to fill its energy needs. Which choice best maintains the style and tone of
7 At the same time, there are compelling the passage?
arguments against the practice. One major concern is A) NO CHANGE
MATH

fracking’s environmental impact. Vast quantities of water B) crummy


are needed: the average water consumption has surged C) messed up
from 5 million to almost 25 million gallons per well. D) harmful
Once the water has been injected into a well, only about
half of the now-contaminated resource is recovered. The
rest is lost and can filter into local water sources. This
potential for contamination drives further concerns
about public health. The chemicals that leach into the
public water system in the aftermath of fracking have
demonstrated 8 junky and long-term effects on the
local populations.

CONTINUE
196  |  © TPR Education IP Holdings, LLC
QUESTIONS PRACTICE

3D printing is also being applied to answer 9

READING
medical needs. The process has already produced usable Which choice provides the best transition from the
medical devices and patient-specific implants. Recently, previous paragraph to this one?
engineers built the first 3D bioprinting production A) NO CHANGE
system, which might someday have the capacity to print B) The waiting lists for people in need of organ
donations can be extremely long, and many
organs and body parts from layers of living tissue. While people die before receiving a transplant.

WRITING AND
LANGUAGE
this technology currently requires further research and C) Some people in the manufacturing sector are
development before it is ready for use, the thought of extremely wary of the changes 3D printing may
someday being able to produce necessary organs for the cause.

many people awaiting organ transplants is exhilarating. D) As with any technology, it’s important to gauge
how unequal access to 3D printing may put
9  Despite the promise of this new already disadvantaged communities even
technology, we cannot allow ourselves to get swept further behind.

MATH
up in the current of innovation without weighing
the consequences. Although the positive aspects
of 3D printing are enormous, there are other, less-
immediate effects to consider. If we are someday able to
manufacture all our goods at home, for example, what
will happen to the manufacturing industry? How will the
people who used to create our clothes and produce our
car parts support themselves and their families?

We live in a golden age of human travel. We can 10


travel vast distances and experience other cultures as Which choice best maintains the style and tone of
readily as our ancestors might have visited a nearby the paragraph?

town. What we might once have known about only A) NO CHANGE


B) also expand our exposure to
through reading, we can now experience firsthand.
C) encounter some nasty little things called
Unfortunately, as we expand our direct exposure to
D) catch all sorts of exotic-sounding
faraway places and people, we 10 catch diseases. As a
result, the work of epidemiologists is increasingly vital.

CONTINUE
© TPR Education IP Holdings, LLC  |  197
SAT MANUAL

There is growing support in America and elsewhere 11


READING

to end the Cuban embargo. Those who favor ending Which choice provides the most effective transition
the embargo question whether, after decades without between ideas in the paragraph?
achieving its intended purpose, the embargo is likely A) NO CHANGE
to ever prove effective in that regard. This argument is B) Because the embargo has proved remarkably
ineffectual at promoting democracy in Cuba,
strengthened by the fact that the spread of communism
WRITING AND

C) Because they want to be able to buy better


LANGUAGE

is itself no longer the perceived or actual threat it once cigars,


was to the United States. 11 Since the embargo also D) Since the embargo has greatly curtailed travel
negatively affects children, who have no role in the between the two countries,
government, critics argue that its limiting effect on
economic development cannot be justified.
MATH

The system of division and term lengths among 12


these governing bodies allows the government to change Which choice provides the most effective transition
in response to the will of the people but also ensures that between ideas in the paragraph?

such change cannot happen so abruptly as to destabilize A) NO CHANGE


B) Given these circumstances, many politicians are
the entire system. As elected officials, Congress and the
influenced by campaign donations, and
president are accountable to the general population.
C) Since the United States generally has relatively
12 Yet since they are elected by different constituencies low voter turnout rates,
and at differing intervals, it is impossible for a D) Because the president is elected by the archaic
single popular vote to replace the entire federal Electoral College system rather than by the
popular vote,
government. Thus the Constitution charted a middle
path between the tyrannical monarchy the Founders
rejected and the Athenian-style democracy whose
volatility they feared. 

CONTINUE
198  |  © TPR Education IP Holdings, LLC
QUESTIONS PRACTICE

READING
ADDING AND DELETING
In December of 1871, Meucci and several Italian 1
associates established the Telettrefono Company and At this point, the writer is considering adding the
filed a caveat, a preliminary type of patent, with the following sentence.
A caveat acted as an official notice
United States Patent Office for his “sound telegraph.”

WRITING AND
that a patent would be filed in the

LANGUAGE
1 Caveats were only valid for one year, but they could future and required a description and
be renewed for an annual fee of ten dollars. drawings of the item patented.
Should the writer make this addition here?
A) Yes, because it more fully explains an important
term within the paragraph.
B) Yes, because it helps to explain why the fee to

MATH
file a caveat was so high.
C) No, because commonly understood terms do
not need to be further explained.
D) No, because it does not match the information
that is provided later in the paragraph.

Clear communication is essential for diplomatic


2
meetings to proceed smoothly. The more languages
At this point, the writer is considering adding the
that are involved in a meeting, the more challenging it following sentence.
is to ensure that clear communication occurs. The task Every day, UN translators do their
becomes even more pressure-filled when the success of work under conditions that make clear
communication essential.
a peace agreement or international settlement is at stake.
For a UN translator, though, such considerations are all Should the writer make this addition here?
A) Yes, because it emphasizes just how difficult the
in a day’s work. 2
job of UN translator can be.
B) Yes, because it makes clear that UN translators
work every day.
C) No, because it repeats information given
elsewhere in the paragraph.
D) No, because it is an inaccurate portrayal of the
job of UN translator based on the description in
the paragraph.

CONTINUE
© TPR Education IP Holdings, LLC  |  199
SAT MANUAL

Henry married Anne after his divorce. After three 3


READING

years of marriage and one daughter, Henry had Anne At this point, the writer is considering adding the
beheaded for adultery, right after he had the marriage following sentence.
annulled. 3 Herein lies a paradox: if the marriage Their daughter would one day become
Queen Elizabeth I, one of England’s
was annulled—and therefore was ruled to have never most beloved monarchs and the last of
actually existed—how could Anne have betrayed it? the Tudor dynasty.
WRITING AND
LANGUAGE

Should the writer make this addition here?


A) Yes, because it helps to explain why people
might find the annulment so surprising.
B) Yes, because to omit that fact would be to ignore
a large part of England’s history directly related
to the topic being discussed.
C) No, because such information is likely to bias
MATH

the reader unfairly against one of the arguments


being presented.
D) No, because the daughter’s identity has no
bearing on the paradox discussed in the
paragraph.

On average, the funding level across states was 4


relatively impressive: $10,774 per pupil per year. Because The writer is considering deleting the underlined
this number is an average, however, it doesn’t tell the sentence. Should the sentence be kept or deleted?
whole story. The difference between the highest amount A) Kept, because it illustrates the differences found
among states’ funding for education.
and lowest amount spent per student by states was
B) Kept, because it is important to judge certain
extreme. 4 For example, a student attending public
states and celebrate others.
school in Tennessee was likely to receive 40% less
C) Deleted, because it is unfair of the author to
funding than a student in Wyoming. compare states to one another.
D) Deleted, because it is unrelated to the
measurement standard mentioned in this
paragraph.

CONTINUE
200  |  © TPR Education IP Holdings, LLC
QUESTIONS PRACTICE

The ADA defines a service animal as “any dog 5

READING
that is individually trained to do work or perform tasks The writer is considering revising the underlined
for the benefit of an individual with a 5 disability.” portion to the following.
This definition is significantly more inclusive than the disability, including a physical,
sensory, psychiatric, intellectual, or
previous one, due to the expanding number of uses other mental disability.”
service animals are being trained for.

WRITING AND
Should the writer make this revision here?

LANGUAGE
A) Yes, because it emphasizes the diverse ways a
service animal can help someone.
B) Yes, because it establishes a contrast with the
idea of inclusion in the following sentence.
C) No, because it takes the focus of the paragraph
away from the general sense of what a disability

MATH
is.
D) No, because the list dilutes the impact of the
word “disability.”

Often, we find it extraordinary that some species 6


are born with certain innate instincts. 6 These turtles The writer is considering adding the following
bury their eggs deep in the sand on beaches and then sentence.
desert them. When the eggs hatch, no adults are present Take sea turtles, for example.
to guide the hatchlings to safety. Guided by instinct Should the writer make this addition here?
alone, a hatchling breaks itself out of its egg, works with A) Yes, because it encourages an active role for the
reader.
nestmates to dig a way out of the nest, and immediately
B) Yes, because it introduces the example the writer
makes its way to the ocean.
uses to support the idea in the first sentence.
C) No, because it is clear from the following
sentence that the writer is discussing sea turtles.
D) No, because the reader may not agree that sea
turtles are extraordinary.

CONTINUE
© TPR Education IP Holdings, LLC  |  201
SAT MANUAL

Solar power is nothing new. It enables us to grow 7


READING

the food we eat, while providing heat that warms our At this point, the writer is considering adding the
atmosphere and light that allows us to see. It even following sentence.
provides us with an important nutrient, vitamin D, Of course, as anyone who has ever
gotten a sunburn knows, the sun can
when we bask in its rays. In these ways, solar power has also cause harm.
long benefited humanity. 7 What is new, though, is
WRITING AND

Should the writer make this addition here?


LANGUAGE

technology that allows us to harness the sun’s energy to A) Yes, because it provides a more balanced view of
an even greater extent. the role of solar energy.
B) Yes, because it provides an additional point
of consideration that should be accounted for
when researching solar energy.
C) No, because it shifts the reader’s focus to a
point that is unrelated to the main topic of the
MATH

paragraph.
D) No, because sunburn can easily be prevented
using sunscreen.

The polar vortices are fueled by the polar jet 8


streams. Jet streams are narrow, high-speed air currents The writer is considering deleting the underlined
that exist in the tropopause, a transitional space between sentence. Should the sentence be kept or deleted?
the troposphere and the stratosphere. 8 Pilots must A) Kept, because it helps the reader understand
how everyday life can be affected by jet streams.
account for jet streams when flying, as planes often
B) Kept, because it clarifies the reason pilots tend
travel through the tropopause. Jet streams are caused to avoid flying into a polar vortex jet stream.
by an interaction between the rotation of Earth along C) Deleted, because air traffic controllers are more
its axis and atmospheric heating. Earth has multiple jet likely to address this issue than pilots are.
streams, the strongest of which are the polar jet streams. D) Deleted, because airplanes are not relevant to
the paragraph’s focus on the role of jet streams
in polar vortices.

CONTINUE
202  |  © TPR Education IP Holdings, LLC
QUESTIONS PRACTICE

Cultural anthropology is the study of human 9

READING
behavior. Cultural anthropologists are trained to observe At this point, the writer is considering adding the
and record facets of human cultures through fieldwork following sentence.
and participant observation. This training enables This commercial aspect of
anthropology may surprise people who
anthropologists to understand consumer behavior when think of anthropologists primarily
it is applied to the marketplace, making them vital assets as people who study mummies and

WRITING AND
pottery shards.

LANGUAGE
for marketers. 9
Should the writer make this addition here?
A) Yes, because it acknowledges that most
people aren’t familiar with the idea of cultural
anthropology.
B) Yes, because it provides additional details about
the role of cultural anthropologists in the global

MATH
marketplace.
C) No, because it is not directly related to the
paragraph’s focus on the work of cultural
anthropologists.
D) No, because it contradicts the information
given earlier in the paragraph about the role of
cultural anthropologists.

As health issues related to diet increase, many 10


government entities have been expanding their efforts At this point, the writer is considering adding the
to address problems related to nutrition, especially in following sentence.
children. Child obesity rates in America have doubled One way to fix this problem would
be to reform school lunch programs
in children and quadrupled in adolescents in the to provide free, healthy meals for all
last 30 years, causing a domino effect of adult health children.
problems that are now beginning to affect the welfare Should the writer make this addition here?
of the country. 10 As our healthcare system is already A) Yes, because school lunches are an important
both overpriced and unstable, many policy makers are source of nutrition for many children.

retaining the services of dietitians to advise on the best B) Yes, because it explains one possible remedy for
the problem of increasing child obesity.
ways to reverse the poor food choices of society.
C) No, because lack of physical activity is a bigger
problem in relation to child obesity than poor
nutrition is.
D) No, because it distracts from the paragraph’s
focus on the role of dietitians in public policy
related to nutrition.

CONTINUE
© TPR Education IP Holdings, LLC  |  203
SAT MANUAL

Communication in a stressful situation is hard 11


READING

enough on 11 anyone. Consequently, a speech The writer is considering revising the underlined
therapist must begin by winning her patients’ trust. portion to the following.
Once that trust is built they can start their real work anyone, and can be especially difficult
for a person who has difficulty
together. Depending on the specific need, that work communicating in ordinary situations.
will vary substantially. If the disorder a patient suffers
WRITING AND

Should the writer make this revision here?


LANGUAGE

from is mentally influenced, they will slowly proceed A) Yes, because the ability to communicate under
through written and spoken exercises together in a quiet stress is an important life skill.
room where no one else is present. That way, the patient B) Yes, because it provides details that clearly
can feel completely free to practice techniques out loud connect the first sentence to the rest of the
paragraph.
without the risk of judgment.
C) No, because it contradicts the idea presented in
the first part of the sentence.
MATH

D) No, because the sentence already makes clear


that communication can be stressful.

The Silent Sentinels, led by Alice Paul, spent 12


months engaged in nonviolent protest in front of the At this point, the writer is considering adding the
White House. In June 1917, the police arrested over following sentence.
The protests that blocked the street
200 women for blocking traffic. 12 Paul and many of
began in early 1917.
her followers underwent a hunger strike during their
Should the writer make this addition here?
incarceration to protest the deplorable conditions of the
A) Yes, because it provides further details about
prison, which resulted in many women being force-fed the protests of the Silent Sentinels.
and Paul being moved to a psychiatric hospital. B) Yes, because Alice Paul is an important figure
in both the history of nonviolent protest and
women’s suffrage.
C) No, because it repeats information given earlier
in the paragraph.
D) No, because it does not give a specific date on
which the protests began.

CONTINUE
204  |  © TPR Education IP Holdings, LLC
QUESTIONS PRACTICE

READING
ORDER
[1] There is a great deal of specialization within 1
the field of dietetics. [2] For example, clinical dietitians To make this paragraph most logical, sentence 3
and consultant dietitians assess and provide nutrition should be placed
therapy in very different contexts. [3] There, dietitians A) where it is now.

WRITING AND
LANGUAGE
may be involved with the treatment of specific food B) before sentence 1.
allergies or the treatment or prevention of food-related C) after sentence 4.
diseases such as diabetes, heart disease, and osteoporosis. D) after sentence 5.
[4] Clinical dietitians are often employed directly by
hospitals or care facilities. [5] Although technically not
medical doctors, clinical dietitians work directly with

MATH
doctors and nurses to serve patients who have specific
dietary needs. 1

[1] Black-and-white colobus monkeys are optimal 2


research subjects. [2] They form small groups that To make this paragraph most logical, sentence 3
consist of one dominant adult male monkey and six to should be placed
eight female or juvenile monkeys. [3] What’s more, they A) where it is now.
are known for their extremely low rates of intragroup B) after sentence 1.
aggression. [4] When male juveniles reach maturity, they C) after sentence 4.
split from their childhood group in order to form groups D) after sentence 5.
of their own. [5] These monkeys are known for being
highly intelligent, skilled at climbing, and rather lazy.
[6] A primatologist might study these monkey groups
in order to learn why aggression within each group is
so low. [7] The methods these monkeys use to avoid
aggression may have human applications. 2

CONTINUE
© TPR Education IP Holdings, LLC  |  205
SAT MANUAL

[1] But this disparity of a few years is nothing 3


READING

compared to the disparity of decades in dating the end of To make this paragraph most logical, sentence 1
World War II. [2] Generally, World War II is thought to should be placed
have spanned from September 1, 1939, to September 2, A) where it is now.
1945. [3] These dates, however, are somewhat arbitrary. B) after sentence 4.
[4] A strong case can be made that the German invasion C) after sentence 5.
WRITING AND
LANGUAGE

of Poland marked the beginning of the war, but several D) after sentence 6.

earlier conflicts deserve consideration. [5] For example,


the Italian invasion of Ethiopia in 1935, the German
involvement in the Spanish Civil War of 1936, and the
Japanese invasion of the Soviet Union in 1938 were all
international events that contributed to the development
MATH

of the greater war. [6] Thus some have argued that


WWII actually began as many as four years before the
generally accepted start date. 3

[1] It took a war to secure the right to vote at all 4


in America. [2] Yet, after so much struggle and sacrifice, To make this paragraph most logical, sentence 2
only some individuals gained the hard-won right: in the should be placed
immediate postwar years, only white, landowning men A) where it is now.

were entitled to vote in the newly-created nation. [3] The B) before sentence 1.

American Revolution was fought to escape an oppressive C) after sentence 3.

monarchy and establish a free democracy in which D) after sentence 4.

individuals were given a say in matters of government.


[4] It took another 100 years—and a great deal more
violence—to secure the vote for African-American
males. [5] The 15th amendment was ratified in 1870
as the nation still reeled in the aftermath of the
Civil War. 4

CONTINUE
206  |  © TPR Education IP Holdings, LLC
QUESTIONS PRACTICE

[1] However, the Constitution most effectively 5

READING
centralized power through the specific structure To make this paragraph most logical, sentence 5
it established for the federal government. [2] The should be placed
Founders allotted distinct powers to the branches of A) where it is now.
the government to create checks and balances, but B) after sentence 1.
they concentrated those powers in the hands of elected C) after sentence 2.

WRITING AND
LANGUAGE
officials, at a remove from the general population. [3] D) after sentence 3.

Four federal decision-making bodies were established:


the House of Representatives, the Senate, the presidency,
and the Supreme Court. [4] This fact, in combination
with the two-year term designated for House members—
as opposed to six years for senators, four years for the

MATH
president, and a life term for Supreme Court justices—
seems to reflect some reservations about the will of the
average voter. [5] Initially, the officials of only one of
those institutions, the House, were elected by popular
vote. 5

[1] This story is memorable but almost certainly 6


not true. [2] The phrase commonly attributed to To make this paragraph most logical, sentence 6
the queen was already in popular circulation by the should be placed
time she was born in 1755. [3] In 1765, Jean-Jacques A) where it is now.
Rousseau, a philosopher and political advisor, wrote in B) after sentence 2.
his Confessions that he had heard the phrase uttered “by C) after sentence 3.

a great princess.” [4] One reason the misattribution is D) after sentence 4.

so pervasive may be that the phrase “let them eat cake”


succinctly epitomized the deep divide between poorer
French people and the aristocracy. [5] Claiming that
Marie Antoinette blithely suggested that the starving
peasants should eat cake if they had no bread served to
underscore the lack of empathy the elites demonstrated
toward the working class in the years before the
Revolution. [6] Though he could not have been referring
to Marie Antoinette, the myth was likely inspired by his
story. 6
CONTINUE
© TPR Education IP Holdings, LLC  |  207
SAT MANUAL

[1] There are three main types of reciprocity. 7


READING

[2] The first is generalized reciprocity, which is basically To make this paragraph most logical, sentence 6
a form of philanthropy. [3] The giver does not expect should be placed
a return on the value of her gift. [4] Then there is A) where it is now.
balanced reciprocity, which is the equal and relatively B) after sentence 2.
timely exchange of goods or services. [5] The last type C) after sentence 3.
WRITING AND
LANGUAGE

is negative reciprocity, which is the attempt to get D) after sentence 4.

something whose value exceeds the value of what is


given for it. [6] The giver expects a return whose value
is roughly equivalent to that of his or her gift, such as in
the exchange of holiday presents. 7
MATH

[1] 3D printing opens up a world of possibilities 8


to us. [2] One of the first uses envisioned for it was To make this paragraph most logical, sentence 6
manufacturing, since the technology allows for the should be placed
mass production of identical objects. [3] 3D printers A) where it is now.

have been used to create everything from clothing to B) after sentence 2.

automobile parts to children’s toys. [4] Think of the C) after sentence 3.

money that could be saved if every household had D) after sentence 4.

access to such technology. [5] What if current efforts


are successful in developing printers large enough to
construct entire buildings? [6] With the push of a button,
you can produce such items for only the cost of raw
materials. 8

CONTINUE
208  |  © TPR Education IP Holdings, LLC
QUESTIONS PRACTICE

[1] It is possible that the tragedy of the commons 9

READING
has become exaggerated to the point of uselessness To make this paragraph most logical, sentence 2
in recent years. [2] It is now used more frequently as should be
justification for the acts it warns against rather than A) placed where it is now.
to prevent them. [3] Even when the theory was first B) placed after sentence 3.
postulated, it may have exaggerated the nature of C) placed after sentence 4.

WRITING AND
LANGUAGE
exploitation. [4] Were it true that all individuals are D) DELETED from the paragraph.

driven to harm society solely by self-interest, there


would be far fewer individuals and societies than there
are at present. [5] The fact that populations somehow
managed to survive to the modern day suggests
that communities are able to successfully share

MATH
resources. 9

[1] It is projected that we will hit that global 10


temperature rise in 2052. [2] In order to prevent this To make this paragraph most logical, sentence 3
from happening, we must limit carbon dioxide emission should be placed
to 46 billion tons by 2030. [3] Scientists have concluded A) where it is now.
that in order to keep global warming in check and B) before sentence 1.
prevent massive changes to our environment, the C) after sentence 1.
temperature cannot rise by another 2°C. [4] At our D) after sentence 4.
current rate, we will exceed that amount by at least 15
billion tons. 10

CONTINUE
© TPR Education IP Holdings, LLC  |  209
SAT MANUAL

[1] The reason for some people’s strong aversion to 11


READING

cilantro has been a puzzle for some time, which is only To make this paragraph most logical, sentence 3
now becoming clearer. [2] Based on the assumption should be
that there might be a genetic link to cilantro aversion, a A) placed where it is now.
study was conducted on sets of identical and fraternal B) placed after sentence 1.
twins. [3] Each of the sets was surveyed on both siblings’ C) placed after sentence 4.
WRITING AND
LANGUAGE

preferences regarding cilantro. [4] The study found D) DELETED from the paragraph.

that 80% of the identical twins were in agreement on


their like or dislike for cilantro, while only 50% of the
fraternal twins agreed. [5] For many scientists, this was
clear evidence that the predisposition against the herb
was genetic. 11
MATH

[1] There are two major concerns about fracking 12


for a growing number of Americans. [2] One of these To make this paragraph most logical, sentence 4
is the effect fracking has on the environment. [3] This should be placed
process uses an enormous quantity of water: up to A) where it is now.
3.5 million gallons are needed for just one well, and B) after sentence 2.
there are hundreds of thousands of wells in the United C) after sentence 5.

States [4] In order to “frack,” fluid must be injected D) after sentence 6.

into underwater wells. [5] Only half of the water


contaminated by fracking use is recovered; the rest
is lost and can filter into local water sources. [6] This
contamination leads to the second major concern: public
health. [7] The chemicals that leach into the public water
system over time in locations where fracking occurs
have demonstrated harmful and long-term effects on the
local populations. 12

CONTINUE
210  |  © TPR Education IP Holdings, LLC
QUESTIONS PRACTICE

READING
COMBINING SENTENCES
Public defenders are sometimes asked how they 1
can justify working hard on behalf of clients they may Which choice most effectively combines the
privately suspect are 1 guilty. Sometimes the clients sentences at the underlined portion?
are accused of committing heinous crimes. A) guilty; sometimes the clients are accused

WRITING AND
LANGUAGE
B) guilty, and sometimes the clients are accused
C) guilty
D) guilty, because sometimes the clients are
accused

MATH
You never know when a fire drill may occur during 2
a lab class full of flaming Bunsen 2 burners. In that Which choice most effectively combines the
situation you have to be ready to roll with the punches. sentences at the underlined portion?
A) burners, and in that situation you have to
B) burners, after which you need to
C) burners, so you must
D) burners, in that case you have to

Biomedical engineers don’t just design and 3


construct items to help the functioning of our Which choice most effectively combines the
3 bodies. On top of that they also design and produce sentences at the underlined portion?

actual body parts. A) bodies: they


B) bodies, because they
C) bodies; they sometimes
D) bodies—as an additional thing they

CONTINUE
© TPR Education IP Holdings, LLC  |  211
SAT MANUAL

In 2012, a team of researchers published a study 4


READING

entitled Is School Funding Fair? A National Report Which choice most effectively combines the
4 Card. In the report, reviews were made of the sentences at the underlined portion?
performances of all fifty states. A) Card, being a review of
B) Card; in the report were reviews of
C) Card, and in it, reviews were published
WRITING AND
LANGUAGE

D) Card, which reviewed

We all know what gift giving 5 is. Gift giving 5


is the presentation of a gift to someone else with the Which choice most effectively combines the
expectation, but not the requirement, of a return in kind. sentences at the underlined portion?
MATH

A) is, being
B) is:
C) is—it is
D) is; namely, it is

People who run often refer to the feeling of 6


euphoria called the 6 “runner’s high.” A runner’s high Which choice most effectively combines the
sentences at the underlined portion?
is a positive feeling that results from the endorphins
A) “runner’s high,” which is
produced by running.
B) “runner’s high.” This is
C) “runner’s high,” because runner’s high is
D) “runner’s high,” which results in

Then you have 7 humans. Humans can’t even 7


hold up their own heads for months after being born. Which choice most effectively combines the
sentences at the underlined portion?
A) humans, who
B) humans, which creatures
C) humans, and they
D) humans, who develop very slowly and

CONTINUE
212  |  © TPR Education IP Holdings, LLC
QUESTIONS PRACTICE

8 Ebola must be spread through bodily fluids, 8

READING
so general awareness of preventative measures helps Which choice most effectively combines the
contain the disease’s spread. Because of the way in which underlined sentences?
A) The average individual is very unlikely to
Ebola is spread, it is unlikely that an average individual
become infected with Ebola through bodily
would become infected. fluids, since the disease must be spread in that
way, and general awareness of preventative

WRITING AND
LANGUAGE
measures helps contain the disease’s spread.
B) A general awareness of the ways to prevent the
spread of Ebola, which happens through bodily
fluids, makes it unlikely that the average person
would become infected.
C) Because a greater awareness of preventative
measures helps to stop an average individual

MATH
from spreading Ebola through bodily fluids,
most people are unlikely to become infected.
D) A general awareness of the role of bodily fluids
in the spread of Ebola means that an average
person who takes preventative measures is
unlikely to spread or be infected by the disease.

9 One of the areas of focus in the field of child 9


welfare is of growing concern. That area explores the Which choice most effectively combines the
relationship between poverty and nutrition. underlined sentences?
A) The relationship between poverty and nutrition
is an area of concern that is growing within the
field of child welfare.
B) A growing concern, in the field of child welfare,
is that of the relationship between poverty and
nutrition.
C) The field of child welfare is growing by being
concerned about the relationship between
poverty and nutrition.
D) A growing area of concern in the field of child
welfare is the relationship between poverty and
nutrition.

CONTINUE
© TPR Education IP Holdings, LLC  |  213
SAT MANUAL

The best way to address global warming has been 10


READING

highly debated among scientists and politicians for quite Which choice most effectively combines the
some 10 time. This debate does not have an end in sentences at the underlined portion?
sight. A) time, with this debate not having an
B) time, and this debate does not have an
C) time, with no
WRITING AND
LANGUAGE

D) time, and there is no

The company could have consulted with a cultural 11


anthropologist familiar with the languages and dialects Which choice most effectively combines the
that exist in 11 China. The differences in languages and sentences at the underlined portion?
A) China, because the differences
MATH

dialects across China are numerous.


B) China, which
C) China; the many different languages and
dialects there
D) China: in that country the languages and dialects

12 No treaty was ever signed by the four Allied 12


Powers and the two German powers. The Allied Powers Which choice most effectively combines the
underlined sentences?
were the United States, the Soviet Union, Great Britain,
A) No treaty, of the four Allied Powers, being the
and France, and the two German powers were the
United States, the Soviet Union, Great Britain,
German Democratic Republic and the Federal Republic and France, was ever signed by the two German
of Germany. powers, which consisted of the German
Democratic Republic and the Federal Republic
of Germany.
B) The treaty, which was never signed, was
between the four Allied Powers, the United
States, the Soviet Union, Great Britain, and
France, and also the two German powers, the
German Democratic Republic and the Federal
Republic of Germany.
C) The two German powers, which were the German
Democratic Republic and the Federal Republic
of Germany, never signed the treaty between the
four Allied Powers, namely, the United States, the
Soviet Union, Great Britain, and France.
D) No treaty was ever signed by the four Allied
Powers, the United States, the Soviet Union,
Great Britain, and France, and the two German
powers, the German Democratic Republic and
the Federal Republic of Germany.
CONTINUE
214  |  © TPR Education IP Holdings, LLC
QUESTIONS PRACTICE

READING
CHARTS AND GRAPHS

Median weekly earnings 1


Unemployment rate
Which choice most accurately and effectively
represents the information in the graph?
Median Weekly Earnings and Unemployment

WRITING AND
A) NO CHANGE

LANGUAGE
Rate by Education Level in 2013
$2,250 16
B) However, those who do not complete high
$1,800 12 school are far more likely to be employed than
$1,350 those who earn a master’s degree.
8
$900 C) As might be expected, earning a doctorate
4 secures the greatest financial benefit.
$450
0 0 D) There is no difference in income and

MATH
unemployment levels between those who
ol

ee

e
leg

re

re

re

re
m
ho

gr

eg

eg

eg

eg
lo

ol

e
Sc

ip

stopped their education after high school and


D

lD

lD
eC
D
h

te

r’s

r’s

na

ra
ig

m
ol

cia

elo

te

to
eH

those who earned an associate degree.


sio
So
ho

as
so

oc
ch

es
Sc
m

As

D
Ba

of
So

Pr
ig
H

People who have earned high school diplomas are


twice as likely to be employed as those who have not.
1 High school graduates will also earn almost twice as
much weekly as those who don’t graduate.

As the figure suggests, 2 the higher the degree a 2


person earns, the more money he or she will earn. Which choice makes the writer’s description of the
figure most accurate?
A) NO CHANGE
B) those people who do not graduate from high
school generally earn the most money.
C) people with professional degrees have a lower
unemployment rate than people with any other
degree.
D) the higher a person’s degree, the better his or
her chances of having a job.

CONTINUE
© TPR Education IP Holdings, LLC  |  215
SAT MANUAL

Primary Energy Consumption in 2013


READING

Global India

3% 2%
4%
4% 2% 7%
WRITING AND
LANGUAGE

30%
35%

31%

49% 8%

25%
MATH

Oil Gas Coal Nuclear Hydro Renewables


Oil Gas Coal Nuclear Hydro Renewables Oil Gas Coal Nuclear Hydro Renewables

Figure 1 Figure 2

As India is already highly dependent on coal, even 3


a modest increase in its coal use would 3 increase Which choice offers an accurate interpretation of
the amount of money used to develop renewable energy the data in Figure 2?
alternatives. A) NO CHANGE
B) make India almost entirely dependent on coal.
C) result in coal providing more than half of the
energy India consumes.
D) reduce the amount of nuclear energy consumed
to almost zero.

Compared to global energy usage, India gets 4 a 4


greater percentage of its energy from gas. Which choice best reflects the information provided
in Figures 1 and 2?
A) NO CHANGE
B) a greater percentage of its energy from
renewables.
C) less of its energy from coal.
D) less of its energy from hydropower.

CONTINUE
216  |  © TPR Education IP Holdings, LLC
QUESTIONS PRACTICE

READING
Per Capita Income
Birth Rate Which choice offers an accurate interpretation of
U.S. Birth Rate in Relation the data in the graph?
to Per Capita Income
4.3 $42,500 A) NO CHANGE
3.4 $41,250 B) Although an indirect relationship between the
$40,000 birth rate and income has not been established,
2.6

WRITING AND
$38,750 C) Household income consistently increases as

LANGUAGE
1.7 $37,500 the birth rate decreases, although this does not
0.0 $36,250 prove one trend influences the other;
0 $35,000
D) Although there is evidence that income
92

94

96
98

00
02

04

06
08
10
and birth rates increase and decrease
19

19

19
19

20
20

20

20
20
20
proportionately, the evidence does not prove
that one directly affects the other;
Fewer children means that people can invest

MATH
their time and productivity in growing their personal
economies. 5 No information is available about this
possible correlation; the trend could be the result of
one or more other factors that affect both income and
birthrate.

Global Reproductive Rates (R) 6


of Infectious Diseases The writer wants to include information from the
Disease Transmission R figure that is most consistent with the situation
Diphtheria Saliva 6–7 described in the first part of the sentence. Which
choice most effectively accomplishes that goal?
Ebola Bodily fluids 1–2
A) NO CHANGE
Hepatitis C Bodily fluids 1–2
B) Americans would be less likely to catch Ebola
HIV/AIDS Bodily fluids 2–5
than any other disease shown in the figure.
Influenza Airborne droplet 2–3
C) Americans would be less at risk of catching
Measles Airborne 12–18 Ebola than any of the airborne diseases shown
Mumps Airborne droplet 4–7 in the figure.
Polio Fecal-oral 5–7 D) more Americans would die of Measles than of
Rubella Airborne droplet 5–7 any other disease, including Ebola.
Smallpox Airborne droplet 5–7
SARS Airborne droplet 2–5

Scientists define a disease’s reproductive rate as the


average number of people who are infected by a single
person. Because of the relatively advanced medical
system in the United States, the reproductive rate
for Ebola is lower than the average reproductive rate
worldwide. Even if Ebola were to spread in the United
States at its global rate, 6 most Americans would only
be infected if a family member became infected first. CONTINUE
© TPR Education IP Holdings, LLC  |  217
SAT MANUAL

7
READING

Oil Yield (per acre per year)


Which choice most accurately and effectively uses
Algae 15,600 information from the figure to support the claim
Biofuel Feedstock Sources

Corn 1,400 the writer has begun to make in this paragraph?


Palm Oil 653
A) NO CHANGE

Rapeseed 127
B) so great a yield per acre that many farmers are
considering switching to algae production.
WRITING AND
LANGUAGE

Sunflower 105
C) from ten to over one hundred times more oil
Soy 51 per acre than other biofuel crops.
0 4,000 8,000 12,000 16,000 20,000
Gallons D) a statistically meaningless amount more oil than
other crops.
Algae possesses many more benefits than other
plant-based biofuels. First of all, it produces 7 a truly
MATH

startling amount of oil.

Hydraulic Fracturing Industry Donations 8


to Candidates
Hydraulic forIndustry
Fracturing U.S. Senate
Donations to Candidates for U.S. Senate The author is considering adding a sentence here.
Which of the following sentences contains accurate
2014 $3,800,000 data based on the graph and would support the
assertion made in the previous sentence?
$900,000
A) For example, in 2014 the fracking industry
2012 $3,100,000 donated approximately four times as much
money to candidates running in states with
$1,100,000
fracking as it did to candidates in states where
2010 fracking does not occur.
$2,250,000
B) The fracking industry is responsible for securing
$1,200,000
several U.S. Senate seats in fracking-friendly
2008 $1,175,000
states due to their generous donations.

$560,000
C) The fracking industry is so forward-thinking
that it has increased donations in states both
2006 $880,000
with and without fracking in the last ten years
in order to secure future governmental favor.
$490,000
D) Since 2010, more and more states have
2004 States with Fracking instituted fracking due to the money available
$550,000
States without Fracking through donations from the fracking industry.
$400,000

Despite the increasing concerns that fracking raises,


the government has been slow to regulate the industry,
perhaps because of the economic support the industry
provides to political candidates. 8

CONTINUE
218  |  © TPR Education IP Holdings, LLC
QUESTIONS PRACTICE

Cilantro Aversion by Geographic Population


9

READING
Which of the following choices is best supported by
Middle East
the data provided in the graph?
Latin America
South Asia A) NO CHANGE
Africa B) a combined half of the entire global population.
Europe
East Asia C) most people in areas where cilantro is not

WRITING AND
LANGUAGE
0 6 11 17 22 28 native.
Percent of Population D) a majority of East Asians.
with Self-Reported Cilantro Aversion

It is no wonder that cilantro is not a key ingredient


in some cuisines, since it is disliked by 9 up to one-
fifth of certain populations.

MATH
Annual CO2 Emissions and Average 10
Global Temperatures, 1880–2020
500 14.9 Which choice offers an accurate interpretation of
14.6 the data in the graph?
Tons in Billions

Degrees Celsius

375 14.4 A) NO CHANGE


14.2
250 B) human activity will have resulted in a total of
14.0
125 13.7 450 billion tons of carbon dioxide emissions.
13.5 C) this figure will have climbed to 450 billion tons
0 13.3
1880 1900 1920 1940 1960 1980 2000 2020 a year.
D) global temperatures will have exceeded carbon
CO2
emissions.
Global Temperature

Since 1959 humans have emitted at least 350 billion


tons of carbon dioxide into the atmosphere annually. It
is expected that by 2020, 10 government regulations
will have resulted in a sharp decline in average global
temperatures.

CONTINUE
© TPR Education IP Holdings, LLC  |  219
SAT MANUAL

Number of Fracking Wells by State 11


READING

The writer wants to include information from


the graph that is consistent with the description
of the quantity of water needed for fracking
in the passage. Which choice most effectively
accomplishes this goal?
A) NO CHANGE
WRITING AND
LANGUAGE

Fracking Wells B) many of the states in the Northeast don’t have


0 to 1
1 to 100
any wells at all.
100 to 1,000
1,000 to 5,000 C) Texas has more wells than any other state.
5,000 to 100,000

D) some individual states contain up to 100,000


wells.
MATH

The fracking process uses an enormous quantity of


water: up to 3.5 million gallons are needed for just one
well, and 11 most of them are located in the lower 48
states.

CONTINUE
220  |  © TPR Education IP Holdings, LLC
QUESTIONS PRACTICE

As people worldwide are getting increasingly 12

READING
concerned about the environmental costs of traditional At this point, the writer wants to add specific
energy sources, 12 solar energy is becoming information that supports the main topic of the
paragraph.
increasingly expensive as compared to nuclear energy.

Cost per kwh by Energy Source,


Historic and Projected

WRITING AND
LANGUAGE
Energy
2000 2005 2010 2015 2020 2025
Source
Solar
30¢ 25¢ 20¢ 15¢ 10¢ 5¢
Cell PV
Nuclear 5¢ 8¢ 12¢ 17¢ 23¢ 30¢
Offshore
6¢ 5¢ 3¢ 3¢ 2¢ 2¢

MATH
Wind

Which choice most effectively completes the


sentence with relevant and accurate information
based on the graph above?
A) NO CHANGE
B) the costs of renewable energy sources, such as
solar and wind power, are dropping.
C) the cost of building nuclear power plants
continues to rise.
D) solar and wind energy will become so
inexpensive that they will eventually replace
nuclear power altogether.

Answers can be found on pages 727–728.

© TPR Education IP Holdings, LLC | 221


PUNCTUATION

www.CartoonStock.com

223
SAT MANUAL
READING

GOALS REVIEW
At the conclusion of this chapter, you will be able to accomplish the following:

• Apply the Basic Approach to questions dealing with punctuation


• Know where to use STOP, HALF-STOP, and GO punctuation
• Know the four situations in which to use a comma
WRITING AND

• Know the two situations in which to use an apostrophe


LANGUAGE

CONNECTING IDEAS, COMMAS, AND


APOSTROPHES
The list of punctuation rules that the SAT tests is relatively short. In this chap-
MATH

ter, we’ll look at punctuation in three main categories. The rules are dependable:
memorize them and apply them, and you won’t miss punctuation questions.

THE BASIC APPROACH


Punctuation questions usually don’t have questions, so remember your Basic
Approach:

1. ____________________________________________________________
Read to the end of the sentence

2. ____________________________________________________________
Look to the answers

POE
3. ____________________________________________________________

224  |  © TPR Education IP Holdings, LLC


PUNCTUATION

READING
CONNECTING IDEAS
Different types of punctuation are used to connect complete and incomplete ideas
to each other.

WRITING AND
• A complete idea can stand on its own, even if it’s part

LANGUAGE
of a longer sentence.
• An incomplete idea cannot stand on its own and must
be appropriately linked to other information to form a
complete sentence.

MATH
Complete Incomplete

i. T’Challa is the king of Wakanda r r


ii. Thor is searching for r r
iii. Has no super powers r r
iv. Because Black Widow closed the wormhole r r
v. He is a hero r r

STOP
STOP punctuation is used to connect two complete ideas.

• Period FANBOYS stands for


For, And, Nor, But, Or,
• Semicolon
Yet, So
• Comma + FANBOYS
• Question mark
• Exclamation point

© TPR Education IP Holdings, LLC | 225


SAT MANUAL

The rules of punctuation can seem 1. A) NO CHANGE


READING

1 confusing, different sources give B) confusing, different


sources,
different guidelines for the same situations.
C) confusing. Different
sources
D) confusing different
WRITING AND

sources
LANGUAGE

punctuation
What’s changing in the answer choices? ____________________________________

Does STOP punctuation appear in any answer choices?________________________


yes
MATH

Vertical Line Test


When you see STOP punctuation in any answer choices, use the Vertical Line
Test.

• Draw a vertical line at the place where the punctuation changes in the
sentence or in the answer choices.
• Identify each part of the sentence as a complete idea or an
incomplete idea.

What comes before the vertical line, a complete or an incomplete idea?

______________________________________________________________________

What comes after the vertical line, a complete or an incomplete idea?

______________________________________________________________________

Can you use STOP punctuation here? _____________________________________

226  |  © TPR Education IP Holdings, LLC


PUNCTUATION

READING
HALF-STOP
HALF-STOP punctuation is used after a complete idea, and before either a
complete or an incomplete idea.

• Colon

WRITING AND
LANGUAGE
• Long dash

Looking at the history of punctuation 2. A) NO CHANGE HALF-STOP punctuation


in the answer choices is
can be 2 helpful, the number, of B) helpful the number

MATH
another indication to use
punctuation marks commonly used today is C) helpful, the number the Vertical Line Test!
much smaller than in the past. D) helpful: the number

What’s changing in the answer choices? ____________________________________

Does STOP or HALF-STOP punctuation appear in any answer choices?

_______________________________________________________________________

Does the Vertical Line Test apply? ________________________________________

© TPR Education IP Holdings, LLC | 227


SAT MANUAL
READING

GO
GO punctuation is used any place STOP punctuation is not used:
complete + incomplete, incomplete + complete, or incomplete + incomplete.

• Comma
WRITING AND
LANGUAGE

• No punctuation

On the SAT, a period and Before the invention of the printing 3. A) NO CHANGE
MATH

a semicolon are used B) the 1400s—books


exactly the same way. press in 3 the 1400s, books were read only
If the only difference by the educated and the wealthy. C) the 1400s; books
between two answer D) the 1400s. Books
choices is a period versus
a semicolon, they’re both
wrong.

What’s changing in the answer choices? ____________________________________

Does STOP or HALF-STOP punctuation appear in any answer choices?

______________________________________________________________________

Does the Vertical Line Test apply? ________________________________________

When you have two Only very wealthy people owned books 4. A) NO CHANGE
incomplete ideas, you because the 4 cost, of the books was so B) cost; of
usually don’t put any
punctuation between great. C) cost of
them. D) cost: of

What’s changing in the answer choices? ____________________________________

Does STOP or HALF-STOP punctuation appear in any answer choices?

______________________________________________________________________

Does the Vertical Line Test apply? ________________________________________

228 | © TPR Education IP Holdings, LLC


PUNCTUATION

These volumes had to be transcribed 5. A) NO CHANGE Whenever you see

READING
FANBOYS underlined,
by 5 hand and they were often also B) hand, and
draw the vertical line
illustrated by hand. C) hand, through it.
D) hand, and,

WRITING AND
LANGUAGE
What’s changing in the answer choices? ____________________________________

Does STOP or HALF-STOP punctuation appear in any answer choices?

______________________________________________________________________

MATH
Does the Vertical Line Test apply? ________________________________________

COMMAS
There are four reasons to use a comma on the SAT. STOP (comma + FANBOYS)
and GO punctuation are two of them.

In a List of Three or More


6. A) NO CHANGE When there’s a list in
Most books prior to the 1400s were a sentence, the “, and”
written in scriptura continua, which had B) capitalization, doesn’t count as STOP
no spaces, no 6 capitalization; and, no C) capitalization, and punctuation.

punctuation marks. D) capitalization, and,

What’s changing in the answer choices? ____________________________________

Is there a list in the sentence? _____________________________________________

© TPR Education IP Holdings, LLC  |  229


SAT MANUAL
READING

Before and After Unnecessary Information


Parentheses or dashes This is not to say that there was no 7. A) NO CHANGE
can also be used to
punctuation before this time, however. B) marks (called dicolons
separate unnecessary
information from the rest As early as the fifth century BCE, the Greeks C) marks, called dicolons,
of the sentence. sometimes used punctuation 7 marks, D) marks called dicolons
WRITING AND
LANGUAGE

called dicolons and tricolons, to help with


the oral transmission of their texts.

What’s changing in the answer choices? ____________________________________


MATH

Which comma rule applies here? __________________________________________

The 8 number of vertically-arranged 8. A) NO CHANGE


dots told speakers the appropriate length of B) number, of vertically-
arranged dots, told
pauses.
speakers
C) number, of vertically-
arranged dots told
speakers
D) number of vertically-
arranged dots, told
speakers

What’s changing in the answer choices? ____________________________________

Which comma rule applies here? __________________________________________

Do You Really Need That?


i. Although rules exist for using commas, anyone __ who has mastered
grammar __ can use a comma in an unexpected way for extra effect.

ii. The Oxford comma __ also known as the serial comma __ is one of
the most highly contested concepts in English punctuation.

iii. The Englishman __ Frederick Howard Collins __ was the first to


include the Oxford comma in a style manual.

230  |  © TPR Education IP Holdings, LLC


PUNCTUATION

READING
iv. The primary argument against the Oxford comma __ which is that
it’s simply not always necessary __ dates back to the days when type-
setting was done by hand and any extra characters made more work
for typesetters.

v. The style manuals __ that advocate for the use of the Oxford comma

WRITING AND
__ include the MLA Handbook and The Chicago Manual of Style.

LANGUAGE
Don’t Comma Round Here No More
Some other languages, such as early 9. A) NO CHANGE
Chinese and Mayan, worked in pictograms or B) syllables, rather than

MATH
individual letters and
9 syllables rather than individual letters,
thus, had no need, for
and thus, had no need for punctuation, at all. punctuation, at all.
C) syllables rather than,
individual letters and,
thus, had no need for
punctuation, at all.
D) syllables rather than
individual letters and
thus had no need for
punctuation at all.

What’s changing in the answer choices? ____________________________________

Do any of the comma rules apply? _________________________________________

There are four reasons to use commas on the SAT:


1. STOP punctuation with one of the FANBOYS
2. GO punctuation
3. In a list of three or more
4. Before and after unnecessary information
If you can’t cite one of these reasons, don’t use a comma!

© TPR Education IP Holdings, LLC | 231


SAT MANUAL
READING

APOSTROPHES
Apostrophes are used for possession and contraction. As with commas, if you can’t
cite a reason to use an apostrophe, don’t use one.

With the introduction of the printing 10. A) NO CHANGE


press, authors and printers began to B) language’s rules’
WRITING AND
LANGUAGE

standardize the 10 language’s rules C) languages’ rule’s


regarding punctuation. D) languages rules

What’s changing in the answer choices? ____________________________________


MATH

Does anything belong to language? ______ What about rules? _________________

Nouns (not including pronouns) with


apostrophes are possessive.

Rewrite the following phrases using apostrophes.


essay's
i. The theme of the essay Ô the ________________________________ theme

ii. octopus's
The garden of the octopus Ô the ____________________________ garden
authors'
iii. The findings of the authors Ô the __________________________ findings
people's
iv. The favorite writer of the people Ô the _________________favorite writer

232 | © TPR Education IP Holdings, LLC


PUNCTUATION

These indicators eventually became the 11. A) NO CHANGE If you’re unsure whether

READING
B) its because of these a pronoun needs an
punctuation marks that we use today, and apostrophe, expand it
marks that writers can
11 it’s because of these marks that writers out! If the verb makes
share there
can share their words with such a vast group sense, use an
C) it’s because of these apostrophe.
of readers. marks that writers can
share they’re

WRITING AND
LANGUAGE
D) its because of these
marks that writers can
share they’re

What’s changing in the answer choices? ____________________________________

MATH
Is either of the pronouns a contraction? ____________________________________

Pronouns with apostrophes are contractions. Possessive


pronouns never have apostrophes.

How Does That Sound?


Your ear won’t help you with apostrophes! Beware of the following commonly
confused words.

Possessive Pronouns Pronoun Contractions Still confused about


• Their • They’re (They are) pronouns? Go to
• Its • It’s (It is/It has) pages 290–293
for a review!
• Your • You’re (You are)
• Whose • Who’s (Who is/Who has)

© TPR Education IP Holdings, LLC | 233


SAT MANUAL
READING

PUNCTUATION DRILL
Time: 6 minutes

Questions 1–11 are based on the following passage.


WRITING AND
LANGUAGE

A Period History, Comma Get Some Knowledge 1


Punctuation had gained traction throughout the A) NO CHANGE
Middle 1 Ages, it was mainly elocutionary (for oral B) Ages, but it
speech) rather than syntactical (for written speech). C) Ages but it
Ben Jonson’s English 2 Grammar, written in 1617 D) Ages. But it
but published in 1640, was among the first works to
MATH

propose standard syntactical punctuation. The idea 2


caught on 3 quickly, because, readers appreciated A) NO CHANGE
the new clarification that came with it. By the 1800s, B) Grammar written in 1617 but published in 1640
enthusiasm for punctuation had grown, and some style C) Grammar, written in 1617, but published in
4 manual’s guideline’s for commas included usages 1640,
that we would find distracting today. D) Grammar written in 1617 but published in 1640,

3
A) NO CHANGE
B) quickly; because
C) quickly. Because,
D) quickly, because

4
A) NO CHANGE
B) manuals guidelines
C) manuals’ guidelines’
D) manuals’ guidelines

234 | © TPR Education IP Holdings, LLC


PUNCTUATION

Punctuation as we use it today was part of a 5

READING
movement that began in Britain in 1906, when the A) NO CHANGE
Fowler brothers, Henry Watson and Francis George, B) its
published 5 they’re guide to The King’s English, which C) their
 
advised light punctuation. By that point, commas, semi- D) there
colons, 6 apostrophes, and colons had become so

WRITING AND
LANGUAGE
arcane that many of the newcomers to literacy, such as
6
those in the growing middle 7 class, were intimidated
A) NO CHANGE
and confused. It is because of the Fowlers that we now
B) apostrophes: and colons
use punctuation only when we have a good reason to do
C) apostrophes, colons
so, in a 8 limited and specific number, of instances.
D) apostrophes; and colons
We can thank the Fowler brothers for the fact that

MATH
the period is much better known 9 today, than the
semicolon is, and for the near death of the subjunctive 7
mood. A) NO CHANGE
B) class were intimidated
C) class—were intimidated
D) class, were intimidated,

8
A) NO CHANGE
B) limited, and specific number:
C) limited and specific number
D) limited—and specific number

9
A) NO CHANGE
B) today than the semicolon is
C) today than the semicolon is,
D) today than the semicolon is;

© TPR Education IP Holdings, LLC | 235


SAT MANUAL

American English has followed much the same 10


READING

course as British 10 English. Though the strictest A) NO CHANGE


grammar treatises tend to come from American sources. B) English, the
Such grammatical precision is nowhere more on display C) English. Whereas the
than in standardized tests for college admissions: each D) English, though the
of these tests measures 11 it’s test takers’ abilities to
WRITING AND
LANGUAGE

follow such rules.


11
A) NO CHANGE
B) they’re
C) their
D) its
MATH

236 | © TPR Education IP Holdings, LLC


PUNCTUATION

Summary
• You should use the Vertical Line Test when
you see ______________________________
in the answer choices.

• What kind of punctuation can only come


between two complete ideas?

_____________________________________

_____________________________________

_____________________________________

_____________________________________

_____________________________________

• What kind of punctuation can never come


between two complete ideas?

_____________________________________

_____________________________________

• HALF-STOP punctuation must come


__________ a complete idea.

• What are the four reasons to use a comma


on the SAT?

_____________________________________

_____________________________________

_____________________________________

_____________________________________

• What are the two reasons to use an


apostrophe on the SAT?

_____________________________________

_____________________________________

• What should you do if you can’t cite a


reason to use a comma or an apostrophe?

_____________________________________

• I have accomplished _________ of the 4


goals stated at the beginning of this chapter.

© TPR Education IP Holdings, LLC | 237


PUNCTUATION
PRACTICE

239
SAT MANUAL
READING

STOP, HALF-STOP, AND GO

Included in these codes was the Sixth 1


1 Amendment: the right to free counsel for indigent A) NO CHANGE
WRITING AND
LANGUAGE

defendants. B) Amendment
C) Amendment;
D) Amendment, and
MATH

A prominent U.S. phone company attempted 2


to market its services in Saudi Arabia through an A) NO CHANGE
advertisement that featured an executive speaking on B) the viewer, because
the phone with his feet propped up on the desk in front C) the viewer because
of him, exposing the soles of his shoes to 2 the viewer, D) the viewer; because
because, the soles of shoes are viewed as unclean in Arab
culture, the ad caused the same sort of offence that an ad
in the United States of someone flipping the bird to the
viewer might cause.

Accurately representing the intended meaning of 3


a word can be very difficult when dealing with what A) NO CHANGE
translators call 3 “UN-speak”; language that can be B) “UN-speak” language
ambiguous and technocratic. C) “UN-speak.” Language
D) “UN-speak”—language

Her specialty within primatology is 4


4 psychology; she is primarily concerned with A) NO CHANGE
studying the behavior of specific monkey groups to B) psychology, she
better understand their psychological processes and C) psychology she
decision-making. D) psychology;

CONTINUE
240  |  © TPR Education IP Holdings, LLC
PUNCTUATION PRACTICE

5
The fairness measurements 5 included; funding

READING
A) NO CHANGE
level, funding distribution, effort, and coverage.
B) included:
C) included
D) including

WRITING AND
LANGUAGE
It is no coincidence that those elected to the 6
House serve the shortest 6 term; a two-year term, as A) NO CHANGE
opposed to six years for senators, four years for the B) term: a
president, and a life term for Supreme Court justices. C) term, a
D) term. A

MATH
The theory pertains to any use of shared 7
7 resources; the office refrigerator, the highway, A) NO CHANGE
or the internet. B) resources. The
C) resources: the
D) resources the

8 Symptoms include: fever, headache, muscle 8


pain, weakness, vomiting, diarrhea, and hemorrhage. A) NO CHANGE
B) Symptoms include,
C) Symptoms include;
D) Symptoms include

Only half of the water contaminated by fracking 9


is 9 recovered; the rest is lost and can filter into local A) NO CHANGE
water sources. B) recovered—the rest,
C) recovered the rest
D) recovered, the rest

CONTINUE
© TPR Education IP Holdings, LLC  |  241
SAT MANUAL

Recently, engineers built the first 3D bioprinting 10


READING

production 10 system, it is hoped that someday it will A) NO CHANGE


have the capacity to print organs and body parts from B) system; it is hoped
layers of living tissue. C) system, it is hoped:
D) system, it is, hoped
WRITING AND
LANGUAGE

There are no tests that can be 11 run; there are no 11


control and test planets waiting for experimentation in a A) NO CHANGE
laboratory closet. B) run;
MATH

C) run; and there are


D) run, there are

Primatologists want to learn why 12 aggression in 12


monkey groups is so low? A) NO CHANGE
B) aggression in monkey groups, is so low?
C) aggression in monkey groups is so low.
D) aggression, in monkey groups, is so low.

Was Henry simply trying to find a way to free 13


himself for marriage to the woman who would become A) NO CHANGE
his second 13 wife, Anne Boleyn? B) wife; Anne
C) wife. Anne
D) wife Anne

Their second point—perhaps the most convincing 14


14 one, is that it is hypocritical of the United States A) NO CHANGE
to limit trade with Cuba based on democratization and B) one, is that:
human rights failings but to trade freely with other C) one—is that
countries that have worse human rights records. D) one—is that,

CONTINUE
242  |  © TPR Education IP Holdings, LLC
PUNCTUATION PRACTICE

To compensate for the loss of cow dung as fuel, 15

READING
which would be a problem if cows were raised for A) NO CHANGE
15 slaughter, India would require 43 million additional B) slaughter—
tons of coal per year. C) slaughter
D) slaughter;

WRITING AND
LANGUAGE
Of course, many scientists say that the changes 16
in weather patterns we are observing now are that A) NO CHANGE
16 proof—that climate change is happening. B) proof; that
C) proof that—

MATH
D) proof, and that

Everything known about global warming is 17


17 based on recorded, observational, data and A) NO CHANGE
projections. B) based on: recorded observational data
C) based on; recorded, observational data
D) based on recorded observational data

Only about three decades 18 old; this technology 18


has exploded in the last third of its life. A) NO CHANGE
B) old; however this
C) old, this
D) old. This

These periods of extreme cold were the result of a 19


break in the 19 Northern Polar Vortex, the polar vortex A) NO CHANGE
is often inaccurately blamed for cold spells caused by B) Northern, Polar Vortex
other factors. C) Northern Polar Vortex: but
D) Northern Polar Vortex, but

CONTINUE
© TPR Education IP Holdings, LLC  |  243
SAT MANUAL

Many bugs also carry aldehyde chemicals on their 20


READING

20 skin. It is this chemical that is responsible for the A) NO CHANGE


well-known and universally loathed aroma of the stink B) skin, it is this
bug. C) skin, it is this:
D) skin. It is this—
WRITING AND
LANGUAGE

Hydraulic 21 fracturing, commonly known as 21


“fracking”) is accomplished by pumping fracturing fluid A) NO CHANGE
into a drilled well in order to increase pressure at a B) fracturing—commonly
specific depth within the well. C) fracturing: commonly
MATH

D) fracturing (commonly

Evolutionary theory is the belief in changing traits 22


22 in, organisms over time due to natural selection. A) NO CHANGE
B) in; organisms
C) in organisms
D) in. Organisms

At the time of that 23 study. This area of the 23


country was experiencing a considerable economic A) NO CHANGE
decline. B) study, this
C) study—this
D) study; this

Solar water heaters are water-filled glass panels that 24


absorb the heat of the sun; they 24 provide, hot water A) NO CHANGE
without the use of gas or electricity. B) provide; hot water
C) provide: hot water
D) provide hot water

CONTINUE
244  |  © TPR Education IP Holdings, LLC
PUNCTUATION PRACTICE

READING
COMMAS
For each answer you choose that contains a comma, indicate which comma rule applies. If no rule applies, choose
the answer with no commas.

COMMA RULES

WRITING AND
LANGUAGE
1. STOP punctuation with one of the FANBOYS
2. GO punctuation
3. In a list of three or more
4. Before and after unnecessary information

MATH
Many advising dietitians work for food service 1
providers for institutions, 1 such as prisons, schools, A) NO CHANGE
hospitals, and nursing facilities. B) such as: prisons, schools, hospitals and nursing
facilities.
C) such as, prisons, schools, hospitals and nursing
facilities.
D) such as prisons, schools, hospitals and nursing,
facilities

Rule: _________

Service animals have been observed to be very 2


2 helpful for example, when working with individuals A) NO CHANGE
diagnosed with intellectual disorders such as autism. B) helpful, for example, when working with
individuals diagnosed with intellectual
disorders, such as,
C) helpful, for example, when working with
individuals diagnosed with intellectual
disorders such as
D) helpful, for example when working with
individuals diagnosed with intellectual
disorders, such as,

Rule: _________

CONTINUE
© TPR Education IP Holdings, LLC | 245
SAT MANUAL

The agreement was 3 provisional, and could only 3


READING

be finalized, once the German government had been A) NO CHANGE


established and then agreed to the terms. B) provisional and could only be finalized,
C) provisional, and could only be finalized
D) provisional and could only be finalized
WRITING AND

Rule: _________
LANGUAGE

This 4 was, of course, the American Revolution, 4


fought to escape an oppressive monarchy and establish a A) NO CHANGE
free democracy in which voices were given equal value. B) was of course
C) was, of course
MATH

D) was of course,

Rule: _________

Fracking has 5 immense, but economic benefits. 5


A) NO CHANGE
B) immense, economic
C) immense economic
D) immense, and economic

Rule: _________

Other powers included controlling the bankruptcy 6


process, coining and regulating 6 money, and, A) NO CHANGE
punishing counterfeiting and piracy. B) money, and punishing counterfeiting and
piracy.
C) money and punishing, counterfeiting, and
piracy.
D) money, and punishing, counterfeiting and
piracy.

Rule: _________

CONTINUE
246  |  © TPR Education IP Holdings, LLC
PUNCTUATION PRACTICE

The 7 boys, evaluated in the study, showed signs 7

READING
of obesity in terms of body mass index and triceps skin- A) NO CHANGE
fold measurements. B) boys—evaluated in the study—
C) boys evaluated in the study
D) boys evaluated, in the study,

WRITING AND
Rule: _________

LANGUAGE
There are great differences of opinion between 8
proponents of the theory of evolution and proponents A) NO CHANGE
of the theory of 8 creationism, the two theories have B) creationism, but
more common ground than is commonly recognized. C) creationism, but,

MATH
D) creationism

Rule: _________

In 9 1890, the Bureau of the Census, declared 9


the end of the land frontier in America. A) NO CHANGE
B) 1890, the Bureau, of the Census declared,
C) 1890 the Bureau of the Census, declared
D) 1890, the Bureau of the Census declared

Rule: _________

States with progressive funding scored higher 10


10 overall, in this category, than did regressive or flat- A) NO CHANGE
rate states. B) overall, in this category
C) overall in this category,
D) overall in this category

Rule: _________

She began to feel 11 better, she was able to return 11


to school the following day. A) NO CHANGE
B) better, and
C) better, but
D) better and

Rule: _________
CONTINUE
© TPR Education IP Holdings, LLC  |  247
SAT MANUAL

An epidemiologist studies the 12 patterns, causes, 12


READING

and effects, of health and disease conditions in specific A) NO CHANGE


populations. B) patterns, causes, and effects, of health,
C) patterns and causes and effects of health
D) patterns, causes, and effects of health
WRITING AND

Rule: _________
LANGUAGE

That is nothing compared to what it would take to 13


run an entire commercial 13 building, let alone a city A) NO CHANGE
or state, on solar energy. B) building, let alone a city, or state
C) building let alone a city, or state
MATH

D) building, let alone a city, or state,

Rule: _________

Even the term “selection” 14 implies, informed, if 14


not intelligent, choice. A) NO CHANGE
B) implies informed, if not intelligent choice.
C) implies informed, if not, intelligent choice.
D) implies informed, if not intelligent, choice.

Rule: _________

Based on these findings, the 15 scientists, 15


conducting the study, determined that the children A) NO CHANGE
demonstrated signs of both under-nutrition and over- B) scientists conducting the study
nutrition concurrently. C) scientists, conducting the study
D) scientists conducting the study,

Rule: _________

Mental 16 well-being—one of the most important 16


aspects of our health—is often overlooked by patients A) NO CHANGE
and doctors alike. B) well-being one of the most important aspects of
our health
C) well-being—one of the most important
aspects—of our health,
D) well-being, one of the most important aspects of
our health

Rule: _________
CONTINUE
248  |  © TPR Education IP Holdings, LLC
PUNCTUATION PRACTICE

READING
APOSTROPHES

Instantaneous interpretation is a demanding task. 1


1 Its even more pressure-filled when the success of a A) NO CHANGE
peace agreement or international settlements is at stake. B) Its’

WRITING AND
LANGUAGE
C) They’re
D) It’s

Pharmaceuticals can be designed in a variety of 2

MATH
ways: 2 their able to pinpoint specific areas of the A) NO CHANGE
body and can time-release differing levels of dosages in a B) it’s
single pill. C) there
D) they’re

Due to the strong support fracking has received 3


based on its economic benefits, 3 its unlikely to end A) NO CHANGE
any time soon. B) they’re
C) it’s
D) their

Polar vortices are repetitive large-scale cyclones 4


that circle 4 its respective geographical poles. A) NO CHANGE
B) their
C) they’re
D) it’s

CONTINUE
© TPR Education IP Holdings, LLC  |  249
SAT MANUAL

5 There are no tests that can be run; there are no 5


READING

control and test planets waiting for experimentation in a A) NO CHANGE


laboratory closet. B) Their
C) They’re
D) They
WRITING AND
LANGUAGE

3D printers have been used to create everything 6


from clothing to automobile parts to 6 childrens toys. A) NO CHANGE
B) childrens’ toy’s.
C) childrens’ toys.
MATH

D) children’s toys.

Each set of twins was surveyed to elicit both 7


7 siblings attitude’s towards cilantro. A) NO CHANGE
B) siblings attitudes
C) siblings’ attitudes
D) sibling’s attitude’s

There was stronger evidence of over-nutrition based 8


on the 8 participant’s diet’s. A) NO CHANGE
B) participants’ diets.
C) participant’s diets’.
D) participants diets.

Scientists define a 9 disease’s reproductive rate 9


as the average number of people who are infected by a A) NO CHANGE
single person. B) diseases’
C) diseases
D) disease

CONTINUE
250  |  © TPR Education IP Holdings, LLC
PUNCTUATION PRACTICE

Without equal numbers of healthy young people 10

READING
paying into an insurance program, the 10 costs of older A) NO CHANGE
peoples health care cannot be met. B) costs’ of older peoples
C) costs of older people’s
D) costs of older peoples’

WRITING AND
LANGUAGE
The size of a 11 countries population is affected 11
by the quality of prenatal and neonatal medical care A) NO CHANGE
available. B) countries’ population
C) countrys population

MATH
D) country’s population

In 2003, one of Meucci’s drawings of a prototype 12


telettrofono was published by the Italian Postal and A) NO CHANGE
Telegraph Service on a stamp commemorating his B) it’s
achievement; 12 its still in use today. C) their
D) they’re

Epidemiologists who work through observation 13


must be descriptive in 13 they’re approach to disease A) NO CHANGE
and health. B) their
C) there
D) its

The only way global warming can be scientifically 14


proven is for it to happen. Scientists say that the changes A) NO CHANGE
we are observing constitute that proof—that 14 its’ B) its
happening right now. C) it’s
D) there

CONTINUE
© TPR Education IP Holdings, LLC  |  251
SAT MANUAL

A polar vortex affects other areas of the globe 15


READING

when there is a break in the polar jet stream: in 1985, A) NO CHANGE


the United States suffered one of 15 its worst weather B) it’s
events due to a break in the arctic polar vortex C) their
D) its’
WRITING AND
LANGUAGE

Some people give money to 16 charities because 16


their invested in a particular cause. A) NO CHANGE
B) charity’s because they’re
C) charity’s because their
D) charities because they’re
MATH

Public defenders save the legal system and the 17


American public a great deal of time and money: A) NO CHANGE
17 there representation of the less fortunate helps B) their
prevent appeals due to inadequate counsel. C) they’re
D) that is

From 1939 to 1999, the Democratic Party 18


experienced a six-point decrease in 18 its membership; A) NO CHANGE
during the same period, Republican Party members B) it’s
decreased by just three points. C) their
D) there

19
The National 19 Womens Party members, led
A) NO CHANGE
by Alice Paul, were arrested while picketing outside the
B) Women’s Party members,
White House for the right to vote.
C) Womens’ Party member’s,
D) Womens Party members’,

CONTINUE
252  |  © TPR Education IP Holdings, LLC
PUNCTUATION PRACTICE

A 20 runners’ high, caused by endorphin’s the 20

READING
body produces in response to exercise, produces a A) NO CHANGE
temporary state of euphoria. B) runners high, caused by endorphins’
C) runners high, caused by endorphins
D) runner’s high, caused by endorphins

WRITING AND
LANGUAGE
It should come as no surprise that countries 21
differ from one another; it could even be said that A) NO CHANGE
21 difference’s are why countries exist. B) difference’s are why countries’
C) differences are why countries

MATH
D) differences are why country’s

22 Dog’s can sense spikes and dips in the blood 22


sugar of a person with diabetes and the onset of a seizure A) NO CHANGE
in a person with epilepsy. B) Dogs can sense spikes
C) Dog’s can sense spike’s
D) Dogs’ can sense spikes

23 Turtles’ eggs’ are buried deep in the sand on 23


beaches and then deserted. A) NO CHANGE
B) Turtles egg’s
C) Turtles’ eggs
D) Turtles eggs

Based on these 24 findings, the scientists


24
conducting the study determined that the children
A) NO CHANGE
demonstrated signs of both under-nutrition and over-
B) finding’s, the scientists
nutrition concurrently.
C) finding’s, the scientists’
D) findings, the scientist’s

Answers can be found on page 729.

© TPR Education IP Holdings, LLC | 253


WORDS
PART II

A synonym is a word you use when you can’t spell the


other one.
—Baltasar Gracián

255
SAT MANUAL
READING

GOALS REVIEW
At the conclusion of this chapter, you will be able to accomplish the following:

• Know what to look for when the number of words changes in the
answer choices and the most concise one doesn’t work
• Know how to make lists and comparisons consistent
WRITING AND

• Know how to choose the most precise words when dealing with
LANGUAGE

idioms and frequently confused words

COMPLETE SENTENCES verbs


In Words Part I, you saw how __________________,
transitions __________________, and
MATH

__________________
pronouns must be __________________
consistent with the sentences in which
they appear and must provide the most __________________
precise meaning.

Sometimes, changing those words in a sentence can make it an incomplete idea or


a run-on sentence.

Game theory is the study of 1. A) NO CHANGE


modern decision-making; | 1 it B) examining
examines how people’s decisions are C) it having examined
influenced by factors such as conflict, D) to examine
cooperation, loss, and gain.

verb/wordchoisce
What’s changing in the answer choices? ____________________________________

How does adding the -ing ending to the verb change the second part of the sen-
tence?

makes it incomplete
_______________________________________________________________________

How does removing the word it change the second part of the sentence?
makes it an incomplete idea
_______________________________________________________________________

256  |  © TPR Education IP Holdings, LLC


WORDS PART II

The origin of modern game theory 2. A) NO CHANGE

READING
can be traced back to the Hungarian- B) he was writing
American mathematician John von C) who wrote
Neumann, 2 he wrote the book D) whose writing of
Theory of Games and Economic Behavior
in 1944.

WRITING AND
LANGUAGE
writting/wrote; pronounce: he/who/whose
What’s changing in the answer choices? ____________________________________

How does changing the pronoun change the second part of the sentence?

MATH
_______________________________________________________________________
removing "he" makes it incomplete

The book described human 3. A) NO CHANGE


behavior as a pattern of moves and B) countermoves,

3 countermoves. Because the goal of C) countermoves; because

such behavior is the greatest individual D) countermoves:

benefit.

punctuation
What’s changing in the answer choices? ____________________________________

Does the Vertical Line Test apply? _________________________________________

What does adding the word because do to the second part of the sentence?

makes it incomplete
_______________________________________________________________________

© TPR Education IP Holdings, LLC  |  257


SAT MANUAL
READING

IDIOMS, HOMOPHONES, AND SLANG


Some problems in Writing and Language ask you to choose words for which there
are no rules: these are called idioms. You can usually spot them because preposi-
tions are changing in the answers.

Not sure what a Pre-Victorian British novels are 4. A) NO CHANGE


WRITING AND
LANGUAGE

preposition is? Go to not the first place most people would B) for mastery of
page 288 for a review!
look 4 to mastery of game theory. C) to mastery in
D) for mastery with

What’s changing in the answer choices? ____________________________________


prepositions/ words
MATH

With idioms, use POE to get rid of the answers you know for sure are wrong. If
you still have more than one answer left, make your best guess.

Jane Austen, who was born 125 5. A) NO CHANGE


years 5 earlier then von Neumann B) sooner then
was, is considered by many a master of C) more early than
game theory. D) earlier than

early/earlier/sooner,..
What’s changing in the answer choices? ____________________________________
than = comparrison then=time
What’s the difference between then and than? _______________________________

Do you know the difference between affect and effect? What about cite, site, and
sight? Principle and principal? Just as with idioms, do your best with POE when
faced with homophones, and move on!

258  |  © TPR Education IP Holdings, LLC


WORDS PART II

Just as economists use game theory 6. A) NO CHANGE See question 5 on

READING
to describe modern consumer behavior, B) work page 172 for the editor
version of this question.
game theory can also be applied to the C) finesse
ways in which Austen’s characters D) toy with
6 engage in the social rituals of
courtship and marriage.

WRITING AND
LANGUAGE
vocabulary change
What’s changing in the answer choices? ____________________________________

Which choices are slangy? ________________________________________________

MATH
You saw questions that asked about style and tone in the Questions chapter; these
problems can also appear without a question.

PARALLELISM
When there’s a list or a comparison in a Writing and Language passage, the things
being listed or compared all need to match.

In Pride and Prejudice, Mrs. 7. A) NO CHANGE


Bennet schemes to catch the handsome, B) and wealthy Mr. Bingley
amiable, 7 and also wealthy Mr. C) Mr. Bingley, who also has
lots of money
Bingley for her eldest daughter Jane.
D) and additionally rich
Mr. Bingley

Bingley's desc
What’s changing in the answer choices? ____________________________________

What is the list in the sentence? ___________________________________________

© TPR Education IP Holdings, LLC  |  259


SAT MANUAL

The strategies Mrs. Bennet uses to 8. A) NO CHANGE


READING

find husbands for her five daughters are B) the game of chess.
similar to 8 chess. C) those used in chess.
D) how she plays chess.

how chess is described


WRITING AND

What’s changing in the answer choices? ____________________________________


LANGUAGE

What two things are being compared? _____________________________________


strategies Mrs bennet uses and strategies of
chess
MATH

Modern game theory usually treats 9. A) NO CHANGE


players as 9 an equally informed B) an equally informative
opponent. opponent.
C) equally informed
opponents.
D) an equally informed
opponents.

opponent vs opponent
What’s changing in the answer choices? ____________________________________

What else in the sentence does the underlined portion need to be consistent with?

_______________________________________________________________________
players

260  |  © TPR Education IP Holdings, LLC


WORDS PART II

READING
MODIFIERS
When the order of words in the answers changes, look for a misplaced modifier—
either a single word or a descriptive phrase. A modifier should be placed as close as
possible to the thing it describes.

Often unaware of the ways in 10. A) NO CHANGE

WRITING AND
LANGUAGE
which they are being manipulated, B) Jane Austen’s characters
10 the hypothetical players and and their situations are
truer to life than the
situations typically discussed in game hypothetical ones typically
theory are less real than Jane Austen’s discussed in game theory.
characters. C) game theory typically
discusses more

MATH
hypothetical players and
situations than the ones
Jane Austen does, which
are truer to life.
D) Jane Austen’s books deal
with truer to life characters
and situations than the
hypothetical ones that
game theory typically deals
with.

What’s changing in the answer choices? ____________________________________

What does the modifying phrase in the non-underlined portion of the sentence
describe?

_______________________________________________________________________

© TPR Education IP Holdings, LLC  |  261


SAT MANUAL

Summary
• Adding or removing which types of words can
make a sentence incomplete or a run-on?

________________________________________

________________________________________

________________________________________

• When prepositions or homophones change in


the answer choices, __________________ and
__________________.

• If a sentence contains a list or a compari-


son, the things listed or compared must be
__________________ with each other.

• When the order of words changes in the


answers, look for _________________________.

• I have accomplished ___________ of the 3


goals stated at the beginning of this chapter.

262 | © TPR Education IP Holdings, LLC


WORDS
PART II
PRACTICE

263
SAT MANUAL
READING

COMPLETE SENTENCES
As consultants, anthropologists provide marketers 1
with valuable 1 feedback, this includes insights into A) NO CHANGE
consumer behavior and strategies to affect that behavior B) feedback. Including
to increase profit. C) feedback, it includes
WRITING AND
LANGUAGE

D) feedback that includes

It pays to know your 2 customers, and also to 2


know what your customers know. A) NO CHANGE
MATH

B) customers, but also to know


C) customers and also
D) customers, you know

The vast majority of patients are children, 3 who 3


see a speech therapist at school at least once a week. A) NO CHANGE
B) they see
C) they are seen by
D) DELETE the underlined portion.

The thought was that if Cuba, 4 it was a 4


valuable asset to the Soviet Union during the Cold A) NO CHANGE
War, suffered enough financially from the embargo, the B) it might be
Cuban government would eventually seek a stronger C) that country was
relationship with the West. D) which was

Although we may not know who first uttered 5


the famous line, 5 understandable that it was A) NO CHANGE
misattributed to Marie Antoinette. B) is understandable that
C) it is understandable that
D) that

CONTINUE
264  |  © TPR Education IP Holdings, LLC
WORDS PART II PRACTICE

The Pomeranian has a reputation as a very yippy 6

READING
dog 6 and they are known to require diligent training. A) NO CHANGE
B) that requires
C) and it requires
D) they require

WRITING AND
LANGUAGE
The important point for evolutionists 7 despite 7
wings leading to an evolutionary benefit for bats, but A) NO CHANGE
that wings were not a predetermined necessity. B) that wings led to an evolutionary benefit for bats
C) is not that wings in bats led to an evolutionary
advantage
D) not being wings leading to an evolutionary

MATH
benefit for bats,

The gene encodes a receptor that is specifically 8


sensitive to 8 aldehydes, which are chemicals in A) NO CHANGE
cilantro that affect how the herb tastes. B) aldehydes;
C) aldehydes; these
D) aldehydes and these are

It makes more financial sense for parents to invest 9


first in their own retirement 9 plans, college students A) NO CHANGE
are eligible for low-interest loans. B) plans, as
C) plans. As
D) plans, however,

The fans suffer just as much as the players 10 a 10


team is unfairly penalized by a referee. A) NO CHANGE
B) although a team is
C) when a team being
D) when a team is

CONTINUE
© TPR Education IP Holdings, LLC  |  265
SAT MANUAL

When you think of a private investigator, you 11


READING

might picture someone sitting in a car trying to be as A) NO CHANGE


inconspicuous as possible 11 he waits for a chance to B) while waiting for
snap incriminating photos. C) as awaiting
D) in which awaiting
WRITING AND
LANGUAGE

There has been increased interest in diet and 12


nutritional food choices over the last few decades. A) NO CHANGE
12 Specific interest being how they interact with B) Specific interest is
weight-related diseases. C) Specific interest being
MATH

D) Of specific interest is how

13 A few days since she has seen this particular 13


group, made up of one male and six females. A) NO CHANGE
B) It has been a few days
C) Several days
D) Having been a few days

Biomedical engineers build physical items for 14


the health and rehabilitation industry, 14 such as A) NO CHANGE
biocompatible prosthetics, which are prosthetics built to B) examples of these are biocompatible prosthetics,
work with the biology of the human body. C) for example, consider the biocompatible
prosthetic,
D) biocompatible prosthetics being an example,

The nonviolent protest ran smoothly for months, 15


15 the police arrested over 200 women for blocking a A) NO CHANGE
public sidewalk in July 1917. B) the police that arrested over 200 women
C) until the police arrested over 200 women
D) over 200 women were arrested by the police

CONTINUE
266  |  © TPR Education IP Holdings, LLC
WORDS PART II PRACTICE

This study demonstrates the extent to which 16

READING
people make decisions about philanthropic giving A) NO CHANGE
based on how a charity or cause makes them feel. Such B) when it is as important
findings reinforce 16 its importance to confirm that an C) it is as important
organization is legitimate and fiscally responsible before D) how important it is
making a donation.

WRITING AND
LANGUAGE
In all 17 likelihood, the brown marmorated stink 17
bug will continue its devastating invasion of the United A) NO CHANGE
States. B) likelihood that the

MATH
C) likelihood, while the
D) likelihood, which

In the 1980s, the United States experienced one 18


of its worst weather events involving the polar vortex, A) NO CHANGE
the 1985 Winter Cold Wave. 18 The event was given B) The event having been given an
an unremarkable name, but the weather conditions it C) Having an
brought were definitely memorable. D) An

Ironically, as a breed becomes more popular, vets 19


see not only more of those dogs but also more of certain A) NO CHANGE
health problems within the 19 breed; as unscrupulous B) breed, as unscrupulous breeders rush
breeders rush to produce more puppies to meet demand. C) breed, unscrupulous breeders rushing
D) breed being that unscrupulous breeders are
rushing

20 It is not a time-intensive practice with 20


negligible effects, composting—especially when A) NO CHANGE
undertaken at the community level—has benefits that B) That’s not
include extending the life of existing landfills, decreasing C) Not that it is
the costs of waste disposal, and reducing the toxicity of D) Far from being
landfill runoff.
CONTINUE
© TPR Education IP Holdings, LLC  |  267
SAT MANUAL

Despite the steep price of game tickets and stadium 21


READING

food, in the case of a globally-recognized sports team A) NO CHANGE


like Manchester United, game-day 21 earnings B) earning
accounting for perhaps only a quarter of the team’s C) earnings account
revenue. D) accountings
WRITING AND
LANGUAGE

If a patient’s state of mind can play a significant 22


role in how treatments are tolerated, these adherents A) NO CHANGE
22 argue. Then the design of hospital spaces becomes B) argue, then the
MATH

not just a matter of practicality but a potential tool for C) argue. The
optimizing patient health. D) argue; the

Students can also identify 23 there being skills 23


they have mastered and which they need to spend A) NO CHANGE
additional time working on. B) that
C) which
D) those are the

Incorporating art installations, maximizing access 24


to natural light, and relocating radiation suites from A) NO CHANGE
the basements they usually occupy are all examples of B) of those
proposed design changes, some 24 of which are too C) changes
costly to be undertaken in standard renovations. D) of them

CONTINUE
268  |  © TPR Education IP Holdings, LLC
WORDS PART II PRACTICE

READING
IDIOMS, HOMOPHONES, AND SLANG
At the time of the Cold War, there was great fear 1
1 for the spread of communism and its threat to
A) NO CHANGE
capitalist and democratic nations.
B) from

WRITING AND
LANGUAGE
C) of
D) DELETE the underlined portion

As diet-related health issues increase, many 2


government entities have been expanding their efforts A) NO CHANGE

MATH
to address the problem of malnutrition, especially with B) for
respect 2 of children. C) to
D) with

Translators must not only understand the literal 3


sense of the words they are translating 3 but must be A) NO CHANGE
able to convey the nuances of the speaker’s word choices B) and must also
as closely as possible. C) but
D) but also

Although master’s degrees in epidemiology are 4


becoming more common, there are very few programs A) NO CHANGE
4 where one can earn a PhD in epidemiology. B) that
C) from
D) in which

Biomedical engineers combine the problem- 5


solving skills and design approaches used in engineering A) NO CHANGE
with medical knowledge in order 5 for advancing B) to advance
healthcare. C) of advancing
D) for the advancement of

CONTINUE
© TPR Education IP Holdings, LLC  |  269
SAT MANUAL

Sometimes either the physical structure 6 nor 6


READING

the neurological process doesn’t work quite right. A) NO CHANGE


B) and the mental process
C) or the mental process
D) or the mental processes
WRITING AND
LANGUAGE

It is the longest-lasting trade embargo 7 between 7


two sovereign nations in modern history. A) NO CHANGE
B) for
C) with
MATH

D) among

A progressive funding system 8 is when poorer 8


areas receive more money than wealthier areas do. A) NO CHANGE
B) is one in which
C) is where
D) that

Fewer children means that people can invest their 9


time and productivity 9 for growing their personal A) NO CHANGE
economies. B) in
C) to
D) of

By sensing physiological changes and alerting an 10


individual 10 of what is about to happen, service dogs A) NO CHANGE
are able to keep people from going into diabetic shock or B) for
being physically harmed by a seizure. C) to
D) at

CONTINUE
270  |  © TPR Education IP Holdings, LLC
WORDS PART II PRACTICE

Additionally, algae needs 11 less natural resources 11

READING
for its cultivation. A) NO CHANGE
B) smaller
C) lower
D) fewer

WRITING AND
LANGUAGE
Solar energy is 12 the most cleanest energy 12
available and has the possibility of becoming the most A) NO CHANGE
convenient. B) the better energy
C) the cleanest energy

MATH
D) the more clean energy

A computer-controlled printer lays down sequential 13


layers of material in order 13 to create an object of any A) NO CHANGE
size or shape. B) of creating
C) for creating
D) from creating

She spent the past six months observing 14 five 14


discreet groups of black-and-white colobus monkeys. A) NO CHANGE
B) five discrete groups
C) different
D) distinct groups of five monkeys

After first being 15 promoted to principle, then 15


assistant superintendent, he was selected for the position A) NO CHANGE
of superintendent of the school district. B) appointed principal,
C) promoted as principal,
D) selected for principle,

CONTINUE
© TPR Education IP Holdings, LLC  |  271
SAT MANUAL

Whether the origin of the disorder is physical, 16


READING

neurological or undetermined, a therapist and client will A) NO CHANGE


16 precede through a series of exercises designed to B) proceed
improve function. C) lead
D) continue
WRITING AND
LANGUAGE

17 One of these is the affect fracking has on the 17


environment. A) NO CHANGE
B) One of these two is the affect
C) One of these is the effect
MATH

D) These are the effect

For many scientists, this suggested a genetic basis 18


for responses to cilantro, a supposition that has since A) NO CHANGE
been 18 pointed to by further research. B) established
C) looked at
D) spotted

These arguments are not as different as they may 19


seem at first glance. These seemingly incompatible A) NO CHANGE
ideologies in fact 19 fundamentally overlap. B) jive.
C) are cool with each other.
D) dig each other.

Many companies 20 crash and burn in new 20


markets due to a poor understanding of how product A) NO CHANGE
names or slogans translate from one language to another. B) bite it
C) falter
D) get schooled

CONTINUE
272  |  © TPR Education IP Holdings, LLC
WORDS PART II PRACTICE

Because we cannot know the secrets of Henry VIII’s 21

READING
heart, this 21 beef about the relationship between his A) NO CHANGE
religious convictions and his need for an expedient way B) dustup over
to remove obstacles from his pursuit of a male heir can C) face-off over
never be fully resolved. D) difference of opinion on

WRITING AND
LANGUAGE
It is estimated that swing voters make up 22 more 22
then twenty percent of the active voting population. A) NO CHANGE

MATH
B) greater then
C) best than
D) more than

A speech therapist who works with children 23


typically 23 sees patients at the schools they attend. A) NO CHANGE
B) see patience
C) has patients
D) has patience

Today, service animals’ functions go far beyond 24


those of seeing-eye dogs that help people who have lost A) NO CHANGE
their 24 site navigate through their communities. B) cite
C) sight
D) sites

PARALLELISM
Public defenders do all of this while earning a 1
median yearly salary of only $51,000 as compared to A) NO CHANGE
1 the $114,000 lawyers. B) the lawyers who make $114,000 a year.
C) a median yearly salary of $114,000 for the
profession as a whole.
D) the profession as a whole, which makes
$114,000.
CONTINUE
© TPR Education IP Holdings, LLC  |  273
SAT MANUAL

You are now as likely to see an anthropologist 2


READING

sitting in a conference room of a Fortune 500 company A) NO CHANGE


2 rather than the jungles of the Amazon. B) rather than in
C) as in
D) as
WRITING AND
LANGUAGE

Formed in 1987, Nirvana featured singer and 3


guitarist Kurt Cobain, 3 bassist Krist Novoselic, and A) NO CHANGE
Dave Grohl on drums. B) bassist Krist Novoselic, and drummer Dave
Grohl.
C) Krist Novoselic on bass, and Dave Grohl on
MATH

drums.
D) Krist Novoselic on bass, and drummer Dave
Grohl.

Many of the skills that are needed as a teacher are 4


the same 4 as being a superintendent. A) NO CHANGE
B) skills that have importance as superintendent.
C) as those needed to be superintendent.
D) as superintendent.

The prefixes 5 meth-, eth-, and prop- refer 5


to molecules with one, two, or three carbon atoms, A) NO CHANGE
respectively. B) meth-, eth, and the prefix prop-
C) meth-, the prefix eth-, and the prefix prop-
D) meth-, the prefix eth-, and prop-

The concept of night as frontier closely mirrors 6


6 land as frontier. A) NO CHANGE
B) that of land as frontier.
C) the land as frontier.
D) land.

CONTINUE
274  |  © TPR Education IP Holdings, LLC
WORDS PART II PRACTICE

Every other factor had a greater influence than 7

READING
did the difference between individualism and 7 the A) NO CHANGE
collective. B) the collectivists.
C) that of the collective.
D) collectivism.

WRITING AND
LANGUAGE
8 Toronto (the capital of Ontario), Montreal, 8
and Vancouver (on the west coast) are the three largest A) NO CHANGE
cities in Canada by population. B) Toronto, Montreal, and Vancouver
C) Toronto, Montreal, and the city of Vancouver

MATH
D) Toronto, Montreal (a city where French is the
official language), and Vancouver

Gua’s physiology was significantly different 9


9 than Donald. A) NO CHANGE
B) from Donald.
C) than that of Donald’s.
D) from Donald’s physiology.

Radio stations can be identified by their call signs. 10


In North America, call signs in the United States begin A) NO CHANGE
with either a W (east of the Mississippi River) or a K B) begin with a C and call signs in Mexico start
with an X.
(west of the Mississippi River), while those in Canada
10 start with a C and the ones in Mexico start with an C) begin with a C and Mexican radio stations begin
with an X.
X.
D) begin with a C and those in Mexico begin with
an X.

The females’ measurements were more variable 11


than 11 the males, with height measurements below A) NO CHANGE
the mean but body mass index and triceps skin-fold B) the males were,
measurements well above average. C) that belonging to the males,
D) those of the males,

CONTINUE
© TPR Education IP Holdings, LLC  |  275
SAT MANUAL

The amount of land required to produce enough 12


READING

algae-based biofuel to replace fossil fuels in the United A) NO CHANGE


States would be less than 12 one-seventh of that B) one-seventh of required corn land.
required for corn. C) corn, which is seven times greater.
D) one-seventh of corn.
WRITING AND
LANGUAGE

Public defenders must meet the same requirements 13


as any other lawyer: they must attend and graduate from A) NO CHANGE
college and law school, pass the bar exam, and 13 they B) they must follow
must abide by a code of ethics. C) abide with
MATH

D) abide by

By using principles of engineering, biomedical 14


engineers can construct tools that function with our A) NO CHANGE
14 body and in our bodies. B) body,
C) bodies
D) bodies,

Scientists now recognize that many traits that used 15


to be considered uniquely human appear in other animal A) NO CHANGE
species, but there are some traits, such as our advanced B) ability to understand others’ thoughts, and
language skills, 15 our ability to understand others’ instinct for cooperation,
thoughts, and instinct for cooperation, that are still C) ability to understand others’ thoughts, and our
instinct for cooperation,
understood to be exclusive to humans.
D) our being able to understand others’ thoughts,
and the instinctiveness of cooperation,

Meucci has been honored posthumously by 16


16 both Italy and the American government for his A) NO CHANGE
efforts. B) both the Italians and the American government
C) both Italian government and the American
government
D) both the Italian and American governments

CONTINUE
276  |  © TPR Education IP Holdings, LLC
WORDS PART II PRACTICE

Those who have earned their 17 high school 17

READING
diploma are twice as likely to be employed as those who A) NO CHANGE
have not. B) high schools diploma
C) high school diplomas
D) high school’s diploma

WRITING AND
LANGUAGE
In Hindu belief, cows are considered 18 a sacred 18
animal, symbols of health and abundance, and it is taboo A) NO CHANGE
to eat them. B) the sacred animal,
C) their sacred animals,

MATH
D) sacred animals,

Dogs have significantly more acute senses 19 than 19


those of humans. A) NO CHANGE
B) than humans do.
C) than the acuteness of human senses.
D) as compared to the senses of humans.

Humans and apes are very good at learning 20


through imitation, or learning to do something by A) NO CHANGE
seeing it done. The better the demonstrator of the act is, B) the much
20 the more better an ape or human will be at learning C) the increasingly
it. D) the

Solar power is what has grown the food we 21


eat, provided our bodies with vitamin D, and 21 is A) NO CHANGE
providing working light and heat for centuries. B) has been providing working light
C) will be providing working light
D) provided working light

CONTINUE
© TPR Education IP Holdings, LLC  |  277
SAT MANUAL

Had he understood how many different skills he’d 22


READING

need, he would have signed up for some marketing and A) NO CHANGE


business classes rather than 22 only music classes. B) that of music
C) those of only music
D) only those
WRITING AND
LANGUAGE

There are two persistent polar vortices; one is 23


located at the North Pole and 23 the other is the South A) NO CHANGE
Pole. B) the other are
C) the other at
D) the other from
MATH

Essential skills for success in life include decision 24


making, 24 assertiveness, and mindfulness. A) NO CHANGE
B) being assertive, and minding one’s self.
C) assertiveness, and being mindful of one’s self.
D) asserting one’s self, and being mindful of one’s
self.

MODIFIERS
Clinical dietitians are often employed directly by 1
hospitals or care facilities. Although technically not A) NO CHANGE
medical doctors, 1 nurses and doctors work with B) working with nurses and doctors helps
clinical dietitians to serve patients who have specific C) clinical dietitians work directly with nurses and
doctors
dietary needs.
D) everyone in healthcare works together

CONTINUE
278  |  © TPR Education IP Holdings, LLC
WORDS PART II PRACTICE

The prime example of what an epidemiologist does 2

READING
is the story of the “father of epidemiology,” Dr. John A) NO CHANGE
Snow. As a practicing physician in London in the 19th B) two neighborhoods in SoHo had an outbreak of
century, 2 Snow was deeply troubled by an outbreak cholera that deeply troubled Snow.
of cholera that occurred in two SoHo neighborhoods. C) an outbreak of cholera that occurred in two
SoHo neighborhoods deeply troubled Snow.

WRITING AND
D) Snow, deeply troubled, observed an outbreak of

LANGUAGE
cholera occurring in two SoHo neighborhoods.

As a field primatologist, 3 research is conducted 3


in the monkeys’ natural habitat. A) NO CHANGE

MATH
B) research is best conducted in the monkeys;
natural habitat.
C) monkeys are best researched in their natural
habitat.
D) she conducts her research in the monkeys’
natural habitat.

A foot high and ten inches wide, 4 he could 4


easily play the eighteen keys of the toy piano. A) NO CHANGE
B) the toy piano had eighteen keys that were easy
for him to play.
C) the eighteen keys on the toy piano were easy for
him to play.
D) he would use both hands to easily press down
on all eighteen keys of the toy piano.

While sad to leave the classroom, 5 the bigger 5


picture of helping students as an administrator is also A) NO CHANGE
important to her. B) she realizes she can affect the lives of more
children as an administrator.
C) the school district needs her as an administrator
more.
D) the students understand she is also good as an
administrator.

CONTINUE
© TPR Education IP Holdings, LLC  |  279
SAT MANUAL

Not having lived in the sixteenth century, 6


READING

6 conversations and interactions may have taken A) NO CHANGE


place that we can never understand all the details of. B) historical figures may have said and done a lot
of things we will never full understand.
C) we can never completely reconstruct all
the details of historical conversations and
interactions.
WRITING AND
LANGUAGE

D) a full understanding of historical conversations


and interactions will remain elusive to us.

Being Italian, 7 the invention called itself 7


MATH

telettrofono. A) NO CHANGE
B) Meucci referred to his invention as the
telettrofono.
C) telettrofono was the name given to the invention.
D) it was named for the Italian word telephone:
telettrofono.

Both immoral and illegal, 8 Dr. Kellogg had to 8


change the set-up of the experiment. A) NO CHANGE
B) the child may have been harmed in the original
version of the experiment.
C) the experiment had to be altered.
D) the monkeys could not be subjected to the
original experiment.

In their published conclusion, 9 the results can 9


be seen as a warning of the consequences of neglecting A) NO CHANGE
the health of the nation’s children and the need to seek B) the data served as a
change through political and social action. C) the scientists described the results as a
D) the children studied were viewed as a

CONTINUE
280  |  © TPR Education IP Holdings, LLC
WORDS PART II PRACTICE

10 After singing for multiple hours a night for 10

READING
three weeks, the band Queen stopped touring so Freddie A) NO CHANGE
Mercury could rest his voice. B) Three weeks of singing for multiple hours a
night,
C) Having sung for multiple hours a night for three
weeks,

WRITING AND
D) Because its lead singer had been singing for

LANGUAGE
multiple hours a night for three weeks,

Having dreamed of a magical land, 11 the poet 11


woke and eagerly began writing a new poem. A) NO CHANGE
B) beginning eagerly, the poet’s new poem

MATH
awakened.
C) the poet’s new poem began after she woke.
D) waking, her new poem was begun by the poet.

12 The warrior’s shield fell with a loud thud, 12


waking the sleeping dragon. A) NO CHANGE
B) With a loud dropping thud was the warrior’s
shield,
C) Falling with a loud thud, the warrior’s shield,
D) A loud thud, the warrior dropped her shield,

Answers can be found on pages 729–730.

© TPR Education IP Holdings, LLC | 281


GRAMMAR 101

Grammar is a piano I play by ear. All I know about


grammar is its power.
—Joan Didion

283
SAT MANUAL
READING

TRANSITIONS
Transitions are words that connect ideas to each other. The Words Part I chapter
includes lists of transitions that indicate whether ideas agree or disagree with each
other, which is often as much as you need to know on the SAT. But sometimes
the type of transition matters, so let’s take a closer look at three different types of
transitions you’ll see on the SAT.
WRITING AND
LANGUAGE

Arrows indicate
direction! Words with Transitions #1: Coordinating Conjunctions
↑ are same-direction Coordinating conjunctions are also known as FANBOYS. There are seven of them,
transitions, and words and FANBOYS is an acronym that can help you remember them. Each one has
with  are opposite- either two or three letters:
direction transitions.
MATH

For ↑ Or
And ↑ Yet 
Nor So ↑
But 

Coordinating conjunctions indicate that ideas have equal standing: that’s why
they work as STOP punctuation when combined with a comma.

Why you should care


When you have the choice to add or remove FANBOYS from a sentence, particu-
larly if there are commas also involved, be sure to use the Vertical Line Test and
check whether you’re correctly connecting complete and incomplete ideas.

Which one is correct?


1. I did all my homework, I got a good score on the SAT.
2. I did all my homework, so I got a good score on the SAT.
3. I did all my homework, but I got a good score on the SAT.

284 | © TPR Education IP Holdings, LLC


GRAMMAR 101

READING
Transitions #2: Subordinating Conjunctions
There are dozens of subordinating conjunctions, but as long as you understand
how they work, you don’t have to memorize them all. Subordinating conjunc-
tions connect an incomplete (or subordinate) idea to a complete idea in the same
sentence.

WRITING AND
1. I got a good Writing and Language score because I studied the

LANGUAGE
grammar rules that get tested on the SAT.
2. Even though I wasn’t sure about some of the answers, I used POE and
got most of the questions right.

The subordinating conjunction because in sentence 1 above makes the second


idea in the sentence (because I studied…) dependent on the first idea (I got a good
score…). In other words, the second part of the sentence is an incomplete idea.

MATH
Same goes for the first part of sentence 2: the subordinating conjunction even
though makes it an incomplete idea.

Here are some of the most common subordinating conjunctions that appear on
the SAT:

Although  Though 
One way you can
As Until  identify subordinating
conjunctions is that they
Because ↑ When generally don’t have
commas after them.
Even though  While
Since ↑

© TPR Education IP Holdings, LLC | 285


SAT MANUAL
READING

Why you should care


What happens to the second part of sentence 1 above if you remove the word
because? It becomes a complete idea! When you have the choice to add or remove
a subordinating conjunction, just like with FANBOYS, use the Vertical Line Test
and check that you’re using the correct punctuation for connecting ideas.
WRITING AND
LANGUAGE

Which one is correct?


1. Although I had always thought commas were confusing, learning the
four comma rules made things much clearer.
2. I had always thought commas were confusing, learning the four
comma rules made things much clearer.
3. Since I had always thought commas were confusing, learning the four
comma rules made things much clearer.
MATH

Transitions #3: Conjunctive Adverbs


ConWHAT? The name doesn’t matter—think of this last category as transition
words that have commas after them. They have commas after them because you
can remove them from a sentence without changing its main meaning. These
are the most common type of transitions you’ll see on the SAT, and the most
important thing to know about them is what they mean.

What comes next?


What comes after each transition word—an example, a conclusion, an additional
point, or a contradiction?

1. However, _________________________________________

2. For example, ______________________________________

3. Therefore, _________________________________________

4. Moreover, _________________________________________

5. Furthermore, ______________________________________

6. In addition, _______________________________________

7. Likewise, _________________________________________

8. Nevertheless, ______________________________________

9. Consequently, _____________________________________

10. Nonetheless, ______________________________________

286  |  © TPR Education IP Holdings, LLC


GRAMMAR 101

READING
11. In fact, ___________________________________________

12. Thus, ____________________________________________

13. Similarly, _________________________________________

WRITING AND
14. Despite this, ______________________________________

LANGUAGE
15. Accordingly, ______________________________________

SUBJECT-VERB AGREEMENT

MATH
A verb is a word that expresses an action (e.g., walk), a feeling (e.g., want), or a
state of being (e.g., is). The subject of a verb is the person or thing that is doing the
action or feeling the feeling. Subjects are usually nouns or pronouns.

The basic rule of Subject-Verb Agreement is relatively simple:


• Singular subjects require singular verbs.
• Plural subjects require plural verbs.

1. The student sharpens her pencil.


2. The teachers announce the rules.

The subject of the first sentence, the student is singular, so sentence 1 requires the
singular verb sharpens. The subject of the second sentence, the teachers, is plural, so
sentence 2 requires the plural verb announce. Don’t be confused by the -s ending
on verbs: that ending only indicates a plural with nouns.

Singular verbs are ones you would use with it,


and plural verbs are ones you would use
with they.

Of course, the SAT has some tricks for making subject-verb agreement more
difficult.

© TPR Education IP Holdings, LLC | 287


SAT MANUAL
READING

Prepositions
1. Only one of the sophomores is / are taking the SAT.

What’s the subject? ______________________________

Singular or plural? ___________________


WRITING AND
LANGUAGE

2. The students in the senior class has / have already taken the SAT.

What’s the subject? ______________________________

Singular or plural? ___________________


MATH

The phrases of the sophomores and in the senior class are prepositional phrases. The
SAT often includes prepositional phrases between the subject and main verb in a
sentence to try to confuse you.

A preposition is a word that describes direction or location.

Common Prepositions:
of with
in on
to at
into by
for from

If you’re trying to identify the subject in a sentence and you see a preposition, look
before the preposition for the subject. In fact, you can use your pencil to cross out
prepositional phrases altogether to keep them from distracting you:

1. Only one of the sophomores is taking the SAT.

2. The students in the senior class have already taken the SAT.

288 | © TPR Education IP Holdings, LLC


GRAMMAR 101

READING
SUBJECT/VERB DRILL
Circle the subject and underline the verb. Some sentences have more than one of
each.

1. My name is Inigo Montoya.

WRITING AND
2. The dude abides.

LANGUAGE
3. If you build it, he will come.

4. Just when I thought I was out, they pull me back in.

5. The stuff that dreams are made of is a bedazzled bird.

MATH
6. Liver with fava beans pairs well with Chianti.

7. Children of the night make beautiful music.

8. They are storming the castle.

9. Building up an immunity to iocane powder could save your life.

10. The people on the jury are faced with a grave responsibility.

11. The jury must decide the defendant’s fate.

© TPR Education IP Holdings, LLC  |  289


SAT MANUAL
READING

PRONOUNS
A pronoun is a word that stands in for a noun. Some pronouns can make subject-
verb agreement tricky.

Personal Demonstrative Indefinite


• I • this • -body (anybody,
WRITING AND

everybody, somebody,
LANGUAGE

• you • that nobody)


• he • -one (anyone, everyone,
• she someone, no one)

• it • -thing (anything,
everything, something,
• me nothing)
Singular
MATH

• him • -where (anywhere, OR


Singular

everywhere, somewhere, plural:


• her nowhere)
• all
• each
• most
• much
• some
• either
• more
• neither
• any
• one
• none
• less
• enough
• little
• we • these • both
• you • those • several
Plural

• they • few
• us • fewer
• them • many

Choose the correct verb for each sentence:

1. Each of the juniors takes / take the SAT at least once.

2. All of the juniors takes / take the SAT at least once.

3. Everybody thinks / think it’s crazy to take the SAT ten times.

290  |  © TPR Education IP Holdings, LLC


GRAMMAR 101

READING
Pronoun Agreement
Any time you have to choose a pronoun on the SAT, you must be able to point to
exactly what noun or other pronoun it refers to, and the pronoun you choose must
match that word not only with respect to singular/plural but also gender.

Subject Object Possessive

WRITING AND
Pronoun Pronoun Pronoun

LANGUAGE
replaces the noun
What It replaces the noun replaces the noun that is
receiving the
Does doing the action showing ownership
action
I me my / mine
you you your / yours
he him his

MATH
she her her / hers
it it its
we us our / ours
they them their / theirs

One thing that’s different on the SAT than in everyday life is that the pronouns
they, them, and their can only be plural. You’re not likely to have to choose a pro-
noun for a person whose gender is unclear, but remember that they can never refer
to a singular noun on the SAT.

1. The writers of the SAT sometimes use confusing sentence structures


because it / they want to confuse the test takers.

What word does the pronoun replace? _____________________________

Singular or plural? ___________________

2. College Board has a lot of power over your college applications


because it / they has a monopoly on AP Exams.

What word does the pronoun replace? _____________________________

Singular or plural? ___________________

© TPR Education IP Holdings, LLC  |  291


SAT MANUAL
READING

Relative Pronouns
A relative pronoun introduces an incomplete idea that gives additional
information about a noun elsewhere in the sentence.

The incomplete idea introduced by a relative pronoun usually answers a question


about a word or phrase elsewhere in the sentence, such as which one:
WRITING AND
LANGUAGE

• I got my highest score on the SAT that I took last month.

Or where:

• I brought ten sharp pencils with me to the test because the sharpeners
at our school, which are mounted on the wall in each classroom, are
often broken.
MATH

Or gives additional information about something:

• The proctor, who is also my AP History teacher, gave a five-minute


warning before the end of each section.

The bolded words in the examples above are all relative pronouns.

Relative Pronouns you’re most likely to see on the SAT include:


Who Which
Whom That

Why you should care


When you change a pronoun from a subject pronoun (I, he, she, they) to a relative
pronoun, it can make an idea incomplete, which means you need to check the
punctuation. Relative pronouns are also often used to introduce unnecessary ideas
that should be set off by commas, so their presence can be an indication that you
need commas in a sentence.

That is especially tricky: Which one is correct?


it can be a demonstrative 1. Roxanne got a perfect score on the PSAT, which qualified her for a
pronoun (which means it National Merit Scholarship.
can act as the subject of a
sentence) or a
relative pronoun. 2. Roxanne got a perfect score on the PSAT, that qualified her for a
National Merit Scholarship.

3. Roxanne got a perfect score on the PSAT, it qualified her for a


National Merit Scholarship.

292 | © TPR Education IP Holdings, LLC


GRAMMAR 101

READING
Who versus Whom
The word whom doesn’t show up very often on the SAT, but if it does, the easiest
way to choose between who and whom is to substitute he/him or they/them in the
sentence. The -m words all go together.

WRITING AND
If you would use he or they in the sentence, use who.

LANGUAGE
If you would use him or them in the sentence, use whom.

1. I had a hard time concentrating during the SAT because the student
who / whom was sitting behind me had a bad cough.

MATH
2. Who / whom are you going to study with?

VERB TENSE
Verb tense is used to indicate when an action occurs. There are three basic tenses:

Past: I studied for the SAT.

Present: I study for the SAT.

Future: I will study for the SAT.

There are more nuances of each tense, however. The ones you’ll most commonly
run into on the SAT are past perfect and present perfect. These tenses are cre-
ated by using the helping verb to have along with the past participle of the verb.

Past perfect tense indicates an action that started in the past, but was interrupted
or ended. To make a past perfect verb, use the past tense of to have plus the past
participle of your verb:

He had studied AP Physics for 20 hours a week before he realized that his
other grades were suffering.

Present perfect tense indicates an action that started in the past and continues
into the present. To make a present perfect verb, use the present tense of to have
plus the past participle of your verb:

They have studied for the SAT every day for the last six weeks.

© TPR Education IP Holdings, LLC | 293


SAT MANUAL
READING

Choose the correct tense for each sentence:

1. I am / was / will be well-prepared to take the SAT today.


2. She takes / took / will take three AP classes next year.
3. They have read / had read at least 50 pages every day this month.
4. Before you took the SAT, you have taken / had taken a practice test
WRITING AND

every Saturday.
LANGUAGE

5. He sends / sent / will send his SAT scores to 12 schools last time he
took the test.
MATH

Answers can be found on page 731.

294 | © TPR Education IP Holdings, LLC


FULL-LENGTH
PRACTICE
PASSAGES

295
SAT MANUAL

Questions 1–11 are based on the following passage. 1


READING

A) NO CHANGE
Passage 1: What Actually is an Actuary? B) their food.
Everyone knows what insurance is: almost every C) that for food.
part of modern life is insured somehow. People have D) all food.
insurance for their houses and cars, insurance for their
WRITING AND
LANGUAGE

health, even insurance for their lives. The total amount


2
people pay for insurance can be greater than 1 food.
A) NO CHANGE
What is less clear is exactly how insurance works.
B) events, such as car accidents,
Insurance programs are based on a very simple
C) events, such as, car accidents
idea. Due to the unpredictability of life, undesirable
D) events such as car accidents,
2 events, such as car accidents are bound to happen.
MATH

A car accident can cause substantial loss, including car


damage, 3 personal injury or death. To help lessen 3
the effect of such loss, people purchase insurance. They A) NO CHANGE
pay premiums to insurance companies, in exchange B) personal injury and death.
for which 4 they provide financial compensation if C) personal, injury, or death.
an accident occurs. The part that gets complicated is D) personal injury, or death.
determining 5 what the probability is an undesirable
event will or could occur and how much of a premium
4
should be paid.
A) NO CHANGE
B) it provides
C) those provide
D) the companies provide

5
A) NO CHANGE
B) the probability of an undesirable event
C) when and how an event may or could occur
D) how likely it is an undesirable event will or
could occur

CONTINUE
296  |  © TPR Education IP Holdings, LLC
FULL-LENGTH PRACTICE PASSAGES

6 Actuaries help to prevent car accidents 6

READING
and other unforeseen events. Actuaries assess risk Which choice provides the best transition from the
previous paragraph to this one?
by examining the statistical data available on past
A) NO CHANGE
undesirable 7 events—or the lack of such events
B) The more likely a car accident is, the more
as the case may be. In order to do this accurately,
insurance you should purchase.
actuaries must create mathematical models based on

WRITING AND
C) Because accidents are unavoidable, it is likely

LANGUAGE
the information available to them. These mathematical that your car will sustain damage and that you
models are then used to determine how much a may even be seriously injured someday.
particular customer’s insurance premiums will cost. 8 D) Actuaries compute the premium rates for
insurance companies.

MATH
A) NO CHANGE
B) events; or
C) events. Or
D) events or

8
At this point, the writer is considering adding the
following sentence.
Many people think their insurance
premiums are too high, but they
don’t realize that different insurance
companies might charge different rates,
so shopping around is a good idea.
Should the writer make this addition here?
A) Yes, because it clarifies how premiums are
determined.
B) Yes, because without it there would be no clear
transition between the previous sentence and
the following paragraph.
C) No, because it is not directly related to the
discussion of how insurance premiums are
determined.
D) No, because it repeats information given in the
following paragraph.

CONTINUE
© TPR Education IP Holdings, LLC  |  297
SAT MANUAL

In order to determine the premium an individual 9


READING

should be charged, actuaries gather information on A) NO CHANGE


that person. The first thing they look at is past events: a B) In fact,
customer’s driving record, for example. The number of C) Moreover,
accidents she has been in or tickets she has received and D) For instance,
their severity will be taken into account in the formula
WRITING AND
LANGUAGE

used. If she has a poor driving record, she is more likely


10
to require an insurance payout and will pay higher
Which choice offers an accurate interpretation of
premiums. 9 However, a customer who was in a the data in the graph?
single fender bender will have lower premiums than one A) NO CHANGE
who has received multiple speeding tickets. B) In general, these risk traits are stable and do not
But what if someone has a clean driving record? change over a person’s lifetime.
MATH

If an indication of risk is not clear from an individual C) Most insurance companies view elderly women
as greater payout risks than elderly men.
history, actuaries turn to statistical data. They look
D) Insurance companies tend to discriminate
at specific factors, such as age, gender, geographical
against teenagers by overcharging them.
location, and make and model of vehicle, and the
correlation of those factors to safe driving behavior in
the general public. Each trait has a different associated 11
level of risk that affects premium payments. 10 For A) NO CHANGE
instance, young men are viewed as greater payout risks B) for determining
than young or middle-aged women. Actuaries enter this C) to help in determining
information into the mathematical model, constructed D) to determine
based on all these varying factors, 11 for to determine
the premium any given individual should pay.

Average Car Insurance Rates by Age and Gender


3,750
Men
3,000 Women
Dollars per year

2,250

1,500

750

0
30

35

45

70
60

65
40

55
20

25

50
26–

31–

41–

66–
56–

61–
36–

51–
16–

21–

46–

Age

CONTINUE
298  |  © TPR Education IP Holdings, LLC
FULL-LENGTH PRACTICE PASSAGES

Questions 12–22 are based on the following passage. 12

READING
Which choice best supports the main point of the
Passage 2: Bag Ban Debate paragraph?
Bans on single-use plastic bags are on the rise, A) NO CHANGE
12 though some businesses object to them. It is not B) with bans in more than 200 cities in 17 states,
far-fetched to believe that someday soon we will be able and legislation in progress elsewhere.

WRITING AND
C) and bans on single-use plastic water bottles may

LANGUAGE
to add single-use plastic bags to the list of things the next
be coming next.
generation may never know, such as landline phones and
D) although the amount of plastic pollution does
wristwatches that don’t count steps. not yet seem to be affected by such legislation.
The main proponents of plastic bag bans are
environmentalists. They see plastic bags as one of the
most ubiquitous pieces of trash in America: plastic 13

MATH
bags are used in many stores and 13 get carried A) NO CHANGE

home by every American. This is no exaggeration—the B) having been carried

United States alone uses 100 billion plastic bags a year. C) carrying

Although plastic bags can technically be recycled, D) carried

14 Americans recycle less than 5% of the plastic bags


they use. This means that the majority of plastic bags 14
are 15 deported in landfills rather than turned into The writer wants to support the paragraph’s main
idea with accurate, relevant information from the
graph. Which choice most effectively accomplishes
Plastic Single-Use Bag Disposal Rates this goal?
100% A) NO CHANGE
B) no country recycles more than 35% of the
75% plastic bags its citizens use.
C) many developed countries already recycle the
50% majority of their plastic bags and yet the bags
are still a problem.
25% D) England, Japan, and Australia are the only
countries that recycle plastic bags at a rate
higher than 10%.
0%
USA China England Japan France Australia

Landfill Recycled 15
A) NO CHANGE
B) imported
C) deposited
D) composted

CONTINUE
© TPR Education IP Holdings, LLC  |  299
SAT MANUAL

something new. 16 Moreover, not all discarded bags 16


READING

even make it to a landfill: the second most common item A) NO CHANGE


of ocean refuse in 2008 was plastic bags. Even if plastic B) Traditionally,
bags do make it into a landfill, they remain toxic after C) Indeed,
they break down, a process that can take up to 1,000 D) As a rule,
years.
WRITING AND
LANGUAGE

[1] 17 Financial cost should also be considered,


17
and some people argue that plastic bags can have a
Which choice best connects the sentence with the
positive economic impact. [2] Plastic bags generally cost previous paragraph?
one-third the amount paper bags do, which means that A) NO CHANGE
when a bag ban is enacted, it can lead to 18 convincing B) Despite all these negatives,
financial hardship for grocers and retailers. [3] When C) Though it’s clear that plastic bags ought to be
MATH

retailers are facing financial hardship, they do the most banned everywhere,
economically logical thing—they pass those costs on D) Although recycling can be expensive,
to the consumers. [4] Consumers then usually react
by buying less. [5] When a consumer decides not to 18
buy that 19 avocado, because it costs 50 cents more, Which choice most effectively establishes that the
avocado farms in Central and South America make cost difference between paper and plastic bags can
less money, and when a farm makes less money, so do have an impact on businesses?
the farm laborers. [6] This is the butterfly effect—the A) NO CHANGE

positive of keeping plastic bags out of landfills and B) heavy

oceans has a negative impact on farming families. C) valuable

[7] There is also an economic impact on plastic bag D) significant

manufacturers when stores suddenly stop using their


products. 20 19
A) NO CHANGE
B) avocado because it costs 50 cents more:
C) avocado because it costs 50 cents more,
D) avocado—because it costs 50 cents more,

20
To make this paragraph most logical, sentence 3
should be placed
A) where it is now.
B) after sentence 1.
C) after sentence 4.
D) after sentence 5.

CONTINUE
300  |  © TPR Education IP Holdings, LLC
FULL-LENGTH PRACTICE PASSAGES

As with so many things, a plastic bag ban is a 21

READING
trade-off, and there’s not necessarily a single perfect A) NO CHANGE
solution. Some cities have implemented fees for plastic B) use, rather than banning them altogether
bag 21 use rather than banning them altogether and C) use, rather than banning them, altogether
some local governments have offered subsidies for D) use, rather than banning them altogether,
plastic bag producers to enable the production of heavier,

WRITING AND
LANGUAGE
reusable plastic bags. Different cities have taken different
22
approaches, but most plastic bag legislation is too new
A) NO CHANGE
for 22 their long-term effects to be fully understood.
B) they’re
C) its
D) it’s

MATH

CONTINUE
© TPR Education IP Holdings, LLC  |  301
SAT MANUAL

Questions 23–33 are based on the following passage. 23


READING

A) NO CHANGE
Passage 3: The Rise of Urban Art
B) medium that
Art is never static. It is a constantly evolving
C) medium. One that
23 medium. That reflects the changing society in
D) medium; that
which it exists. One recent change in the art world is
WRITING AND
LANGUAGE

the emergence of urban art and the market that has


developed around it. 24

Urban art has its roots in what is sometimes known A) NO CHANGE


as street art or graffiti. This is art that has been created B) a mistake
on physical pieces of public or private property and C) a nuisance

is freely viewed by the public at large. Historically, it D) an offense


MATH

has been viewed at best as 24 a bore and at worst


as destruction of public property. But urban art has 25
increased both in popularity and notoriety in the early Which choice provides the best supporting example
years of the 21st century. for the idea in the first part of the sentence?
One reason urban art is becoming more popular A) NO CHANGE
is its ability to capture the popular imagination. Many B) urban artists tend to hide their work out in the
open so it is tricky to find.
people are drawn to urban art because it is accessible to
C) although in the business of day-to-day life it is
them, both literally and figuratively. Literally, because
easy to walk by urban art without noticing it.
it is physically in front of them during their daily lives—
D) where it adds beauty to otherwise boring parts
25 on the commute to and from work, on a walk with of cities.
the dog, or on a night out on the town. Figuratively,
because urban art typically serves as social commentary.
26
Its subject matter is most often about the space in which
Which choice most effectively combines the
it 26 exists. That space is society, which is both the
sentences at the underlined portion?
subject and the consumer of urban art.
A) exists, which is
B) exists—
C) exists; that space is
D) exists—that space being

CONTINUE
302  |  © TPR Education IP Holdings, LLC
FULL-LENGTH PRACTICE PASSAGES

[1] Up until the last half decade, most urban art 27

READING
was transitory. [2] Because it was created on exposed A) NO CHANGE
surfaces, 27 it tended to disappear. [3] As more works B) they
of urban art are 28 described to famous artists, cities C) that
are not only commissioning but also preserving the art, D) those
either for the sake of the art itself or for the possible

WRITING AND
LANGUAGE
monetary value it holds. [4] But the growing interest in
28
the genre has changed the art form into a permanent
A) NO CHANGE
one. [5] 29 More and more increasingly, cities are
B) described as
jumping on the urban art bandwagon and actually
C) ascribed to
commissioning pieces of urban art to be created in
D) defined as
public areas. 30

MATH
29
A) NO CHANGE
B) Increasingly greater numbers of
C) More increasingly,
D) More and more

30
To make this paragraph most logical, sentence 3
should be placed
A) where it is now.
B) after sentence 1.
C) after sentence 4.
D) after sentence 5.

CONTINUE
© TPR Education IP Holdings, LLC  |  303
SAT MANUAL

The presence of urban art in the auction house has 31


READING

exploded in recent years. This has led to a fascinating A) NO CHANGE


debate over the ownership of urban art: does it belong B) should it belong to its environment?
to the artist who created it, the owner of the property on C) its existence in a particular place?
which it was created, or 31 maybe to the environment D) the environment in which it exists?
in which it exists? Does the artist have any right to a
WRITING AND
LANGUAGE

work on public property? Many in the art world are


32
against the monetization of urban art, including some
A) NO CHANGE
of the most prominent 32 creator’s of such art. The
B) creators’ of that type of
famous street artist Banksy, for example, views the
C) creators of such
selling of street art as an undemocratic process that
D) creators of those types of
epitomizes social greed. 33
MATH

33
The writer wants to conclude the passage with
a sentence that emphasizes the unclear future
of urban art. Which choice most effectively
accomplishes this goal?
A) As the art world continues to evolve, it will be
interesting to see how this debate evolves with it.
B) Hopefully in the future artists will choose to
create art on surfaces that are easier to remove
for selling.
C) If people don’t want artists to create works on
their property, they should put up polite signs.
D) It is likely that artists will focus on more
experiential art forms such as performance art
in the future.

CONTINUE
304  |  © TPR Education IP Holdings, LLC
FULL-LENGTH PRACTICE PASSAGES

Questions 34–44 are based on the following passage. 34

READING
A) NO CHANGE
Passage 4: The Science of Self-Talk
B) though
Mental well-being is one of the most important
C) because
aspects of our health, 34 and it is often overlooked by
D) however,
patients and doctors alike. In fact, our mental states are

WRITING AND
LANGUAGE
intimately connected to our physical states. The mind
35 standardizes the body and keeps it functioning 35

properly. It stands to reason that if we are not as mentally A) NO CHANGE


well 36 as you could be, we will not be as physically B) conducts
well either. C) regulates

It turns out that this connection affects more than D) supervises

MATH
just health. The way people think about themselves while
performing different tasks has an impact on 37 they’re 36
physical abilities. This is known as “self-talk.” Self-talk is A) NO CHANGE
the active, instructive commentary that plays silently B) as you could be, you will not
38 through people’s heads without making noise when C) as we could be, you will not
D) as we could be, we will not

37
A) NO CHANGE
B) their
C) there
D) they are

38
A) NO CHANGE
B) without noise on the inside of people’s heads
C) through people’s heads internally and noiselessly
D) through people’s heads

CONTINUE
© TPR Education IP Holdings, LLC  |  305
SAT MANUAL

they engage in difficult or unfamiliar tasks. When 39


READING

learning to drive, a person might mentally recite “turn At this point, the writer is considering adding the
on blinker, check mirrors, check blind spot, and slowly following sentence.
move into the lane.” 39 This mental narrative is easier to follow
if there is no other noise, which is why
There is a cycle that connects thought and action, many states have restrictions on the
and self-talk is an essential component of that cycle. The number of passengers teenage drivers
WRITING AND

can have in the car.


LANGUAGE

first stage is forethought: setting a goal and planning


how to attain it. 40 Then there is performance. This Should the writer make this addition here?

second step of performance is comprised of enacting A) Yes, because it clarifies the importance of self-
talk when learning a new task.
the plan. The last stage is self-reflection, a careful review
B) Yes, because it helps explain why many states
of how the plan was performed and how it could be place restrictions on young drivers.
adjusted in the future. Self-talk, in particular the way C) No, because it is not directly related to the
MATH

people phrase their self-talk, is vital to the success of all paragraph’s focus on the mind-body connection.
three parts of this cycle. D) No, because it is already clear that restrictions
on teenage drivers are important.

40
Which choice most effectively combines the
underlined sentences?
A) Then there is performance, in which the
planning is executed.
B) Then there is performance; this step includes
executing the plan.
C) Then there is performance, the second step of
the execution of the plan.
D) Then there is performance, executing the plan.

CONTINUE
306  |  © TPR Education IP Holdings, LLC
FULL-LENGTH PRACTICE PASSAGES

The best predictor of success in performing a 41

READING
task is the noun or pronoun a person uses in self-talk. A) NO CHANGE
The use of the word “I” in self-talk is detrimental B) study—conducted at the University of Michigan,
compared to “you,” “one,” or the person’s own name. C) study conducted at the University of Michigan,
In one 41 study conducted at: the University of D) study, conducted at, the University of Michigan,
Michigan, half of the participants were asked to solve

WRITING AND
LANGUAGE
a puzzle while mentally referring to themselves as
42
“I,” while the other half were asked to use “you.” The
Which choice most effectively concludes the
“you” participants were more successful, 42 though paragraph?
researchers aren’t quite sure why. A) NO CHANGE
The second-best indicator of success is 43 the B) with eighty-eight percent completing the task
tone of one’s self-talk. The more positively people frame as compared to only sixteen percent of the “I”

MATH
participants.
their inner narrative, the more successful they tend to be.
C) and they also seemed to enjoy the puzzle more
In a weight loss study conducted in Greece, participants
than the “I” participants did.
who were instructed to frame their thinking as “I am
D) although several participants from both groups
at this weight; I would prefer not to be; I can take steps failed the task completely.
to change” lost more weight than did participants
instructed to frame their thinking as “I am at this weight;
43
I am unhappy.” 44 Fortunately, the more precise,
Which choice most effectively sets up the main idea
succinct, and consistent self-talk is, the more effective it
of the following two sentences?
will be.
A) NO CHANGE
B) whether a person has just a few pounds to lose,
or more than 50.
C) how aware a person is of the importance of self-
talk.
D) how physically active one is.

44
A) NO CHANGE
B) Nevertheless,
C) Generally,
D) Surprisingly,

CONTINUE
© TPR Education IP Holdings, LLC  |  307
SAT MANUAL

Questions 1–11 are based on the following passage. 1


READING

The writer wants to support the paragraph’s main


Passage 5: Forensic Anthropology idea with accurate, relevant information from the
Forensic anthropology is the application of the graph. Which choice most effectively accomplishes
this goal?
science of anthropology in a legal setting. This most
A) Two years after each show premiered,
commonly means the study of human remains as
graduation rates in forensic anthropology had
WRITING AND
LANGUAGE

it relates to the criminal justice system. The field of nearly doubled.


forensic anthropology has had a surge in popularity in B) Initially after each show premiered there was
the last two decades, in part due to shows such as Law & a decline in interest in forensic anthropology,
followed by a surge.
Order and CSI. 1
C) Although earlier in the twentieth century more
Graduates with BS Degrees in Forensic women than men graduated with degrees in
Anthropology from 1985–2003 forensic anthropology, that trend has now
MATH

reversed.
23 D) However, as the popularity of television shows
Law and Order premiers CSI premieres
18 Law and Order highlighting forensic studies wanes so does
premieres student interest.
Frequency

14
9
5 2
0 A) NO CHANGE
19 1
92
19 3
19 5
86
19 7

03
88
19 9
19 0

19 7
98
20 9
00
20 1
20 2
94
19 5
19 6

B) On the other hand,


9

9
8

8
9

0
0
9
9
19
19

19

19

19

20
19

Female Male C) For example,


D) Similarly,

2 Surprisingly, it has also been over the last


two decades that the field of forensic anthropology 3
has experienced the greatest shifts in technique and A) NO CHANGE
perspective. These changes have not been universally B) then it was twenty years ago.
celebrated: there is some argument within the field over C) rather than twenty years prior.
whether the discipline is more vital 3 then it was D) than it was twenty years ago.
twenty years prior.

CONTINUE
308 | © TPR Education IP Holdings, LLC
FULL-LENGTH PRACTICE PASSAGES

There are six key developments that have changed 4

READING
forensic anthropology. Two of these are external A) NO CHANGE
developments that are generally agreed to be detrimental B) field. The advent
to the 4 field; the advent and increased applicability C) field, the advent
of DNA analysis and changes in requirements for D) field: the advent
evidence in the legal system. DNA analysis is seen

WRITING AND
LANGUAGE
by many as a threat to the relevance of the field of
5
forensic anthropology. Because the evidence it provides
A) NO CHANGE
is scientifically certain, it renders the expertise of a
B) For instance,
forensic anthropologist unnecessary. 5 Similarly, legal
C) Indeed,
challenges have recently shifted the focus of court cases
D) Fortunately,
to the methods used and the reliability of findings from

MATH
forensic scientists rather than on the credentials and
experience of the scientists themselves.

CONTINUE
© TPR Education IP Holdings, LLC  |  309
SAT MANUAL

6 These changes can make it difficult for forensic 6


READING

anthropologists to stay focused on their jobs. The other Which choice best establishes the main idea of the
four key developments that occurred are improvements paragraph?
A) NO CHANGE
in forensic taphonomy, quantitative methods, forensic
B) While the debate over the legal reliability of
archaeology, and human skeletal trauma analysis.
forensic anthropology continues to rage, other
Improvements in forensic taphonomy—the study of changes have happened also.
WRITING AND
LANGUAGE

human 7 decomposition, and quantitative data C) Not all of the changes affecting forensic
have led to more reliable analysis techniques based on anthropology have been negative, however.
modern skeletal samples. Many of these improvements D) These changes may make forensic anthropology
unappealing to future generations.
are due to the expansion of teaching facilities. It was
not until 1981 that the first Anthropological Research
Facility opened at the University of Tennessee at 7
MATH

Knoxville. The facility accepts donated human remains A) NO CHANGE


for the study of decomposition, which has 8 obtained B) decomposition—
it the nickname of “the body farm.” It wasn’t until 25 C) decomposition:
years later, in 2006, that Western Carolina University D) decomposition;
opened the second human decomposition lab. These
days, five such facilities operate in the United States,
8
9 and it seems unlikely that any more will open
A) NO CHANGE
anytime soon.
B) derived
C) earned
D) pulled in

9
Which choice most effectively concludes the
sentence and the paragraph?
A) NO CHANGE
B) collectively educating over 2,000 students on
the most up-to-date forensic methods.
C) with applications from prospective students
increasing every year.
D) four more than can be found in Europe.

CONTINUE
310  |  © TPR Education IP Holdings, LLC
FULL-LENGTH PRACTICE PASSAGES

Developments in forensic archaeology 10 has 10

READING
helped to advance the field through new methods and A) NO CHANGE
technologies that relate to both archaeological sites and B) have helped to advance
11 forensics. And human skeletal trauma analysis has C) has been advancing
broadened the scope of forensic anthropologists and D) was advanced in
their usefulness in specific forensic settings, such as fatal

WRITING AND
LANGUAGE
fire scenes.
11
Only time will tell whether the business of
A) NO CHANGE
DNA analysis will eliminate the need for forensic
B) sites relating to forensics.
anthropology. Due to advances in both fields, it is likely
C) forensic sites.
that a combination of the two will be the legal way
D) those situations requiring forensics.
forward.

MATH

CONTINUE
© TPR Education IP Holdings, LLC  |  311
SAT MANUAL

Questions 12–22 are based on the following passage. 12


READING

A) NO CHANGE
 assage 6: A Whale of a Tale—The Myth of
P
B) relate
Christopher Columbus
C) relates
One of the most enduring myths ever spread
D) would have related
12 had related to the reason for Christopher
WRITING AND

Columbus’s voyage to the New World. Despite the


LANGUAGE

lingering belief that Columbus took to the sea to 13


disprove the notion that the Earth was flat, there was no A) NO CHANGE
such notion to disprove in the 15th century. Since the B) BCE, the
time of the ancient Greeks, scholars had been aware that C) BCE; when the
the Earth was spherical, just like the moon and visible D) BCE—the
MATH

planets. As early as 276 13 BCE; the mathematician


Eratosthenes had calculated the circumference of
14
the Earth with impressive precision. In fact, during
Which choice most effectively supports the idea
Columbus’s time, 14 scholars of ancient Greek were that the concept of a spherical Earth was widely
familiar with Eratosthenes’s calculation. accepted in Columbus’s time?
A) NO CHANGE
B) mapmakers were producing globes in addition
to flat maps of the world.
C) Columbus himself had been to sea before so he
was especially confident that the Earth was a
sphere.
D) it was not widely known that the Vikings had
reached the New World centuries previously.

CONTINUE
312  |  © TPR Education IP Holdings, LLC
FULL-LENGTH PRACTICE PASSAGES

The myth that Columbus was trying to disprove the 15

READING
notion of a flat Earth did not actually originate until the A) NO CHANGE
19th century, in an American work of historical fiction. B) As the most popular rendition of the Columbus
Washington Irving published a highly fictionalized tale,
biography of Columbus, The History of the Life and C) As the discoverer of the New World,
Voyages of Christopher Columbus, in 1828. 15 The D) As the author of the most widely read version of

WRITING AND
Columbus’s story,

LANGUAGE
most widely read version of the story, Irving became
one of the most notorious mythmakers in history. Many
took his story quite literally, and eventually Irving’s 16
version became popular belief. But the popularity of the Which choice most effectively introduces the topic
tale went even further: people also began to question that is discussed in the remainder of the essay?
A) What were the motivations behind Irving’s
whether the world was, in fact, round. 16

MATH
desire to mislead the world as to the shape of
Just a few decades after Irving first established the Earth?
this myth in print, an Englishman named Samuel B) Who would believe the Earth was flat when
Rowbotham published a 16-page pamphlet entitled there was so much evidence against that theory?
Zetetic Astronomy: Earth Not a Globe, which he later C) What are the other most prevalent myths from
the last few centuries?
expanded to a 430-page book. The term “Zetetic
D) What is the scientific evidence that people use
Astronomy” comes from the Greek word zetein,
to determine the shape of the Earth?
which means “proceeding by 17 inquiry.” The
term was meant to bolster the scientific legitimacy of
Rowbotham’s research. Rowbotham believed that the 17
Earth was a flat disc with the North Pole at the center. Which choice most effectively combines the
sentences at the underlined portion?
The disc was surrounded by a 150-foot wall of ice, which
A) inquiry” and
we know as Antarctica.
B) inquiry”; this officious-sounding term
In 1873, the American Universal Zetetic Society
C) inquiry”—the purpose of this term
was founded based on the work of Rowbotham. Ten
D) inquiry,” which term
years later, the British chapter was established by Lady
Elizabeth Blount. The Universal Zetetic Society had a
18 devout, albeit small group, of followers, and 18
published two periodicals during its existence: The Earth A) NO CHANGE
Not a Globe Review and Earth: A Monthly Magazine of B) devout, albeit small group of followers,
Sense and Science. C) devout, albeit small, group of followers
D) devout, albeit small group, of followers

CONTINUE
© TPR Education IP Holdings, LLC  |  313
SAT MANUAL

Interest in the Universal Zetetic Society waned in 19


READING

the aftermath of World War I, but it was 19 revived A) NO CHANGE


and renewed under the guise of the International Flat B) brought back to life again
Earth Society in 1956. Despite the seemingly irrefutable C) reestablished and reenergized
evidence that came from the launch of the first satellite D) revived
and the Apollo missions in the subsequent decades,
WRITING AND
LANGUAGE

the society 20 did not waver in its beliefs. Members


20
of the society, commonly referred to as “flat-earthers,”
Which choice most effectively establishes a contrast
dismissed satellite photos of a spherical Earth as between the evidence mentioned in the first part of
Hollywood hoaxes and government cover-ups. 21 As a the sentence and the views of the flat-earthers?
rule, the group boasted over 3,500 members. By 1980 the A) NO CHANGE
group had dwindled to 200 members and was officially B) did not uphold
MATH

dissolved in 2001. But the flat Earth myth persists: the C) continued to change
organization was revived again in 2009, and today has D) began to question
about 500 members 22 worldwide.

21
Nationality of Self-identified Flat-earther Twitter Users
A) NO CHANGE
B) In addition,
United States
United Kingdom C) In its prime,
Canada
India
Japan D) Then,
Australia
Indonesia
South Africa
New Zealand
France
0% 10% 20% 30% 40% 50%
22
Twitter Average Flat Earthers Size of Difference Which choice provides an accurate interpretation of
the graph?
A) worldwide, primarily from non-English-
speaking countries.
B) worldwide, with members evenly spread across
nationalities.
C) worldwide, with the lowest percentage of
members in Australia.
D) worldwide, though a greater percentage of those
members live in the United States than anyplace
else.

CONTINUE
314  |  © TPR Education IP Holdings, LLC
FULL-LENGTH PRACTICE PASSAGES

Questions 23–33 are based on the following passage. 23

READING
Which choice most effectively combines the
Passage 7: The Compassionism Model sentences at the underlined portion?
There’s a new economic system in 23 town. A) town, called
It is called “compassionism.” Compassionism is a B) town; they call it
combination of capitalism and compassion. It has C) town, which is

WRITING AND
LANGUAGE
emerged from the idea of for-profit charities. Unlike D) town:
traditional nonprofit charities, 24 who raise money
for charitable causes and retain only enough profit
24
to maintain their operations, for-profit charities sell
A) NO CHANGE
consumer goods with the primary goal of making a
B) which raise money
profit. A secondary goal of such organizations is to
C) who donate money

MATH
contribute a portion of that profit to a charitable cause.
D) that are raising money
A good example of compassionism 25 is: the shoe
company, TOMS. TOMS manufactures shoes that are
sold in developed countries. Although it is a for-profit 25
company, it uses a portion of its revenue to manufacture A) NO CHANGE
and distribute shoes to people in developing countries. B) is, the shoe company,
The promise that, for every pair of shoes purchased, a C) is the shoe company
pair will be donated to a person in need is an integral D) is the shoe company,
part of the company’s marketing plan. Although the
company is upfront about its for-profit approach,
26
26 they focus their messaging heavily on the charitable
A) NO CHANGE
aspect of its business. When a customer signs up for the
B) they focus the
TOMS newsletter, he is asked to check a box that says,
C) it focuses their
“I love companies that give back and use business to
D) it focuses its
improve lives.”

CONTINUE
© TPR Education IP Holdings, LLC  |  315
SAT MANUAL

The management of TOMS argues that the for- 27


READING

profit business model is the most sensible way to create A) NO CHANGE


and sustain charitable giving. In the traditional model B) a volatile, unpredictable future
of nonprofit charities, organizations are primarily C) a volatile future
dependent upon donated funds. This creates D) an unknown future that might be volatile
27 a volatile and uncertain future for such
WRITING AND
LANGUAGE

organizations. It also makes it very difficult to attract


28
and retain valuable employees in the long term. With
A) NO CHANGE
the compassionism model, 28 however, the companies
B) yet,
are profit-based and therefore are not reliant on outside
C) moreover,
funding to operate. Since a for-profit business is more
D) for example,
sustainable than a nonprofit charity, 29 it is better.
MATH

And, of course, it’s hard to argue with anyone, capitalists


or otherwise, attempting to do something for 30 more 29
fortunate people in the world. Which choice most effectively completes the
explanation of the benefits of a for-profit charity
model?
A) NO CHANGE
B) a for-profit company is better able to do more
for more people.
C) a for-profit company is different from a charity.
D) it makes more of a profit than a charity does.

30
A) NO CHANGE
B) less fortunate
C) less unfortunate
D) more prosperous

CONTINUE
316  |  © TPR Education IP Holdings, LLC
FULL-LENGTH PRACTICE PASSAGES

31 That’s not to say there aren’t arguments against 31

READING
compassionism. Many view compassionism as not only a Which choice is the best introduction to the
flawed system but also a hypocritical one. If a company’s paragraph?
marketing plan is to make consumers feel they are doing A) NO CHANGE

good by purchasing a product, it would be difficult to B) But it’s not always good to do things for the less
fortunate.
move away from that plan at any point. That means that,

WRITING AND
C) However, there is a lot more that can be done

LANGUAGE
32 in order to be successful, in the long term, a outside of compassionism.
company like TOMS needs a poor population to D) Yet the model of compassionism may not be as
donate its products to in order to sell its products to the profitable as was first thought.
more advantaged. Many think it would be a 33 most
charitable and least selfish approach for companies to
32
donate money directly to populations in need rather

MATH
A) NO CHANGE
than donating products that generate profits for those
B) in order, to be successful in the long term
companies elsewhere in the world.
C) in order, to be successful, in the long term,
D) in order to be successful in the long term,

33
A) NO CHANGE
B) more charitable and less selfish
C) more charitable and lesser selfish
D) most charitable and less selfish

CONTINUE
© TPR Education IP Holdings, LLC  |  317
SAT MANUAL

Questions 34–44 are based on the following passage. 34


READING

A) NO CHANGE
Passage 8: A Plethora of Senses
B) In fact,
Ask any grade school child how many senses
C) Therefore,
humans have, and the answer will be five. This belief is
D) Unfortunately,
so widespread that we refer to people with heightened
WRITING AND
LANGUAGE

or odd abilities as having a sixth sense. 34 For instance,


there are many more than five or six human senses; there 35

could be as many as twenty-one. A) NO CHANGE


The five senses were first established by Aristotle in B) he
his work De Anima, 35 which he attempted to describe C) in which he

all forms of perception: seeing, hearing, touching, D) in that book he


MATH

tasting, and smelling. His grasp of how each of these


senses was experienced was understandably limited by 36
a lack of knowledge of human biology, but the fact that A) NO CHANGE
these five senses 36 are considered the sum of human B) will be considered as
physical experience for centuries speaks to the brilliance C) being considered
of Aristotle’s scientific observation. With our modern- D) were considered
day understanding of physiology, however, we now
37 know that humans, experience sensation in many
37
ways. In addition to Aristotle’s five senses, it is generally
A) NO CHANGE
agreed that humans have four additional senses:
B) know that humans experience sensation
nociception, thermoception, equilibrioception, and
C) know that, humans experience sensation
proprioception.
D) know that humans, experience sensation,

CONTINUE
318  |  © TPR Education IP Holdings, LLC
FULL-LENGTH PRACTICE PASSAGES

Nociception is the sense of pain felt in skin, joints, 38

READING
and body organs. 38 It is named after a nociceptor, A) NO CHANGE
the nerve cell receptors that send signals to the spinal B) They are named after nociceptors,
cord and brain when the body experiences potentially C) It is named after nociceptors,
damaging stimuli. The brain then decides whether the D) It is named after the nociceptor,
body is in danger, and if so, sends pain signals to attract

WRITING AND
LANGUAGE
attention to the danger. Thermoreception, which refers
39
to the 39 sense, of heat on skin, acts in a manner
A) NO CHANGE
similar to nociception. Thermorceceptors are sensory
B) sense of heat on skin,
receptors that react to changes in temperature. Unlike
C) sense, of heat on skin
nociceptors, thermoreceptors do not require physical
D) sense of heat on skin
contact with a heat source in order to be stimulated,

MATH
which is why thermoception and nociception are
considered distinct senses.

CONTINUE
© TPR Education IP Holdings, LLC  |  319
SAT MANUAL

40 Equilibrioception is the sense of balance. 40


READING

Equilibrioception is controlled by fluid-containing Which choice most effectively combines the


cavities of the inner ear. Without the ability to sense underlined sentences?
A) Equilibrioception is the sense of balance, being
balance, the human body would be unable to perform
controlled by those cavities of the inner ear that
basic actions such as running, walking, or even standing. contain fluid.
The sense of body awareness, or proprioception, is B) Equilibrioception is the sense of balance; this
WRITING AND
LANGUAGE

closely connected to equilibrioception. 41 Body sense is controlled by fluid-containing cavities


of the inner ear.
awareness means that humans know where each body
C) Equilibrioception is the sense of balance, which
part is in relation to another without the benefit of any
is controlled by fluid-containing cavities of the
other sense, such as seeing. For example, when walking, inner ear.
42 the subconscious knows where each human foot, D) Equilibrioception is the sense of balance; it is
leg, and arm is in relation to the others. This enables a controlled by fluid contained in some cavities of
MATH

the inner ear.


person to walk fluidly without conscious effort.

41
The writer is considering deleting the underlined
sentence. Should the sentence be kept or deleted?
A) Kept, because it gives additional information
that clarifies what proprioception does.
B) Kept, because without it there are only three
new senses mentioned, not four.
C) Deleted, because the example that comes after it
clearly demonstrates how proprioception works.
D) Deleted, because it blurs the paragraph’s focus
on the sense of equilibrioception.

42
A) NO CHANGE
B) each human foot, leg, and arm knows where it
is
C) the subconscious knows where feet, legs, and
arms are
D) humans know where each foot, leg, and arm is

CONTINUE
320  |  © TPR Education IP Holdings, LLC
FULL-LENGTH PRACTICE PASSAGES

Many other senses have been proposed that are 43

READING
not yet as widely acknowledged. Some scientists wonder A) NO CHANGE
whether chronoception, the sense of the passing of B) should be considered a sense.
time, 43 should be considered to be a sense. Others C) should be considered as being a sense?
argue that the senses of hunger and thirst and the D) should be considered a sense?
opposing sense of satiation are also distinct senses. As

WRITING AND
LANGUAGE
our understanding of the human body grows deeper, the
44
number of senses may continue to be 44 devalued.
A) NO CHANGE
B) revitalized.
C) expanded.
D) embellished.

MATH

CONTINUE
© TPR Education IP Holdings, LLC  |  321
SAT MANUAL

Questions 1–11 are based on the following passage. 1


READING

A) NO CHANGE
Passage 9: The Ethics of Photojournalism
B) Due to the
In the midst of the digital age, the field of
C) Because the
photojournalism, a form of reporting in which a
D) In that photojournalists have the
story about a particular time or place is told through
WRITING AND
LANGUAGE

photographic images, has expanded. 1 The ability to


distribute photos more easily than ever before through 2

the 24-hour news cycle and the internet, the demand for A) NO CHANGE
such images has increased. Photojournalism plays an B) in that it allows
important role in news media, 2 as it allows for and C) allowing

makes possible objective reflections of real events to be D) allowing as it does


MATH

conveyed to the public. It provides visual evidence to


enhance the written telling of a story. Photojournalism 3
helps make the realities of war, famine, and disease more At this point, the writer is considering adding the
immediate to people who don’t directly experience following sentence.
them. 3 We often think of disease and famine
as things that only happen in distant
countries, but they are prevalent in
many American cities, though often
well hidden from the view of many
citizens.
Should the writer make this addition here?
A) Yes, because it helps educate the reader about
problems of urban poverty.
B) Yes, because it reinforces the impact
photojournalism can have when combined with
print journalism.
C) No, because it distracts from the paragraph’s
emphasis on war as a social problem.
D) No, because it is not directly related
to the paragraph’s focus on the role of
photojournalism.

CONTINUE
322  |  © TPR Education IP Holdings, LLC
FULL-LENGTH PRACTICE PASSAGES

The same technological advances that 4 have 4

READING
been increasing the demand for photojournalism have A) NO CHANGE
also simplified the process of capturing the pictures the B) increases
public wants. Smaller and more precise cameras make C) have increased
working in areas of conflict easier, 5 but the ability to D) would have increased
send, print, or publish images in a much shorter time

WRITING AND
LANGUAGE
has enabled photojournalists to increase the time they
5
spend taking photographs.
A) NO CHANGE
However, the changes that have enhanced the field
B) however,
of photojournalism are also hurting it. Due to advances
C) for example,
in technology, it is not only easy to take and send
D) and
pictures but also to manipulate them. A photojournalist

MATH
is a journalist 6 their job is to relate events objectively,
as they actually occurred. Journalists of all types have 6
always struggled with this kind of objectivity. The A) NO CHANGE
unethical manipulation of news photographs used B) they should
to be limited to the practice of staging photographs. C) whose job is to
Now, 7 therefore, photographers have the ability to D) the job comes with the responsibility to
change actual photographs in ways that can be difficult,
or even impossible, to detect. An image can be easily 7
manipulated to either add or omit details, 8 which A) NO CHANGE
changes the way a photograph looks. B) however,
C) moreover,
D) for example,

8
Which choice most effectively supports the central
point of the paragraph?
A) NO CHANGE
B) which destroys the journalistic integrity of the
photograph.
C) depending on what the journalist wants to show.
D) a technique that is also used by artists working
with digital images.

CONTINUE
© TPR Education IP Holdings, LLC  |  323
SAT MANUAL

[1] Having time to consider the implications of 9


READING

publishing such photos helps a photojournalist make A) NO CHANGE


well-reasoned decisions about whether a particular B) In sum,
photograph contributes to a story or simply serves to C) On the other hand,
shock. [2] A conscientious photojournalist considers not D) Therefore,
only the integrity of her photographs but also the impact
WRITING AND
LANGUAGE

they will have. [3] 9 Many times, the images that a


10
photojournalist captures include sensitive or difficult
A) NO CHANGE
material. [4] However, the quick turnaround expected
B) public, and thus, provided by news outlets
by the 10 public, and thus provided by news outlets,
C) public and thus, provided by news outlets,
diminishes a photojournalist’s ability to determine
D) public, and thus provided by news outlets
the appropriateness of her photographs. [5] Media
MATH

consumers can support ethical photojournalism by


considering the same questions about whether images 11
are meant to educate or to shock and by supporting To make this paragraph most logical, sentence 1
should be placed
those publications and journalists whose work they
A) where it is now.
admire. 11
B) after sentence 2.
C) after sentence 3.
D) after sentence 4.

CONTINUE
324  |  © TPR Education IP Holdings, LLC
FULL-LENGTH PRACTICE PASSAGES

Questions 12–22 are based on the following passage. 12

READING
A) NO CHANGE
Passage 10: The Work of Upton Sinclair
B) published, in 1906,
When people think of Upton Sinclair, they usually
C) published: in 1906,
associate him with the book he is most famous for,
D) published in 1906,
The Jungle. They may consider him a vital player in

WRITING AND
LANGUAGE
establishing current food safety standards, but they fail
to recognize him as one of the most important shapers 13

of public policy in America in the early 20th century. Which choice most effectively combines the
sentences at the underlined portion?
Upton Sinclair was born in Baltimore, Maryland
A) “muckraking,”
in 1878. Although best known for The Jungle,
B) “muckraking,” which refers to
12 published in 1906 Sinclair published over 100
C) “muckraking”; the term refers to

MATH
books in his lifetime. The majority of them were written
D) “muckraking,” which is
in a style known as 13 “muckraking.” Muckraking
refers to a form of investigative writing that serves to
expose corruption, audit public offices, or 14 entertain 14
its readers. This type of writing 15 is very popular in Which choice is most consistent with the previous
America in the early 20th century, in both books and examples in the sentence?
A) NO CHANGE
newspapers.
B) monitor social ills.
C) showcase the author’s writing style.
D) bore high school students.

15
A) NO CHANGE
B) had been
C) was
D) will be

CONTINUE
© TPR Education IP Holdings, LLC  |  325
SAT MANUAL

In The Jungle, Sinclair wrote about the 16


READING

16 appalling conditions that shocked people in the A) NO CHANGE


American meatpacking industry in the early 1900s. B) appalling and shocking conditions
His goal was to expose the harsh working conditions C) conditions that shocked and appalled people
of the immigrant workers in the industry, but most D) appalling conditions
Americans were 17 less surprised about the unhygienic
WRITING AND
LANGUAGE

procedures of the meatpacking industry also revealed


17
in his book. The Jungle caused public outrage over the
Which choice results in a sentence that best
lack of sanitation and government oversight, which supports the point developed in this paragraph?
prompted Congress to pass the Pure Food Act and the A) NO CHANGE
Meat Inspection Act later that same year. B) more concerned
C) more complacent
MATH

D) less anxious

CONTINUE
326  |  © TPR Education IP Holdings, LLC
FULL-LENGTH PRACTICE PASSAGES

18 Including his commercial and financial 18

READING
success with The Jungle, Sinclair moved on to other areas A) NO CHANGE
of corruption. His second major novel, The Brass Check, B) Unlike
was published in 1919. In this work Sinclair focused C) After
on the unlawful limitations imposed on the free press D) Despite
and harshly criticized the sensation-based, rather than

WRITING AND
LANGUAGE
truth-based, approach to journalism 19 taken from 19
many American newspapers. Sinclair considered this A) NO CHANGE
book 20 the most important, and dangerous one he B) taken by
ever wrote, and it did have quite an impact. Within half C) taken to
a decade after the book was published, the first code of D) given up
ethics for journalism 21 were established.

MATH
[1] Sinclair’s interest in politics and public policy
20
went beyond his publishing career. [2] He established
the California chapter of the Americans Civil Liberties A) NO CHANGE

Union and was a political activist for the majority B) the most important
C) the most, important
of his life. [3] He was even arrested while protesting
D) the most important—
for industrial workers’ rights. [4] Despite these failed
attempts, Sinclair has arguably left a greater mark on
American public policy than many who have actually 21
served in public office. [5] He ran unsuccessfully A) NO CHANGE
for public office as a Socialist three times—once for B) was established.
the United States House of Representatives, once for C) have been established.
the United States Senate, and once for Governor of D) was being established.
California. 22

22
To make this paragraph most logical, sentence 4
should be placed
A) where it is now.
B) before sentence 1.
C) after sentence 2.
D) after sentence 5.

CONTINUE
© TPR Education IP Holdings, LLC  |  327
SAT MANUAL

Questions 23–33 are based on the following passage. 23


READING

A) NO CHANGE
Passage 11: The Pros and Cons of Urban Farming
B) feed the chickens, and pigs
Many people daydream about an idealized life on
C) feed the chickens and pigs,
a farm: waking every day to milk the cows, 23 feed
D) feed the chickens and, pigs,
the chickens, and pigs, and tend the garden. 24 Many
WRITING AND
LANGUAGE

city dwellers are beginning to make this dream a reality


through the practice of urban homesteading. Urban 24

homesteading is the practice of growing food and raising Which choice most effectively combines the
underlined sentences?
animals on one’s property for personal consumption.
A) Many city dwellers are beginning to make this
Urban homesteading has many 25 extras. First dream a reality through the practice of urban
of all, it’s local. As 26 humanities carbon footprint homesteading: this is the practice of growing
MATH

expands in part due to shipping food long distances, food and raising animals on one’s property for
personal consumption.
people are putting more effort into eating locally-
B) Many city dwellers are beginning to make
sourced foods. What could be more local than one’s this dream a reality through the practice
own backyard? Urban farmers know exactly where of urban homesteading; by way of doing
their food is coming from and reduce their personal this, city dwellers practice growing food and
raising animals on their property for personal
carbon footprints. Perhaps just as importantly, urban consumption.
homesteaders know just what is going into their food. C) Many city dwellers are beginning to make this
dream a reality through the practice of urban
homesteading; urban homesteading is the
practice of growing food and raising animals on
one’s property for personal consumption.
D) Many city dwellers are beginning to make
this dream a reality through the practice
of urban homesteading, growing food and
raising animals on one’s property for personal
consumption.

25
A) NO CHANGE
B) benefits.
C) benevolences.
D) goods.

26
A) NO CHANGE
B) humanity
C) humanities’
D) humanity’s
CONTINUE
328  |  © TPR Education IP Holdings, LLC
FULL-LENGTH PRACTICE PASSAGES

For people who are concerned about the increasing 27

READING
use of pesticides and insecticides in food production, A) NO CHANGE
avoiding the 27 use, of those products, in their own B) use of those products in their own gardens
gardens easily ends that worry. C) use of those products in their own gardens,
Both of these benefits still hold true for urban D) use, of those products, in their own gardens,
farmers who raise meat as well as vegetables. As the

WRITING AND
LANGUAGE
figure shows, 28 meat products have as great a carbon
28
footprint as all other food products combined.
Which choice makes the writer’s description of the
29 Consequently, commercially raised animals are figure most accurate?
given hormones and medications that, like pesticides A) NO CHANGE
and insecticides, some would prefer to avoid. By raising B) meat accounts for roughly two-thirds of the
animals, urban farmers can control exactly what they are total food production carbon footprint.

MATH
putting in so they know what they are getting out. C) white meat from chicken, pork, and fish is
particularly bad for the environment in terms of
carbon emissions.
Carbon Emissions by Food Type
D) beef and lamb create a carbon footprint four
Beef/Lamb times greater than that of vegetables and fruit
Chicken/Pork/Fish combined.
Dairy
Cereals/Breads
Vegetables 29
Fruits
Oils/Spreads
A) NO CHANGE
Snacks/Sugar B) Next,
Drinks
C) Moreover,
0 2 4 6 8 10 12 14 16
D) Subsequently,
Grams of carbon dioxide per
kilocalorie of food eaten

CONTINUE
© TPR Education IP Holdings, LLC  |  329
SAT MANUAL

30 For consumers concerned about chemicals in 30


READING

their food, a Community-Supported Agriculture (CSA) Which choice most effectively anticipates and
share is a good alternative to starting a garden. An addresses a relevant counterargument to the
argument in favor of urban homesteading
urban setting is likely to have pollutants present in the described in the passage?
air and/or the soil from either the past (such as lead A) NO CHANGE
contamination from lead-based paint) or the present
WRITING AND

B) Just deciding to start an organic garden won’t


LANGUAGE

31 (like smokestacks). While a farmer may purchase make it happen though—would-be urban
the best organic fertilizer on the market, it won’t make homesteaders need to be mindful that their
yards may already be contaminated with
his tomatoes organic if the soil is contaminated or chemicals that could make the food they grow
contains chemical fertilizer residue from 32 previous less safe than conventionally-farmed options.
years in the past. C) A potential urban homesteader should keep in
mind that using his yard as a farm will be much
These kinds of concerns shouldn’t necessarily
MATH

more labor intensive than just growing grass.


prevent anyone from trying urban homesteading,
D) Although many people imagine that farms
though. The more people who are paying attention to are idyllic and beautiful places, the reality is
the possible 33 presence of contaminants in their somewhat different, and urban homesteaders
should realize that their neighbors may object
neighborhoods, the more likely the contamination is
to the way their yards look.
to be remedied. A group of urban homesteaders who
provide their families and neighbors with fresh produce
and eggs could have the power to mobilize a city to 31
regulate factory emissions or provide environmental Which choice provides information that is most
consistent in style and content with the information
remediation. Although urban homesteading is not
about possible past contaminants?
without potential challenges, its benefits most likely
A) NO CHANGE
outweigh the drawbacks for those who are seriously
B) (such as polluting factories)
invested in it.
C) (for example, factory exhaust that contains
chemicals)
D) (such as noxious exhaust from a nearby factory)

32
A) NO CHANGE
B) time past in previous years.
C) previous years.
D) the actions of previous homeowners in years
past.

33
A) NO CHANGE
B) presents of
C) present’s for
D) presence to CONTINUE
330  |  © TPR Education IP Holdings, LLC
FULL-LENGTH PRACTICE PASSAGES

Questions 34–44 are based on the following passage. 34

READING
A) NO CHANGE
Passage 12: The Final Mosquito Solution
B) example: despite
It is a great paradox that humanity is so efficient
C) example; despite,
at destroying the organisms it wants and needs but
D) example, despite,
so inefficient at destroying organisms it doesn’t want.

WRITING AND
LANGUAGE
Mosquitoes are a prime 34 example, despite our deep-
seated dislike of the annoying pests, we are just now 35

beginning to find a way to meaningfully 35 deal with A) NO CHANGE


them. However, if history has taught us anything, it’s B) operate
that we should carefully consider the potential negative C) control

consequences of extermination, even for an organism as D) allocate

MATH
widely considered a pest as the mosquito.
There are over 3,000 species of mosquitoes, 36
approximately 36 200 bite humans. These pests A) NO CHANGE
wreak havoc on humanity, killing one person every 12 B) 200 of them
seconds through disease transmission. Mosquitoes can C) 200 of which
transmit more than 100 fatal diseases, including malaria, D) 200 are ones that
yellow fever, encephalitis, elephantiasis, and West Nile
virus. Malaria alone kills one million people a year
37
globally. Because of this threat, public health officials
A) NO CHANGE
have been trying to kill mosquitoes for quite some time.
B) are
Historically, the preferred methods of control 37 was
C) had been
harsh chemicals and pesticides. Although these methods
D) were
were effective, they caused such severe health and
environmental issues that they are now banned in most
countries.

CONTINUE
© TPR Education IP Holdings, LLC  |  331
SAT MANUAL

Development of other, less toxic mosquito control 38


READING

methods is underway. The first patent was recently A) NO CHANGE


granted for RNA interference (RNAi) pesticides. B) sequins to effect
These pesticides consist of RNA strands that are C) sequins to affect
38 sequenced to effect a particular mosquito species. D) sequenced to affect
When the targeted species ingests the right RNAi, its
WRITING AND
LANGUAGE

genes are silenced 39 . Think of it as a computer system


39
that internally self-destructs when supplied with the
At this point, the writer is considering adding the
proper code. This technology has so far been focused following.
—in other words, the species’ genetic
code is literally disabled
Should the writer make this addition here?
MATH

A) Yes, because it clarifies what RNAi does when


used as a pesticide.
B) Yes, because it emphasizes the ethical concerns
raised by the use of RNAi.
C) No, because understanding how RNAi works
distracts from the paragraph’s focus on which
species of mosquito are most deadly.
D) No, because it mischaracterizes scientists’
attitudes towards any life that is not human.

CONTINUE
332  |  © TPR Education IP Holdings, LLC
FULL-LENGTH PRACTICE PASSAGES

on the mosquito species that carry the deadliest diseases, 40

READING
including Anopheles, Culex, and Aedes, but it could easily At this point, the writer is considering adding the
be adapted for widespread application. 40 following graphic.

Principal Characters for Identifying


Mosquitoes of General Importance
ANOPHELES AEDES CULEX

WRITING AND
LANGUAGE
EGGS

Laid singly Has floats Laid singly No floats Laid in rafts No floats
LARVAE Rest at an angle
Air tube

Rest parallel to water surface


No air tube Air tube long and
Head rotated 180° Air tube short and
stout with one pair slender with several
when feeding of hair tufts pairs of hair tufts

MATH
PUPAE

Pupae differ slightly


ADULTS
Maxillary palps Maxillary palp
as long as proboscis shorter than proboscis

Wings
spotted Wings clear

Abdominal tip Abdominal tip


pointed rounded

Proboscis and Proboscis and body in two axes


body in one axis

Should the writer make this addition here?


A) Yes, because it shows the physical differences
between the mosquito species mentioned in the
passage.
B) Yes, because it could help the reader identify
which mosquito species are most likely to carry
disease.
C) No, because identifying mosquito species is
not directly related to the paragraph’s focus on
mosquito control.
D) No, because most of the most dangerous
mosquito species are not found in the United
States.

CONTINUE
© TPR Education IP Holdings, LLC  |  333
SAT MANUAL

We must carefully consider, 41 however, whether 41


READING

widespread eradication of mosquitoes is a good idea. A) NO CHANGE


The main concern with the extinction of any species is B) but
the effect its absence will have on its ecosystem. There is C) although
a fierce divide 42 between biologists over what kind of D) therefore,
effect mosquito extinction might have. Many proponents
WRITING AND
LANGUAGE

of the plan argue that mosquitoes do not fill a vital niche


42
in most ecosystems, or at least not one that couldn’t
A) NO CHANGE
easily be filled by another species. But many others
B) in
disagree. For example, a study published in the Journal
C) among
of Applied Ecology reported that French house martins,
D) through
which 43 depend on mosquitoes as their primary food
MATH

source, laid two fewer eggs on average when mosquitoes


were removed from their environment. The bigger point 43

opponents of mosquito eradication make is, even if there A) NO CHANGE


is no substantial proof, you can never really know how B) depend, on mosquitoes, as their primary food
source,
much harm will be caused by removing a species
C) depend on mosquitoes as their primary food
44 and making it go extinct until it is too late. source

D) depend on mosquitoes, as their primary food
source,

44
A) NO CHANGE
B) thereby driving it to extinction
C) and getting rid of it altogether
D) DELETE the underlined portion.

CONTINUE
334  |  © TPR Education IP Holdings, LLC
FULL-LENGTH PRACTICE PASSAGES

Questions 1–11 are based on the following passage. 1

READING
Which choice most effectively sets up a contrast in
Passage 13: Transportation Engineers the sentence?
America has a lot of people living in cities. Its A) NO CHANGE
urban residential rate is a whopping 81%, 1 which is B) meaning that roughly 8 out of every 10 people
relatively high. It is estimated that for every 1,000 urban live in a city.

WRITING AND
C) although that can be attributed to America’s

LANGUAGE
dwellers there are 685 urban-dwelling drivers, a rate that
status as a developed nation.
has risen considerably in the last 50 years. That’s a lot
D) as compared to a worldwide rate of 54%.
of cars. In order to accommodate the growing number
of drivers in American cities, state Departments of
Transportation are hiring more and more transportation 2
engineers. Transportation engineers apply 2 scientific A) NO CHANGE

MATH
principals and new technologies to the planning, B) principally scientific
operation, and management of transportation. Their C) scientific principles
goal is to make transportation as convenient, efficient, D) the principles of transportation science
and affordable for both citizens and governments as
possible. Transportation engineers hired for the purpose
3
of addressing traffic and roadway 3 congestion, are
A) NO CHANGE
generally specialists in highway engineering. Highway
B) congestion
specialists must be able to do a great many things with
C) congestion;
not a lot of money. 4
D) congestion—

4
At this point, the author is considering adding the
following sentence.
Working within a budget is a skill that
more engineering schools should focus
on as an essential job skill.
Should the writer make this addition here?
A) Yes, because it makes an important point that
the reader is unlikely to already know.
B) Yes, because working within a budget is
an essential skill in many fields, not just
engineering.
C) No, because it repeats information given earlier
in the paragraph.
D) No, because it blurs the paragraph’s focus on
the increasing importance of transportation
engineers.

CONTINUE
© TPR Education IP Holdings, LLC  |  335
SAT MANUAL

The first step in updating a highway is to survey 5


READING

the environment for factors that influence the road’s A) NO CHANGE


fundamental design. The height, width, shape, and B) weather, patterns, types of natural disasters, and
direction of highways are all determined by what fits physical
best in a specific environment. Elevation, common C) weather patterns, types of natural disasters, and,
physical
5 weather patterns, types of natural disasters, and
WRITING AND

D) weather patterns, types of natural disasters, and


LANGUAGE

physical obstacles are the true designers of our highways. physical,


Along with considering how the environment
will shape the highway, transportation engineers
6
6 will be considering how the highway will shape
A) NO CHANGE
the environment. 7 Accordingly, they examine the
B) considering
impacts the highway may have on things like air, water,
MATH

C) consider
and noise pollution. Additionally, we want our highways
D) have considered
to be aesthetically pleasing. 8
Highway engineers must also be able to make
7
accurate predictions. It is important when designing a
highway to base the design not only on current needs A) NO CHANGE

but also on anticipated future needs. Most highways are B) In conclusion,

built with an 9 intended life span of at least 50 years C) For starters,

or more, assuming they are properly maintained. D) However,

8
The writer is considering deleting the previous
sentence. Should the sentence be kept or deleted?
A) Kept, because it emphasizes an important value
in American culture.
B) Kept, because it demonstrates the versatile skills
necessary to be able to design highways the
public will accept.
C) Deleted, because it implies the American public
is overly concerned with trivial matters.
D) Deleted, because it detracts attention from
the primary considerations of transportation
engineers.

9
A) NO CHANGE
B) intended lifespan of 50 years
C) intended lifespan that should last 50 years
D) minimum intended lifespan of 50 years

CONTINUE
336  |  © TPR Education IP Holdings, LLC
FULL-LENGTH PRACTICE PASSAGES

An additional consideration that highway engineers 10

READING
must always take into account is safety. Highways are A) NO CHANGE
far and away the leading location of injury and death in B) in others
developed nations. More people are injured or hurt in C) by other types
highway accidents than 10 others in the United States. D) in other types of accidents
Transportation engineers have a professional and ethical

WRITING AND
LANGUAGE
obligation to design our systems of transportation to be
11
as safe as they possibly can be.
Which choice most effectively sets up the
Transportation engineers must incorporate all these information that follows?
considerations in highway design while also being good A) NO CHANGE
stewards of taxpayers’ money. A highway 11 should be B) is one of the few public places that is used by
a pleasure to drive on. It must be easy to access, navigate, nearly every person in a society.

MATH
and maintain, and it must be all that for the smallest tax- C) is often the fastest way to get from place to place.
funded amount possible. D) can contribute an important element to the
visual impression of a city.

CONTINUE
© TPR Education IP Holdings, LLC  |  337
SAT MANUAL

Questions 12–22 are based on the following passage. 12


READING

Which choice provides the most effective transition


Passage 14: Behind Every Good Movement… between ideas in the paragraph?
The Civil Rights Movement is usually presented in A) NO CHANGE
history classes as a united effort, a single great push by B) Not everyone in the Civil Rights Movement
the disenfranchised all working in harmony. But this shared the same ideologies, and some were
more peacefully inclined than others.
WRITING AND
LANGUAGE

is an oversimplified vision of what actually occurred.


C) Geography played a major role in how
Within the larger movement there were many factions, much people contributed to the Civil Rights
all working towards the same overall goal but in different Movement.
ways and to different extents. 12 One factor that D) Not everyone was in favor of rocking the
determined the extent to which people could contribute political boat.
to the Civil Rights Movement was gender. Women
MATH

generally did not hold major public positions in the Civil 13


Rights Movement. Even though women suffered the A) NO CHANGE
same injustices that 13 they’re fathers, husbands, and B) the
brothers did, they mostly worked in behind-the-scenes C) there
and limited roles when it came to addressing those D) their
indignities.
One unsung hero of the Civil Rights Movement
14
was Dorothy Height. Born in Virginia and raised
A) NO CHANGE
in Pennsylvania, Height was an exceptional student.
B) Height was hired by the New York City Welfare
Earning a master’s degree in educational psychology
Department as a case worker.
from New York University, 14 the New York City
C) case worker was the position Height was given
Welfare Department hired Height as a case worker. She at the New York City Welfare Department.
turned her focus from social work to political activism D) her career began as case worker at the New York
15 when joining the National Council of Negro City Welfare Department.
Women in 1937 at the age of 25. She went on to serve as
that organization’s president 16 for forty years, from 15
1957 to 1997. A) NO CHANGE
B) having joined
C) joining
D) when she joined

16
A) NO CHANGE
B) for the forty years from 1957 to 1997.
C) from 1957 to 1997, a period of forty years.
D) from 1957 to 1997.

CONTINUE
338  |  © TPR Education IP Holdings, LLC
FULL-LENGTH PRACTICE PASSAGES

Height took on a major behind-the-scenes role 17

READING
in the Civil Rights Movement, specifically focusing on Which choice most effectively sets up the examples
17 the issue of voting rights. She wrote a column that follow in the next two sentences?
titled “A Woman’s Word” for a weekly African-American A) NO CHANGE
newspaper in New York. She instituted a tradition called B) supporting people who were dedicated to
nonviolent protest.
“Wednesdays in Mississippi,” a meeting of prominent

WRITING AND
C) the ways women could contribute to the

LANGUAGE
black and white women from both sides of the Mason- movement.
Dixon 18 line. These meetings were designed to build D) ending racial segregation.
positive relationships and create constructive dialogues.
19
18
Which choice most effectively combines the

MATH
sentences at the underlined portion?
A) line; these meetings were
B) line: the meetings were
C) line,
D) line, which were

19
At this point, the writer is considering adding the
following sentence.
In the decades that followed the Civil
Rights Movement, Height served on
three presidential committees and was
awarded the Medal of Freedom.
Should the writer make this addition here?
A) Yes, because it clarifies the relationship between
Height and the president of the United States.
B) Yes, because it adds additional details about
Height that further clarify the impact she had.
C) No, because it repeats information given earlier
in the paragraph.
D) No, because it blurs the paragraph’s focus on
African-American newspapers.

CONTINUE
© TPR Education IP Holdings, LLC  |  339
SAT MANUAL

Height was often frustrated by the lack of a public 20


READING

voice she and other women in the movement were A) NO CHANGE


afforded. One of the primary examples of this was the B) but they were not speaking.
exclusion of female speakers from the pivotal March on C) but no speaking role.
Washington in 1963. When Height made clear this was D) but no opportunities for speaking.
unacceptable to the women of the movement, she was
WRITING AND
LANGUAGE

offered a seat on the platform 20 but no speaking roles.


21
[1] Height is just one example of many women
A) NO CHANGE
within the movement who were not given their due.
B) Rather,
[2] We must not only stand up against the visible
C) Nevertheless,
inequality around us; we must fight the unconscious
D) Therefore,
tendencies towards inequality we carry within ourselves.
MATH

[3] Her story should not be told to scold or shame those


who were responsible for keeping her behind the scenes. 22
[4] 21 For instance, we should take her story and those To make the paragraph most logical, sentence 2
should be placed
like it as a call for continued action. 22
A) where it is now.
B) before sentence 1.
C) after sentence 3.
D) after sentence 4.

CONTINUE
340  |  © TPR Education IP Holdings, LLC
FULL-LENGTH PRACTICE PASSAGES

Questions 23–33 are based on the following passage. 23

READING
A) NO CHANGE
Passage 15: America’s Love Affair with Cars
B) the Model T rolled, off
America has a love affair with cars, or so the saying
C) the Model T, rolled off,
goes. It has now been over one hundred years since the
D) the Model T rolled off,
first automobile made for mass consumption,

WRITING AND
LANGUAGE
23 the Model T, rolled off the Ford manufacturing
floor. For a piece of machinery that was supposedly built 24

to serve humans, we have dedicated quite a bit of time, A) NO CHANGE


money, and 24 a lot of effort to accommodating the B) quite a bit of effort
automobile. C) also effort

When the automobile first made an appearance in D) effort

MATH
American society in 1908, society was not particularly
well prepared. Although cities and towns were built to 25
accommodate non-mechanized transportation, the car At this point, the writer is considering adding the
was something completely new. 25 In the majority of following sentence.
urban settings, streets were not paved. Intersections did Cars needed different types of
infrastructure than horses and wagons
not have any standardized system of right of 26 way. did to keep both their drivers and
The traffic before cars came along was not heavy or fast pedestrians safe.
enough to necessitate such a system. All that changed Should the writer make this addition here?
quickly, however, in the years after the Model T was A) Yes, because it provides an important
introduced. counterpoint to the first part of the paragraph.
B) Yes, because it gives further details about an
idea presented in the previous sentence.
C) No, because it is not directly related to the
paragraph’s main focus on the state of American
society in 1908.
D) No, because it repeats details given in the first
paragraph.

26
Which choice most effectively combines the
sentences at the underlined portion?
A) way; the traffic before
B) way, since traffic
C) way, being that the pre-car traffic
D) way, although traffic

CONTINUE
© TPR Education IP Holdings, LLC  |  341
SAT MANUAL

[1] The popularity of cars was immediate; within 27


READING

twenty years of the production of the first Model T, A) NO CHANGE


household ownership of automobiles increased from B) front runner
essentially zero to forty-five percent of the American C) forerunner
public. [2] This local road-building was the D) leader
27 winner of the construction of the interstate highway
WRITING AND
LANGUAGE

system, which began another 20 years later. [3] With a


28
percentage that big—and still growing—changes to the
A) NO CHANGE
infrastructure of roads and cities 28 was necessary.
B) would have been
[4] Cities sprang to action by expanding roads and
C) would be
paving them. [5] Thanks in part to all of this road
D) were
building, by 2007 there was a historical high of 843 cars
MATH

on the road for every 1,000 Americans. 29


29
To make this paragraph most logical, sentence 2
should be placed
A) where it is now.
B) after sentence 3.
C) after sentence 4.
D) after sentence 5.

CONTINUE
342  |  © TPR Education IP Holdings, LLC
FULL-LENGTH PRACTICE PASSAGES

30 By 2012, the number was down to 789 cars 30

READING
on the road for every 1,000 Americans. Increasingly, Which choice best establishes the argument that
cities are moving their efforts away from implementing follows?
A) However, that trend appears to be changing.
infrastructure changes to accommodate cars and
B) There does not appear to be an end in sight to
focusing instead 31 in improving mass transit. No
the increasing number of cars on our roads.
one argues that America never had a love affair with

WRITING AND
C) One of the newest changes in car technology is

LANGUAGE
cars. But the very nature of that love affair makes it electric and hybrid cars.
hard to tell whether it is really still 32 going on or D) The infrastructures of cities have changed in a
happening: because of the infrastructure we created to variety of ways over that time period, not just
for cars.
accommodate cars, we are basically stuck with them. For
this reason, some scholars claim that the love affair is
waning faster than the data would lead us to believe. As 31

MATH
historian Peter Norton has put it, if you locked someone A) NO CHANGE
in a 7-Eleven for a week, used that week as a window B) by
into her preferred diet, and then came to the conclusion C) on
that she preferred processed food to fresh food, your D) from
data may be a little flawed. 33

32
A) NO CHANGE
B) going on in the present day:
C) continuously happening:
D) going on:

33
At this point, the writer wants to further explain
the connection of the 7-Eleven example to the
main idea of the paragraph. Which choice most
effectively accomplishes this goal?
A) Just because people drive doesn’t mean they
have to like it.
B) Likewise, it may be that Americans, rather than
being in love with cars, are just trapped in an
infrastructure created by a car-loving society.
C) The fact that it is so easy to obtain processed
food at fast food restaurant drive-up windows
when driving in a car could also skew a person’s
preferences.
D) People need to have a variety of choices in order
to thrive, and a car-dependent society may not
effectively give that to us.

CONTINUE
© TPR Education IP Holdings, LLC  |  343
SAT MANUAL

Questions 34–44 are based on the following passage. 34


READING

A) NO CHANGE
 assage 16: There May Not Be Life on Mars, But at
P
B) as well as
Least There’s Gas
C) along with
In 2013, NASA announced that its most recent
D) and
Mars rover, the Curiosity, had measured surprising levels
WRITING AND

of methane gas on Mars. Methane gas is a chemical


LANGUAGE

compound made from one atom of carbon 34 and 35


also from another four atoms of hydrogen. On Earth, A) NO CHANGE
methane is commonly used as an energy source and is B) was recording
the main component of natural gas. Curiosity C) recorded
35 records levels of methane gas between 5.5 and 7 D) will have recorded
MATH

parts per billion on Mars. This was very exciting news


for the scientific community, as previous measurement
36
indicated no meaningful presence 36 of, methane gas
A) NO CHANGE
on Mars, only 0.7 parts per billion.
B) of methane gas on Mars, only 0.7 parts per
billion.
C) of methane gas, on Mars, only 0.7 parts per
billion.
D) of, methane gas on Mars only 0.7 parts per
billion.

CONTINUE
344  |  © TPR Education IP Holdings, LLC
FULL-LENGTH PRACTICE PASSAGES

37 Methane can be made from inorganic 37

READING
chemical processes, generally it is made by living A) NO CHANGE
microbes. These microbes, known as methanogens, B) Nevertheless, methane
produce methane through a process of anaerobic C) Because methane
respiration, 38 where respiration occurs without D) Although methane
oxygen. This process is known as methanogenesis. The

WRITING AND
LANGUAGE
reason the discovery of methane on Mars is 39 cool is
38
that living microbes, in the past or present, could explain
A) NO CHANGE
the presence of methane. Though they are not exactly
B) in which
the walking, talking, green-tinted aliens that the movies
C) when
may have led us to expect, microbes are a form of life.
D) by means of which
Their presence on Mars would firmly 40 establish the

MATH
existence of life on Mars, something scientists have been
39
Which choice best maintains the style and tone of
the passage?
A) NO CHANGE
B) no big deal
C) significant
D) boss

40
A) NO CHANGE
B) install
C) authorize
D) build

CONTINUE
© TPR Education IP Holdings, LLC  |  345
SAT MANUAL

trying to prove for decades. 41 For these scientists, 41


READING

what might be a clear indication of life on Mars is a The writer wants a conclusion to the paragraph
cause for celebration. that logically completes the discussion of the
possibility of life on Mars and provides an effective
[1] Other scientists are quick to remind them, transition to the next paragraph. Which choice best
however, that Curiosity’s methane measurements do not accomplishes these goals?
necessarily prove the existence of life on Mars. [2] There A) NO CHANGE
WRITING AND
LANGUAGE

are other ways in which methane can be produced that B) It is now only a matter of time until we find
we know about. [3] It is also entirely possible that, due to evidence of the larger life forms we have been
imagining on Mars.
differences between the atmosphere of Earth and
C) Without the proof of methane, it is likely
42 Mars, methane is or was produced on Mars by a scientists would have abandoned their pursuit
process completely unknown to us. 43 of life on Mars.
One thing that all scientists agree on is that the D) Now that evidence of life on Mars has been
MATH

found, we are one step closer to establishing a


presence of methane on the surface of Mars is significant,
human colony on the planet.
even if 44 their not sure why. Additional information
about the red planet will enhance our knowledge of its
present state, supplement our guesses about its past, and 42
inform our plans for its possible use in the future. A) NO CHANGE
B) that of Mars,
C) the planet Mars,
D) the surface of Mars,

43
To improve the cohesion and flow of this paragraph,
the writer wants to add the following sentence.
For example, methane can also be
produced by chemical reactions.
The sentence would most logically be placed
A) before sentence 1.
B) after sentence 1.
C) after sentence 2.
D) after sentence 3.

44
A) NO CHANGE
B) there
C) they’re
D) they aren’t

CONTINUE
346  |  © TPR Education IP Holdings, LLC
FULL-LENGTH PRACTICE PASSAGES

Questions 1–11 are based on the following passage. 1

READING
A) NO CHANGE
Passage 17: Physical Therapy
B) bath houses and steam rooms,
The field of physical therapy is probably the oldest
C) bath houses and steam rooms
medical field still in practice today. While little was
D) bath houses, and steam rooms
known in ancient times about the internal organs and

WRITING AND
LANGUAGE
nothing was known of bacteria, viruses, and diseases,
many ancient people took a keen interest in the health of 2

limbs, muscles, and joints. Evidence of such interest can At this point, the writer is considering adding the
following sentence.
be found in the 1 bath houses, and steam rooms, of
The ancient Greeks were limited in
both ancient Greece and the Roman Empire. 2 their knowledge of body systems
[1] Modern physical therapy began in earnest after because they believed that it was

MATH
immoral to dissect a human body.
the polio epidemic of the early twentieth century. [2] The
Army was the first to train and employ “reconstruction Should the writer make this addition here?
aides” to rehabilitate soldiers. [3] Large portions of A) Yes, because it demonstrates why physical
therapy is an older form of medicine than
the population were affected by the disease and others.
3 will require aid in relearning how to use muscles B) Yes, because it contributes to the reader’s
that had atrophied or deformed. [4] This epidemic was understanding of the history of medicine.
quickly followed by World War I, which resulted in C) No, because it is too gruesome a fact to include
in an article written for young people.
many returning soldiers who required the same kind
D) No, because it is not directly related to the main
of specialized medical care. [5] Thus was born modern
point of the paragraph.
physical therapy. 4

3
A) NO CHANGE
B) were requiring
C) required
D) requires

4
To make this paragraph most logical, sentence 2
should be
A) placed before sentence 1.
B) placed before sentence 4.
C) placed before sentence 5.
D) DELETED from the paragraph.

CONTINUE
© TPR Education IP Holdings, LLC  |  347
SAT MANUAL

Today, physical therapists are experts 5 and the 5


READING

professional authority on rehabilitation and physical A) NO CHANGE


management of injuries and chronic conditions. Physical B) and the professional authorities
therapists typically work directly with patients in C) and professional authority
private clinics, hospitals, or nursing facilities. Some of D) DELETE the underlined portion.
the specializations within the field of physical therapy
WRITING AND
LANGUAGE

include geriatric and neuromuscular.


6
Physical therapy is a growing field. It is estimated
A) NO CHANGE
that from 2012 to 2022 the demand for physical
B) 36%, and that is significantly
therapists will grow by 6 36%, this is significantly
C) 36%, but this is significantly
higher than the expected average growth for all
D) 36%, significantly
professions. With that level of projected growth, physical
MATH

therapy is the perfect field for someone who has an


interest in the medical field but doesn’t want to spend 7

the time and money required to become a physician. The A) NO CHANGE


growing 7 request for physical therapy is attributed B) demand
to the large number of baby boomers who have begun C) desire
to enter the older demographic, those aged 65 and older. D) want
As of 2009, there were 39.6 million older Americans. 8
Baby boomers are not just living longer than previous 8
generations did, 9 and they are staying more active. At this point, the writer is considering adding the
This puts them at a much greater risk of injury and following sentence.
chronic conditions and therefore in need of physical By 2030, that number is projected
to nearly double, to 72 million older
therapy.
Americans.

Number of Americans Aged 65 and Older Should the writer make this addition here?
80 A) Yes, because it addresses a possible counterclaim
72.1
to the writer’s argument.
Number of Americans

60 54.8
B) Yes, because it provides additional support for
the main point of the paragraph.
(in millions)

40.3 C) No, because it is not directly relevant to the


40 35.0
31.2 main focus of the paragraph.
25.5
20 16.6
D) No, because it provides an inaccurate
9.0 interpretation of the graph.
3.1 4.9
0
1890 1910 1930 1950 1970 1990 2000 2010 2020 2030
Year 9
A) NO CHANGE
B) and
C) they are also
D) but they also
CONTINUE
348  |  © TPR Education IP Holdings, LLC
FULL-LENGTH PRACTICE PASSAGES

The field of physical therapy expands and changes 10

READING
along with innovations in other medical fields, making Which choice best fits with the tone of the rest of
for a challenging and 10 overwhelming profession. In the passage?
the 1940s the use of enhanced prosthetics changed the A) NO CHANGE
field, and in the 1960s full joint replacement surgery B) ever-changing
changed it again. Now that the first face and arm C) overpowering

WRITING AND
LANGUAGE
transplants have been successfully completed, physical D) erratic

therapy for those patients is driving changes yet again.


11 As needs for rehabilitation grow, who knows what 11
the future of physical therapy will hold? The writer wants to conclude the passage with a
sentence that emphasizes the future possibilities
for the field of physical therapy. Which choice best

MATH
accomplishes this goal?
A) NO CHANGE
B) If surgeons can replace limbs, who knows what
else they can do?
C) Leg transplants cannot be far off.
D) Physical therapists must do a lot of reading
when new developments occur.

CONTINUE
© TPR Education IP Holdings, LLC  |  349
SAT MANUAL

Questions 12–22 are based on the following passage. 12


READING

A) NO CHANGE
Passage 18: The Legacy of Watergate
B) major
The Watergate scandal, which encompassed a
C) necessary
break-in at the Democratic National Committee’s offices
D) simple
and the fallout from President Nixon’s involvement
WRITING AND
LANGUAGE

in the ensuing cover-up, was a 12 basic turning


point for America. Watergate both brought the federal 13

government under greater scrutiny and also left many A) NO CHANGE


Americans with a disenchanted view of the moral B) it’s
character of 13 its government and politicians. But C) their

these are not the only ways in which the legacy of D) they’re
MATH

Watergate has affected America.


The most important effect Watergate had on the 14
federal government was bolstering the authority of the A) NO CHANGE
Supreme Court. 14 Following Watergate, the Supreme B) After
Court had always had the power to hold the other C) Notwithstanding
branches of government accountable, but Watergate D) Before
tested the high court’s authority in a new way. Nixon’s
defense was that he acted within the presidential right of
15
“executive power.” Despite the fact that his actions were
A) NO CHANGE
15 illegal, he argued, that his position superseded the
B) illegal; he argued
law. The Supreme Court disagreed, holding that not even
C) illegal, he argued
the president was above the law. This check on executive
D) illegal he argued,
power has helped shape how modern presidents conduct
themselves.
Another crucial legacy of Watergate was the 16
creation of the Foreign Intelligence Surveillance A) NO CHANGE
Act (FISA) of 1978. During the Watergate hearings, B) had been claiming
information surfaced regarding the FBI’s unauthorized C) had claimed
surveillance of private citizens. The FBI 16 claims D) claimed
innocence on the grounds that no entity other than
the President had the power to authorize (or forbid)
surveillance. FISA created courts to oversee surveillance
for national security purposes.

CONTINUE
350  |  © TPR Education IP Holdings, LLC
FULL-LENGTH PRACTICE PASSAGES

[1] The fallout from the Watergate scandal is also 17

READING
partly responsible for the lawyer joke trope. [2] The legal Which choice adds the most relevant supporting
profession’s reputation suffered when it came to light information to the paragraph?
that many of the people involved in the scandal A) NO CHANGE
17 were Republicans. [3] Many claimed attorney-client B) were lawyers.
privilege as a defense, even though they had knowingly C) had, in fact, broken the law.

WRITING AND
LANGUAGE
participated in criminal activity. This led to a large-scale D) may not have realized the extent of the
conspiracy.
transformation of the legal profession. [4] By the late
1970s, every law school in the country 18 offered a
course of professional responsibility. [5] The American 18
Bar Association also revised its code of ethics and added A) NO CHANGE
questions on ethics and professional responsibility to the B) gave a course to

MATH
bar exam. 19 C) offered a course on
D) proposed a course for

19
To make this paragraph most logical, sentence 4
should be placed
A) where it is now.
B) after sentence 1.
C) after sentence 2.
D) after sentence 5.

CONTINUE
© TPR Education IP Holdings, LLC  |  351
SAT MANUAL

20 The congressional Watergate hearings were 20


READING

broadcast by the major television networks, including At this point, the writer is considering adding the
Public Broadcasting System 21 (PBS), public television following sentence.
The legacy of Watergate has influenced
had never before broadcast programs related to public
popular culture as well.
affairs. When 85% of the American public tuned in
Should the writer make this addition here?
to watch the hearings, 22 it became clear that: this
WRITING AND
LANGUAGE

A) Yes, because it gives important details about


was a viable programming model. The PBS NewsHour the influence of Watergate on television
and C-SPAN have President Nixon and his Watergate programming.
scandal to thank for their existence. B) Yes, because it introduces the main idea of this
paragraph.
C) No, because it blurs the focus of the paragraph
by introducing unrelated information.
MATH

D) No, because it contradicts the information that


follows.

21
A) NO CHANGE
B) (PBS), which had
C) (PBS); although public television had
D) (PBS); having

22
A) NO CHANGE
B) it became clear, that
C) it became clear, that,
D) it became clear that

CONTINUE
352  |  © TPR Education IP Holdings, LLC
FULL-LENGTH PRACTICE PASSAGES

Questions 23–33 are based on the following passage. 23

READING
A) NO CHANGE
Passage 19: The Disappearing Bee
B) It is an abrupt disappearance that cannot be
Colony Collapse Disorder (CCD) is the predicted.
phenomenon of the sudden and unexpected C) CCD is unforeseen and happens abruptly.
disappearance of bees from their hives. 23 The D) DELETE the underlined portion.

WRITING AND
LANGUAGE
disappearance is unforeseen and abrupt. There have
been recorded instances of the phenomenon since 1869,
24
although it was not until recently that it was named. In
A) NO CHANGE
2006, when both North America and Europe saw a
B) decrease
24 stabilization in the disappearance of bee colonies,
C) surplus
CCD was named and recognized as a serious problem.
D) surge

MATH
We depend heavily on bees for food production: the net
worth of food crops pollinated by honey bees totaled
$274 billion in 2012. Large portions of the farming 25
industry depend on honey bees for their livelihood, and Which choice best introduces the paragraph?
the entire world depends on the crops that bees pollinate A) NO CHANGE
to eat. B) though several theories have been proposed.
The causes of CCD are unknown, 25 and we C) but one theory stands out as the most likely
culprit.
will never find the true cause. The most likely possible
D) just like so many other recent devastating
culprits that have been presented are genetics, mites,
ecological changes.
neonicotinoids, and antibiotic use. The causes can be
divided into two distinct 26 camps; historical factors
versus modern factors. 26
A) NO CHANGE
B) camps historical
C) camps: historical
D) camps. Historical

CONTINUE
© TPR Education IP Holdings, LLC  |  353
SAT MANUAL

27 The people in the historical factors camp 27


READING

believes that genetics and known predators such as mites A) NO CHANGE


are the likely causes of CCD. They point to the historical B) Those in the historical factors camp believes
instances in which CCD has been observed and note C) Those in the historical factors camp believe
that the viruses carried by mites that infect honey bees D) They on the side of the historical factors believe
28 would naturally have evolved over time just as the
WRITING AND
LANGUAGE

honey bee has. This would cause the viruses to have a


28
more destructive effect now than has ever been seen
Which choice most effectively sets up the main idea
before. Such a change would naturally result in higher of the following two sentences?
rates of disease and death in bees. A) NO CHANGE
[1] These arguments are rejected by those who B) have demonstrated genetic mutations that
believe that modern developments are causing CCD. heighten their resistance to modern pesticides
MATH

and result in abnormally high lethality.


[2] Bees ingest these chemicals during pollination, but
C) are probably still a factor in bees’ health, just as
do not immediately die. [3] Researchers believe that the
they were in the 19th century.
chemicals alter the behavior and development of the
D) can attack a variety of other pollinating insects
bees and that other bees could be infected as well based in addition to bees.
on contact. [4] One possible modern cause of CCD is
neonicotinoids, a type of 29 pesticides that are used
29
in seed treatment and remains in the growing plant.
A) NO CHANGE
[5] Another potentially harmful modern practice
B) pesticides that is
30 would be the increased use of antibiotics by
C) pesticide that are
both commercial and private apiarists in their bee
D) pesticide that is

30
A) NO CHANGE
B) is an increase
C) was the increase in
D) is the increased

CONTINUE
354  |  © TPR Education IP Holdings, LLC
FULL-LENGTH PRACTICE PASSAGES

populations. [6] Although antibiotics have been shown 31

READING
to protect against pathogens such as mites, it is unclear To make this paragraph most logical, sentence 4
whether the use of such medications has left honeybees should be placed
vulnerable to previously unknown predators. 31 A) where it is now
It is most likely 32 an addition of several of B) before sentence 1.
the factors listed above that is causing the dramatic C) after sentence 1.

WRITING AND
LANGUAGE
rise of CCD. Researchers are hard at work developing D) after sentence 2.

remedies to reverse CCD, but the possibility of multiple


causes makes it difficult to approach the problem in a 32
systematic way. The dire consequences of losing our A) NO CHANGE
pollinators, however, means that a solution 33 has B) a sequence
gotta be found. C) a combination

MATH
D) a reason

33
A) NO CHANGE
B) could
C) will maybe
D) must

CONTINUE
© TPR Education IP Holdings, LLC  |  355
SAT MANUAL

Questions 34–44 are based on the following passage. 34


READING

A) NO CHANGE
Passage 20: Do You See What I See?
B) reflection because it
Echolocation is a familiar concept to most people.
C) reflection and
Echolocation is the process of locating objects by sound
D) reflection even though it
34 reflection, and is primarily associated with animals
WRITING AND
LANGUAGE

such as bats and dolphins. An echolocating animal


emits sounds that bounce off the objects around it, 35

so it can create a mental map of its environment. Which choice most effectively sets up the example
in the following sentence?
35 Because bats are nocturnal, they have poor eyesight
A) NO CHANGE
and rely primarily on echolocation for navigation. A
B) This type of mental map is fundamentally
blind person can produce sound by tapping a cane, different from the way we see the world, but is
MATH

snapping fingers, or clicking with the tongue against the still an effective tool for navigating through a
roof of the mouth and then use the echoes that reflect given environment.

off nearby objects to mentally 36 visualize and make a C) Sonar and radar, which are two tools commonly
used by ships for navigation, are also forms of
map of the environment. echolocation.
Sighted people are often 37 surprised to learn that D) What many people are less familiar with is the
echolocation, which is also called acoustic wayfinding, is fact that humans, most commonly those with
visual impairments, can also use echolocation.
a feasible way to navigate the world. An examination of
the mechanics of the human brain, 38 for real, shows
how well-suited humans are to using echolocation. 36
A) NO CHANGE
B) visualize
C) make images of and map out
D) map and picture

37
A) NO CHANGE
B) shocked and awed
C) gobsmacked
D) flabbergasted

38
A) NO CHANGE
B) indeed,
C) however,
D) similarly,

CONTINUE
356  |  © TPR Education IP Holdings, LLC
FULL-LENGTH PRACTICE PASSAGES

The processes behind vision and hearing are quite 39

READING
similar: they both function through reflected waves At this point, the writer is considering adding the
of energy. 39 Consequently, when sound waves are following sentence.
used for spatial purposes, the vision area of the brain Vision processes light waves and
hearing processes sound waves, but
processes them in the same way it processes light waves. either type of wave can be processed in
This connection between echolocation and sight the area of the brain that is generally

WRITING AND
associated with vision.

LANGUAGE
was 40 accepted by a 2014 study that recorded the
clicks that blind people made while using echolocation Should the writer make this addition here?

and the resulting echoes. The sounds were then played A) Yes, because it adds details that help to explain
the connection between the processes of sight
back while the subjects were in an MRI machine. The and hearing.
resulting images of blind subjects’ brains showed B) Yes, because it helps to clarify the paragraph’s
41 activity, in the part of the brain typically associated focus on how humans and bats are similar.

MATH
with sight, while no corresponding activity was recorded C) No, because it is only loosely related to the
paragraph’s focus on new developments in
in the brains of subjects 42 who had normal sight.
acoustic wayfinding.
D) No, because it does not address whether light
waves can be processed in the area of the brain
usually associated with hearing.

40
A) NO CHANGE
B) conformed
C) confirmed
D) guaranteed

41
A) NO CHANGE
B) activity in the part,
C) activity in the part
D) activity: in the part

42
A) NO CHANGE
B) whom
C) that
D) which

CONTINUE
© TPR Education IP Holdings, LLC  |  357
SAT MANUAL

One reason people find the similarity between 43


READING

vision and echolocation surprising is that they don’t A) NO CHANGE


fully understand how their own sight works. The B) is
notion that we actually see the objects in front of us is C) was
inaccurate. What we really see 43 are the reflection D) were
of light waves off those objects. Our sight is nothing
WRITING AND
LANGUAGE

more than a mental picture, not necessarily a reflection


44
of the physical world. A good example of this principle
Which choice most effectively concludes the
is the stick-in-water phenomenon. A stick that appears paragraph?
straight when held up in front of us will appear to be A) NO CHANGE
broken or bent when it is dipped into a glass of water. B) Of course, perception and reality are always
This is because water distorts the light reflections our different to some degree.
MATH

brains receive. 44 The stick has not changed, but our C) This is why it is important to pay attention to
as many details as possible, whether blind or
perception of it has.
sighted.
D) The more scientists learn, the more they will be
able to enhance the skill of human echolocation
for the blind.

Answers can be found on pages 732–735.

358 | © TPR Education IP Holdings, LLC


MATH

Yes Calculator
1
2
3
4
5
6
7
8
9
10
11
12
13
No Calculator 14
15
1 16
2 17
3 18
4 19
5 20
6 21
22
7
23
8 24
9 25
10 26
11 27
12 28
29
13
30
14 31
Grid-In
15 32
Grid-In 16 33
17 34
18 35
36
19
37
20 38
20 Questions / 38 Questions /
25 minutes 55 minutes

359
GLOBAL
TECHNIQUES

361
SAT MANUAL
READING

GOALS REVIEW
At the conclusion of this chapter, you will be able to accomplish the following:

• Name how many questions to do in the Math sections to reach your


goal score
• Use Pacing, POOD, and LOTD to achieve your goal score
WRITING AND

• Methodically work through word problems using the Word Problems


LANGUAGE

Basic Approach
• Use your calculator wisely and approach questions without it
MATH

362  |  © TPR Education IP Holdings, LLC


GLOBAL TECHNIQUES

READING
SCORING
Small changes to the number and type of questions you attempt can have a huge
impact on your score. The best place to start is with a score improvement goal.

WRITING AND
LANGUAGE
Answer this many questions
Section 3: Section 4:
No Calculator Yes Calculator
To get: You need 15 30 8 Total #
(scaled to earn: questions 5 questions questions questions of questions
score) (raw points) MC Grid-Ins MC Grid-Ins to attempt
350 12 5 1 9 1 16

MATH
400 16 7 2 11 2 22
450 20 9 2 13 3 27
500 26 10 2 18 3 33
550 32 11 2 21 5 39
600 39 12 3 24 6 45
650 44 13 4 26 7 50
700 50 14 5 29 8 56
750 54 15 5 30 8 58
800 58 15 5 30 8 58

POOD
So how do you know which questions to do and which ones to skip? Make sure to
follow your POOD and focus on these two ideas.

Do questions that can be answered quickly and accurately.


Do questions that can be made easier using TPR strategies.

© TPR Education IP Holdings, LLC | 363


SAT MANUAL
READING

GUESSING AND PACING


Remember to guess on any questions that you don’t have time to legitimately
complete. Just make sure to always guess the same letter, your Letter of the Day
(LOTD).
WRITING AND
LANGUAGE

RTFQ
There will often be a lot of extraneous words in the set-up to Math questions. Read
the final question carefully before doing any calculations.

What would you do if your math teacher asked you to solve these questions?
MATH

If 5a – 4 = 2a + 11, _____________________?

3
If a = 35 and a = b , _____________________?
7

364  |  © TPR Education IP Holdings, LLC


GLOBAL TECHNIQUES

READING
Now look at these SAT questions.

16. If 5a – 4 = 2a + 11, what is the value of a – 5 ?


A) −2
B) 0

WRITING AND
LANGUAGE
C) 3
D) 5

3 b +1
5. If a = 35 and a = b , what is the value of ?
7 4
A) 15

MATH
B) 9
C) 5
D) 4

Always make sure to

R_____________
ead ________

T_____________________
he

F_____________________
ull

Q____________________
uestion

© TPR Education IP Holdings, LLC  |  365


SAT MANUAL
READING

POE
On the SAT, there are more wrong answers than right ones. When you find a
wrong answer, cross it off !
WRITING AND
LANGUAGE

WORD PROBLEMS
For the many word problems on the SAT, make sure to use a consistent approach.
Start with RTFQ to avoid getting lost in the text.
MATH

WORD PROBLEM BASIC APPROACH


1. Read the Final Question—Read and underline the actual
question that is being asked.
2. Let the Answers Help—Look for clues on how to approach
the question and opportunities to use POE.
3. Work in Bite-Sized Pieces—Start with the most straight-
forward piece of information.

366 | © TPR Education IP Holdings, LLC


GLOBAL TECHNIQUES

READING
BALLPARKING AND ESTIMATING
Use Ballparking or Estimating to eliminate answers, which is even more important
when calculator use is not allowed.

WRITING AND
LANGUAGE
Try to eliminate answer choices that can’t possibly be correct
before calculating anything.

13. Aaron wants to make as many gluten-free, low-carb cupcakes as he


3

MATH
can for a bake sale. His recipe for 12 such cupcakes requires of a cup
8
of cocoa. Assuming Aaron has enough of the remaining ingredients,
which of the following is closest to the number of cupcakes that he
could make if he has 2 cups of cocoa?
A) 10
B) 30
C) 60
D) 100

Rounding the numbers in the question to estimate your answer can also help
save time. Don’t multiply out awkward numbers on paper if you can estimate the
answer.

x2 + y2 = 125
y = –2x

11. If (x, y) is a solution to the system of equations above, what is the


positive value of x ?
A) –5
B) 5
C) 10
D) 25

© TPR Education IP Holdings, LLC | 367


SAT MANUAL
READING

BITE-SIZED PIECES
Deal with one small piece of information at a time, eliminating answers as you go.
WRITING AND
LANGUAGE

When there is something to figure out, STOP!


Figure it out and try to use POE before you move on.
MATH

What’s 8n 3 + 21n 3? 6. 2(4n3 – 2n2 + n + 8) – 3(−7n3 + 2n2 − 5n + 9) =


How does knowing the
A) –13n3 + 10n2 – 17n + 11
answer help with POE?
B) –13n3 – 10n2 + 17n – 11
C) 29n3 + 10n2 – 17n + 11
D) 29n3 – 10n2 + 17n – 11

What should you do after


7. Which of the following ordered pairs (x, y) satisfies the inequality
testing one of the points?
2x + 7y > −1 ?
I. (1, −2)
II. (−3, 1)
III. (4, −1)

A) I only
B) III only
C) I and II only
D) II and III only

368 | © TPR Education IP Holdings, LLC


GLOBAL TECHNIQUES

10. Natalia is joining an online music service that charges a monthly membership When does the $15 fee

READING
fee of $5.95. A tax of 9% is applied to the monthly membership fee, and an come into play? How
additional one-time initiation fee of $15 is charged at the beginning of the does that help with POE?
membership. Which of the following represents Natalia’s total charge, in
dollars, for a membership lasting m months?
A) 1.09(5.95m + 15)
B) 1.09(5.95m) + 15

WRITING AND
LANGUAGE
C) 1.09m(5.95 + 15)
D) 0.09m(5.95) + 15

MATH
POE POINT—When attacking the problem in Bite-Sized Pieces,
don’t forget to stop between each piece to see which answers
can be eliminated.

© TPR Education IP Holdings, LLC | 369


SAT MANUAL
READING

CALCULATOR USE
On Section 4, you’re allowed to use your calculator, but that doesn’t mean that it
will be useful on every question in that section. When you do want to use your
calculator, remember the following:

1. Read the question.


WRITING AND

2. Set up the problem.


LANGUAGE

Only pick up your calculator after you’ve made sure to RTFQ and
set the problem up on paper.
MATH

33. If f(x) = 14x + 5[6 – (2x + 3)]2, what is the value of f(–2) ?
When using a calculator,
follow the rules of
PEMDAS!

P _______________
E _______________
M _______________
D _______________
A _______________
S _______________

Even the questions that allow calculator usage are often written so that using it
may actually end up being less effective or efficient.

370 | © TPR Education IP Holdings, LLC


GLOBAL TECHNIQUES

Summary
• By slowing down, I can ___________________.

• Which questions should you focus on in the


Math sections?
_____________________________________
_____________________________________
• When should you guess on a Math question?
_____________________________________
• What does RTFQ stand for?
_____________________________________
• What does that mean?
_____________________________________
• What are two reasons to use POE?
_____________________________________
_____________________________________
• What is the Basic Approach for Word Problems?
_____________________________________
_____________________________________
_____________________________________
• What is Ballparking?
_____________________________________
• What does it mean to solve a question in
Bite-Sized Pieces?
_____________________________________
• When do you use Bite-Sized Pieces?
_____________________________________
• What is important to keep in mind about
calculator use?
_____________________________________
_____________________________________
• I have accomplished ______ of the 4 goals
stated at the beginning of this chapter. © TPR Education IP Holdings, LLC | 371
PLUGGING IN

Algebraic symbols are used when you do not


know what you are talking about.
–Philippe Schnoebelen

373
SAT MANUAL
READING
WRITING AND
LANGUAGE
MATH

GOALS REVIEW
At the conclusion of this chapter, you will be able to accomplish the following:

• Identify when Plugging In is possible


• Use Plugging In to solve problems
• Understand why Plugging In is such a powerful technique

374  |  © TPR Education IP Holdings, LLC


PLUGGING IN

READING
PLUGGING IN
College Board makes a big deal about the differences between Heart of Algebra
questions and Passport to Advanced Math questions, but those differences don’t
matter too much to us. If there are variables in a question, turn it into an arithme-
tic question by plugging in numbers.

WRITING AND
LANGUAGE
9. Which of the following is equivalent to the expression x2 – 4x + 11 ?
A) (x + 2)2 – 7
B) (x + 2)2 + 7
C) (x – 2)2 + 7
D) (x – 2)2 – 7

MATH
PLUGGING IN BASIC APPROACH
1. Identify the variable(s).
2. Plug in a number for the variable(s).
3. Work the steps of the problem.
4. Circle the Target Value.
5. Plug into all four answers, eliminating any that do not
match the Target Value.

6k 2 + 11k
3k + 1

11. The expression above can be rewritten as which of the following?


3
A) 2k  3 
3k  1
3
B) 2k 
3k  1
C) 2k + 11

D) 2k

© TPR Education IP Holdings, LLC | 375


SAT MANUAL
READING

WHAT TO PLUG IN
Why would it be a bad idea to plug in x = 37 on question 9? What happens if you
plug in a number that results in a denominator of 0 on question 11? Choose num-
bers that make the arithmetic as straightforward as possible.

w 4
8. Which of the following must be true if  ?
WRITING AND

What’s a good number to


LANGUAGE

w  3v 13
plug in for w?
w 12
A) =
v 17
v 4
B) =
w 3
v 3
C) =
MATH

w 4
w
D) 4
vw

PLUGGING IN TIPS
• If the question is about a relationship between
numbers, plug in! The question may contain variables,
fractions, equivalent expressions, or the phrase “in terms
of.”
• Try numbers that make the math as straightforward
as possible, especially when calculator use is not allowed.
Numbers like 2, 3, 5, and 10 work great, and 100 is good for
percent questions.
• As long as your numbers fit the requirements of the
problem, your numbers will be good, viable numbers to
use. Even a weird target number will probably only match
one answer choice.
• Check all four answer choices if they have variables. If
two of them work, plug in again!

376 | © TPR Education IP Holdings, LLC


PLUGGING IN

READING
r
23. If  r  s  t , where r is a positive integer and s is a negative integer,
s
then which of the following must be true?
A) t = 0
B) t = −5

WRITING AND
LANGUAGE
C) The value of t is negative.
D) The value of t is positive.

Sometimes you’ll plug in values from a chart or graph, rather than your own
numbers.

MATH
Metric versus U.S. measurements

2 Liters (2,000 milliliters) 67.63 ounces


591.47 milliliters 20 ounces
3,785.41 milliliters 1 gallon (128 ounces)
473.81 milliliters 1 pint (16 ounces)

The table above shows the conversion between metric and U.S.
22.
measurements of common beverage sizes. If m milliliters is equal to o
ounces, which best represents the relationship between m and o ?
A) m = 29.57o
B) o = 29.57m
C) m = 0.034o
D) mo = 0.034

© TPR Education IP Holdings, LLC  |  377


SAT MANUAL
READING

Plugging In can be a great tool for word problems too.

Are there variables? 28. If the average (arithmetic mean) of 8 and q is a, the average of 12 and
Plug in!
1 1
q is b, and the average of 16 and q is c, what is the average of a, b,
2 3
WRITING AND
LANGUAGE

and c, in terms of q ?

11
A) q + 36
6
11
B) q + 18
12
MATH

11
C) q+9
24
11
D) q+6
36

HIDDEN PLUG-INS
Plugging In is not just for questions with variables in the answer choices—be on
the lookout for questions that ask about changes to values but don’t provide the
actual values. Instead of trying to imagine how the numbers behave, plug in num-
bers and see what happens.

13. Pyramid Q has a rectangular base and a volume of 96 cubic


inches. Pyramid R has a rectangular base that is one-half the base area
of pyramid Q and a height that is 3 times the height of pyramid Q.
What is the volume of pyramid R?
A) 64
B) 144
C) 288
D) 432

378  |  © TPR Education IP Holdings, LLC


PLUGGING IN

31. A person’s height H, in centimeters, can be estimated based on the

READING
length of the femur bone f, in centimeters, using the formula
H = 2.71f + 45.86. According to the model, for every increase of
5 centimeters in the length of the femur, by how many centimeters
will the person’s estimated height increase? (Round your answer to the
nearest tenth.)

WRITING AND
LANGUAGE
MATH
PLUG AND PLAY
On occasion, a question will contain a variable or an unknown quantity, but it is
not clear exactly what to plug in. When that happens, play around with the num-
bers until you find one that works.

2x 2y = 162

17. If the equation above is true for some positive integer values of x, what
is one possible value of y ?

© TPR Education IP Holdings, LLC  |  379


SAT MANUAL
READING

PLUGGING IN DRILL
Time: 8 minutes

Unless you are aiming for a top score, don’t try all the questions! Use your POOD to choose the best ones for you.
WRITING AND
LANGUAGE

9 20
If y = (cd) , x = c – d , and c ≠ d, which of the
2 2 2
v m2  6m  9
xy  0
following is equivalent to
c −d
?  m  3  m  33
A) c 3d 2 + c 2d 3
MATH

B) 2c 2d 2 If the equation above is true, and the variable m ≠ 3,


then what is the value of constant v ?
C) d 2 – c 2
D) c 3d 2 – c 2d 3

380  |  © TPR Education IP Holdings, LLC


PLUGGING IN

10 27

READING
A marble rolls down a track at an average speed of
S
P 3 p
SF s centimeters per second, where s = . Which
p
A tennis league uses the formula above to of the following gives the distance traveled by the
determine a player’s first serve practice score, P,

WRITING AND
marble, in centimeters, over p seconds?

LANGUAGE
based on the number of successful first serves,
S, and the faults on first serves, F. Which of the
following expresses the number of successful serves, p
A)
in terms of F and P ? 3 p

A) S  FP B) p
1 P
FP

MATH
B) S  C) 3 p
P 1
F
C) S  D) p
P 1
F
D) S 
1 P

29
11x + 5 = 9x + p
21 11y + 5 = 9y + q

The intensity of light, I, is inversely proportional to In the equations above, p and q are constants. If
the square of the distance, d, from the light source, p = q + 4, which of the following is true?
as given by the equation I1(d1)2 = I2(d2)2. If Viewer A
is 5 times as far from the light source as Viewer B, A) x = y – 2
the intensity of light seen by Viewer A is what B) x = y + 2
fraction of the intensity of light seen by Viewer B?
C) x = y + 4
1
A) D) x = y + 8
25

B) 1
5
5
C)
1
25
D)
1

© TPR Education IP Holdings, LLC  |  381


SAT MANUAL

Summary
• What are the advantages of Plugging In?

______________________________________

______________________________________

• What are some clues that you can Plug In?

______________________________________

______________________________________

______________________________________

______________________________________

• What are the steps for Plugging In?

______________________________________

______________________________________

______________________________________

______________________________________

______________________________________

• What are some good numbers to use


when Plugging In to make the math more
convenient?

______________________________________

• I have accomplished ______ of the 3 goals


stated at the beginning of this chapter.

382 | © TPR Education IP Holdings, LLC


PLUGGING IN
PRACTICE

383
SAT MANUAL

THE PRINCETON REVIEW GUIDE TO GRIDDING


READING

The Grid-In Questions


Some of the questions on the SAT will not be multiple choice. We call these “grid-
ins” because you have to come up with your own answer and enter it into a special
grid. Otherwise, these questions are just like the normal four-choice problem solv-
WRITING AND
LANGUAGE

ing questions—with the answer choices missing, of course.

College Board’s directions for gridding in your answers take up two-thirds of a


page, and reading them during the test can eat up some serious time. We’ll show
you how to grid in your answers so that you’ll never have to look at College Board’s
directions. You can simply fill in your response because you’ll know what to do.
MATH

Rules for Gridding


1. Write your answer in the top of the grid.  Because the answer sheet is
scored by a machine, you must fill in the bubbles in order to receive credit for
answering the question. However, entering your answer into the spaces at the top
of the grid box will keep you from making careless mistakes when you fill in the
bubbles.

2. You can use either fractions or decimals.  By letting you use fractions or
1
decimals, these questions offer a bit of flexibility. If the correct answer is , you
2
can enter it as

1 / 2 or . 5

3. Don’t grid in mixed numbers.  The computer can’t recognize mixed num-
3
bers. If your answer is 5 , you can either grid in 5.75 or you can convert the
4
mixed number to an improper fraction. In this case, your answer would be marked
23
correct if you gridded in . If the improper fraction doesn’t fit in the grid, you’ll
4
have to use a decimal.

384  |  © TPR Education IP Holdings, LLC


PLUGGING IN PRACTICE

4. Start at the far left and use all four spaces if necessary.  Not all your

READING
answers will take up all four spaces but it’s a good idea to always start at the far
left of the grid. Why? Well, suppose that your answer is some long decimal such
as .142856, which doesn’t fit into the grid. College Board wants the most accu-
rate answer possible. If you just grid in .1 or .14, you’ll be marked wrong. Those
answers aren’t as accurate as possible. If you start at the far left, you can grid in
more of the digits from your answer.

WRITING AND
LANGUAGE
Here’s what College Board would credit as a correct response:

. 1 4 2 or . 1 4 3

MATH
5. Lop, don’t round.  As you just saw, the answer .142856… can be entered in
two different ways. College Board doesn’t care whether you round up the last digit
of a decimal that doesn’t fit in the grid. Save yourself a little time and don’t worry
about rounding; just enter the digits that fit and ignore the rest. Be aware, though,
that strange decimals are uncommon on the SAT.

4
6. Don’t bother reducing fractions that already fit.  If you get as your
28

answer, you might be tempted to reduce it. Don’t waste your time! It already fits in

the grid and College Board will give you credit for any answer that’s equivalent to

4 2 1
. Why bother figuring out that you could also grid in or ? Reducing
28 14 7
10
is just one more place to make a mistake. However, if your answer is , that
70

won’t fit in the grid. Now, you’ve either got to reduce or convert your fraction to a

decimal.

7. There’s no bubble for this!?!  The grid box can’t handle negative numbers,
variables, or square root signs. If any of these appear in your answer, go back and
check your work.

If you get an answer that has π in it, you should also check your work. Don’t
simply multiply your answer by 3.14. College Board would consider it unfair to
expect students without calculators to know the value of π and to multiply that
value as part of their answer.

© TPR Education IP Holdings, LLC  |  385


SAT MANUAL
READING

Quick, what’s 7 ? (Not fair to use your calculator.) If you didn’t know, then
you’re like most people. If you get a weird square root as an answer, you should
also check your work. Something’s wrong.

8. Just drop the percent sign.  If you get 75% as your answer, just drop off
WRITING AND

the percent sign and grid in 75. Don’t convert it to a decimal. Don’t convert it to a
LANGUAGE

fraction. If you grid in .75, the machine will think that you mean .75 percent and
mark you wrong.

Here’s what your answer should look like:


MATH

7 5

If any of this information is unclear, be sure to ask your instructor about it at the
next class.

386  |  © TPR Education IP Holdings, LLC


PLUGGING IN PRACTICE

READING
PLUGGING IN

BASIC APPROACH

WRITING AND
1. Identify the variable(s).

LANGUAGE
2. Plug in a number for the variable(s).
Pick a number that fits the requirements and will make the
math straightforward.
3. Work the steps of the problem.
4. Circle the Target Number.

MATH
5. Plug into all four answers, eliminating any that do not match the
Target Value.

3 5
The distance d that a car can travel in a certain Hector’s father gives Hector a piggybank that
amount of time t can be found by multiplying the contains $10. If Hector adds $3 every two weeks to
rate of speed r by the travel time. If a car has already the piggy bank, how much money M, in dollars,
traveled 4 miles, the formula for the total distance will the piggy bank contain at the end of w weeks,
traveled is d = rt + 4. Which of the following gives where w is a positive even integer?
the value of r, in terms of d and t ?
3
A) M = w + 10
t 2
A) r 
d4
B) M = 3w + 10
t
B) r 
d4
C) M = 6w + 10
d4
C) r 
t D) M = 6w + 20

d4
D) r 
t

CONTINUE
© TPR Education IP Holdings, LLC | 387
SAT MANUAL

6 10
READING

Which of the following is equivalent to John’s car can travel m miles on one gallon of
2 3 1 gasoline. After filling his gas tank, John drove x
+ 2 + ? miles. If gasoline costs $2.75 per gallon, which of
x x 2x
the following represents how much it will cost John,
6 in dollars, to refill his gas tank?
A) 2
x + 3x
WRITING AND
LANGUAGE

2.75x
A)
2x + 3 m
B)
x2
2.75m
B)
5x + 6 x
C)
2x 2
C) 2.75x – m
2
4x + 6
MATH

D)
6x 6 D) 2.75mx

9
The sum, s, of 4 consecutive odd integers can be
represented by the equation s = 4n – 12. What does
n represent?
A) The least of the 4 integers
B) The average of the 4 integers
C) The mode of the 4 integers
D) The greatest of the 4 integers

CONTINUE
388  |  © TPR Education IP Holdings, LLC
PLUGGING IN PRACTICE

11 19

READING
If a watch store paid $125 per watch for a shipment A new radioactive element is found on Mars. Every
of watches, and sold all but 15 watches from the 10 years, the concentration of the element is one-
shipment for $150 per watch, then, in terms of
the number of watches in the shipment, y, what quarter of what it was at the start of the 10-year
t
function describes the watch store’s profit, P, from period. If the decay rate is given by C  C0  x 10 ,
the sales?

WRITING AND
where C0 is the initial concentration of the element,

LANGUAGE
A) P(y) = 125(y – 15) – 150y
C is the final concentration of the element, and t is
B) P(y) = 15(125 – y) – 150y
the time in years, what is the value of x ?
C) P(y) = 150(y – 15) – 125y
D) P(y) = 15(150 – y) – 125y

MATH
13
If the expression x3 – x2 – 32x + 60 can be written in
the equivalent form K(x – 2), which of the following
is equivalent to K ?
A) x2 – x + 28
B) x2 + x – 30
C) x2 – x + 32
D) x2 + x + 34

CONTINUE
© TPR Education IP Holdings, LLC  |  389
SAT MANUAL

1 6
READING

Which of the following is equivalent to Which of the following is equivalent to


c(3c + 6) – 5c ? m 6m  4
 ?
A) –c + 6 2 3m  1
B) 3c2 – c
2m
C) 3c2 + c A)
6m + 2
WRITING AND
LANGUAGE

D) 3c2 + 6c
2
B) 2m - 4m
2

6m2 - 4
C)
6m + 2
4 6m2 - 4m
MATH

D)
Which of the following expressions is equivalent to 6m + 2
1
2a 2  ?
1
A) a 4 2

7
B) a 2
In football, a player’s statistic for offensive yards
1 is the sum of his rushing yards and his receiving
C) 2a 4 yards in a game. Ali is the running back on a
football team. He averages 60 yards rushing and 20
yards receiving per game during the regular season.
D) 2a During playoff games, Ali’s rushing yards are 12%
greater on average than his rushing yards in regular
season games. Which of the following represents
Ali’s total number of offensive yards if he plays in p
playoff games?
A) 0.12(60p + 20)
B) 1.12(60p) + 20
C) 1.12(60p + 20p)
D) 1.12(60p) + 20p

CONTINUE
390  |  © TPR Education IP Holdings, LLC
PLUGGING IN PRACTICE

12 17

READING
Which of the following is equivalent to Weights of Nine Models of Hammer
(y + 8)(2y3 + y – 5) ? Sold by Manufacturer X
A) 2y4 + 16y3 + y2 – 3y – 40
Model number Weight, in ounces
B) 2y4 + 16y3 + y2 + 3y – 40 1 6
C) 18y4 + y2 + 3y – 40 2 8

WRITING AND
LANGUAGE
D) 19y + 8y – 40
3
3 10
4 13
5 16
6 19
7 23
8 27
14

MATH
9 30
The power in watts, P, that is generated by a certain
electric circuit depends on the current in amperes, For a homework assignment, Shapour is given the
i, and can be modeled by the equation P = 20(i – 3)2 table above and asked to write a function relating
+ 180, where i > 3. Which of the following gives the the weight of a hammer in ounces, W(m), to the
value of i in terms of P ? model number of the hammer, m. If the function
Shapour writes is W(m)  = 3m + 1, for which of the
A) i  3  2 5(P  180) following ranges of model numbers is his function
valid?
5 A) 1 through 3
B) i  3  2
P  180 B) 4 through 6
C) 7 through 9
1 P  180 D) The function is valid for all models.
C) i  3 
2 5

1 P  180
D) i  3 
2 5

CONTINUE
© TPR Education IP Holdings, LLC  |  391
SAT MANUAL

19 22
READING

The graph below shows the percentage of American The following bar graph shows the percentage of
households that owned landline telephones from the population of adults in each age group in King
2004 to 2010. County, Georgia, that received flu shots in 2007 and
Percentage of American Households 2008.
with Landline Telephones
90 Flu Shots in King Country
WRITING AND
LANGUAGE

80 60

Percent Receiving Flu Shots


% of Households

70
60 50
50
40 40
30
20 30 2007
10 2008
0 20
2000 2001 2002 2003 2004 2005 2006
10
MATH

Year
0
If t = 0 refers to the year 2000 in the graph above, 18–24 25–34 35–44 45–54 > 54
years years years years years
which of the following equations could describe the old old old old old
percentage of American households with landline Age Group
telephones, P, as a function of time in years, t, from
2000 to 2006?
A) P(t) = –5t + 85 Given that the number of people aged 18-24 in King
B) P(t) = 5t + 85 County was three times the number of people aged
35–44 in 2008, which of the following statements is
C) P(t) = –5t – 85 true?
D) P(t) = 5t – 85 A) Approximately twice as many flu shots were
given to people aged 35–44 than people aged
18–24 in King County in 2008.
B) Approximately six times as many flu shots were
given to people aged 18–24 than people aged
35–44 in King County in 2008.
20 C) Approximately 50% more people aged 18–24
If the expression x + 10x + 15 can be factored into
2 received flu shots than people aged 35–44 in
(x + 4)(x + 5) + Z, what is Z in terms of x ? King County in 2008.
A) –x D) Approximately 50% more people aged 35–44
received flu shots than people aged 18–24 in
B) x + 5
King County in 2008.
C) x – 5
D) x – 9

CONTINUE
392  |  © TPR Education IP Holdings, LLC
PLUGGING IN PRACTICE

24 26

READING
Lemon Tree Fruit Weight Marble Ramp Rolling Times

6.5 100
90

Length (in centimeters)


80
6.0
70

WRITING AND
Weight in ounces

LANGUAGE
60
5.5 50
40
5.0 30
20
10
4.5 0
0 5 10 15
Time (in seconds)

MATH
4.0
55 60 65 70 75 80 85
Days A student is rolling a marble down ramps of
varying lengths. The scatterplot above shows the
The scatterplot above shows the weight, in ounces, time, in seconds, it takes the marble to roll down
of the fruits on a certain lemon tree from days 55 each ramp. If l is the length of the ramp and t is
to 80 after flowering. If w = weight and d = days, the time in seconds, which of the following could
which of the following could be the equation of the be the equation of the curve of best fit to the data
line of best fit to the data represented, as shown on represented, as shown on the graph?
the graph? A) l = 0.2t
A) w = 0.075d B) l = 0.5t2
B) w = 0.08125d – 0.375 C) l = 2t2
C) w = 0.5d + 55 D) l = 2.5t3
D) w = 1.5d + 80

CONTINUE
© TPR Education IP Holdings, LLC  |  393
SAT MANUAL

27 29
READING

Which of the following is the result of dividing The graph below shows the partial trajectory of a
2x2 – 11x – 20 by 2x + 3 ? projectile thrown from an elevated platform.

8 Partial Trajectory of a Projectile


A) x  4 

Vertical Distance (in feet)


2x  3 40
35
WRITING AND

32 30
LANGUAGE

B) x  4  25
2x  3
20
1 15
C) x  7  10
2x  3 5
41 0
D) x  7  0 0.5 1.0 1.5 2.0 2.5 3.0 3.5
2x  3 Horizontal Distance (in feet)
MATH

If y represents vertical distance and x represents


horizontal distance, then which of the following
equations could represent the trajectory of the
projectile shown above?
28 A) y = –5x2 + 20x + 15
2 B) y = 5x2 + 20x – 15
y  ( x  a)
3 C) y = –5x2 – 20x – 15
1
y   ( 4 x  7a ) D) y = 5x2 + 20x + 15
5

If a is a constant in the system of equations above,


and a ¹ 0, which of the following ordered pairs
(x, y) is the solution to the system of equations, in 32
terms of a ? An after-school sewing club has 120 yards of fabric
to divide equally among the members of the club.
 1  When two more students join the club at the last
A)   a, 2a  minute, each member gets 5 fewer yards of fabric
 2 
to work with. How many members were in the club
1  before the two additional students joined?
B)  a, a 
2 

C) (2a, a)

D) (4a, 6a)

Answers can be found on page 735.

394 | © TPR Education IP Holdings, LLC


PLUGGING IN
THE ANSWERS

The most dangerous phrase is


“We’ve always done it that way.”
–Grace Hopper

395
SAT MANUAL
READING

GOALS REVIEW
At the conclusion of this chapter, you will be able to accomplish the following:

• Identify when Plugging In the Answers is possible


• Follow the steps of PITA to accurately and efficiently solve problems
• Explain why PITA is such a powerful technique
WRITING AND
LANGUAGE
MATH

396  |  © TPR Education IP Holdings, LLC


PLUGGING IN THE ANSWERS

READING
PLUGGING IN THE ANSWERS (PITA)
When the question asks for a specific amount and the answer choices are presented
in numerical order, Plug In the Answers.

10. Which of the following is a possible solution to the equation


20 18

WRITING AND
- 2 ?

LANGUAGE
a -1 a 1

A) 4
B) 5
C) 6
D) 7

MATH
PITA BASIC APPROACH
1. Label the answer choices—what do the answers represent?
2. If the question asks for the greatest or smallest value, start
there. Otherwise, start with one of the answers in the middle.
3. Use Bite-Sized Pieces to work the steps of the problem.
4. Eliminate answers that are too big or small.
5. When one of the answers works, STOP.

3 RTFQ! What do the


9. If  2  y , then which of the following is a possible value of 2 – y ?
2 y answer choices
represent? What does
A) 3 that mean about y?
B) 3
C) 2 + 3
D) 9

© TPR Education IP Holdings, LLC | 397


SAT MANUAL
READING

By plugging in the answers, you can avoid doing time-consuming algebra and fall-
ing for trap answers.

Why can you eliminate x2  7x x


21. If  10 and x > 0, what is the value of ?
(A) immediately? 6 6
A) –5
WRITING AND
LANGUAGE

B) 2
C) 12
D) 24
MATH

2
 y 1 x
3

y+4=x

Plug the points into the 4. Which of the following ordered pairs (x, y) is the solution to the system
simpler equation first. of equations above?
A) (1, –3)
B) (2, –6)
C) (5, 1)
D) (13, –21)

8. What is the solution set to the equation a  5  7  3a  10 ?


A) {–3, –1, 2}
B) {–3, 2}
C) {–3}
D) {2}

398  |  © TPR Education IP Holdings, LLC


PLUGGING IN THE ANSWERS

16. If 8x2 – 14x – 15 = 0, then which of the following is the least value of x ? When the question asks

READING
for the greatest number
5
A) - or the maximum value,
2 plug in that answer
choice first.
3
B) -
4

WRITING AND
3

LANGUAGE
C)
4
5
D)
2

MATH
CLUES THAT YOU CAN USE PITA
• The question asks for a specific amount—“how much,” “how
many,” or “what is the value of…?”
• You get the urge to write your own equation.
• All the answer choices are numbers.

PITA often works for word problems too!

18. A car begins at rest 200 meters from the finish line of a straight
segment of track. If the car accelerates from rest at a constant rate of
4 meters per second each second, its distance from the finish line, x, is
given by the equation x = 200 – 2t2, where 0 ≤ t ≤ 10. After how many
seconds has the car travelled 72 meters?
A) 2
B) 6
C) 8
D) 12

11. A preschool teacher gave out a total of 65 cookies to her class during
snack time. She allowed each of her 25 students to choose either 3 small
cookies or 2 large cookies. Assuming all the children in the class chose
one of these two options at snack time, how many of the students in her
class chose to have 3 small cookies?
A) 10
B) 15
C) 18
D) 20

© TPR Education IP Holdings, LLC | 399


SAT MANUAL
READING

PLUGGING IN THE ANSWERS DRILL


Time: 8 minutes

Unless you are aiming for a top score, don’t try all the questions! Use your POOD to choose the best ones for you.
WRITING AND
LANGUAGE

4 6
If the expression n2 – 2n – 8 represents the net profit
4 2
on the sale of n units of a product, which of the 
following values represents the number of units a  5 3a
sold for which the net profit equals 0 dollars?
MATH

Given the equation above, what is the value of 3a ?


A) 2
A) 9
B) 4
C) 6
B) 3
D) 8
C) 1

1
D)
3

400  |  © TPR Education IP Holdings, LLC


PLUGGING IN THE ANSWERS

7 15

READING
–5 < –2x + 4y 3
If  13  10 x , then x could equal which of the
x
Which of the following ordered pairs satisfies the following?
inequality above?
A) 0.2
I. (2, 3) B) 0.5

WRITING AND
LANGUAGE
II. (4, 2) C) 0.6
III. (6, 1) D) 1.5

A) I only
B) III only
C) I and II only

MATH
D) II and III only
25
A leader of a scouting troop is planning to decorate
vests for the members of the troop. If he buys x
beads to divide among the vests, he can put 4
beads on each and still have 4 beads left over. If
he misplaces 12 beads, however, he can only put 3
11 beads on each vest and will have 9 beads left over.
The Eiffel Tower was constructed for the Exposition How many vests is the troop leader planning to
Universelle, a world’s fair held in Paris in 1889. Four decorate?
years later, the first Ferris Wheel was constructed
for the World’s Columbian Exhibition in Chicago, A) 12
Illinois. The combined total height of these two B) 13
structures was 1,248 feet. If the Eiffel Tower is 720 C) 17
feet taller than the Ferris Wheel was, how tall was
D) 22
the Ferris Wheel?

A) 264
B) 528
C) 678
D) 980

© TPR Education IP Holdings, LLC  |  401


SAT MANUAL

Summary
• What are the advantages of Plugging In
the Answers?

______________________________________

______________________________________

• What are some clues that you can PITA?

______________________________________

______________________________________

______________________________________

______________________________________

• What are the steps for PITA?

______________________________________

______________________________________

______________________________________

______________________________________

______________________________________

• I have accomplished ______ of the 3 goals


stated at the beginning of this chapter.

402 | © TPR Education IP Holdings, LLC


PLUGGING IN
THE ANSWERS
PRACTICE

403
SAT MANUAL

PLUGGING IN THE ANSWERS


READING

BASIC APPROACH
1. Label the answer choices.
WRITING AND
LANGUAGE

2. If the question asks for the greatest or smallest value, start there.
Otherwise, start with one of the answers in the middle.
3. Work the steps, using Bite-Sized Pieces.
4. Eliminate answers that are too big or small.
5. When one of the answers works—STOP.
MATH

2 8
2a  1 1 2f 8 4
If  , what is the value of a ? If 4 £ £ , which of the following
5a  1 3 3 3
A) –4 accurately expresses all possible values of f ?
B) –1
A) –24 ≤ f ≤ 8
C) 3
B) –12 ≤ f ≤ 4
D) 4
C) –6 ≤ f ≤ 2
D) 2 ≤ f ≤ 10

4
y = –x
y2 – 6 = x

If (x, y) is a solution to the system of equations


above, which of the following includes all possible
values of x ?
A) x = {2, 3}
B) x = {2, –3}
C) x = {–2, 3}
D) x = {–2, –3}

CONTINUE
404 | © TPR Education IP Holdings, LLC
PLUGGING IN THE ANSWERS PRACTICE

10 15

READING
In her Physics class, Tori conducted a number of (ax + 2)(x – 5) = 2x2 – kx – 10
experiments to determine the correct equation to
describe the trajectory of a ball that was thrown Which of the following values for constants k and a
straight up from a height of four feet above the will make the equation above true?
ground with an initial velocity of 20 feet per second. A) a = 2; k = 2
If the height of the ball is given by the equation

WRITING AND
B) a = 3; k = 5

LANGUAGE
h = 4 + 20t – 5t2, where h = height and t = time (in
seconds), at which of the following values for t will C) a = 2; k = 8
the ball have the greatest height? D) a = 3; k = 8
A) t = 0
B) t = 1
C) t = 2
D) t = 3

MATH
13
If a2 = (2a – 9)2, which of the following includes all
possible values of a ?
A) a = {–9, –3}
B) a = {–9, 3}
C) a = {–3, 9}
D) a = {3, 9}

CONTINUE
© TPR Education IP Holdings, LLC  |  405
SAT MANUAL

3 8
READING

If f(x) = x , what is one possible value of x for which


2
y+x>4
4 < f(x) < 21 ? y – 2x ≤ 5
A) –2
B) –3 Which of the following points in the xy-plane is in
the solution set of the systems of inequalities above?
C) –5
A) (1, 2)
WRITING AND
LANGUAGE

D) –7
B) (2, 2)
C) (2, 3)
D) (3, 12)

6
MATH

If (z – 4)2 – 121 = 0 and z > 0, what is the value of z ?


A) 7
13
B) 11
1
C) 15
7  6(5n) 3  37
D) 16
In the equation above, what is the value of n ?
A) 10
B) 25
C) 43
D) 68

CONTINUE
406  |  © TPR Education IP Holdings, LLC
PLUGGING IN THE ANSWERS PRACTICE

15 21

READING
If –5 + 17q ≤ –6(13q – 15), what is the greatest Santi and Vlado drove together from New York to
possible integer value of q ? New Orleans. Both drove the same amount of time,
A) –1 but Santi drove 75% as quickly as Vlado. If the drive
took a total of 24 hours and New Orleans is 1,304
B) 0
miles from New York, approximately how many
C) 1 miles did Santi drive?

WRITING AND
LANGUAGE
D) 2 A) 326
B) 560
C) 744
D) 978

18

MATH
y = (x + 5)2
–y = x – 1
27
In the solutions to the system of equations above,
what is the smallest value of x ? In dry conditions and with normal reaction
A) –8 times, the safe stopping distance d of a car can be
1
calculated as d  v 2  v , where v is the velocity of
B) –3 20
the car in miles per hour at the time the brakes are
9 applied. Given a stopping distance of 59.8 feet for a
C)
2
car traveling in dry conditions, what was the car’s
D) 5
velocity when the brakes were applied?
A) 13
B) 20
C) 26
D) 46

Answers can be found on page 735.

© TPR Education IP Holdings, LLC | 407


TRANSLATING
AND MEANING
IN CONTEXT

First, learn the meaning of what you say,


and then speak.
– Epictetus

409
SAT MANUAL
READING

GOALS REVIEW
At the conclusion of this chapter, you will be able to accomplish the following:

• Translate English into math


• Use translation, Bite-Sized Pieces, and POE to solve translation
questions
WRITING AND

• Create and solve equations and inequalities to answer Grid-In


LANGUAGE

questions
• Apply the Meaning in Context Basic Approach to solve Meaning in
Context questions
MATH

410  |  © TPR Education IP Holdings, LLC


TRANSLATING AND MEANING IN CONTEXT

READING
TRANSLATING 101
Many SAT problems require you to turn English into math. Let’s start by going
over some of the more common translations that you’ll need to know.

English Math Equivalent


what

WRITING AND
LANGUAGE
is, are, were, did, does
more than
less than
at least
no more than
how much greater than

MATH
of
out of
per
percent
sum
difference
product
quotient

Let’s apply some of these terms to a sample word problem.

5. A student has $60 and must purchase textbooks costing a total of $42.
What percent of the $60 does the student spend on textbooks?
A) 70%
B) 60%
C) 51%
D) 42%

Sometimes you’ll be asked about increasing or decreasing an amount by a percent.

English term Operation


increase by
decrease by

© TPR Education IP Holdings, LLC  |  411


SAT MANUAL

3. A pair of boots costs $50 and a certain coat costs $150. If the cost of the
READING

boots is increased by 20% and the cost of the coat is decreased by 30%,
what is the sum of their new costs?
A) $165
B) $200
C) $225
WRITING AND
LANGUAGE

D) $265

You may also be asked to calculate the percent increase or decrease within a word
problem. Use the following formula.
MATH

Percent Change = Difference × 100


Original

31. In the 1990s, the park rangers at Yellowstone National Park implemented
a program aimed at increasing the dwindling coyote population in
Montana. If there were 20 coyotes in the park in 1995 and 70 in 1997,
by what percent did the coyote population increase in that time span?
(Disregard the percent sign when gridding your answer.)

412 | © TPR Education IP Holdings, LLC


TRANSLATING AND MEANING IN CONTEXT

READING
DON’T GET LOST IN TRANSLATION
You will also be asked to translate word problems into equations or inequalities.
Work these in Bite-Sized Pieces, using POE after you figure out one piece of
information.

18. On a certain airline, the standard price for a one-way flight between

WRITING AND
LANGUAGE
Toronto and Ottawa (in either direction) is $179. However, for members
of the airline’s Frequent Flyer club, the price is $155. Membership in
the Frequent Flyer club costs $985 each year. Which of the following
inequalities can be solved to find f, the minimum number of flights
that one would need to take per year between Toronto and Ottawa to
save money with a Frequent Flyer club membership, assuming no other
benefits from the membership?

MATH
179
A) 985 < f
155
B) 985 > (179 – 155)f

C) 985 < (179 – 155)f

D) 985 – 179 > 155f

(179-155)f > 985

© TPR Education IP Holdings, LLC  |  413


SAT MANUAL
READING

When you have systems of equations or inequalities in the answer choices, use
Bite-Sized Pieces, start with the most straightforward piece of information, and
use POE.

What’s the most 15. Nathalie is selling jewelry at a trade show. She has 30 necklaces and
straightforward place to 50 rings available for sale, and she wants to sell at least $1,000 worth
WRITING AND
LANGUAGE

start translating? of jewelry to cover the costs of attending the trade show. Nathalie
sells necklaces for $25 each and rings for $15 each. If n represents the
number of necklaces sold and r represents the number of rings sold,
which of the following systems of inequalities represents this situation?
A) n + r ≥ 1,000
n ≤ 30
r ≤ 50
MATH

B) 25n + 15r ≥ 1,000


n ≤ 30
r ≤ 50
C) 30n + 50r ≥ 1,000
n ≤ 25
r ≤ 15
D) 40(n + r) ≥ 1,000
n ≤ 30
r ≤ 50

Sometimes you’ll need to translate and then simplify to determine the correct
answer.

What happens to the 21. At the beginning of a certain week, Bowley has 18 pages of reading
number of pages to do for a certain class. Each day she reads 8 pages. Bowley’s teacher
Bowley has left to read assigns 6 more pages of reading every day. Which of the following
each day? equations best models the number of pages, p, that Bowley has
remaining to read for the class d days since the beginning of the week?
A) p = 18 – 2d

B) p = 2d + 18

8
C) p = d + 18
7
D) p = 18 – d

414  |  © TPR Education IP Holdings, LLC


TRANSLATING AND MEANING IN CONTEXT

READING
Finally, sometimes you’ll need to set up an equation or inequality and solve to
answer a Grid-In question.

19. In a certain video game, characters can make various items using bars of
copper and tin. An axe handle requires 2 bars of copper and 1 bar of tin, and
an axe head requires 6 bars of copper and 4 bars of tin. Armok, a character in

WRITING AND
LANGUAGE
this game, has 150 bars of copper and 152 bars of tin. What is the maximum
number of axes that Armok can make if each axe requires 1 axe handle and 2
axe heads?

MATH

© TPR Education IP Holdings, LLC  |  415


SAT MANUAL
READING

THE SEARCH FOR MEANING (IN CONTEXT)


Some SAT Math questions will ask what a number or expression means in the
context of a word problem. When asked to find meaning in context, use the fol-
lowing approach.
WRITING AND
LANGUAGE

MEANING IN CONTEXT BASIC APPROACH


1. Read the Final Question—Know what part of the equation or
expression the question is asking about.
2. Label the Expression—Start with the most straightforward
piece of information and translate using Bite-Sized Pieces.
MATH

Label what you can in the expression.


3. Use POE—Eliminate answer choices that don’t make sense
with the labels.
4. Plug-and-Play—Plug in numbers to determine if any remain-
ing answer choices don’t make sense.
5. Guess and Go—If you have more than one answer remaining,
pick one of the remaining answers and move on!

17. The total number of wolves, w, in Idaho, Montana, and Wyoming


from 1995 to 2005 can be approximated using the equation
w = 71.3y + 48, where y is the number of years since 1995. Which
of the following is the best interpretation of the number 71.3 in the
context of the problem?
A) The average annual increase in the total number of wolves in Idaho,
Montana, and Wyoming from 1995 to 2005
B) The average total number of wolves in Idaho, Montana, and
Wyoming from 1995 to 2005
C) The number of years it took the total number of wolves in Idaho,
Montana, and Wyoming to increase by 48
D) The total number of wolves in Idaho, Montana, and Wyoming in
1995

416 | © TPR Education IP Holdings, LLC


TRANSLATING AND MEANING IN CONTEXT

2 2

READING
x x 1
 3   5   3
   

10. Two fungal species of Aspergillus flavus, AF-36 and AF-42, were grown in liquid

cultures at a temperature of 37oC and spun at 60 rotations per minute. The

WRITING AND
LANGUAGE
AF-36 culture grew at a slower rate than the AF-42 culture, and together the

cultures produced in 3 days the amount of fungus required for experiments.

The equation above represents the experiment described. Which of the


2
x
following describes what the expression   represents in this equation?
5

MATH
A) The average amount of AF-36 fungus that is produced in one day
B) The time, in days, that it takes for AF-36 to produce the total amount of
fungus alone
C) The time, in days, that it takes for AF-42 to produce the total amount of
fungus alone
D) The average amount of AF-42 fungus that is produced in one day

© TPR Education IP Holdings, LLC  |  417


SAT MANUAL
READING

TRANSLATING DRILL
Time: 8 Minutes

Unless you are aiming for a top score, don’t try all the questions! Use your POOD to choose the best ones for you.
WRITING AND
LANGUAGE

2 15
A teacher has 51 items to grade, divided into Olivia is earning money for summer camp by
p papers and q quizzes. It takes the teacher selling cookies. She earns a portion of the profits
12.4 minutes to grade a paper and 7.75 minutes from each box of cookies she sells, and if she sells
to grade a quiz. The teacher grades for a total of 100 boxes of cookies, her portion will be $72.80.
MATH

465 minutes. Which of the following systems of Summer camp costs $185.75, and Olivia’s parents
equations can be used to find the number of quizzes have promised to pay the difference between the
graded? cost of the camp and the amount Olivia earned by
A) 12.4p + 7.75q = 465 selling boxes of cookies. Which of the following
p + q = 51 functions gives the cost, C, in dollars, that Olivia’s
parents will pay towards summer camp if Olivia
B) 7.75p + 12.4q = 465 sells b boxes of cookies?
p + q = 51
A) C(b) = 185.75 – 7,280b
C) 12.4p + 7.75q = 51
p + q = 465 B) C(b) = 185.75 – 72.8b
D) 7.75p + 12.4q = 51 C) C(b) = 185.75 – 0.728b
p + q = 465 D) C(b) = 185.75b

418  |  © TPR Education IP Holdings, LLC


TRANSLATING AND MEANING IN CONTEXT

9 26

READING
A 150 mL beaker of water is sitting in a room kept The cost of producing a certain number of units of
at a constant temperature and humidity. Each day Product A decreases until a certain point, at which
approximately 7.3 mL of water evaporates from the cost of producing more units of Product A
the beaker, and no water is added. Which of the increases due to storage and additional production
following functions best approximates the volume v, costs. The cost, C, in dollars, for producing x units
in mL, of water remaining in the beaker d days of Product A can be modeled by the equation

WRITING AND
LANGUAGE
after the beginning of the experiment, where C = 0.07x2 – 456x + 750,000. What does the number
0 ≤ d ≤ 20.5 ? 750,000 represent in the equation?
A) v(d) = 150 + 7.3d A) The maximum cost, in dollars, of producing
B) v(d) = 150 + 20.5d Product A

C) v(d) = 150 – 20.5d B) The increase in cost, in dollars, for producing an


additional x units of Product A
D) v(d) = 150 – 7.3d
C) The number of units of Product A that

MATH
minimizes the cost per unit
D) The fixed costs associated with production of
Product A regardless of the number of units
produced

12
A certain country experienced approximately
linear growth in per capita gross domestic product
from 2000 to 2008. The function f, defined by
f(y) = hy + k, models the per capita gross domestic
35
product, in U.S. dollars, y years after 2008, where
0 ≤ y ≤ 8 and h and k are constants. What does A restaurant is ordering vegetables for the
k represent? upcoming week. Potatoes are shipped in 30-pound
A) The approximate increase in per capita gross bags and onions are shipped in 5-pound bags. If the
domestic product, in U.S. dollars, each year restaurant receives 1,800 pounds of potatoes and
from 2000 to 2008 onions and there are 25 more bags of potatoes than
bags of onions, how many pounds of onions are
B) The approximate per capita gross domestic received?
product, in U.S. dollars, in 2000
C) The approximate per capita gross domestic
product, in U.S. dollars, in 2008
D) The approximate total increase in per capita
gross domestic product, in U.S. dollars, from
2000 to 2008

© TPR Education IP Holdings, LLC  |  419


SAT MANUAL

Summary
• What is the formula for percent change?

______________________________________

• What steps should you follow when asked


to identify a system of equations or
inequalities?

______________________________________

______________________________________

______________________________________

• What are the steps to tackling Meaning in


Context questions?

______________________________________

______________________________________

______________________________________

______________________________________

______________________________________

• I have accomplished ______ of the 4 goals


stated at the beginning of this chapter.

420 | © TPR Education IP Holdings, LLC


TRANSLATING
AND MEANING IN
CONTEXT
PRACTICE

421
SAT MANUAL

TRANSLATING
READING

8 10
Nissa is selling advertising time on a radio A shipping company is delivering boxes of
show. She sells 45-second spots for $400 each chocolate that weigh 5 pounds each and boxes of
and 60-second spots for $500 each. For each pralines that weigh 9 pounds each. In total, the
WRITING AND
LANGUAGE

hour of airtime, up to 15 minutes are allotted delivery truck is carrying 180 boxes that weigh a
for commercials, and Nissa hopes to sell at least total of 1,440 pounds. At the first stop, the driver
$8,000 worth of advertising time for each hour of delivers 40 boxes, reducing the total weight of the
airtime. Solving which of the following systems of packages on the truck by 15%. Solving which of the
inequalities yields the number of 45-second spots, following systems of equations yields the number of
x, and the number of 60-second spots, y, that Nissa boxes of chocolate, c, and boxes of pralines, p, that
can sell for one hour of airtime? remain on the truck after the first stop?
3 A) c + p = 140
MATH

A) x  y  15 5c + 9p = 1,224
4
400x + 500y ≥ 8,000 B) c + p = 140
5c + 9p = 1,440
3
B) x  y  15 C) c + p = 180
4
5c + 9p = 1,224
500x + 400y ≥ 8,000
D) c + p = 1,224
5 5c + 9p = 140
C) x  y  15
4
500x + 400y ≥ 8,000

D) 45x + 60y ≤ 15

400x + 500y ≥ 8,000

CONTINUE
422  |  © TPR Education IP Holdings, LLC
TRANSLATING AND MEANING IN CONTEXT PRACTICE

MEANING IN CONTEXT

READING
17 8
At Dina’s Diner, an order of 2 bags of French fries In a hot dog eating competition, c contestants eat a
and 1 burger costs $5.80. An order of 1 bag of total of cx hot dogs. What does x represent?
French fries and 2 burgers costs $6.20. What is A) The number of contestants

WRITING AND
LANGUAGE
the cost of an order of 3 bags of French fries and 3
B) The total number of hot dogs eaten
burgers, in dollars?
C) The median number of hot dogs eaten per
contestant
D) The average number of hot dogs eaten per
contestant

MATH

CONTINUE
© TPR Education IP Holdings, LLC  |  423
SAT MANUAL

TRANSLATING
READING

7 31
A teacher is ordering lab kits for a chemistry Dr. Khan is combining a 20% acetic acid solution
course. Each lab kit costs $18. There is an additional with a 60% acetic acid solution. She wants to obtain
shipping charge of $12 per total order placed. The 10 liters of a 40% acetic acid solution. How many
WRITING AND

liters of the 20% acetic acid solution does Dr. Khan


LANGUAGE

professor estimates that there will be 15 to 20


students in the class. If only one order, costing a need to create the 40% acetic acid solution?
total of t dollars, is placed for lab kits, which of the
following inequalities includes both the maximum
and minimum values of t, in dollars?
A) 189 £ t £ 318
B) 270 £ t £ 360
MATH

C) 282 £ t £ 372
D) 350 £ t £ 600

21
Marcus is evaluating two different types of savings
accounts. Account A will steadily increase his
savings over time at a constant rate and requires
a small initial deposit, while Account B will 34
exponentially increase his savings but requires a A baseball team has won 18 of the 30 games it has
larger initial deposit. Based off of this information, played during the season. What is the least number
which of the following can be concluded about the of additional games the team must win in order to
two savings accounts and their graphs? raise its winning record above 75% ?
A) On the day of account opening, x = 0, the
y-value of Account A will be greater than the
y-value of Account B.
B) On the day of account opening, x = 0, the
y-value of Account A will be smaller than the
y-value of Account B.
C) On the 364th day after account opening,
x = 365, the y-value of Account A and Account
B will be equal.
D) On the 364th day after account opening,
x = 365, the y-value of Account A will be exactly
triple the y-value of Account B.

CONTINUE
424  |  © TPR Education IP Holdings, LLC
TRANSLATING AND MEANING IN CONTEXT PRACTICE

MEANING IN CONTEXT

READING
22 27
The cost of a cell-phone call using a certain carrier
M = (278 . 10 –23) . 0.968t is $0.20 per minute for the first 5 minutes, and
$0.15 per minute for any time over 5 minutes. If

WRITING AND
LANGUAGE
Scientists studying changes in the nucleus of the cost in dollars, C, of a call that lasts t minutes,
an unstable atom noticed that the mass of the where t is an integer greater than 5, can be
atom, in grams, at a given time declined at a rate represented by the equation C = 0.15x + 1, which of
proportional to the mass of the atom at that time, the following must be true?
such that the mass decreased by 3.2% of the present A) x = t + 5
mass of the sample, per second. They further B) x = t
found that the mass of the atom, M is given by
C) x = t – 1

MATH
the equation above when t ³ 0, where t is time in
seconds. What does the quantity 278 . 10–23 from D) x = t – 5
the expression above represent?
A) The mass in grams lost between t = 0 and t = 1
B) The mass in grams of the atom after t seconds
C) The initial mass in grams of the atom
D) The mass in grams lost between t = 1 and t = 2

Answers can be found on page 736.

© TPR Education IP Holdings, LLC | 425


SOLVING
101

A good foundation in the fundamentals is


key to improving your math score. In this
chapter, you’ll review the terms and rules
you will need for the SAT.

427
SAT MANUAL
READING

ORDER OF OPERATIONS
Write in the math operations for these terms. If you get stuck, check the glossary
at the end of this chapter.

Term Definition
WRITING AND
LANGUAGE

Sum

Difference

Product
MATH

Quotient

What is the order of operations?

_______________________________________________________________________________

What’s one famous saying to help you remember the order of operations?

_______________________________________________________________________________

Absolute Value
What does the absolute value sign do to a number?

_______________________________________________________________________________

1. −3 + −5 = _________________________________________________________

2. 4  7  __________________________________________________________

3. − −2 + −3 =________________________________________________________

4. −8 + 5 = ___________________________________________________________

Now, define absolute value.

Term Definition

Absolute Value

428  |  © TPR Education IP Holdings, LLC


SOLVING 101

READING
Rational Numbers (and Their Irrational Friends)
1. Write .5 as a fraction. ________________________________________________

2. Write 3 as a fraction. _________________________________________________

WRITING AND
3. For the number .83, what digit is coming next? _______________________

LANGUAGE
4. For the number .57, what digits are coming next? ______________________

5. For π, do the digits follow a pattern? __________________________________

6. For 2 , do the digits follow a pattern? _______________________________

MATH
A rational number is any number that can be written as a
fraction; that includes integers and repeating decimals.

We ♥ Our Calculators In Section 4, you can use


On your calculator, you should be able to convert a decimal into a fraction if the your calculator to convert
decimals to fractions.
number is rational. If you don’t know how to do it, dig up the guide that came
with your calculator, search for it online, or ask your math teacher. Use your cal-
culator to turn the following into fractions (if possible).

7. Type .375 into your calculator. ______________________________________

8. Type .1666666666666 (until end of screen). _________________________

9. Type π. _____________________________________________________________

10. Type .1572751.______________________________________________________

Irrational numbers are like irrational people: you never know


what they’re going to do next.

© TPR Education IP Holdings, LLC | 429


SAT MANUAL
READING

MANIPULATING EQUATIONS
Solving for a variable is a key skill you can expect to see the SAT test you on
directly. But solving for an unknown (i.e., variable) is at the heart of many word
problems too.
WRITING AND
LANGUAGE

Linear Equations

SOLVING AN EQUATION
Isolate the variable.
Get the variable on one side of the equals sign and the
MATH


numbers on the other.
• Add, subtract, divide, or multiply both sides of the
equation to gather like terms.
Treat both sides equally.
• Perform the same operations on both sides of the equation.

Solve for x in each of these equations.

1. 6 x + 3 = 15

3x
2. + 9 = 33
4

3. 3( x + 5) = 23

4. 12( x − 4 ) = 36

5. 2( x − 7 ) = 6 x + 10

2x
6. −5=9
4

7. 4( x − 5) + x = 25

430 | © TPR Education IP Holdings, LLC


SOLVING 101

READING
Inequalities
Solving an inequality is just like solving an equation. There’s just one more rule to
follow.

If you multiply or divide both sides by a negative

WRITING AND
LANGUAGE
number, flip the inequality sign.

1. 3 x − 8 < 12 + 5 x

MATH
2. 7 x − 5 < 13 + 4 x

7 − 2x
3. < −5
3

Systems of Equations
Double your fun! If there are two variables and two equations, stack them. Add or
subtract to make one variable cancel out.

3x + 2 y = 7
− ( 2 x + 2 y = 9)
x = −2

SOLVING A SYSTEM OF EQUATIONS


• Pay attention to which variable to isolate.
• Add or subtract to eliminate the other variable.
• Multiply by a constant if necessary.

© TPR Education IP Holdings, LLC | 431


SAT MANUAL
READING

Use this method of stacking and solving to find the value for x, then solve for y.

1. 2 x + 3 y = 25 and 3 x − 3 y = 25
WRITING AND
LANGUAGE

2. − x − y = −2 and 2 x − y = −11
MATH

3. 2 x + 4 y = 32 and 4 x + 2 y = 34

On many systems of equations questions on the SAT, PITA may be faster and
more accurate. If the question asks for values of x, y, or both (rather than some-
thing weird like x – y), then use PITA.

Answers can be found on page 736.

432 | © TPR Education IP Holdings, LLC


SOLVING 101

READING
GLOSSARY
The following list contains useful math terms for the SAT. The ones covered in
the previous chapter will be the most important to improving your score, but the
others here are good to know as well. A few topics, such as exponents and imagi-
nary numbers, will be explored more in later chapters.

Absolute Value:  The distance of a number from zero on the number line

WRITING AND
LANGUAGE
Consecutive:  Numbers that follow one another from smallest to largest or
largest to smallest (The numbers 2, 3, and 4 are consecutive integers, while
2, 4, and 6 are consecutive even integers.)
Decimal:  A way of expressing a fraction in which numbers are divided by ten,
one hundred, one thousand, and other powers of ten
Denominator:  The bottom number in a fraction

MATH
Difference:  The result of subtraction
Digit:  A whole number 0 through 9 that is part of a larger number
(The number 12 contains the digits 1 and 2.)
Distinct:  Different
Divisible:  An integer that can be divided by another integer evenly, with no
fraction or decimal left over (The number 12 is divisible by 3.)
Even:  An integer that is divisible by 2
Exponent/Power:  A number that indicates how many times to multiply a base
number or variable by itself (The number 24, where 2 is the base and 4 is the
exponent, becomes 2 × 2 × 2 × 2.)
Factors:  Numbers that a given number is divisible by (The number 4 is a factor
of 24.)
Fraction:  A way of expressing the division of numbers as a ratio by stacking
one over the other
Greatest Common Factor:  The largest number that divides into two other
numbers (The greatest common factor of 30 and 42 is 6.)
Imaginary:  The square root of a negative number
Integers:  Commonly known as whole numbers, or all real numbers that are
not decimals or fractions (The numbers 0, 10, and –1,000 are integers.)
Irrational:  A number that can be expressed as a decimal but cannot be
expressed as a fraction
Least Common Multiple:  The smallest common multiple of two numbers
(The least common multiple of 30 and 40 is 120.)
Multiples:  Numbers that are divisible by a given number (The number 24 is a
multiple of 4.)

© TPR Education IP Holdings, LLC  |  433


SAT MANUAL

Negative:  All real values less than zero


READING

Numerator:  The top number in a fraction


Odd:  An integer that is NOT divisible by 2
Order of Operations:  Parentheses, Exponents, Multiplication, Division,
Addition, Subtraction (PEMDAS)
WRITING AND
LANGUAGE

Positive:  All real values greater than zero


Prime:  A number that has itself and 1 as its only factors (Some examples of
prime numbers are 2, 3, 5, 7, 11, and 79. The number 1 is not a prime
number.)
Product:  The result of multiplication
Quotient:  The result of division
MATH

Real:  All positive and negative numbers, zero, fractions, decimals, and irratio-
nal numbers
Radical:  A term to express the sign
Rational:  A number that can be expressed as a fraction
Reciprocal:  The inverse of a fraction—flip the numerator and denominator
2 5 1
(The reciprocal of is , and the reciprocal of 5 is .)
5 2 5
Remainder:  The number left over when a number is not divisible by another
number
Sum:  The result of addition

434  |  © TPR Education IP Holdings, LLC


SOLVING
(LINEAR)

The essence of mathematics is not to make simple


things complicated, but to make complicated things
simple.
–Stan Gudder

435
SAT MANUAL
READING

GOALS REVIEW
At the conclusion of this chapter, you will be able to accomplish the following:

• Know when to solve for a variable or variables


• Solve for a variable or variables in equations
• Solve equations involving absolute values
WRITING AND

• Solve simultaneous equations and inequalities


LANGUAGE
MATH

436  |  © TPR Education IP Holdings, LLC


SOLVING (LINEAR)

READING
BEYOND PLUGGING IN
Plugging In and Plugging In the Answers are great tools for a wide variety of ques-
tions, as seen in the first two math chapters. But what do you do when Plugging
In isn’t an option or PITA seems too complicated or time-consuming?

WRITING AND
LANGUAGE
Finding the Solution
If you can’t use Plugging In or PITA and need to solve for a variable, you need to
isolate that variable. We covered the basics of solving in Solving 101, so this chap-
ter will focus on some of the strange ways College Board might present solving
questions.

MATH
Isolate the variable. Perform the same operations
on both sides of the equation.

v = 13.5 + 1.25t

4. A particle travels in a straight line at a constant speed of Which is faster here—


PITA or solving?
13.5 meters per second. When the particle begins a constant
acceleration of 1.25 meters per second, the particle travels at
a velocity of v meters per second at time t seconds as shown
in the equation above. What is t when v is 58.3 ?
A) 13.50
B) 35.84
C) 57.44
D) 86.38

© TPR Education IP Holdings, LLC | 437


SAT MANUAL

6. Gibbs free energy, G, is a thermodynamic quantity that is equal to the


READING

enthalpy of a system minus the product of the entropy and the absolute
temperature. The equation G = U + PV – TS illustrates how internal energy
(U), pressure (P), volume (V), temperature (T), and entropy (S) are all used
to determine Gibbs free energy. Which of the following expressions best
describes how to find the absolute temperature, T, that is necessary for a given
system?
WRITING AND
LANGUAGE

G  U  PV
A) T 
S

G  U  PV
B) T 
S

U  PV  G
MATH

C) T 
S

U  PV  G
D) T 
S

2. If 7y – 2 = 15, what is the value of 21y – 1 ?


A) 17
B) 31
C) 45
D) 50

You don’t always need to completely isolate the


variable to answer the question. RTFQ!

438 | © TPR Education IP Holdings, LLC


SOLVING (LINEAR)

READING
INEQUALITIES
Working with inequalities is a lot like working with equations, in that you must
preform the same operations on both sides of the inequality. However, there is one
important difference.

WRITING AND
LANGUAGE
If you multiply or divide both sides by a negative
number while working on an inequality, remember to
flip the inequality sign.

MATH
–10x + 4y > 40

6. Which of the following inequalities is equivalent to the inequality


above?
A) x – y > –4
B) x – y > –10
C) 5x – 2y < –20
D) –5x + 2y < 20

32. In a particular board game, silver pieces are worth 4 points and gold If the inequality is not
given to you, you may
pieces are worth 9 points. If Byron has between 125 and 135 points
have to write it out.
with s silver pieces and 10 gold pieces, what is one possible value of s ?
There’s more on this
skill in the Translating
chapter.

© TPR Education IP Holdings, LLC | 439


SAT MANUAL
READING

RATIONAL EQUATIONS
College Board also likes to test rational equations, which are equations made up of
fractions. Rational equations can often be solved by Plugging In and PITA. How-
ever, when this is too complicated, cross-multiplying is a good alternative.

Which is faster here— 2m m  3


8. If , what is the value of m ?
WRITING AND


LANGUAGE

PITA or cross-multiplying 5 12
and solving? 3
A)
19
1
B)
3
15
C)
29
MATH

15
D)
19

Solving with pencil and paper can sometimes be


faster and more accurate than using a calculator.

Another way College Board may present solving with fractions may look like the
one below. To solve this one, first find a common denominator for the two frac-
tions using the Bowtie Method.

7 1 1 3
18. If   y  y , what is the value of y ?
9 6 8 8

440 | © TPR Education IP Holdings, LLC


SOLVING (LINEAR)

READING
BOWTIE METHOD
1. Multiply diagonally up (opposing denominators and
numerators).

42 9
7 1

WRITING AND
-

LANGUAGE
9 6

2. Carry up the addition or subtraction sign.


42 - 9
7 1
-
9 6

MATH
3. Add or subtract across the top.

42 - 9
7 1 33
- =
9 6

4. Multiply across the bottom.


42 - 9
7 1 33
- =
9 6 54

5. Reduce, if necessary.

42 - 9
7 1 33 11
- = =
9 6 54 18

Now set that equal to the other side of the equation and solve for y!

© TPR Education IP Holdings, LLC | 441


SAT MANUAL

5
READING

34. Which of the following is a value of y for which the expression 3 5


y−
4 8
is undefined?
WRITING AND
LANGUAGE
MATH

Rational expressions are undefined when the


denominator is equal to 0.

442 | © TPR Education IP Holdings, LLC


SOLVING (LINEAR)

ABSOLUTE VALUE

READING
Some questions will ask about an absolute value, which is the distance a value is
from 0 on the number line. For positive numbers and 0, the absolute value is equal
to the value of the number. For negative numbers, find the absolute value by drop-
ping the negative sign.

WRITING AND
LANGUAGE
2 = |n – 7|

17. The value of one solution to the equation above is 9. What is value of the other Remember to follow the
solution? order of operations!

MATH
If |x| = c, then x = c or x = –c.

2. Which of the following expressions could be equal to 1 for some value of a ? What must be true of
the absolute value of any
A) |a + 2| + 2 expression?
B) |a – 2| + 2
C) |2 – a| – 2
D) |2 – a| + 2

© TPR Education IP Holdings, LLC | 443


SAT MANUAL
READING

SIMULTANEOUS EQUATIONS
When given two equations with two variables, there are a number of approaches
for solving them. The way the equations are written and what the question is ask-
ing will determine whether you use elimination or substitution to solve.

4x – 5y = 15
WRITING AND
LANGUAGE

2x – y = 9

Make sure to RTFQ! 5. In the system of equations shown above, what is the value of 3x – 3y ?
A) 9
B) 12
C) 15
MATH

D) 24

2y + 9x = 8
–3x + y = –11

8. What is the solution (x, y) to the system of equations shown above?


A) (–3, 7)
B) (–2, 13)
C) (2, –5)
D) (3, 8)

Stack the equations up, then add or subtract. If you


need to solve for one variable, try to make the other
one disappear! Sometimes, you may need to
manipulate one of the equations first.

444 | © TPR Education IP Holdings, LLC


SOLVING (LINEAR)

y = 9(x – 2)

READING
y
15 =
x
2. If (x, y) is a solution to the system of equations above, what is the value What strategy can you
of x ? use when asked for a
specific value? Why is

WRITING AND
A) –45

LANGUAGE
that not the best
B) –15 approach on this
question?
C) –9
D) –3

MATH
On more complex questions, you need to use a method
other than stacking and solving. Sometimes
substitution is needed.

© TPR Education IP Holdings, LLC | 445


SAT MANUAL

SOLVING (LINEAR) DRILL


READING

Time: 8 minutes

Unless you are aiming for a top score, don’t try all the questions! Use your POOD to choose the best ones for you.
WRITING AND
LANGUAGE

4 17

x 1 3x + y = –13
If  4  2  , then which of the following is
y y x + y = –3
equivalent to x, in terms of y ?
MATH

According to the system of equations above, what is


A) x = y + 6 the value of y ?
6
B) x =
y
C) x = 6y + 1

1
D) x = y –
6

446  |  © TPR Education IP Holdings, LLC


SOLVING (LINEAR)

3 18

READING
If 7(x – y) = 2, what is the value of x – y ? If 2 ≥ 5p + 7, what is the greatest possible value of
5p – 7 ?
2 A) –1
A)
7 B) –5
B) 2 C) –7

WRITING AND
LANGUAGE
D) –12
7
C)
2

D) 7

29

MATH
Which of the following equations can be solved to
find the points that are a distance of 5 units away
from the point 2 on the number line?
5
A) |x – 5| = 2
5 2 B) |x + 5| = 2

3x  2 4  5x
C) |x – 2| = 5
In the equation above, what is the value of x ? D) |x + 2| = 5

24
A) –
19
16
B) –
19
16
C)
31
24
D)
19

© TPR Education IP Holdings, LLC  |  447


SAT MANUAL

Summary
• What is one essential math strategy you
can use when Plugging In and PITA are not
effective options?

______________________________________

______________________________________

• What is the ultimate goal when solving for


a variable?

______________________________________

• When solving an inequality, what must


you remember to do when you multiply or
divide by a negative number?

______________________________________

• When do you use the Bowtie Method?

______________________________________

• What are two ways to solve simultaneous


equations without using a calculator?

______________________________________

______________________________________

• I have accomplished _________ of the 4


goals stated at the beginning of this chapter.

448 | © TPR Education IP Holdings, LLC


SOLVING (LINEAR)
PRACTICE

449
SAT MANUAL

SOLVING EQUATIONS
READING

3 9
The time it takes a car to accelerate from a complete The formula for acceleration can be expressed as
1
d  vt  at 2 , where d is the distance traveled, v is
stop to a speed of 60 miles per hour is based on the 2
WRITING AND

the initial velocity, t is the time taken, and a is the


LANGUAGE

velocity, V, in meters per second, the car’s average


acceleration. Which of the following expresses a in
power, P, in watts, and the mass of the car, m, in terms of d, v, and t ?
3PT
kilograms. If V = , which of the following 2d 2v
m A) a  2

t t
gives the value of T, the time in seconds, in terms of
d v
MATH

V, P, and m ? B) a  2

2t 2t

3V 2 m 2d 2v
A) T = C) a  2

P t t

3V 2 D) a = 2dt2 – 2vt3
B) T =
mP

V 2P
C) T =
3m

V 2m
D) T =
3P

CONTINUE
450  |  © TPR Education IP Holdings, LLC
SOLVING (LINEAR) PRACTICE

SOLVING INEQUALITIES

READING
20

23 3 1
− < − c −2< −
4 4 2

WRITING AND
LANGUAGE
What is one possible nonnegative integer solution to
the inequality above?

-21 < -3c -8 < -2

MATH
-15<-3c<6

CONTINUE
© TPR Education IP Holdings, LLC  |  451
SAT MANUAL

SOLVING RATIONAL EXPRESSIONS


READING

2 17

11 2 1
2  1
k1 2z  2 z  1
WRITING AND
LANGUAGE

In the equation above, what is the value of k ? In the equation above, what is the value of z ?
A) –11

13
B) -
2
C) –6
MATH

9
D) -
2

15

5(x  2)  6 5x

3x 11  3(2  x )

In the equation above, what is the value of x ?

13
A) -
7

20
B) -
37
8
C)
7
20
D)
13

CONTINUE
452  |  © TPR Education IP Holdings, LLC
SOLVING (LINEAR) PRACTICE

SOLVING EQUATIONS

READING
3 34
The kinetic energy, KE, of an object can be
1 12 2
determined by the equation KE = mv 2 , where m (r - 2)  3r - 2(2r - 1)
2 5 3

WRITING AND
is the mass of the object and v is the velocity of the

LANGUAGE
In the equation above, what is the value of r ?
object. Which of the following gives the value of v,
in terms of KE and m ?

m
A) v =
2KE

MATH
KE
B) v =
2m

2KE
C) v =
m
2KE
D) v =
m

13

 1
7  3w   - 3w  1
 4

In the equation above, what is the value of w ?

1
A) -
24
1
B) -
28
1
C) -
32

3
D)
28

CONTINUE
© TPR Education IP Holdings, LLC  |  453
SAT MANUAL

SOLVING INEQUALITIES
READING

6 33
If –3(j + 4) < 6, which of the following accurately
3 a3 a 1
expresses all possible values of j ? If − ≤ , what is the least possible
8 4 2
A) j < –6
WRITING AND

integer value of a ?
LANGUAGE

B) j > –6
C) j < 2
D) j > 2
MATH

28

5 3
If - ≤ 3s – 2 ≤ - , which of the following
3 2
accurately expresses all possible values of –12s + 9 ?

31
A) –11 ≤ –12s + 9 ≤ −
3
17
B) 5 ≤ –12s + 9 ≤
3
20
C) 6 ≤ –12s + 9 ≤
3
23
D) 7 ≤ –12s + 9 ≤
3

CONTINUE
454  |  © TPR Education IP Holdings, LLC
SOLVING (LINEAR) PRACTICE

SOLVING RATIONAL EXPRESSIONS

READING
10 35

x 1 6 1 3 2b
If  , what is the value of ? If   2 , what is the value of b ?
x 5 x 2b b  3

WRITING AND
LANGUAGE
1
A)
5
5
B)
6

C) 5

MATH
D) 6

CONTINUE
© TPR Education IP Holdings, LLC  |  455
SAT MANUAL

SIMULTANEOUS EQUATIONS
READING

5 34
7w + 2z = 16 At a corner bakery, 6 croissants and 3 cups of coffee
5w = 2z + 8 costs $25.95. An order of 4 croissants and 6 cups of
coffee costs $26.70. What is the cost of an order of 1
WRITING AND

croissant and 1 cup of coffee? (Disregard the dollar


LANGUAGE

Based on the system of equations above, what is the


value of 12w ? sign when gridding your answer.)
A) 2
B) 8
C) 16
D) 24
MATH

21
4y – 3x = 8
ax + 4 = 2y

If the system of equations above has infinitely many


solutions, what is the value of constant a ?

3
A) -
2

B) 3
2

C) 1

D) 2

Answers can be found on page 737.

456 | © TPR Education IP Holdings, LLC


SOLVING
201

The Solving 101 chapter provided you with a


foundation in the linear fundamentals key to
improving your math score. In this chapter,
you’ll review the more advanced terms and rules
related to non-linear concepts you will need for
the following chapter.

457
SAT MANUAL
READING

QUADRATICS
Consider the following equation.

x + 12 = 7x

You may be able to solve this fairly easily, but what if the question were a bit
WRITING AND

harder?
LANGUAGE

x 2 + 12 = 7x

This is an example of a quadratic equation. Quadratic equations are equations


that can be put into the following form:

ax 2 + bx + c = 0
MATH

This type of equation appears frequently in the SAT Math sections. Fortunately, it
is one that you can learn to solve.

FACTORING
The first thing to do is move everything to one side of the equation so that 0 is on
the other side. In this equation, subtract 7x from both sides to get

x 2 – 7x + 12 = 0

Now, factor the left side. The three terms do not have any common factors, but
that’s not a problem. First, write down the following:

(x )(x )=0

The next thing to notice is the sign of the c term. If the c is positive, the signs in
the factors match each other and the sign of the b term. If, instead, the c term is
negative, the signs in the factors will be different. In this case, c is positive and b is
negative, so you can add the minus signs to each factor.

(x − )(x − )=0

Now, find two factors of 12 (the c term) that have a sum of 7 (the b term). The fac-
tors of 12 are 1 and 12, 2 and 6, and 3 and 4. Of these three pairs, only 3 and 4
have a sum of 7. Therefore, the factored equation is

(x – 3)(x – 4) = 0

458  |  © TPR Education IP Holdings, LLC


SOLVING 201

READING
So how do you solve this equation? Remember that if a product equals 0, at least
one of the two factors must be 0. Set each factor equal to 0 and solve.

(x – 3) = 0 or (x – 4) = 0

WRITING AND
LANGUAGE
x=3 or x=4

3 and 4 are the solutions, or roots, of the equation. With quadratic equations,
there may be (and often will be) two distinct solutions. Let’s look at another
example.

x2 + x – 6 = 0

MATH
This time, the sign of the c term is negative. Remember, when this happens, the
signs of the factors should be different, so write

(x + )(x − )=0

Because the signs are different, you also need two factors of 6 that have a differ-
ence of the b term, which in this case is 1. The factors of 6 are 1 and 6 or 2 and 3.
Because 2 and 3 have a difference of 1, these must be the factors. The larger
factor always gets the sign of the b term. Since b is positive, factor this as

(x + 3)(x – 2) = 0

Now just set each factor equal to 0 and solve. What are the solutions to this
equation?

FOIL
Sometimes, the SAT will give the factored form and ask for the expanded form. In
this case, use the FOIL method. FOIL stands for

First

Outer

Inner

Last

© TPR Education IP Holdings, LLC  |  459


SAT MANUAL
READING

If you’re given the expression

(x + 5)(x – 2)

First, multiply the first terms


WRITING AND

(x + 5)(x – 2) = x2 …
LANGUAGE

Then, multiply the outer terms

(x + 5)(x – 2) = x 2 – 2x …

Then, multiply the inner terms


MATH

(x + 5)(x – 2) = x 2 – 2x + 5x …

Then, multiply the last terms

(x + 5)(x – 2) = x 2 – 2x + 5x – 10

Finally, combine like terms to get

(x + 5)(x – 2) = x 2 + 3x – 10

On the SAT, it can also be helpful to memorize three common quadratics. They
show up often in difficult quadratic problems. You can always factor or FOIL, but
having these memorized can save time and energy on the test.

COMMON QUADRATICS
(x + y)2 = x 2 + 2xy + y2

(x − y)2 = x 2 − 2xy + y2

(x + y)(x – y) = x 2 – y2

If you come across a difficult quadratic question, look for one of these three
quadratics.

460 | © TPR Education IP Holdings, LLC


SOLVING 201

READING
QUADRATICS PRACTICE
Solve the following quadratic equations.

1. x 2 + 3x + 2 = 0 6. x 2 + 10x = 24

2;-12

WRITING AND
LANGUAGE
-1; -2

2. x 2 – 6x + 5 = 0 7. 5x = 14 – x 2

MATH
5;1
2;-7

3. x 2 + 2x – 8 = 0 8. x(x + 6) = −9

2;-4

-3

4. x 2 – 3x – 10 = 0 9. x 2 – 4 = 0

+-2
5;-2

x −8 4
5. x 2 = 5x + 6 10. =−
4 x

6;-1
4

© TPR Education IP Holdings, LLC  |  461


SAT MANUAL
READING

EXPONENTS
Exponents are a shorthand way of indicating that a number (known as the base) is
multiplied by itself: the exponent tells you how many times. 73 = 7 × 7 × 7.

Put these exponent


WRITING AND

Whenever the bases are the same


LANGUAGE

rules all together


and the acronym is
MADSPM. • to multiply the quantities, keep the base and add the
M ______________ exponents.
A ______________ • to divide the quantities, keep the base and subtract the
D ______________
S ______________
exponents.
P ______________ • to raise the quantity to another power, multiply the
M ______________ exponents.
MATH

If quantities with the same base and exponent are added or sub-
tracted, just add or subtract the coefficients and do nothing to the
base or exponent.

Basic Rules
1. ( x 2 )( x 3 ) = 5. (2 x ) 2 3
=

x6 8x 3
2. = 6. =
x2 4x 2
3. (x ) 4 2
= 7. 3 x 2 + 5x 2 =

4. 2x 2 × 6 y3 = 8. 2x 2 + 4 y3 =

Special Rules
x5
1. 5
= 5. x1 = 9. (−2 )3 =
x
2
1
x0 = 6. =   =
1, 276
2. 1 10.
2
2
x
3. = 7. 0243 =
x4
4. x −2 = 8. (−2 )2 =

462 | © TPR Education IP Holdings, LLC


SOLVING 201

READING
11. Any number to the exponent 0 is________________________________________

12. A negative exponent means______________________________________________

13. Any number to the exponent 1 is________________________________________

WRITING AND
14. 1 to any exponent is_____________________________________________________

LANGUAGE
15. 0 to any exponent (besides 0) is__________________________________________

16. A negative number raised to an even exponent is_________________________

17. A negative number raised to an odd exponent is__________________________

MATH
18. A fraction between 0 and 1 raised to a positive exponent gets_____________

© TPR Education IP Holdings, LLC  |  463


SAT MANUAL
READING

EXPONENTS PRACTICE
Simplify and solve all of the following expressions.

57
1. 6. (3x4 + 2x3)x 2
54
WRITING AND
LANGUAGE
MATH

x3x4 53 × 6 4
2. 7.
x2 25 × 6 2

7 2 × 75
3. (3x y z )
3 6 5 4
8.
74

15 x 5 y 3
4. 53 – 33 9.
3x 3 y 5

9x 3 y 4
5. 10. a-4b-1d -3g 9 × d -2a5c6e-3 × b3d7fg-11 × e4c-5b-1g 3d -1
3 xy 2

464  |  © TPR Education IP Holdings, LLC


SOLVING 201

READING
Working with Fractional Exponents
Remember that mathematicians use exponents and roots as shortcuts to represent
repetitive multiplication and division. Well, sometimes you need to symbolize a
square root as an exponent. The way you do this is with a fractional exponent.
Consider the following example.

WRITING AND
1

LANGUAGE
42

1
Notice that the exponent in this case is . This is the way to symbolize a square
2 1
root, so the solution to this expression is 4 , or 2. What if you’re given 64 3 ? The
number on the bottom of the fractional exponent tells you what root you need.
Therefore,

MATH
1
64 3 = 3 64

Sometimes, you will see a fractional exponent with a number other than 1 in the
numerator, like this one:

3
42

3
This time, the fraction is a little more complicated. The exponent represents a
2
two-part calculation: the 3 represents the exponent by which the base, which in

this case is 4, is raised.

The 2 represents a square root, just like before. So the solution will look something
like the following.
3
42

43

64
8

For fractional exponents, you can choose to apply either the exponent or the root
first. This can also be solved as ( 4 )=
3
2=
3
8.

i. How would you solve the expression 3 2 ? ________________________

© TPR Education IP Holdings, LLC  |  465


SAT MANUAL
READING

FRACTIONAL EXPONENTS PRACTICE


Simplify and solve all of the following expressions.

1
 12  3
WRITING AND

1. 64d 4 6.  64 
LANGUAGE

 
MATH

1 5

2. 81 2 7. x 2

3

2
 − 12  2
3. 8 3 8.  25 
 

1
25 2
4. 1
9. ( 3x-3 y5 )–2
125 3

1

5. (16 x 2 y6 ) 2 2

10. (216 x 9 y6 ) 3

466  |  © TPR Education IP Holdings, LLC


SOLVING 201

READING
ROOTS
In the same way that division is the opposite of multiplication and subtraction is
the opposite of addition, finding the root of a number is the opposite of raising a
number to an exponent. Therefore, you can use roots to solve equations involving
exponents. For instance, what is the value of x in this equation?

WRITING AND
x 2 = 81

LANGUAGE
What does the equation say? There is some number, x, that when multiplied by
itself is 81. What number multiplied by itself is 81?

Another way to describe what you just did is finding the square root. The
symbol, called a radical, is used to represent square roots. So 81 is another way

MATH
to write “the square root of 81.”

You may have noticed that some square roots are easy to figure out, like the square
root of 100, which is 10. Numbers with square roots that are whole numbers are
called perfect squares.

One way to find the square root of a number that is not a perfect square is by
breaking the number down into two factors, one of which is a perfect square.
Consider the following example.

75 = 25 × 3

Break up the square root into two square roots. Note that you can break apart
square roots only with multiplication and division.

75 = 25 × 3

Now, find the square root of 25.

75 = 5 × 3

Finally, write the 5 right next to the square root of 3 to represent the multiplication.

75 = 5 3

The square root is now in its most reduced form.

© TPR Education IP Holdings, LLC  |  467


SAT MANUAL
READING

Rationalizing the Denominator


In order for a fraction to be in its simplest form, there cannot be a root in the
denominator (bottom) of the fraction. Let’s say that the final step after working a
problem is the following:
5
2
WRITING AND
LANGUAGE

You aren’t allowed to leave that 2 in the denominator, so you need to get rid of
2
it. To do this, multiply by . Because any number divided by itself is 1, you
2
aren’t actually changing the value of your original fraction; you’re just playing

with its formatting. Then, just multiply across to get the simplified fraction.
MATH

5 2 5× 2 5 2
× = =
2 2 2× 2 2

Your new answer has the same value as the original, but it doesn’t have a root in
the denominator, so it’s in its simplest form.

6
ii. Can you simplify ? _______________________________________
3

Combining Roots
You can add or subtract square roots only when the numbers under the square root
sign are the same.

1. 4 x + 2 x =

2. 9 x − 3 x =

Multiplication and division are more flexible: different values can be combined
under the root.

3. ( x )( y ) =
x
4. =
y

468  |  © TPR Education IP Holdings, LLC


SOLVING 201

READING
We ♥ Our Calculators
If a question on Section 4 of the SAT asks you to simplify exponents or roots with
numbers instead of variables, you can use your calculator. Just be careful with
those parentheses!

5. 4 12 × 2 3 =

WRITING AND
LANGUAGE
32
6. =
8
( )
2
7. 75 + 12 =
1

8. ( 529 − 361 ) 2
=

MATH

© TPR Education IP Holdings, LLC  |  469


SAT MANUAL
READING

ROOTS PRACTICE

48
1. 100b 2 6.
3
WRITING AND
LANGUAGE

121
2. 3
125 7.
MATH

169

64
c6
3. 8.
4 c4

63 x 4 y 3
4. 31 × 31 9.
7x2 y

36
x2 10. −
5. 49
y2

Answers can be found on pages 737–738.

470 | © TPR Education IP Holdings, LLC


SOLVING
(NONLINEAR)

Mathematical science shows what is. It is the language


of unseen relations between things. But to use and
apply that language, we must be able fully to appreciate,
to feel, to seize the unseen, the unconscious.
—Ada Lovelace

471
SAT MANUAL
READING

GOALS REVIEW
At the conclusion of this chapter, you will be able to accomplish the following:

• Successfully work with exponents and roots


• Solve quadratics through factoring
• Use the quadratic formula when factoring is difficult
WRITING AND

• Combine the rules of exponents to solve more difficult root and


LANGUAGE

exponent questions
• Solve questions related to exponential growth and decay
MATH

472  |  © TPR Education IP Holdings, LLC


SOLVING (NONLINEAR)

READING
WORK YOUR QUAD(RATIC)S
Questions that ask for the specific factors of or solutions to quadratics can often be
solved with Plugging In and PITA. Harder questions may ask about some math-
ematical operation like the sum or product of those roots.

11. What is the sum of the distinct possible values of x for the equation When a quadratic is in

WRITING AND
LANGUAGE
x2 + 3x – 10 = 0 ? the form ax2 + bx + c = 0:
A) –3 The sum of the solutions
B) –2 b
equals - .
a
C) 3
The product of the
D) 5 c
solutions equals .
a

MATH
Try to use Plugging In or PITA on quadratics.
When a quadratic is difficult to factor, remember
the quadratic formula:

-b ± b 2 - 4ac
x=
2a

4 2 1
12. What are the solutions for x if x  2 x  ?
5 5

5 29
A) x   
4 4

5 21
B) x   
4 4

21
C) x  1 
4

5 21
D) x  
4 4

© TPR Education IP Holdings, LLC | 473


SAT MANUAL
READING

More advanced questions on quadratics do not necessarily require the use of more
advanced math. Rather, they require trickier applications of the basics.

25x4 + 40x2y2 + 16y4


WRITING AND

5. Which of the following expressions is equivalent to the expression above?


LANGUAGE

Start by looking for per-


fect squares in the first A) (25x + 16y)4
and last terms.
B) (25x2 + 16y2)2
C) (5x + 4y)4
D) (5x2 + 4y2)2
MATH

(x + d)(x + e) = x2 + fx + 33

If you need to multiply 14. In the equation above, d, e, and f are positive constants. Which of the
the factors of a following are the possible integer values of f ?
quadratic, use FOIL. A) f = 0 or 1
B) f = 1 or 33
C) f = 2 or 16.5
D) f = 14 or 34

474  |  © TPR Education IP Holdings, LLC


SOLVING (NONLINEAR)

READING
EXPONENTS AND ROOTS
Many questions about exponents and roots can be solved with Plugging In and
PITA, especially when calculator use is allowed. However, sometimes that is not
an option, as in the following question.

10. If a5 = −4, what is the value of a15 ? Remember MADSPM!

WRITING AND
LANGUAGE
M ______________
A) −64
A ______________
B) −12 D ______________
C) 16 S ______________
P ______________
D) 60 M ______________

MATH
We went over negative and fractional exponents in Solving 201. College Board will
make things more difficult by combining these concepts.

c 3d −3
3
cd 2

15. Which of the following is an equivalent form of the expression above for all
positive values of c and d ?
A) c2d−5

7 13
B) c6d 6

7
c6
C) 13
d6
7
c6
D) 5
d6

With fractional exponents, the numerator is the


exponent, and the denominator is the root. You can apply
either one first to make the calculations easier. For negative
exponents, calculate the positive exponent, then take the
reciprocal.

© TPR Education IP Holdings, LLC | 475


SAT MANUAL
READING

16a
13. Which of the following is equivalent to the expression ?
64b
a
 1 b
A)  
4
WRITING AND
LANGUAGE

2a 3b
1
B)  
4
a
C) 4 b
MATH

D) 42a – 3b

476  |  © TPR Education IP Holdings, LLC


SOLVING (NONLINEAR)

READING
GROWTH AND DECAY
Another type of question related to exponents may ask about the exponential
growth or decay of a population or group over time. If something is increasing or
decreasing by a constant percent or multiplier over a set period, use the growth/
decay formula.

WRITING AND
LANGUAGE
EXPONENTIAL GROWTH OR DECAY FORMULAS
When the growth is a percent of the total population, use
final amount = original amount (1 ± rate)number of changes.
When the growth is a multiple of the total population, use
final amount = original amount (multiplier)number of changes.

MATH
13. Ruwanthi’s car was worth $5,000 when she bought it. Over the next Knowing the formula
several years, the car’s value decreased by 10 percent per year. Which of will help you use POE.
the following functions gives the value, v, in dollars, of the car after n If calculator use were
years at this rate? allowed, what other
strategy could you use?
A) v(n)  5, 000  0.9n

B) v(n) = 5, 000(0.9)n

C) v(n) = 5, 000(0.1)n

D) v(n) = 5, 000(1.1)n

36. A couple buys a certain home that is worth $120,000. A real estate
agent tells the couple that the value of the home will increase by 12%
per year for the next ten years. The real estate agent uses the equation
H = 120,000(k) y to model the value, H, of the home after y years. What
value should the real estate agent use for k ?

© TPR Education IP Holdings, LLC | 477


SAT MANUAL
READING

Some exponential growth or decay questions will ask you to adapt formulas for
different units of time.

B = 100(1.09) h
WRITING AND
LANGUAGE

24. The equation above models the number of microbes, B, in a bacteria


sample h hours after observation begins. Of the following, which
equation models the number of microbes of bacteria in the sample m
minutes after observation begins?
A) B = 100(176) m
MATH

B) B = 100(1.0015) 60m

C) B = 100(1.09) 60m
m
D) B  100 1.09  60

27. In selecting a long-term investment, a stock broker predicts that the


value of a particular mutual fund will increase by 15% every ten years.
If the present value of the mutual fund is $20 per share, which of the
following expressions represents the stock broker’s prediction of the
value per share of the mutual fund y years from now?

y
A) 20 1.15 10

B) 20( 1.15)10y

y
C) 20  0.15 10

D) 20(0.15)10y

478  |  © TPR Education IP Holdings, LLC


SOLVING (NONLINEAR)

READING
Some questions will test exponential growth or decay without directly referenc-
ing any formula. Use your knowledge of the formula to solve questions about the
parts of the scenario.

36. A town has a population of p people based on a 2010 census. According


to research conducted by the town council, the population of the city is

WRITING AND
LANGUAGE
expected to increase by 5 percent per year. According to the estimate,
the expected population of the town in 2020 is 810. To the nearest
whole number, what is the value of p ?

MATH

© TPR Education IP Holdings, LLC  |  479


SAT MANUAL
READING

SOLVING (NONLINEAR) DRILL


Time: 8 minutes

Unless you are aiming for a top score, don’t try all the questions! Use your POOD to choose the best ones for you.
WRITING AND
LANGUAGE

12 14
Which of the following is the product of all values If r and s are constants in the equation
of x that satisfy the equation 5x2 – 20x – 10 = 0 ? x2 – rx = –4s, then what are the values of x ?
A) –2
r r 2  16s
MATH

B) −5 6 A) x  
2 2
C) 5 6
r r 2  16s
B) x  
D) 2 2 2

r 2  16s
C) x  r 
2

r r 2  16s
D) x   
2 2

480  |  © TPR Education IP Holdings, LLC


SOLVING (NONLINEAR)

8 20

READING
A grocery store wishes to increase its number of Which of the following expressions is equivalent to
customers by p percent per month. The grocery 1

store currently has c customers. Which function  x 9 y 27 3


 3  ?
f best represents the number of customers the  z 
grocery store wishes to have m months from now?
x3 y9
A)

WRITING AND
z

LANGUAGE
A) f(m) = pm – c
x3 y9
B)
z3
B) f(m) = pm + c
x 27 y 81
C)
 p 
m z9
C) f (m)  c  1  
 100 

MATH
z
D)
m x3 y9
 c 
D) f (m)  p  1  
 100 

25
3
c 4 = 2d 8
13
In the equation above, c and d are positive real
2
If x  6 x  66  5 , then what are the roots of the numbers. In terms of d, what is the value of c3 ?
equation? A) 16d32
A) x = –7 and x = 13 B) 8d32
C) 2d24
B) x = 7 and x = –13 D) 2d6

C) x = –7 and x = –13

D) x = 7 and x = 13

© TPR Education IP Holdings, LLC  |  481


SAT MANUAL

Summary
• What is the standard form of a quadratic
equation?

________________________________________

• What is the quadratic formula?


________________________________________

• When solving a quadratic equation using


FOIL, what are the four steps?

F_______________________________________

O ______________________________________

I _______________________________________

L_______________________________________

• What are the basic rules for manipulating


exponents?

M______________________________________

A ______________________________________

D______________________________________

S ______________________________________

P ______________________________________

M______________________________________

• What do the numerator and denominator


represent in fractional exponents?

________________________________________

• What do you do to manipulate negative


exponents?

________________________________________

• What formula do you use when a population


is increasing or decreasing by a percent
over time?

________________________________________

482 | © TPR Education IP Holdings, LLC


SOLVING (NONLINEAR)

• What formula do you use when a population


is increasing or decreasing by a multiple
or fraction over time?

______________________________________

• I have accomplished _________ of the 5


goals stated at the beginning of this chapter.

© TPR Education IP Holdings, LLC | 483


SOLVING
(NONLINEAR)
PRACTICE

485
SAT MANUAL

SOLVING QUADRATICS
READING

16 19
If t2 + 16t = 2t + 32, and t > 0, what is the value of t ? A right triangle with legs of lengths (x + 1) and
(2x – 2) has an area of 80. What is the length of the
shorter leg?
WRITING AND
LANGUAGE
MATH

CONTINUE
486  |  © TPR Education IP Holdings, LLC
SOLVING (NONLINEAR) PRACTICE

EXPONENTS AND ROOTS

READING
5 18
Which of the following expressions is equivalent to If 3(4 s  3)  2s , what is the value of s ?
1
(3x 2 y 3 ) 2 ?

WRITING AND
LANGUAGE
1
A)
9x 4 y 6
3
1 2
B) xy
9
3

MATH
C) xy 2 3

D) 9x4y6

CONTINUE
© TPR Education IP Holdings, LLC  |  487
SAT MANUAL

SOLVING QUADRATICS
READING

32 35
If x2 – 4x – 5 = 0, what is the sum of the possible
values of x ? (x  a)(x  3)  x 2  11ax  k
WRITING AND
LANGUAGE

In the equation above, a and k are constants. If the


equation is true for all real values of x, what is the
value of k ?
MATH

CONTINUE
488  |  © TPR Education IP Holdings, LLC
SOLVING (NONLINEAR) PRACTICE

SOLVING RATIONAL EXPRESSIONS EXPONENTS AND ROOTS

READING
23 15
Which of the following is equivalent to (5a – 7)–2 ?
h2  5h  6
h2  9 2 1
A)

WRITING AND


LANGUAGE
h2 3 5a - 7

In the equation above, what is the value of h ? 1


B) 2
25a - 70a + 49
7
A) − 1
3 C) 2
25a - 35a + 49
B) − 3

MATH
2
D) 25a2 – 70a + 49
11
C)
3
9
D)
2

CONTINUE
© TPR Education IP Holdings, LLC  |  489
SAT MANUAL

GROWTH AND DECAY


READING

6 23
Researchers are studying a tide pool that is home A radioactive element decays at a rate of 16% every
to 10 starfish. The population is monitored once three years. If a sample of that element contains 150g
per year. After several years of collecting data, in 2015, how many grams will remain after y years?
WRITING AND
LANGUAGE

researchers conclude that the population quintuples


y
every year. If t is the time in years, which equation A) 150(0.16) 3
for the population, S(t), best models their data? y
A) S(t) = 50t B) 150(0.84) 3
B) S(t) = 10(5)t
C) 150(0.16)y
C) S(t) = 50t
D) S(t) = 10 + 5t D) 150(0.84)y
MATH

15
Joy received a gift card to her favorite store that had
an initial value of $75. After she received it, the card
decreased in value by 4% for every month it was not
used. Unfortunately, she misplaced the card and
didn’t find it again until the value of the card was
$24.91. If n is the number of months the card was
unused, which of the following equations could be
used to solve for n ?
A) 24.91 = 75(1 + 0.04)n
B) 24.91 = 75(1 – 0.04)n
C) 75 = 24.91(1 + 0.04)n
D) 75 = 24.91(1 – 0.04)n

CONTINUE
490  |  © TPR Education IP Holdings, LLC
SOLVING (NONLINEAR) PRACTICE

26

READING
Time (hours) Concentration (mg)
0 100
1 79.37
2 62.966
3 50

WRITING AND
LANGUAGE
4 39.685
5 31.498
6 25

The half-life of a medication is the amount of time it


takes for the concentration of the drug in the body to
be reduced by one-half. Maddie took a 100-mg dose

MATH
of a particular medication and its concentration in
her body was measured every hour. The results are
shown in the table above. If c represents the drug’s
concentration in Maddie’s body and m represents
the time, in minutes, since she initially took the
medication, then which of the following equations
accurately expresses the drug’s half-life?
m
 1 180
A) c  100  
 2 m
12
B) c  100  
 2 m
1
C) c  100  
 2 3 m
1
D) c  100  
2

Answers can be found on page 738.

© TPR Education IP Holdings, LLC | 491


ADVANCED
SOLVING

The infinite in mathematics is always unruly


unless it is properly treated.
—James Newman

493
SAT MANUAL
READING

GOALS REVIEW
At the conclusion of this chapter, you will be able to accomplish the following:

• Use the discriminant to determine the number of solutions to a


quadratic
• Answer questions about the relationship between factors and roots of
WRITING AND

polynomials
LANGUAGE

• Factor or divide polynomials in more advanced ways


• Perform mathematical operations on imaginary and complex
numbers
MATH

494  |  © TPR Education IP Holdings, LLC


ADVANCED SOLVING

READING
MORE SOLVING (NONLINEAR)
In the last chapter, we looked at the most common ways that the SAT tests con-
cepts of polynomial equations. If you are aiming for a top score, you will also want
to know how to tackle these additional ideas that come up occasionally on the test.

WRITING AND
LANGUAGE
MORE ABOUT THE SOLUTIONS TO A POLYNOMIAL
You may see questions that ask about the number of solutions instead of what the
actual solutions are. If it is a system of equations, you still have to solve it to find
how many solutions you get.
y = 3x – 3

MATH
y = −2x2 + 7x − 1

13. The system of equations above has exactly how many solutions? What’s the best method
A) None to solve this system of
equations?
B) 1
C) 2
D) 3

If a question asks about the number of solutions to a quadratic, use the discrimi-
nant, which should look familiar as part of the quadratic formula.

For a quadratic in the form ax2 + bx + c, where a, b, and c are


real numbers, the discriminant is D = b2 – 4ac.
• If the discriminant is positive, the quadratic has two real
roots.
• If the discriminant equals zero, the quadratic has one real
root.
• If the discriminant is negative, the quadratic has no real
roots.

© TPR Education IP Holdings, LLC | 495


SAT MANUAL

14. If m is a variable and n is a constant, and 6m2 + 5m = n, then which of the


READING

following values for n would result in an equation with no real solutions?

6
A) n =
5
B) n = 1 D is Discriminant
WRITING AND
LANGUAGE

5 6m^2+5m - n=0
C) n  
6
6
D) n  
5

When you are solving a polynomial for its roots or solutions, you factor the poly-
MATH

nomial and set each factor equal to zero. As a result, knowing the solutions or
roots of a polynomial also tells you the factors of the polynomial, and vice versa.

x g(x)
0 –1
1 0
3 5
4 6

6. Abby is using the polynomial function g(x) to model the data she records,
which is shown in the table above. Which of the following must be a factor of
g(x) ?
A) x + 1
B) x – 1
C) x – 3
D) x – 4

For a given polynomial, if x = a is a solution or root, then


(x – a) is a factor.

496 | © TPR Education IP Holdings, LLC


ADVANCED SOLVING

px³ + qx² + rx + s = 0

READING
11. The percent increase of solar energy production in the United States
can be modeled by the equation above, in which p, q, r, and s are
rational number constants, and x is the number of months after
January of 1979. If the equation has factors of (x + 5), (x – 3), and
(x – 7), which of the following is a solution of the equation?

WRITING AND
LANGUAGE
A) 5
B) –3
C) –5
D) –7

MATH
On the topic of factoring, the SAT will sometimes give you equations that are
third-degree or higher and thus harder to factor. When this happens, try to group
similar terms together to factor something out.

x³ – 9x² + 3x – 27 = 0

17. For the equation above, there is one real root at x = a. What is the value What can be factored
of a ? out of the terms x³ – 9x²?
What can be factored
out of 3x – 27? What’s
left over?

You may even see a question that asks what combination of two different polyno-
mials can be factored by a certain binomial.

27. If h(x) = 3x3 + 18x2 + 15x, and j(x) = x2 + 6x + 5, then which of the Although Plugging In is
following functions is divisible by 3x + 1 ? possible on this one, it
could take a while and
A) t(x) = h(x) + 2j(x) may result in more than
B) u(x) = 2h(x) + j(x) one answer that works.
C) v(x) = 2h(x) + 2j(x)
D) w(x) = 3h(x) + j(x)

© TPR Education IP Holdings, LLC  |  497


SAT MANUAL
READING

Hard SAT questions may expect you to do long division on polynomials in a way
that makes Plugging In the Answers difficult. It helps to know a bit about how to
do one like this with algebra.

26 16b2  4b  28
 4b  2 
cb  1 cb  1
WRITING AND
LANGUAGE

15. In the equation above, b is a variable and c is a constant.


1
If b ≠ , then what is the value of c ?
c

A) −7
B) −4
MATH

C) 4
D) 7

To divide one polynomial by another, look at the leading terms in


both polynomials. The value that results from dividing the lead-
ing term in the numerator by the leading term in the
denominator is the first term of the quotient.

One last polynomial concept that may come up is the polynomial remainder
theorem. It states that the remainder when polynomial f(x) is divided by a linear
polynomial x – a is equal to f(a).

30. Which of the following must be true regarding d(x) if d(x) is a


polynomial and d(−9) = 1 ?
A) When d(x) is divided by x + 9, the remainder is 1.
B) x – 9 is a factor of d(x).
C) x + 9 is a factor of d(x).
D) x + 10 is a factor of d(x).

498 | © TPR Education IP Holdings, LLC


ADVANCED SOLVING

READING
IMAGINARY NUMBERS CAN GET COMPLEX
Occasionally, a question will contain an i. This isn’t a variable but rather an imagi-
nary number—the result of taking an even root of a negative value.

WRITING AND
LANGUAGE
The imaginary number i = −1 .
Treat i just like a variable, except that i 2 = –1.
a + bi is a complex number, where a is real and bi is imaginary.
Many calculators have an i button and an a + bi mode, but

MATH
these concepts are more likely to appear on Section 3, where
calculator use is not allowed.

Sometimes, solving questions with i is as simple as substituting – 1 for i 2. Practice


that on the following questions.

(2 + i)(2 – i) = ___________________________________________________

(3 + i)2 = _______________________________________________________

(7 – 2i) – (4i + 8i 2)

3. Which of the following complex numbers in the form a + bi is equivalent to


the expression above, for i = −1 ?
A) 15 – 6i
B) 1 + 2i
C) –1 – 2i
D) –15 + 6i

© TPR Education IP Holdings, LLC | 499


SAT MANUAL
READING

On hard questions, it may be necessary to multiply by the conjugate of the


denominator (switch the sign between terms) to make the i term in a denominator
disappear.

3i

3  i
WRITING AND
LANGUAGE

Remember—whatever 12. Which of the following complex numbers is equivalent to the above
you do to the denomina- expression? (Note: i = −1 )
tor must also be done to
the numerator. 4 3i
A) − −
5 5
4 3i
B)  
MATH

5 5
4 3i
C) −
5 5
4 3i
D) +
5 5

500  |  © TPR Education IP Holdings, LLC


ADVANCED SOLVING

Summary
• For a quadratic, the discriminant is
defined as _____________________________.

• What can you tell about the roots of a


quadratic based on the discriminant?

______________________________________

______________________________________

______________________________________

• For a given polynomial, if x = a is a solution


or root, then ______ is a factor.

• How do you factor by grouping?

______________________________________

______________________________________

______________________________________

• What is the first step of polynomial long


division?

______________________________________

______________________________________

• The polynomial remainder theorem states


that when f(x) is divided by ____________,
the remainder is equal to ________________.

• The imaginary number i is equal to


____________ and i2 is equal to __________.

• I have accomplished _________ of the 4


goals stated at the beginning of this chapter.

© TPR Education IP Holdings, LLC | 501


ADVANCED
SOLVING
PRACTICE

503
SAT MANUAL

SIMULTANEOUS EQUATIONS COMPLEX NUMBERS


READING

14 14
y = x2 + x – 10 If s, t, and v are real numbers and (s  2i)2  t  vi ,
y = 3x – 2 which of the following must be equal to t ?
(Note: i  1 )
WRITING AND
LANGUAGE

The equations above intersect each other at two A) s2 + 2


points. Which of the following is true about both
points of intersection? B) s2 – 2
A) x > –3 C) s2 + 4
B) x > –2 D) s2 – 4
C) y < 4
D) y > 5
MATH

CONTINUE
504  |  © TPR Education IP Holdings, LLC
ADVANCED SOLVING PRACTICE

COMPLEX NUMBERS

READING
10 21
If x = 5i, and i2 = –1, what is the value of x3 ? If the product of a and b is a real number and
A) –5 i2 = –1, which of the following could be values for
a and b ?
B) –125

WRITING AND
LANGUAGE
A) a = 3i and b = 6
C) –125i
B) a = 3i3 and b = 4
D) 125i
C) a = 2i and b = 4i4
D) a = i and b = 4i

MATH

Answers can be found on page 739.

© TPR Education IP Holdings, LLC | 505


CHARTS AND
DATA 101

Many questions on the SAT involve charts or


graphs of real-world data. This chapter will give
you the tools to dissect and analyze the variety
of figures you’ll see on the test.

507
SAT MANUAL
READING

SCATTERPLOTS
Graphs can present data in a variety of ways. In the scatterplot graph on this page,
each dot represents one data point. Sometimes, a line or curve “of best fit” will
be drawn to represent the equation that most closely matches the data. The term
scatterplot isn’t important, but the text on the graph is very important. Read
the titles of all graphs, look for a key if there is one, and notice the units before
WRITING AND

answering any questions.


LANGUAGE

When working with graphs, always read the


title, key or legend, variables, and units before
working any questions.
MATH

Grams of Protein and Grams of Fiber


in Twelve Brands of Whole-Wheat Bread
8
7
6
Fiber (grams)

5
4
3
2
1
0
0 2 4 6 8 10 12 14
Protein (grams)

1. What are the variable and units along the horizontal (x) axis? _____________

2. What are the variable and units along the vertical (y) axis? ________________

3. How much fiber is in the bread with 6 grams of protein? __________________

4. How much protein is in the bread with 7 grams of fiber? __________________

5. According to the line of best fit, approximately how many grams of fiber
would be in the bread with 10 grams of protein? _________________

6. How many grams of protein are in the bread that is closest to the line of
best fit? ___________________

508 | © TPR Education IP Holdings, LLC


CHARTS AND DATA 101

READING
BAR GRAPHS AND HISTOGRAMS
A bar graph is another way to represent data. Rather than giving points, a bar
graph represents the values with a bar. A histogram is just a bar graph in which
each bar represents a range of values rather than a specific value. The height or
the length of the bar corresponds to a value on the perpendicular axis. As with
scatterplots, read the title, key or legend, variables, and units before working any

WRITING AND
questions.

LANGUAGE
Unemployment Rate in the United States
12

10
% Unemployment

MATH
8

0
2006 2007 2008 2009 2010 2011
Year

1. What is the variable along the horizontal (x) axis? ____________________

2. What is the variable along the vertical (y) axis? _______________________

3. What is the lowest unemployment rate shown? _______________________

4. For what year was the unemployment rate the highest? ________________

5. What was the unemployment rate during the last year shown? __________

© TPR Education IP Holdings, LLC  |  509


SAT MANUAL
READING

TWO-WAY TABLES
Two-way tables give counts for data according to two variables. Much as how the
previous graphics had one variable along the x-axis and one along the y-axis, two-
way tables have categories listed across the top and down the left side of the table.
Again, before answering any questions, read the headings and note the units.
WRITING AND
LANGUAGE

Preferred Beverage by Gender

Coffee Tea Hot Chocolate Total


Men 923 254 89 1,266
Women 655 362 193 1,210
Total 1,578 616 282 2,476
MATH

1. What is the variable along the top of the table? _______________________

2. What is the variable along the left side of the table? ___________________

3. What is the total number of people represented in the table? ___________

4. What is the total number of people who preferred tea? _________________

5. How many women are represented in the table? ______________________

6. How many men preferred hot chocolate? ____________________________

7. How many women preferred coffee? ________________________________

510  |  © TPR Education IP Holdings, LLC


CHARTS AND DATA 101

READING
FREQUENCY TABLES
The SAT will sometimes provide data in a frequency table. The table provides a
shorthand for listing out many values. A frequency table has two columns: one
column contains the values and the other column contains the number of times
each value occurs—or its frequency. To answer a particular question, it may be
best to list out all of the values contained in the table.

WRITING AND
LANGUAGE
In order to conduct a tree survey, a forester divides a certain section
of a forest into 50-foot by 50-foot plots. The forester then counts the
number of holly trees within each plot. The findings are shown in the
frequency table below.

Number of Holly Trees Number of Plots


0 2

MATH
1 0
2 5
3 7
4 2
5 3
6 1

1. How many total plots were surveyed? _______________________________

2. List the number of trees in each of these plots. ________________________

________________________________________________________________

3. How many plots contain 5 holly trees? ______________________________

4. How many total holly trees are in those plots? _______________________

5. What is the most frequent number of holly trees in a plot? _____________

6. What is the greatest number of holly trees in any single plot? ___________

7. What is the range of the number of holly trees in the plots? ____________

8. How many total holly trees are in all the surveyed plots? _______________

_________________________________________________________________

9. What is the average number of holly trees per plot? ___________________

© TPR Education IP Holdings, LLC  |  511


SAT MANUAL
READING

LINE GRAPHS
Line graphs show the change in one variable per another variable. As with all fig-
ures, start with the title, key or legend, variables, and units before working the
questions.

y
WRITING AND

60
LANGUAGE

50
Population 2

Area (square inches)


40

30 Population 1
MATH

20

10

0 x
0 10 20 30 40 50 60
Time (days)

1. What are the variable and units along the horizontal (x) axis? ___________

2. What are the variable and units along the vertical (y) axis? _____________

3. What area does Population 1 cover at day 0? _________________________

4. What area does Population 2 cover at day 0? _________________________

5. On what day does the area of Population 1 equal Population 2? _________

Answers can be found on page 739.

512 | © TPR Education IP Holdings, LLC


CHARTS AND DATA 101

READING
STEM-AND-LEAF PLOTS
Once in a while, College Board may ask you about a stem-and-leaf plot or a box
plot (also known as a box-and-whisker plot). The good news is that these questions
are usually pretty straightforward if you understand the basic concepts.

Suppose that a class earned these quiz scores: 65, 70, 70, 78, 80, 81, 84, 86, 89, 89,

WRITING AND
93, 93, 93, 98, 100.

LANGUAGE
A stem-and-leaf plot would illustrate the data as follows:

6 5
7 0 0 8
8 0 1 4 6 9 9

MATH
9 3 3 3 8
10 0

BOX PLOTS
A box plot shows the data broken into quartiles. Using our fifteen quiz scores, the
box plot would be illustrated as follows:

65 78 86 93 100

Here is what all the parts of the box plot represent.

Box
Whisker Whisker

Min Lower Upper Max


Quartile Median Quartile
Q1 Q2 Q3

© TPR Education IP Holdings, LLC  |  513


SAT MANUAL
READING

For the quiz scores, 86 is the median, and this is the line inside the box, also
known as the second quartile. The lower quartile and upper quartile are the medians
of the lower and upper halves of the data, respectively, and are represented by the
ends of the box.

Q1 Q2 Q3
WRITING AND
LANGUAGE

65 78 86 93

The horizontal lines on a box plot, called the whiskers, extend to the lowest data
point on the left and the highest one on the right. Here, those points are 65 and
100, respectively.
MATH

Minimum Q1 Q2 Q3 Maximum

65 78 86 93 100

A box plot shows the spread of the data by the width of the whiskers or halves of
the box. Here, because the left whisker appears to be the longest, we know that
the data points in the lowest 25% are spread out the most. Also, the interquartile
range is the range of the middle 50%: Q3 – Q1, or the width of the box.

From a stem-and-leaf plot or a box plot, you can determine the median and range
of the set of data. It is also possible to calculate the mode and mean from a stem-
and-leaf plot and the interquartile range from a box plot. We’ll cover some of these
concepts in greater detail in the Word Problems chapter.

514  |  © TPR Education IP Holdings, LLC


CHARTS AND DATA 101

READING
WHAT MAKES A GOOD STUDY?
Some questions will ask for the most reasonable conclusion drawn from the data
of a study. Other questions will ask what factors may cause the results of the study
to be unreliable. There are a few terms that are helpful in tackling these questions.

Sample Size The number of experiments run or people sur-

WRITING AND
veyed. Generally, the larger the sample size, the

LANGUAGE
better.
Population Size The total number of people that the survey is
taken from. Be careful with predictions about
the population based on a sample, especially if
those predictions are strongly worded.
Unrepresentative sample/ Some aspect of the survey or experiment is
biased sample flawed, so the results do not represent the popu-

MATH
lation. Look for factors such as where and when
the survey was given or who was asked to com-
plete the survey. If there’s a mismatch between
who was asked and who the conclusion is about,
the sample is unrepresentative.
Biased question The survey question is written such that one
response becomes more likely than another.
Causation Claiming that one thing was the cause of
another. Be suspicious of inferences that claim
causation; there are many possible issues that
prevent a study or survey from proving
causation.
Correlation Two events occurring in similar circumstances.
This does NOT prove causation (though it may
support causation).

© TPR Education IP Holdings, LLC  |  515


CHARTS AND
DATA

Theories come and go, but fundamental data


always remains.
—Mary Leakey

517
SAT MANUAL
READING

GOALS REVIEW
At the conclusion of this chapter, you will be able to accomplish the following:

• Identify the correct data needed to answer Charts and Graphs questions
• Determine the best graph to represent a situation
• Evaluate the results of a survey or experiment
WRITING AND

• Make accurate conclusions based on survey data


LANGUAGE
MATH

518  |  © TPR Education IP Holdings, LLC


CHARTS AND DATA

READING
FINDING THE RIGHT DATA
As you saw in Charts and Data 101, some SAT questions will tell you exactly what
data to look for. However, other questions will require you to determine what data
the question is asking about.

Temperature versus Precipitation

WRITING AND
LANGUAGE
Average Annual Precipitation (inches)

70
60
50
40
30

MATH
20
10
0
20 30 40 50 60 70 80
Average Annual Temperature (degrees Fahrenheit)

14. The scatterplot above displays the average annual temperature in What points do you need
degrees Fahrenheit and the average annual precipitation in inches for from the scatterplot?
12 U.S. states. Approximately how many more inches of precipitation
fall in the warmest state than in the coldest state, on average?
A) 22
B) 33
C) 51
D) 55

© TPR Education IP Holdings, LLC  |  519


SAT MANUAL
READING

MAKING PREDICTIONS
You may be asked to predict a value that is off the chart. Use your scantron as a
straightedge to ballpark the answer and then use POE.

Income Data for Selected Part-Time Employees at Coffee Planet


WRITING AND

340
LANGUAGE

320

Paycheck Amount ($)


300
280
260
240
MATH

220
200
20 21 22 23 24 25 26 27
Number of Hours Worked

Be sure to follow the line 11. The scatterplot above shows the income data for 24 selected part-time
of best fit! employees at Coffee Planet during a one-week period in March 2014.
If the trend in the relationship between the number of hours worked
and paycheck amount remains consistent with the data above for part-
time employees who worked more than 27 hours during the week in
question, which of the following would most likely be the paycheck
amount earned by an employee who worked 28 hours during that week?
A) $290
B) $300
C) $330
D) $380

520  |  © TPR Education IP Holdings, LLC


CHARTS AND DATA

READING
HARDER TWO-WAY TABLES
Harder questions involving two-way tables require reading carefully and identify-
ing the correct pieces of information from the table. Be sure to read (and under-
line) the final question and work in Bite-Sized Pieces!

Greenhouse Plant Inventory

WRITING AND
LANGUAGE
Spring Summer Fall Total
Blooming Blooming Blooming
Annual 6,700 2,500 1,200 10,400
Perennial 3,200 3,500 5,300 12,000
Total 9,900 6,000 6,500 22,400

MATH
18. A horticulturalist working in a greenhouse is planning when to move
plants from the production greenhouse to the store. The inventory of What numbers do you
need from the table?
plants in the production greenhouse is shown in the two-way table
above. What proportion of perennials are summer blooming?

5
A)
32

25
B)
224

7
C)
24

7
D)
12

© TPR Education IP Holdings, LLC  |  521


SAT MANUAL
READING

REPRESENTING DATA
Some questions will ask you to determine the graph that best represents a word
problem. As always, work in Bite-Sized Pieces and use POE.

Right after school, how 15. Everett lives ten miles from school. He was driving home after class on
far is Everett from his a typical weekday afternoon when he suddenly realized that he had
WRITING AND
LANGUAGE

house? What answers forgotten his calculator in his locker. He decided to return back to
can be eliminated at school and spent some time there talking to his Pre-Calculus teacher,
that point? whom he met in the hallway. After about twenty minutes at school,
Everett got back into his car and drove straight home. Which of the
following graphs best represents Everett’s afterschool activity?

A) Distance from home B)

Distance from home


15 15
MATH

10 10
(miles)

(miles)
5 5
0 0
0 20 40 60 80 100 0 20 40 60 80 100
Time Time
(minutes) (minutes)

C) D)
Distance from home

Distance from home


15 15
10 10
(miles)

(miles)
5 5
0 0
0 20 40 60 80 100 0 20 40 60 80 100
Time Time
(minutes) (minutes)

522  |  © TPR Education IP Holdings, LLC


CHARTS AND DATA

READING
DRAWING CONCLUSIONS FROM DATA
As seen in Charts and Data 101, some questions will ask about problems with a
survey or experiment. Look for the size of the sample, the representativeness of the
sample (who was asked compared to who the conclusion is about), and whether
the question itself was biased.

WRITING AND
LANGUAGE
12. A bicycling organization, hoping to determine how to encourage more
people to ride bikes, surveyed its members on what motivates them
to ride their bikes. A total of 249 members responded to the survey,
while the remaining members did not respond. Which of the following
factors most calls into question the organization’s ability to make a
reliable conclusion about how to encourage more people to ride bikes?
A) The group affiliation of the respondents

MATH
B) The number of members who did not respond to the survey
C) The size of the survey sample
D) The method of distributing the survey

Other questions will ask for the best conclusion drawn from a survey or experi-
ment. Be suspicious of any answers that make absolute claims about a population
based on a sample or infer causation from correlation.

25. Seeking to determine the average amount of pizza eaten on a weekly


basis by people in his neighborhood, Darren visited his local pizzeria
every day for a week and asked the customers how much pizza they
consume each week. He found that they consumed an average of
2.7 slices of pizza per week. Which of the following statements must be
true?
A) The method of sampling is flawed and may generate a biased
estimate of the average (arithmetic mean) number of slices of pizza
eaten per person each week in the neighborhood.
B) The method of sampling is not flawed and likely generated an
unbiased estimate of the average (arithmetic mean) number of
slices of pizza eaten per person each week in the neighborhood.
C) People in the neighborhood consume an average of 2.7 slices of
pizza per week.
D) Residents of the neighborhood who didn’t visit the pizzeria
consumed less pizza than those who did.

© TPR Education IP Holdings, LLC  |  523


SAT MANUAL
READING

MEANING IN CONTEXT FROM CHARTS AND GRAPHS


Some Charts and Graphs questions will ask for what some feature of the graph
means in context. Use a modified form of the Meaning in Context Basic
Approach for these questions.
WRITING AND
LANGUAGE

MEANING IN CONTEXT CHARTS APPROACH


1. Read the Final Question—Know what part of the chart or
graph the question is asking about.
2. Label the Figure—Start with the most straightforward
piece of information and use Bite-Sized Pieces. Label what
you can in the figure.
3. Use POE—Eliminate answer choices that don’t make sense
MATH

with the labels.


4. Plug-and-Play—Choose values from the figure to determine
whether any remaining answer choices don’t make sense.
5. Guess and Go—If you have more than one answer choice
remaining, pick one of the remaining answer choices and
move on!

v Car Value Based on Age


Value (in thousands of dollars)

12

10 •


8 •


6


4

2 •


y
5 10 15 20 25
Age (years)

17. The scatterplot above shows the age and value of ten cars
produced by a certain manufacturer. The line of best fit is also shown.
What does the slope of the line of best fit represent?
A) The estimated value, in thousands of dollars, of a car y years old
B) The estimated initial value of the car, in thousands of dollars
C) The estimated age at which the car has a value of zero dollars
D) The estimated change in the car’s value, in thousands of dollars,
each year
524 | © TPR Education IP Holdings, LLC
CHARTS AND DATA

READING
For linear graphs that represent an amount over time,
• the slope represents the rate of change.
• the y-intercept represents the initial amount.
• the x-intercept represents the time at which the amount

WRITING AND
is 0.

LANGUAGE

Questions 13 and 14 refer to the following information.

f Depth of Flood Waters Over Time

MATH
6
Water depth (inches)

5
4

3
2

1
0 d
0 1 2 3 4 5 6 7 8 9 10 11 12 13 14
Time (days)

13. The graph above displays the depth of flood waters f, in inches, d days
after a certain hurricane. What does the f-intercept represent in the graph?

A) The depth of the flood waters after 9 days


B) The decrease in floodwater depth for each day
C) The number of days until the flood waters subside
D) The initial depth of the flood waters

14. Which of the following represents the relationship between d and f ?

2
A) d   f
3
2
B) f   d 6
3
C) f = –2d + 6

D) f = 6d

© TPR Education IP Holdings, LLC | 525


SAT MANUAL
READING

CHARTS AND DATA DRILL


Time: 8 minutes

Use your POOD! Unless you’re aiming for a top score, don’t attempt every question! Calculators are permitted on all
questions in this drill.
WRITING AND
LANGUAGE

2 5

Age versus Capacity A school administrator conducted a survey to


determine whether students would be interested
1,400 in taking an elective class on graphic design. The
1,300
1,200 administrator surveyed 10 students out of a school
MATH

1,100 population of 2,195. Which of the following factors


1,000
900 most prevents the administrator from drawing a
Capacity (W)

800 reliable conclusion about students’ interest in the


700
600 class?
500 A) The grade level of students surveyed
400
300 B) The sample size of the survey
200
100 C) The difficulty in hiring a graphic design teacher
0 2 4 6 8 10 12 14 D) The time of day the survey was given
Age (years)

Becca created the scatterplot above to examine


the relationship between the age and capacity of
8 appliances in her home. What is the capacity, in
watts, of the newest appliance?
A) 120
B) 250
C) 1,100
D) 1,400

526  |  © TPR Education IP Holdings, LLC


CHARTS AND DATA

17 28

READING
A company wished to determine whether its
Tuition Cost per Credit Hour and investors were satisfied with the new CEO’s plans
Number of Students to reduce environmental harm. The company sent
Number of Students Enrolled

for Eleven Community Colleges in 2018 out a web survey to all investors who had provided
25,000 the company with their email addresses. According
20,000 to the survey, 43% of respondents reported that

WRITING AND
LANGUAGE
they were satisfied with the plans. Based on this
15,000
information, which of the following must be true?
10,000
A) The sample size is too small to produce an
5,000 accurate determination of the percent of
0 satisfied investors.
$0 $50 $100 $150 $200 $250 $300
B) A total of 43% of investors are satisfied with the
Cost per Credit Hour ($)
new CEO’s plans to reduce environmental harm.

MATH
The scatterplot above displays data for eleven C) The survey is likely to result in an unbiased
community colleges along with a line of best fit. estimate of the percent of satisfied investors
For the community college with the lowest number because the sampling method is not flawed.
of students enrolled, what is the approximate D) The survey may result in a biased estimate of
difference between the actual number of students the percent of satisfied investors because the
and the number predicted by the line of best fit? sampling method is flawed.
A) 200
B) 5,000
C) 10,000
D) 15,000

© TPR Education IP Holdings, LLC  |  527


SAT MANUAL

35 37
READING

10
Team Team Team Team Total
Location (cm)

8 A B C D
0 Articles 1 0 4 3 8
6
1 Articles 11 17 20 8 56
2 Articles 19 7 3 19 48
WRITING AND

4
LANGUAGE

3 Articles 3 6 5 5 19
2
Time (s) 4 Articles 1 5 3 0 9
0 2 4 6 8 10 12 14 16 18
Total 35 35 35 35 140

–2
Four teams—Team A, Team B, Team C, and
–4 Team D—worked on a particular day writing
MATH

articles for various websites. Each team had a total


–6
of 35 members, and each of those members wrote
–8 0, 1, 2, 3, or 4 articles that day. What percent, to the
nearest tenth, of the members who wrote at least 3
articles were members of either Team B or Team
C? (Note: Disregard the % sign when gridding your
The movement of a tuned mass damper located at answer.)
the top of a 101-story building was measured during
a minor earthquake. The location of the tuned mass
damper, in centimeters from the resting location, is
graphed in time, in seconds, after the start of the
earthquake. How many times was the damper 3 or
−3 centimeters from the resting location after the
start of the earthquake?

528  |  © TPR Education IP Holdings, LLC


CHARTS AND DATA

Summary
• When working with any type of graphic,
what four things should you look at before
working through the questions?

______________________________________

______________________________________

______________________________________

______________________________________

• When you’re answering a question, be sure to


__________________, __________________,
and __________________________________.

• When asked to find a point not on the


graph, _____________________.

• When asked to determine the best graph


to represent a word problem, ________,
_________________, and ________________.

• What are the five steps to interpreting the


meaning of part of a graph in context?

______________________________________

______________________________________

______________________________________

______________________________________

______________________________________

• What three things should you consider


when determining whether a survey or
experiment was well done?

______________________________________

______________________________________

______________________________________

• I have accomplished _________ of the 4


goals stated at the beginning of this chapter.

© TPR Education IP Holdings, LLC | 529


CHARTS AND DATA
PRACTICE

531
SAT MANUAL

SCATTERPLOT GRAPHS
READING

q 18
Based on the line of best fit to the data represented,
Questions 18–20 refer to the following information. which of the following is the closest to the expected
miles per gallon, rounded to the nearest whole
WRITING AND

Fuel Efficiency by Vehicle Weight


LANGUAGE

number, of a minivan weighing 5,000 pounds?


(One ton is equal to 2,000 pounds.)
A) 6
35 B) 15
C) 28
30
D) 50
Miles per gallon
MATH

25
19
20
Based on the line of best fit to the data represented,
which of the following is the closest to the expected
15 miles per gallon of a vehicle weighing 1 ton?
A) 55.6
10 B) 46.8
1.5 2.0 2.5 3.0 3.5 4.0
C) 41.4
Weight (in Tons)
D) 37.0

The scatterplot above shows the fuel efficiency, in


miles per gallon, of a variety of vehicles weighing 20
between 1.5 and 4 tons. Based on the line of best fit to the data represented,
which of the following is the closest to the decrease
in the fuel efficiency of a vehicle per additional ton
of weight?
A) 0.67
B) 1.60
C) 3.67
D) 8.80

CONTINUE
532  |  © TPR Education IP Holdings, LLC
CHARTS AND DATA PRACTICE

q 21

READING
Based on the curve of best fit to the data
Questions 20–22 refer to the following information. represented, which of the following is the closest to
the average change in ramp length per second of
Marble Ramp Rolling Times roll time for the first 10 seconds?
A) 3.50

WRITING AND
100 B) 4.00

LANGUAGE
90 C) 4.25
Length (in centimeters)

80
D) 5.00
70
60
50
40
30

MATH
20
10 22
0
0 5 10 15 Based on the curve of best fit to the data
Time (in seconds) represented, what is the difference between the
predicted time for the ramp that was 70 centimeters
in length and the actual observed time, in seconds?
A student is rolling a marble down ramps of A) 0.5
varying lengths. The scatterplot above shows the B) 1.0
time, in seconds, it takes the marble to roll down C) 10
each ramp.
D) 12

20 p
Based on the curve of best fit to the data
represented, which of the following is the closest to
the expected length, in centimeters, of a ramp if a
marble took 15 seconds to roll down it?
A) 95.0
B) 105.0
C) 112.5
D) 125.0

CONTINUE
© TPR Education IP Holdings, LLC  |  533
SAT MANUAL

OTHER LINE GRAPHS


READING

7 22

g(x) Maggie’s and Glenn’s Distances


from Home During Jog
WRITING AND
LANGUAGE

3500
Revenue earned ($)

3000

Distance in meters
h(x)
2500
2000
Maggie
1500 Glenn
1000
MATH

500
Time (months) 0
0 3 6 9 12 15
Time in minutes
In the figure above, the revenues earned by two
different ice cream cake companies, Betsy Cakes Maggie and Glenn both leave from the same house
and Frozalicious, are denoted by the functions to go for a jog along a trail. Shortly after leaving,
g(x) and h(x), respectively. Which of the following Maggie realizes she forgot her iPod and returns
statements is true? home to find it before heading back out onto the
A) Betsy Cakes consistently earned more revenue same trail. The graph above shows how far each
than did Frozalicious. of them is from home for the first fifteen minutes
of their jogs. Which of the following statements is
B) Betsy Cakes initially earned more revenue than
supported by the information in the graph?
did Frozalicious but was eventually overtaken.
A) Maggie jogs faster than Glenn for approximately
C) Frozalicious consistently earned more revenue
the first 5 ½ minutes, but then Glenn jogs faster
than did Betsy Cakes.
for the remainder of the 15 minutes.
D) Frozalicious initially earned more revenue than
B) Glenn is always in front of Maggie on the trail.
did Betsy Cakes but was eventually overtaken.
C) Maggie’s average speed for the fifteen minutes
is higher than Glenn’s average speed over the
15 minutes.
D) After 15 minutes, Maggie has run further from
home than Glenn.

CONTINUE
534  |  © TPR Education IP Holdings, LLC
CHARTS AND DATA PRACTICE

BAR GRAPHS

READING
q 7
According to the data above, the number of
Questions 6–8 refer to the following information.
earthquakes measuring 5.0–5.9 magnitude in 2001
was approximately what percent of the number of

WRITING AND
LANGUAGE
The bar graph below shows the number of earthquakes earthquakes measuring 4.0–4.9 magnitude in 2005?
measuring 4.0–4.9 and 5.0–5.9 on the Richter scale A) 12%
from the year 2000 to the year 2005. B) 14%
C) 16%
Earthquakes in the United States
D) 18%
600
536 541
Number of Earthquakes

500

MATH
400 8
345
300 281 290 284
If the total number of earthquakes of all
200 magnitudes in the years 2000–2005 was 18,210,
and the number of earthquakes measuring 4.0–5.9
100 63 63 54 47
41 25 magnitude in 2003 accounted for approximately
0 20% of all the earthquakes in that year, then the
2000 2001 2002 2003 2004 2005
perce

Year number of earthquakes measuring something other


than 4.0–5.9 magnitude in 2003 was approximately
Earthquakes Measuring
4.0–4.9 Magnitude what percent of the total number of earthquakes of
Earthquakes Measuring all magnitudes in the years 2000–2005?
5.0–5.9 Magnitude
A) 8%
B) 13%
C) 21%
6 D) 56%
According to the data above, which of the following
most likely represents the number of earthquakes
measuring 5.0–5.9 magnitude in 2006? p
A) 44
B) 47
C) 56
D) No such prediction can be made on the basis of
the data.

CONTINUE
© TPR Education IP Holdings, LLC  |  535
SAT MANUAL

TWO-WAY TABLES
READING

q 12
Based on the data shown, which of the following
Questions 11–13 refer to the following information. cities has the highest percentage of commuters who
rely on personal vehicles?
WRITING AND
LANGUAGE

A survey company gathered data regarding people’s A) Washington, D.C.


transportation habits in four major U.S. cities. The B) New York
survey asked participants in each of these cities to
C) Boston
indicate whether they regularly used a personal
vehicle, public transportation, or neither to commute. D) San Francisco
Participants were not limited to one response and could
check both personal vehicle and public transportation.
The results are shown below.
MATH

Public
Personal Transport- Neither Total
13
Vehicle ation Responses
Washington, D.C. 5,687 3,134 1,232 8,505 How many people in Washington, D.C. responded
New York 2,476 5,738 1,459 7,789
that they regularly used both personal vehicles and
public transportation?
Boston 5,281 3,504 1,025 7,556
A) 916
San Francisco 4,122 4,629 1,192 7,934
B) 1,548
C) 2,493
D) 8,505

p
11
If an equal number of surveys were given in each of
the four cities, which city had the lowest response
rate?
A) Washington, D.C.
B) San Francisco
C) New York
D) Boston

CONTINUE
536  |  © TPR Education IP Holdings, LLC
CHARTS AND DATA PRACTICE

q 7

READING
Based on the table above, which of the following
Questions 5–7 refer to the following information. statements can be concluded?
A) At RG University, the amount of money
The table below shows the relative scholarship amounts
awarded in scholarships to the sophomore class
awarded at RG University by class. The total amount
in 2012 was less than the amount awarded in
awarded in 2002 was $18 million, and the total amount
scholarships to the sophomore class in 2002.

WRITING AND
awarded in 2012 was $26 million.

LANGUAGE
B) In 2012, the average scholarship amount
awarded to a senior at RG University was
Class 2002 2012 greater than the average scholarship amount
Freshmen 0.52 0.54 awarded to a sophomore at RG University.
Sophomores 0.25 0.18 C) At RG University, the junior class has the fewest
Juniors 0.04 0.06 number of students.
Seniors 0.19 0.22 D) At RG University, the freshmen class was

MATH
Total 1.00 1.00 awarded 50 percent more money in scholarships
in 2012 than in 2002.

p
5
How much money was the junior class awarded in
scholarships in 2002?
A) $720,000
B) $1,560,000
C) $3,420,000
D) $4,500,000

6
How much more money, in dollars, was the senior
class awarded in scholarships in 2012 than in 2002?
A) 540,000
B) 780,000
C) 1,620,000
D) 2,300,000

CONTINUE
© TPR Education IP Holdings, LLC  |  537
SAT MANUAL

TRANSLATING FIGURES
READING

8 16
Emma takes a break at a rest-stop on the way Kyle is training for a marathon and runs daily to
home from a long vacation. Upon leaving the rest- build up his endurance. On a particular day, he
stop, she gradually accelerates until she reaches a leaves his house and runs at a constant rate for 2
WRITING AND
LANGUAGE

constant speed on the highway. A few miles down hours, and then returns home at twice the rate that
the road, however, she encounters a bad traffic jam he ran for the first 2 hours. Which of the following
and fairly quickly has to slow down significantly. graphs could represent Kyle’s entire run?
After a few minutes driving slowly in the traffic
jam, she is forced to stop completely. Which of the A) 14

Distance from home


following graphs could represent Emma’s journey, 12
10
as described above?

(miles)
8
A) 6
MATH

70
4
Speed (mph)

60
50 2
40 0
30

140
160
0
0
0

60

0
20
40
80

12

18
10
20 Time
10 (Minutes)
0
0 1 2 3 4 5 6 7 8 9 10 B) 14

Distance from home


Time 12
(minutes)
10
(miles)
B) 70 8
Speed (mph)

60 6
50 4
40 2
30 0
20

140
160
0
0
0

60

0
20
40
80
10

12

18
10
0 Time
0 1 2 3 4 5 6 7 8 9 10 (Minutes)
Time C)
(minutes) 14
Distance from home

12
C) 70 10
(miles)
Speed (mph)

60 8
50 6
40 4
30 2
20 0
10
140
160
0
0
0

60

0
20
40
80

0
12

18
10

0 1 2 3 4 5 6 7 8 9 10 Time
Time (Minutes)
(minutes)
D) D) 14
Distance from home

50
Speed (mph)

40 12
30 10
(miles)

20 8
10 6
0 4
2
0
0 1 2 3 4 5 6 7 8 9 10
Time
140
160
0
0
0

60

0
20
40
80
12

18
10

(minutes) Time
(Minutes)

CONTINUE
538  |  © TPR Education IP Holdings, LLC
CHARTS AND DATA PRACTICE

24

READING
A reaction between two chemical reactants is
endothermic, which means it consumes heat from
its environment. As the reactants are consumed,
the rate of reaction slows and the amount of heat
being consumed decreases. Which of the following
graphs, showing temperature in degrees Celsius as

WRITING AND
LANGUAGE
a function of time in seconds, best illustrates the
temperature graph of the reaction?

A)
Temperature (°C)

MATH
Time (s)

B)
Temperature (°C)

Time (s)

C)
Temperature (°C)

Time (s)

D)
Temperature (°C)

Time (s)

CONTINUE
© TPR Education IP Holdings, LLC  |  539
SAT MANUAL

JUSTIFYING CONCLUSIONS
READING

3 17
A newspaper advertisement for Alex’s Funeral Mark is researching the effects of high fructose corn
Home states the following: “A survey of clients of syrup on American obesity. He discovers that use of
Alex’s Funeral Home shows that 84% of clients high fructose corn syrup (HFCS) increased 1,000%
WRITING AND

between 1970 and 1990 in the United States, and


LANGUAGE

would use Alex’s Funeral Home in the future.”


Which of the following conclusions is most valid that BMI (body mass index) increased significantly
based on this advertisement? in the United States and worldwide over the same
A) Approximately 84% of respondents to the time period. Which of the following is the most
survey would use Alex’s Funeral Home in the reasonable conclusion that can be drawn from the
future. data?
B) Competitors to Alex’s Funeral Home have A) As HFCS consumption increased worldwide,
a lower satisfaction rate than Alex’s Funeral there was an increase in BMI worldwide.
MATH

Home. B) Increased consumption of HFCS causes


C) 16% of Alex’s Funeral Home’s clients were very increased BMI in American citizens.
upset with the service they received. C) As HFCS consumption continues to increase,
D) Alex’s Funeral Home is the best option for there will be a continued increase in BMI.
everyone’s funeral home needs. D) Increased HFCS consumption occurred at the
same time as increased BMI among American
citizens during 1970–1990.

21
A recent survey of a random sample of 200 high
school seniors in a town indicated that 143 of them
preferred cats and 57 preferred dogs. If there are
45,376 high school seniors in the town, which of the
following statements is most likely to be accurate?
A) Approximately 13,000 high school seniors in the
town will report that they prefer cats.
B) Approximately 18,000 high school seniors in the
town will report that they prefer cats.
C) Approximately 29,000 high school seniors in the
town will report that they prefer cats.
D) Approximately 32,000 high school seniors in the
town will report that they prefer cats.

CONTINUE
540  |  © TPR Education IP Holdings, LLC
CHARTS AND DATA PRACTICE

DATA COLLECTION METHODS

READING
7 23
A nationwide club of over 1 million members Danielle is conducting a survey of 500 people who
wishes to survey the opinions of its members love astronomy. She asks the participants to decide
on topics relevant to the club’s membership. It whether they think that manned or unmanned

WRITING AND
LANGUAGE
is determined that directly surveying all of the space exploration will yield the most scientific
members of the club would be too costly. Which of value. The survey results appear in the following
the following alternative surveying methods would table.
result in the most accurate representation of the
club’s membership’s opinions? Profession of Manned Unmanned
Undecided
participants is better is better
A) Survey members who attend the club’s annual
meeting. Typical attendance at this meeting is Student 65 17 18
10,000 members. Engineer 91 50 34

MATH
B) Randomly select 10,000 members to send Artist 47 44 44
surveys to. Politician x y 15
C) Place a link to an online survey on the club’s
website. Upon reviewing the data, Danielle concludes
D) Survey members of the club’s executive that most people in the United States believe that
committee. manned space exploration is more important
scientifically. Assuming all of the following
statements are true, which provides the best reason
to disregard Danielle’s conclusion?
A) Of the politicians surveyed, the number who
prefer unmanned exploration is twice the
number who prefer manned exploration.
B) The survey participants are drawn from people
who have a vested interest in astronomy.
C) Danielle did not list values for the politicians
who prefer manned and unmanned exploration.
D) 14,000 people in another similar study
conducted said they preferred unmanned
exploration.

Answers can be found on pages 739–740.

© TPR Education IP Holdings, LLC | 541


WORD
PROBLEMS 101

Even the simplest-looking word problems can


be deceptively tricky. A good review of some
key skills can leave you much better prepared to
tackle even the most difficult word problems.

543
SAT MANUAL
READING

PARTS & WHOLES


Fractions, decimals, percents, and ratios all give us relative values.

1. A friend tells you she has read half of her homework


assignment. Is that many or few pages?
WRITING AND
LANGUAGE

A. Many.
B. Few.
C. Who knows?

2. Another friend tells you he has done 100 pages worth


of his homework assignment. Is he almost done?

A. Yes.
MATH

B. No.
C. Who knows?

3. You tell your mother you’ve spent 2 hours on


homework. How many more hours are left to finish?

A. A ton.
B. None—it’s time to play some video games.
C. Who knows?

4. You tell your mother you’ve done 75% of your


homework. Does she know what portion you have left
to do?

A. Yes.
B. No.
As you can see, statistics mean very little out of context. Unless you know the
total pages of homework assigned, you can’t evaluate whether “half” is impres-
sive (1,000 pages assigned) or lame (2 pages assigned). By the same logic, either
100 pages or 2 hours could be just the beginning or close to the end. Fractions
and percents, however, have one chief advantage: you always know what fraction/
percentage is left.

544  |  © TPR Education IP Holdings, LLC


WORD PROBLEMS 101

READING
Fractions & Decimals
Fractions and decimals are two different ways of expressing the same value of a
part
relationship.
whole
part 5
= 0.5 (five-tenths) = = 1
whole 10 2

WRITING AND
LANGUAGE
Decimals are fractions whose denominators are powers of ten and whose numera-
tors are expressed by digits placed to the right of the decimal point.

Calculators

MATH
On Section 4, use your calculator wisely. If a question uses decimals, stick with
decimals. If a question uses fractions, stick with fractions: use parentheses around
the fraction, and use your calculator to get the answer in a fraction.

Mixed Numbers 3
Questions on the SAT may use mixed numbers in word problems: 1 cups of
4
2 7 8
flour, 2 mile run, etc. You have to convert to an improper fraction  ,  ,
3 4 3

convert to a decimal (1.75, 2.667 ) , or know how to use your calculator to do so.

This is how to convert a mixed fraction to an improper fraction.

3 3 4 3 7
1 = 1+   = +   =
4 4 4 4 4

This is how to perform operations with mixed fractions.

1 7   1    7    24 1   16 7   25   23 
4 − 2 = 4 +    −  2 +    =  +  −  +  =   −  
6 8  6   8  6 6 8 8 6 8

Then you can use your calculator if it is allowed to get the answer or get a common
denominator of 24.

 25   4   23   3   100   69  31
    −     =  − =
6 4 8 3 24   24  24

© TPR Education IP Holdings, LLC  |  545


SAT MANUAL
READING

Section 3 may have mixed numbers that you’ll need to convert by hand. Convert
the following, adding or subtracting to simplify as needed. Use your calculator to
check your answers.

1
WRITING AND

1. 5 = ____________________________________________________
LANGUAGE

3
2. 4 = ____________________________________________________
7

2 3
3. 3 + 1 = _________________________________________________
5 8
MATH

3 4
4. 4 − 1 = ________________________________________________
11 5

7 2
5. 5 − 2 = ________________________________________________
8 9

5 1
6. 1 + 3 = _________________________________________________
8 6

2
7. Erin is baking cookies and needs 6 cups of flour to make two batches. If she
2 3
has 4 cups of flour in her pantry, then how many more cups of flour does
7
she need to make both batches of cookies?

1
A) 2
21
8
B) 2
21
9
C) 2
21
13
D) 2
21

546  |  © TPR Education IP Holdings, LLC


WORD PROBLEMS 101

READING
Percents part
Percents are another way of expressing a relationship, except that the
whole
“whole” is always 100. Percent actually means “out of 100.”

If you’re comfortable with your method of working with percents, keep using

WRITING AND
that method. If this is something you struggle with, try translating the English to

LANGUAGE
math, as you did in the Translating chapter.

English term Operation/Symbol

of

MATH
is

what percent

2. If x is equal to 60% of 15% of 800, what is the value of x ?


A) 7
B) 72
C) 120
D) 280

15. If 20 percent of a number N is 36, what is 0.2 percent of N ?


A) 7.2
B) 1.8
C) 0.72
D) 0.36

© TPR Education IP Holdings, LLC  |  547


SAT MANUAL
READING

Retail Therapy
There are lots of real-life scenarios that involve percents, so College Board has lots
of material for word problems. Some questions involving percents require several
steps, so always pay careful attention to what the question is asking.

1. At a sale at a local department store, a shirt that


WRITING AND
LANGUAGE

normally costs $100 is on sale for 25% off.

i. How much do you pay for the shirt?_____________________

ii. How much do you save?_______________________________

iii. What operation(s) do you perform to figure this out? _________


MATH

––––––––––––––––––––––––––––––––––––––––––––––––

College Board can assume that you know a lot about how money works in the real
world. Think about what these terms mean in terms of percents—do you multiply
and then add or multiply and then subtract?

Real Life Percent Operation


Discounts
Delivery fees
Sales taxes
Income taxes
Interest

11. Danielle’s bank pays 6.5% interest annually on Certificates of Deposit


(CDs). If she purchases a $25,000 CD from her bank, how much will
she have after one year?
A) $ 1,625.00
B) $16,250.00
C) $25,006.50
D) $26,625.00

548  |  © TPR Education IP Holdings, LLC


WORD PROBLEMS 101

READING
Ratios
Ratios are another way to express a relative relationship of numbers. Instead of a
part part
relationship, however, they show a relationship. Ratios will always be
whole part
defined as a ratio or described as a part-to-part relationship; other relationships are
part

WRITING AND
likely fractions. Ratios can be written as or as part:part. You can treat them

LANGUAGE
part
like fractions in your calculator—just divide the first part by the second.

1. Marie lives with her mother, father, one sister, and five
brothers. Everyone in her family is either a woman or a
man.

MATH
i. What is the ratio of women to men in the family?___________

ii. What is the ratio of children to parents?__________________

iii. How many people are there in the family in total?___________

iv. What fraction of the family is men?______________________

Ratios are always given in their most reduced form. Sometimes a word problem
will make you use the ratio to figure out what the real values are. When things get
complicated, you can use a Ratio Box to keep track of all the information.

© TPR Education IP Holdings, LLC  |  549


SAT MANUAL
READING

Ratio Box
Whenever you need to convert from a ratio in its most reduced form to real-life
numbers, there are two key steps.

• Always add the ratio numbers to get a whole.


WRITING AND
LANGUAGE

• Find the multiple that connects a ratio number to its real-


life counterpart. All of the ratio numbers get multiplied by
this number to convert to real-life numbers.

When things get complicated, you can use a Ratio Box.


MATH

2. In Mr. Peterson’s class of 48 students, the ratio of boys to girls is 3:5.

Boys Girls Whole

3 + 5 = 8 Ratio
× × ×
= = 6 Multiply by
= = =
+ = Actual
48 number

i. How many girls are in the class? ________________________


ii. How many boys are in the class? _______________________
iii. Boys make up what fractional part of the class? ____________
18
iv. If you answered above, what does that reduce to? _______
48

13. In a parking lot, there are 3 cars for every 7 SUVs. If there are 150 cars
and SUVs parked in the lot, how many are SUVs?
A) 10
B) 15
C) 86
D) 105

Answers can be found on page 740.

550 | © TPR Education IP Holdings, LLC


WORD PROBLEMS 101

READING
VOCABULARY
There are a few additional terms that can show up in word problems. Some of
these are key terms that indicate a specific topic or formula, while others are more
real-life terms that College Board expects you to know.

Term Definition

WRITING AND
LANGUAGE
Constant difference a-b=Aa-Ab

Constant ratio

Direct variation

Financing

MATH
Gross revenue

Inverse variation

Mean

Median

Mode

Net profit

FORMULAS
Rarely will you need to call on these formulas; most times, you should apply com-
mon sense and PITA skills to solve. But in case you just want to know...

x1 y1
• Direct Variation: = or y = kx
x 2 y2
k
• Inverse Variation: x1 y1 = x 2 y2 or y =
x

© TPR Education IP Holdings, LLC  |  551


SAT MANUAL
READING

GLOSSARY
Constant difference:  A fixed amount added or subtracted to each term in a
sequence to get the next term
Constant ratio:  A fixed amount used to multiply each term in a sequence to get
the next term
WRITING AND

Direct variation:  A proportion or a relationship between two variables in which


LANGUAGE

the ratio is constant


Financing:  To borrow money for a purchase and pay back with interest
Gross revenue:  The amount of money made without subtracting any costs
Inverse variation:  A relationship between variables in which the product is con-
stant
MATH

Mean:  The average of a list of numbers


Median:  The middle number in a sequential list of numbers. If there is an even
number of numbers in the list, the median is the average of the two middle
numbers.
Mode:  The number that occurs most often in a list
Net profit:  The amount of money left over after costs are subtracted from gross
revenue

552  |  © TPR Education IP Holdings, LLC


WORD
PROBLEMS

If a hen and a half lay an egg and a half in a day


and a half, how long does it take a monkey with a
wooden leg to kick the seeds out of a dill pickle?
—Tom Robbins

553
SAT MANUAL
READING

GOALS REVIEW
At the conclusion of this chapter, you will be able to accomplish the following:

• Consistently apply the Basic Approach to word problems


• Use the strategies of Plugging In, PITA, and POE to more efficiently
answer word problems
WRITING AND

• Understand and apply concepts of mean, median, mode, range, and


LANGUAGE

standard deviation
• Understand and apply concepts of rates, proportions, ratios, and
percentages
MATH

BASIC APPROACH REVIEW


1. __________________________________________________

2. __________________________________________________

3. __________________________________________________

554  |  © TPR Education IP Holdings, LLC


WORD PROBLEMS

READING
WORD PROBLEMS, PLUGGING IN, AND
PLUGGING IN THE ANSWERS

Plugging In

WRITING AND
As you’ve already seen, Plugging In is often the most effective way to work a word

LANGUAGE
problem. Remember to look for variables or relationships between numbers.

14. Each state in the United States has r members of the House of
Representatives. The relationship between the number of Electoral
College votes, v, that each state has and the number of members it has
in the House of Representatives is represented by the equation

MATH
v – 2 = r. In 1840, Kentucky lost three Electoral College votes.
Which of the following accurately describes the change in the
number of members of the House of Representatives for Kentucky
in 1840?
A) The number increased by three.
B) The number remained the same.
C) The number decreased by two.
D) The number decreased by three.

Plugging In the Answers


If you find yourself tempted to write your own equation for a word problem and
there are numbers in the answer choices, ask yourself whether Plugging In the
Answers would be a better approach.

14. Faraz paid $48.00 before tax for a new coat that was marked 20 percent
off of its original price. What was the original price of the coat?
A) $38.40
B) $57.60
C) $60.00
D) $72.00

© TPR Education IP Holdings, LLC  |  555


SAT MANUAL
READING

MEAN, MEDIAN, MODE, RANGE

Mean = Average
The average is the sum of the elements divided by the number of elements. If you’re
WRITING AND

given the average and the number of elements, you can find the total by multiply-
LANGUAGE

ing the average and the number of elements (T = AN).

25. Mei has an average score of 83 on five equally-weighed quizzes. The


teacher decides to remove the lowest quiz score for each student in the
class. Mei’s average for the four remaining quizzes is now 89. What was
Mei’s lowest quiz score before the teacher decided to remove that score?
MATH

A) 59
B) 65
C) 70
D) 77

Median = Middle
The median is the middle number in an ordered list of numbers. If there is an even
number of elements in the list, the median is the average of the two numbers in
the middle.

19.
An element’s valence is a measure of the combining power of an
element. Valence is the number of electrons the element can displace or
combine with. The frequency table below shows the valence of the 108
elements with known valency.
Valence Number of elements
0 3
1 9
2 13
3 28
4 20
5 12
6 19
7 4

Based on the table, what is the median valence for these 108 elements?
A) 3
B) 3.5
C) 3.7
D) 4

556  |  © TPR Education IP Holdings, LLC


WORD PROBLEMS

READING
Mode = Most
The mode is the number that appears most often in a list of numbers.
Incorrect Answers on Math Quiz
Incorrect Answers Number of Students
0 1

WRITING AND
LANGUAGE
4 1
5 2
6 7
7 5
9 6

MATH
22. The table above shows the distribution of incorrect answers among
the 22 students of a certain math class. Which of the following is true
based on the table?
A) The mode is greater than the median, and the median is greater than
the mean.
B) The median is greater than the mean, and the mean is greater than
the mode.
C) The mean is greater than the median, and the median is greater
than the mode.
D) The mean, median, and mode are equal.

Range = Greatest – Least


Range is the difference between the greatest and least elements of a list of numbers.

Test Scores from First Practice Test


1 21 22 24 25 26
26 27 28 30 31 33

11. A class of 12 students took their first practice test. The scores earned
by the students are in the table above. Later, it is determined that the
student who scored a 1 used a pen instead of a pencil, making the
score invalid. If the score of 1 is removed from the data, which of the
following would change the most?
A) Mean
B) Median
C) Mode
D) Range

© TPR Education IP Holdings, LLC  |  557


SAT MANUAL
READING

Standard Deviation = Spread


Standard deviation is a measure of the spread of a group of numbers. A small
standard deviation means most of the elements are close to the average; a large
standard deviation means the numbers are spread out farther away from the
average.
WRITING AND
LANGUAGE

29. A game designer is determining whether to use 3 four-sided dice


(Option A), 2 six-sided dice (Option B), or 1 twelve-sided die
(Option C) in her science fiction adventure game. To determine the
differences among the three options, she created dot plots of the
number of ways to roll a certain total for each combination of dice.
Option A Option B Option C
Total Number Total Number Total Number
MATH

of ways to of ways to of ways to


roll roll roll
3 • 2 • 1 •
4 ••• 3 •• 2 •
5 •••••• 4 ••• 3 •
6 •••••••••• 5 •••• 4 •
7 •••••••••••• 6 ••••• 5 •
8 •••••••••••• 7 •••••• 6 •
9 •••••••••• 8 ••••• 7 •
10 •••••• 9 •••• 8 •
11 ••• 10 ••• 9 •
12 • 11 •• 10 •
12 • 11 •
12 •


If sA, sB, and sC are the standard deviations of Group A, Group B, and
Group C, respectively, and rA, r B, and rC are the ranges of Group A,
Group B, and Group C, respectively, which of the following contains
true statements about the standard deviations and ranges of the three
groups?
A) sA < sC
rA < rB < rC
B) sC > sB
rC < rB < rA
C) sA = sC
rA < rB < rC
D) sB = sC
rA < rB < rC

558  |  © TPR Education IP Holdings, LLC


WORD PROBLEMS

READING
PROBABILITY = CHANCE

# of outcomes that fit requirements


Probability = �
total # of possible outtcomes

WRITING AND
LANGUAGE
Probability of a Single Outcome
q

MATH
Questions 8 and 9 refer to the following information.

Preferred Beverage by Gender


Coffee Tea Hot Chocolate Total
Men 923 254 89 1,266
Women 655 362 193 1,210
Total 1,578 616 282 2,476

The student union at a local college sent a survey to all members of the sophomore
class in order to learn more about the students’ preferences regarding hot drinks.
The students were asked to choose their top choice among three different beverage
options: coffee, tea, and hot chocolate. The respondents’ answers were counted and
are shown in the two-way table above.

8. According to the table, if a man from among these respondents were


chosen at random, what is the probability that he prefers coffee?

343
A)
1, 266
923
B)
2, 476
655
C)
1, 210
923
D)
1, 266

© TPR Education IP Holdings, LLC | 559


SAT MANUAL
READING

Probability of This OR That


For those questions that ask for the probability of either of two different outcomes
occurring, find the “number of outcomes that fit the requirements” for both situ-
ations. Then, add those together to get the numerator of the probability fraction.
The “total # of possible outcomes” is still the total number of people or things from
which you are choosing.
WRITING AND
LANGUAGE

9. According to the table, if a sophomore from among these respondents


were chosen at random, what is the probability that the chosen person
is a man who prefers hot chocolate or a woman who prefers tea?

89
A)
1, 266
MATH

447
B)
1, 266

451
C)
2, 476
898
D)
2, 476

560  |  © TPR Education IP Holdings, LLC


WORD PROBLEMS

READING
RATES AND UNIT CONVERSIONS
Rates are related to averages. Divide work or distance by time to find the rate. As
with an average, you can rearrange the formula to get a total; in this case, distance
or amount of work = rate × time (D = RT or W = RT ).

13. Claudia’s computer can download a 6-megabyte file in one second. If Make sure to use

WRITING AND
LANGUAGE
Claudia is shopping in an online music store and decides to purchase consistent units.
and download sixteen full-length music albums averaging 110
megabytes each, approximately how long, in minutes, would the
download process take?
A) 1
B) 5

MATH
C) 50
D) 300

35. Lindsey and Stephen work at a factory. Lindsey can complete one full What is the combined
job in 3 hours, and Stephen can complete the same job in 5 hours. If rate of Lindsey and
Lindsey and Stephen work together on the job for 1 hour, then how Stephen?
long, in minutes, will it take Stephen to finish the job by himself?

© TPR Education IP Holdings, LLC  |  561


SAT MANUAL
READING

PROPORTIONS
Unit conversion is related to proportional relationships—there are 12 inches in
every foot, 60 seconds in every minute. Proportions can be used to figure out
relationships, including between different rates and between a scale drawing and
reality.
WRITING AND
LANGUAGE

14. Neb works an average of 35 hours a week over four days each week. Neb
would like to increase his weekly wage by $25. Assuming Neb makes an
average of $125 per day, which equation can Neb use to determine how many
more hours, h, he should work each week?

A) 25 
 4 125 h
35
MATH

B) 25 
 35125 h
4

C) 150 
 35125 h
4

D) 150 
 4 125 h
35

562  |  © TPR Education IP Holdings, LLC


WORD PROBLEMS

READING
Direct Variation
Direct variation means that as one variable increases, the other also increases.

When x and y vary directly, y = kx


x
or x 1 = 2 .

WRITING AND
LANGUAGE
y1 y2

MATH
32. Newton-meters and foot-pounds are measures of torque. They are
directly proportional such that 8 newton-meters is equivalent to
5.90 foot-pounds. How much torque, in newton-meters, is 10.33 foot-
pounds, rounded to the nearest integer?

© TPR Education IP Holdings, LLC | 563


SAT MANUAL
READING

Inverse Variation
Inverse variation means that as one variable increases, the other decreases.

k
When x and y vary inversely, y =
x
WRITING AND
LANGUAGE

or x1y1 = x2y 2.
MATH

32. The volume of hydrogen in a balloon varies inversely with the applied
pressure. At an applied pressure of 200 torrs, the volume of hydrogen
in the balloon is 3 cubic feet. What is the applied pressure, in torrs,
when the volume of hydrogen in the balloon is 40 cubic feet?

564 | © TPR Education IP Holdings, LLC


WORD PROBLEMS

READING
RATIOS
Ratios relate one part to another part, not the part to the whole. To compare
ratios, divide the first number in the ratio by the second and compare the resulting
numbers. Even though calculator use is allowed, consider rounding ugly numbers
when possible to save time. Only calculate exact ratios when there are two or more
close answers.

WRITING AND
LANGUAGE
Reported Favorite Movie Genre by Age Group

Age Action Comedy Drama Romance Thriller


15–25 30,432 18,835 6,984 5,395 9,926
26–35 35,967 16,923 34,451 20,916 20,510
36 –50 8,123 12,531 30,792 25,520 11,647

MATH
51–70 5,291 15,515 22,591 17,043 4,750

17. The movie-watching preferences of a randomly selected group of


Americans aged 15–70 are represented in the table above. Survey
participants were asked to choose their favorite genre of movie among
the following categories: Action, Comedy, Drama, Romance, and
Thriller. According to the data above, the ratio of moviegoers who
prefer Action to those who prefer Romance is greatest for which of the
following age categories?
A) 15–25
B) 26–35
C) 36–50
D) 51–70

© TPR Education IP Holdings, LLC  |  565


SAT MANUAL
READING

PERCENTAGES
Unlike ratios, percentages relate the part to the whole. Percent means “out of 100,”
so you can convert a percentage into a decimal by dividing by 100.

What technique can 22. Giovani paid $55,000 for a new car. The amount Giovani paid includes
you use when you see a 15 percent sales tax and a 10 percent delivery fee, charged on the
WRITING AND
LANGUAGE

numbers in the answer after-tax amount. Which of the following best approximates the cost of
choices? Giovani’s car before the sales tax and delivery fee?
A) $42,100
B) $43,500
C) $44,000
D) $47,800
MATH

Sometimes percentage questions will ask you to extrapolate a population from a


representative sample.

24. Out of the 18 employees of the Toronto office of a certain company,


ESSAY

27.8 percent have at least two dogs. The average number of employees in
each office of this company is 18. If there are 30 offices in this company
and the Toronto office is representative of the offices in this company
with regards to dog ownership, which of the following best estimates
the number of employees of this company who have fewer than two
dogs?
A) 150
B) 390
C) 540
D) 2,700

566  |  © TPR Education IP Holdings, LLC


WORD PROBLEMS

READING
Percent Change
Percent change is a measure of change relative to the starting number. The best
way to avoid trap answers is to use the formula.

WRITING AND
LANGUAGE
Percent Change = Difference × 100
Original

MATH
29. In January, a certain physicians’ group treated 180 people for the flu. In
February, that group treated 144 people for the flu. The group manager
believes that the percent decrease in the number of people treated for
the flu by the group from January to February would be half of the
percent decrease in the number of people treated for the flu by the
group from February to March. Which of the following is closest to the
group manager’s prediction for the number of people treated for the flu
by the group in March?
A) 86
B) 108
C) 115
D) 126

© TPR Education IP Holdings, LLC | 567


SAT MANUAL
READING

WORD PROBLEMS DRILL


Time: 8 minutes

Unless you are aiming for a top score, don’t try all the questions! Use your POOD to choose the best ones for you.
All the questions in this drill are calculator permitted (though Word Problems appear on the non-calculator section
of the SAT as well!).
WRITING AND
LANGUAGE

q 6

Questions 5 and 6 refer to the following information: The 2011 per capita income of a fifth country, New
Zealand, was $31,250. If this data is combined
MATH

The table below shows the yearly per capita income in


U.S. dollars of four countries in select years, based on with the data in the table, what was the average
data from the World Bank. (arithmetic mean) per capita income of all five
countries in 2011, rounded to the nearest whole
dollar?
Country 2007 2009 2011 2013
Afghanistan $374 $451 $614 $665 A) $19,126
Colombia $4,664 $5,105 $7,125 $7,831 B) $20,258
Denmark $58,501 $57,896 $61,304 $59,382 C) $25,323
Kenya $847 $930 $998 $1,246 D) $101,291

p
5

According to the data in the table above, the range


of Kenya’s per capita income from 2007 to 2013 was
how much less than the range of Colombia’s per
capita income from the same time period?

A) $399
B) $2,768
C) $3,167
D) $6,585

568  |  © TPR Education IP Holdings, LLC


WORD PROBLEMS

15 20

READING
Annalee bicycles down a long, straight road at a Car Value Based on Years Since Manufacture
constant rate of 22 feet per second. How many 20

Value (in thousands


hours will it take Annalee to ride her bicycle for 18
16
30 miles at this rate? (5,280 feet = 1 mile)

of dollars)
14
12

WRITING AND
A) 0.5 10

LANGUAGE
8
B) 1 6
C) 1.5 4
2
D) 2 0
0 1 2 3 4 5 6 7 8 9
Years since manufacture

MATH
The scatterplot above shows the value for a certain
model of car based on its age. Which of the
following would be closest to the percent decrease
in the value of a car from 2011 to 2013 if that car
were manufactured in 2009?

A) 70%
B) 40%
C) 35%
D) 20%

© TPR Education IP Holdings, LLC  |  569


SAT MANUAL

q 38
READING

Questions 37 and 38 refer to the following A physicist is comparing the power flowing through
information.
two circuits. Both circuits have equal resistance.
V = IR The first circuit has a current of I, and the second
3I
circuit has a current of . What is the ratio of the
WRITING AND

P = I 2R 4
LANGUAGE

power of the second circuit to the power of the first

V2 circuit?
P=
R

The above equations relate voltage V, current I,


resistance R, and power P to one another. The first
MATH

is known as Ohm’s law, and the second and third


equations use the definition of power and Ohm’s Law to
relate power to V, I, and R.

37

A high school physics lab is testing 10 wires of


various resistances. Each wire is tested using a p
current of 5 units. If the 10 wires have resistances of
1, 2, 3, 4, 5, 6, 7, 8, 9, and 10, what is the probability
that a wire tested at random will have a voltage of at
least 20 units?

570  |  © TPR Education IP Holdings, LLC


WORD PROBLEMS

Summary
• What strategies are useful on word problems?

______________________________________

______________________________________

• What are the definitions of mean, median,


and mode?

______________________________________

______________________________________

______________________________________

• What formula can you use to organize your


information on average questions?

______________________________________

• What is the definition of range?

______________________________________

• What is the definition of standard


deviation?

______________________________________

• What is the formula for probability?

______________________________________

• How do you find the probability of this


OR that?

______________________________________

• What two formulas can you use to organize


your information on rate questions?

______________________________________

______________________________________

• What are the formulas for direct and


inverse variation?

______________________________________

______________________________________

© TPR Education IP Holdings, LLC | 571


SAT MANUAL

• What does a ratio represent?

______________________________________

• What is the formula for percent change?

______________________________________

• I have accomplished _________ of the 4


goals stated at the beginning of this chapter.

572 | © TPR Education IP Holdings, LLC


WORD PROBLEMS
PRACTICE

573
SAT MANUAL

MEAN, MEDIAN, MODE, AND RANGE


READING

q 13
Which cookie type had the greatest median number
Questions 13, 14, and 15 refer to the following
of cookies per student?
information.
A) Chocolate Chip
WRITING AND
LANGUAGE

B) Peanut Butter
Four students agreed to make cookies to sell at the
homecoming game. The table below shows the number C) Oatmeal
of cookies made by each of the four students and the D) Snickerdoodle
number of calories per cookie for each of the four
flavors of cookies sold.
MATH

Chocolate Peanut Oatmeal Snicker-


Chip Butter doodle
Calories per
130 140 100 120 14
cookie
Number of How much greater is the mode of the number of
cookies made by 22 16 0 32 oatmeal cookies made per student than the mean
Adam
of the number of chocolate chip cookies made per
Number of student?
cookies made by 0 0 40 7
Brett A) 0
Number of B) 0.5
cookies made by 20 10 15 11
Chuck C) 3
Number of D) 12
cookies made by 16 24 15 20
Dave

15
What is the mean number of calories per cookie for
all of the cookies made by Chuck?
A) 14
B) 121.8
C) 122.5
D) 1,705

CONTINUE
574  |  © TPR Education IP Holdings, LLC
WORD PROBLEMS PRACTICE

23 27

READING
11 8
12 1 4 4 7
13 0 1 2 11 27 43 63 76
14 0 3
15 1

WRITING AND
LANGUAGE
The boxplot shown above displays the total
The stem-and-leaf plot shown above displays the snowfall, in inches, for thirteen cities last winter.
times of eleven contestants in a footrace, where each Which of the following statements must be true?
stem represents seconds and each leaf represents I. The mode is less than the mean.
tenths of a second. Which of the following can be II. The median is 43.
determined? III. The interquartile range is 36.
I. The mode

MATH
II. The median
A) I only
III. The mean
B) II only
A) I only
C) I and III only
B) II only
D) II and III only
C) II and III only
D) I, II, and III

CONTINUE
© TPR Education IP Holdings, LLC  |  575
SAT MANUAL

29 31
READING

A biology lab is growing mold spores in two Sales Totals Last Year Sales Totals This Year
different incubators: one in constant darkness and
$22,500 $36,000
one in constant light. There are 6 dishes of mold
spores in the dark incubator and 4 dishes of mold $42,600 $12,650
spores in the light incubator. The dishes in the dark $31,450 $39,400
incubator contain a mean of 120 mold spores. If the $15,000 $29,400
WRITING AND
LANGUAGE

mean for both incubators combined is 240 mold $28,000 $22,000


spores, what is the mean number of spores for the
dishes in the light incubator?
A manager conducts a review of his sales associates’
A) 180
performance each year. The manager has had
B) 360 the same five associates working for him for the
C) 420 past three years. For each associate, the manager
considers the number of sales made, the dollar
D) 540
MATH

amount of each sale, and the total dollar amount


of all sales. The total amounts of all sales for each
of the five associates is shown in the table above.
The median of the total dollar amounts of all sales
for this year is how much greater than the range of
the total dollar amounts of all sales for last year (in
dollars)? (Disregard the dollar sign when gridding
your answer.)

CONTINUE
576  |  © TPR Education IP Holdings, LLC
WORD PROBLEMS PRACTICE

PROBABILITY

READING
19 28
A survey was conducted among a randomly A consumer survey was performed in three cities to
chosen sample of full-time salaried workers about determine if people preferred a name brand product
satisfaction in their current jobs. The table below over a generic one. In Angleton, 40% of consumers

WRITING AND
LANGUAGE
shows a summary of the survey results. preferred the name brand, in Bracketsville, 55%
of consumers preferred the name brand, and in
Calculopolis, 60% of consumers preferred the
Reported Job Satisfaction by Education Level
name brand. The total population of each of the
(in thousands)
three cities is 240,000 in Angleton, 360,000 in
Bracketsville, and 200,000 in Calculopolis. If a
Highest Level Satisfied Not No Total
consumer prefers the name brand, what is the
of Education Satisfied Response
probability that he lives in Bracketsville?

MATH
High School 17,880 12,053 2,575 32,508
Diploma A) 0.42
Bachelor’s 24,236 8,496 3,442 36,174 B) 0.45
Degree C) 0.48
Master’s 17,605 5,324 1,861 24,790 D) 0.55
Degree
Doctoral 12,210 2,081 972 15,263
Degree
Total 71,931 27,954 8,850 108,735

What is the probability that a randomly selected


full-time salaried worker has a bachelor’s degree
as his or her highest level of education and is not
satisfied with his or her job?
A) 0.032
B) 0.078
C) 0.223
D) 0.235

CONTINUE
© TPR Education IP Holdings, LLC  |  577
SAT MANUAL

RATES AND UNIT CONVERSION


READING

7 21
Rick, Shane, and Darryl work at a widget factory. Paolo is visiting Mexico and needs to fill his gas
The table below shows the number of hours they tank. Paolo finds that the daily exchange rate for his
each spent at the factory on a given day, the number currency is 30.77 Mexican pesos to the U.S. dollar.
WRITING AND

If the gas prices are 15.20 pesos per liter, and Paolo
LANGUAGE

of widgets they produced, and the number of


15-minute breaks they took while they were at the pays $1.80 per gallon at home, how much more, in
factory. Each man works at a constant rate. dollars, does it cost Paolo to completely fill his 14
gallon tank?
Widgets Hours Breaks (1 liter = 0.26 gallons)
Rick 28 4 2
Darryl 42 6 4 A) 0.70
MATH

Shane 27 5 2 B) 1.40
C) 6.90
If he works without a break, approximately how
D) 18.20
long will it take Rick to produce 14 widgets?
A) 1 hour and 30 minutes
B) 1 hour and 45 minutes
C) 2 hours
D) 2 hours and 15 minutes

18
It took Adeline 20 minutes and 18 seconds to run a
5 kilometer race. If one kilometer is approximately
0.62 miles, what was Adeline’s approximate average
speed, in miles per hour?
A) 5
B) 7
C) 9
D) 11

CONTINUE
578  |  © TPR Education IP Holdings, LLC
WORD PROBLEMS PRACTICE

30 32

READING
An engineer is designing a large steel pad to be A block of wood has a mass of 20 grams and is
installed on the deck of an aircraft carrier. Its total 4 cm wide, 3 cm thick, and 12 cm long. What is the
volume will be 18 yd3. A scale model made of the density of the block of wood, in g/cm3 ?
same material measuring 6 feet by 4.5 feet and 4 (Density = mass / volume)
inches thick weighs 75 lbs. If the steel costs $55/
lb, how much will the material for the full-sized

WRITING AND
LANGUAGE
steel pad cost?
(1 yard = 3 feet, 1 foot = 12 inches)

A) $12,375.00
B) $55,687.50
C) $82,500.00

MATH
D) $222,750.00

CONTINUE
© TPR Education IP Holdings, LLC  |  579
SAT MANUAL

PROPORTIONS AND VARIATION


READING

6 14
The distance that Olivia walks is directly The time it takes a factory to produce x units of
proportional to the time she spends walking. Product A is inversely proportional to the number
Which of the following equations could show the of workers on the production line at a given time.
WRITING AND

relationship between D, the distance, in kilometers, Last week, the factory employed 88 workers for
LANGUAGE

that she walks, and m, the time, in minutes, that she a total of 875 work hours to produce x units of
spends walking? Product A. If the factory is to produce x units of
A) D = 10 – m Product A in 1,000 work hours this week, how
many workers must be on the line this week?
B) D = m – 10 A) 12
B) 77
MATH

C) D = m + 10 C) 82
m D) 88
D) D =
10

CONTINUE
580  |  © TPR Education IP Holdings, LLC
WORD PROBLEMS PRACTICE

PERCENTS

READING
2 5
Greg has $1,985 dollars in his savings account. Each The following bar graph shows the percentage of
month, he gains 1% in interest. After 2 months, the population of adults in each age group in King
Greg deposits $800 into his account. Which is County, Georgia, that received flu shots in 2007 and

WRITING AND
LANGUAGE
closest to the amount of money in Greg’s savings 2008.
account after two months? Flu Shots in King Country
A $1,987.54 60

Percent Receiving Flu Shots


B) $2,024.89 50
C) $2,804.85
40
D) $2,824.89
30 2007

MATH
2008
20
10
0
18–24 25–34 35–44 45–54 > 54
years years years years years
old old old old old
Age Group

For which of the following age groups is the percent


increase in the proportion of people receiving flu
shots from 2007 to 2008 the highest?
A) 18–24 years old
B) 35–44 years old
C) 45–54 years old
D) > 54 years old

CONTINUE
© TPR Education IP Holdings, LLC  |  581
SAT MANUAL

22 28
READING

A gaming company conducted a study to find out When a particular metal is heated to 600°C, its
what age groups preferred which types of games. tensile strength drops by 50%. For every 5°C its
The table below outlines the survey results. temperature rises after 600°C, the tensile strength
decreases by 35%. This metal has a tensile strength
of 280 MPa below 600°C and is used in the
First- construction of industrial ovens. If the minimum
Age Sports Adventure
WRITING AND

Person Total
LANGUAGE

Group
Shooters
Games Games safe tensile strength of this material is 38 MPa, what
is the maximum safe temperature of the oven, in
9- to 13-
year olds
16,000,000 9,000,000 25,000,000 50,000,000 degrees Celsius?
14- to 18- A) 605
48,000,000 13,000,000 31,000,000 82,000,000
year olds B) 610
19- to 22-
year olds
38,000,000 27,000,000 19,000,000 84,000,000 C) 615
D) 620
MATH

23- to 60-
8,000,000 3,000,000 10,000,000 21,000,000
year olds

After the initial survey, the gamers each play a game


of their choosing, which is one of the 3 types listed
in the table. If a follow-up survey of 170 of the 19- to
22-year olds reveals that 44 of them decided to play
an adventure game, which of the following is the
best approximation of the number of 19- to 22-year
olds who decided to play one of the other two game
types?
A) 62,000,000
B) 63,000,000
C) 64,000,000
D) 65,000,000

CONTINUE
582  |  © TPR Education IP Holdings, LLC
WORD PROBLEMS PRACTICE

EXTENDED THINKING

READING
q 38
The student responsible for carrying the ballots
Questions 37 and 38 refer to the following down the hallway accidentally dropped the ballot
information. box, causing all 117 ballots to scatter all over the

WRITING AND
LANGUAGE
floor. Assuming the information from Question 37
In a recent poll, the 117 students of Ocapo High School’s is true, what is the probability that the first ballot
senior class were given ballots and asked to vote for the student picks up will contain a vote for Dory but
the name of the school’s new mascot. Students were NOT for Randy?
allowed to vote for both, one, or neither of the two
finalists. The results of the ballots cast are shown in the
table below.

MATH
Randy, the Loveable Dory, the Playful
Rottweiler Dragon
Yes 87 68
No 30 49

37
After the results came out, exactly 15 students p
revealed that they voted for neither Randy nor
Dory. Based on this information, how many votes
were cast only in favor of Randy?

CONTINUE
© TPR Education IP Holdings, LLC  |  583
SAT MANUAL

q 38
READING

Upon further review, Daniel and Kelly realize that


Questions 37 and 38 refer to the following
they left out a third category: primetime dramas.
information.
They add the category and hours watched per week
per person to the table as seen below.
Kelly and Daniel are charting their TV watching habits
as part of an extra credit assignment for Health class. Sports Sitcoms Primetime
WRITING AND
LANGUAGE

The students must compile their total number of hours Entertainment Dramas
watched in a given category weekly. They note two Kelly 3 hours 5 hours 4 hours
common categories between them, sitcoms and sports Daniel 7 hours 3 hours 2 hours
entertainment, and chart approximately how many
hours each week they each watch of those categories as
seen below. Given this new data, Daniel wants to make the
percent of his total time of sports entertainment
programming watched equal to the percent of the
MATH

Sports Entertainment Sitcoms


total time that Kelly spends watching primetime
Kelly 3 hours 5 hours dramas. How many additional hours of sitcoms
Daniel 7 hours 3 hours does Daniel need to watch to accomplish this goal?

37
What is the difference between the percent of the
total time that Daniel spends watching sports
entertainment and the percent of the total time
that Kelly spends watching sports entertainment?
(Disregard the percent symbol when gridding your
answer.)

Answers can be found on pages 740–741.

584 | © TPR Education IP Holdings, LLC


FUNCTIONS

What underlies great science is what underlies


great art, whether it is visual or written, and that
is the ability to distinguish patterns out of chaos.
—Diana Gabaldon

585
SAT MANUAL
READING

GOALS REVIEW
At the conclusion of this chapter, you will be able to accomplish the following:

• Interpret function notation in a variety of questions


• Interpret graphs and their values in the xy-plane
• Solve questions about multiple and compound functions
WRITING AND
LANGUAGE
MATH

586  |  © TPR Education IP Holdings, LLC


FUNCTIONS

READING
FUNCTION BASICS
A function is a machine for producing ordered pairs. An x-value is put into the
function and the corresponding y-value comes out. This y-value is usually referred
to as f(x). The f in f(x) is not a variable; it’s just the name of the function.

1. If f (x) = x2 + 8x + 2, what is the value of f(3) ?

WRITING AND
LANGUAGE
A) 29
B) 33
C) 35
D) 53

MATH
The x goes in the function machine, and the y
comes out, so f(x) = y.

College Board could also ask for the x-value that must be put into the function to
get a certain y-value out.

15. The function w is defined by w(x) = 6 + 3x . If 4 • w(z) = 96, what is the


value of z ?
A) 3
B) 6
C) 10
D) 78

3
1
12. If f (x) =   , what is one possible value of x for which  What should you do
x when you see numbers
1 1 in the answer choices?
< f (x) <  ?
216 64

A) 3
B) 4
C) 5
D) 6

© TPR Education IP Holdings, LLC | 587


SAT MANUAL
READING

FUNCTIONS IRL
Functions may show up in the form of lengthy word problems as well.

What technique can you 3. Jack’s band charges by the performance. Jack’s share P, in dollars, for x
use with numbers in the performances is given by the function P(x) = 1,200x – 60. If Jack earned
answer choices? $4,740 one month playing with his band, how many performances did
WRITING AND
LANGUAGE

the band give?


A) 3
B) 4
C) 5
D) 6
MATH

What technique can you 14. Catherine is filling her swimming pool. The water is currently at a level
use with variables in the of c inches, and the level of the water rises by i inches every r minutes.
question and answer Which of the following functions represents the water level, in inches,
choices? after Catherine fills the pool for an additional m minutes?

i
A) f  m   c  m  
r
r
B) f  m   c  m  
i
r 
C) f  m   c  i  
m

D) f(m) = c + imr

For word problems containing functions, read


carefully for key words, and look for ways to
Plug In or PITA.

588 | © TPR Education IP Holdings, LLC


FUNCTIONS

READING
FUNCTION JUNCTION
Sometimes College Board will ask you to examine more than one function at a
time. Just apply the same rules that you would use for a single function, and you’ll
have the answer in no time.

WRITING AND
s(x) = 3x2 – 4

LANGUAGE
t(x) = 7 + s(x)

13. The functions s and t are defined above. What is the value of t(1) ?

A) 8
B) 6

MATH
C) 5
D) 2

x –4 –2 0 2 4
f(x) –1 1 3 5 7
g(x) –9 –6 –3 0 3

9. The table above shows some values of the functions f and g. For which value of
x is f(x) – g(x) = x ?

A) –2
B) 0
C) 2
D) 4

© TPR Education IP Holdings, LLC  |  589


SAT MANUAL
READING

FUNKY FUNCTIONS
Some function questions can get a little tricky by combining factoring or other
algebra work into the question. Try using Plugging In combined with your knowl-
edge of functions to find the right answer.

p(x) = x3 – x2
WRITING AND
LANGUAGE

q(x) = x3 + 3x2 – 4x

What strategy can q(x )


you use when you see
7. Which of the following expressions is equivalent to , for x > 1 ?
p(x )
variables in the answer
choices? A) x + 4
x
B) x − 4
MATH

x
C) x + 4

D) x – 4

19. The function g is defined by g(m) = (m – 7)(m – 5)3. If g(p + 4) = 0, what is one
possible value of p ?

590  |  © TPR Education IP Holdings, LLC


FUNCTIONS

READING
COMPOUND FUNCTIONS
A compound function is a combination of two or more functions in sequence. It’s
basically a function of a function—the output from the first function is the input
for the second function. Start with the innermost part and work your way out.

1
18. If f(x) = 5 – 2x and g  x   , what is the value of f(g(–2)) ? Work from the inside out

WRITING AND
LANGUAGE
x2 on compound functions.
1
A)
81

B) 1
9
C)
5
9

MATH
D)
2

FUNCTIONS AND GRAPHS


Because functions yield ordered pairs in the xy-plane, many function questions
will involve graphing. Here are the basics of graphs of functions in the xy-plane.

1
O
x
–5 –3 –1 –1 1 3 5

–3

2. The figure above shows the graph of the function f. Which of the
following is closest to f (–3) ?
A) –2
B) –1
C) 1
D) 2

If the question gives a number inside the f(x)


parentheses, such as f(3), it means x = 3. From
there, see what y-value intersects the line at x = 3.

© TPR Education IP Holdings, LLC | 591


SAT MANUAL
READING

1
WRITING AND

O x
LANGUAGE

–3 –2 –1 –1 1 2 3

–2

–3

Remember: y = f(x) 4. The graph of y = h (x) is shown above. If h (x) = –1, which of the following is a
MATH

possible value of x ?

A) –2
B) –0.5
C) 0.5
D) 2

If the question gives a value for f(x), such as


f(x) = 3, it means y = 3. From there, see what
x-value intersects the line y = 3.

592 | © TPR Education IP Holdings, LLC


FUNCTIONS

READING
GET THE FUNC OUT
Let’s look at some other ways the SAT tests functions and graphs.

WRITING AND
LANGUAGE
(0, 4)
• (1, 5)

O x

MATH
11. If the figure above is the graph of y = f (x), which of the following could Try plugging points
be the equation of f (x) ? from the graph into the
function.
A) f (x) = (x + 2)2
B) f (x) = (x – 2)2
C) f (x) = (x + 4)2
D) f (x) = x2 + 4

© TPR Education IP Holdings, LLC  |  593


SAT MANUAL
READING

y = c(x)
WRITING AND
LANGUAGE

1
x
O 1
MATH

15. In chemistry class, Cole models some experimental data in the


xy-plane using the function c shown above. For what value of x is the
value of c(x) at its maximum?
A) 5
B) 1
C) –3
D) –6

If a question asks for the maximum of a function, look for the


greatest y-value on the graph.
If a question asks for the minimum of a function, look for the
smallest y-value on the graph.

594 | © TPR Education IP Holdings, LLC


FUNCTIONS

y y = f(x)

READING
• •

WRITING AND
LANGUAGE
• x
O 1

x 
17. If g (x) = f   1  and the graph of f(x) is shown in the figure above,
3 
then what is the value of g (6) ?

MATH
y

h(x)

k(x)

15. The graphs of the functions h and k are shown in the xy-plane above.
For which of the following values of x does h(x) + k(x) = 0 ?
A) 1
B) 2
C) 3
D) 4

© TPR Education IP Holdings, LLC  |  595


SAT MANUAL
READING

FUNCTIONS DRILL
Time: 8 minutes

Unless you are aiming for a top score, don’t try all the questions! Use your POOD to choose the best ones for you.
WRITING AND
LANGUAGE

5 7
On a recent hiking trip in the High Atlas For all values of x, if the function f(x) is defined as
Mountains, Mackenzie starts her trek one morning x
from a basecamp that is 2,000 feet above sea level. f(x) = , then which of the following is the value of
2
If she ascends an additional 300 feet from sea level
MATH

every hour, then which of the following functions f(2a – 4b) ?


accurately describes Mackenzie’s altitude, A, in feet,
A) a + 2b
as a function of her time, t, in hours?
a−b
300 B)
A) h(t )  2, 000  4
t
B) h(t) = 2,000 + 300t C) a – 4b

C) h(t) = 300t – 2,000 D) a – 2b

D) h(t) = 2,000t + 300

596  |  © TPR Education IP Holdings, LLC


FUNCTIONS

8 28

READING
For function a, the value of a(1) = 4 and a(8) = 2, x 1 2 3 4 5 6 7
and for function b, the value of b(4) = 7 and b(1) = 8.
t(x) –2 0 2 4 6 8 10
What is the value of b(a(1)) ?

A) 2
y
B) 4

WRITING AND
C) 7

LANGUAGE
w(x)
D) 8

1
x
1
13

MATH
If f(x) = 3 – 4x, what is the value of the expression
–2f(–2) – 3f(–3) ?

A) –67 The complete graph of the function w and a table


B) –37 of values for the function t are shown above. The
C) –17 maximum value of w is m. What is the value of t(m) ?
D) 37
A) 4
B) 5
C) 6
D) 7

35
In the function f(x) = 3x2 + 4x – 7, for how many
integer values of x between –3 and 3 is f(x) ≤ 0 ?

© TPR Education IP Holdings, LLC  |  597


SAT MANUAL

Summary
• Given a function, you put __________ in
and get _________ out.

• For word problems contain-


ing functions, read carefully for
_________________, and look for ways
to ways to _______________________,
__________________, or ________________.

• For questions about the graphs of


functions, remember that f(x) = _________.

• What can you do if the graph contains a


labeled point?
_____________________________________

• On compound function questions, work

_____________________________________

• I have accomplished _________ of the


3 goals stated at the beginning of this
chapter.

598 | © TPR Education IP Holdings, LLC


FUNCTIONS
PRACTICE

599
SAT MANUAL

FUNCTIONS AND GRAPHS VOCABULARY


READING

Term Definition Examples/Draw It


Perpendicular
Parallel
WRITING AND

Intersection (of lines)


LANGUAGE

Bisect
Midpoint
Origin
x-axis
y-axis
MATH

Slope
x-intercept
y-intercept
Constant
Function
y = f(x)
Interval
Symmetric
Reflection
Rotation

CONTINUE
600  |  © TPR Education IP Holdings, LLC
FUNCTIONS PRACTICE

FUNCTION BASICS

READING
7 16
If f(x) = 3x + 1, what is one possible value of x for The function f is defined by f(x) = 2x – 7 for all
which 6 < f(x) < 22 ? values of x. What is the value of f(3) + f(5) ?
A) 3

WRITING AND
LANGUAGE
B) 7
C) 9
D) 10

MATH
11
x f(x)
–2 4
–1 0
0 –2
1 –2
2 0

A function is plotted in the xy-plane. The function


passes through the points in the table above. Which
of these equations could represent the function?
A) f(x) = 2x + 8
B) f(x) = 2x2 – 2
C) f(x) = (x – 2)(x + 1)
D) f(x) = x2 – 3x + 2

CONTINUE
© TPR Education IP Holdings, LLC  |  601
SAT MANUAL

FUNCTIONS IRL
READING

1 19
Priya is estimating how much candy she will collect P(x) = –ax + k
on Halloween. Priya’s total pieces of candy C, for
b blocks trick-or-treated is given by the function
The number of skaters at an outdoor skating rink
WRITING AND

C(b) = 8b + 15. If Priya collected 135 pieces of


LANGUAGE

candy, how many blocks did she trick-or-treat? can be modeled by the function above, where x is
A) 12 the average daily temperature in degrees Fahrenheit
and a and k are constants. When the average daily
B) 15
temperature is 15°, there are twice as many skaters
C) 16
as when the average daily temperature is 24°. What
D) 18 a
is the value of ?
k
MATH

42"
36"

Figure I Figure II Figure III

Note: Figure not drawn to scale.

The figures above show three stacks of identical


storage containers. Figure I illustrates how a
container stacks inside the one below it. Figure II
shows a stack of four containers with a height
of 36 inches, and Figure III shows a stack of six
containers with a height of 42 inches. Which of the
following functions gives the height in inches, f(n),
of a stack of n storage containers, for all n ≥ 1 ?
A) f(n) = 3n + 12
B) f(n) = 3n + 24
C) f(n) = 3n + 27
D) f(n) = 6n + 12

CONTINUE
602  |  © TPR Education IP Holdings, LLC
FUNCTIONS PRACTICE

FUNCTIONS AND GRAPHS

READING
5 16

y y
5 6

WRITING AND
4

LANGUAGE
4
3
2
2
1
0 x 0 x
1 2 3 4 5 6
–1 2 4 6 8 10
–2 –2

MATH
–3
–4 –4
–5
–6

Which of the following equations best describes the


figure above? The parabola and vertical line intersect at
A) y = 2x + 1 x = 6. What is the y-coordinate of the point of
intersection?
B) y  x  4

1
C) y = x
3
D) y = 2 x

CONTINUE
© TPR Education IP Holdings, LLC  |  603
SAT MANUAL

FUNCTION BASICS
READING

3 34
If f is a linear function and f(3) = 7, which of the If f(x) = ax + b, where a and b are constants, and
following could be the definition of f ? if f(0) = 4 and f(2) = 6, for what value of x does
A) f(x) = –x + 10 f(x) = 10 ?
WRITING AND
LANGUAGE

B) f(x) = –x2 + 16
C) f(x) = x2 – 2
D) f(x) = 2x + 2
MATH

23
If f(x) is a linear function such that f(2) = 2 and
9
f (5) = , which of the following is f(x) ?
2
5 1
A) f(x) = x+
6 3
B) f(x) = x + 2

9 2
C) f(x) = x+
2 3 35
D) f(x) = 5x + 6 If f(x) ≥ |x| + 10, then which of the following is the
LEAST possible value of f(x) ?

CONTINUE
604  |  © TPR Education IP Holdings, LLC
FUNCTIONS PRACTICE

FUNCTIONS IRL

READING
8 29
Sara drops a coin down a wishing well, which she The profit of a certain manufacturing company
knows to be 60 feet deep. She hears it hit the bottom can be expressed using the quadratic function
at t = 5 seconds after she dropped it. If the coin P(x), where P is the company’s profit, in thousands

WRITING AND
LANGUAGE
passed the halfway mark at approximately t = 3.5 of dollars, and x is the number of units sold, in
seconds, which equation, as H(t), best represents the thousands. If the maximum profit of the company
distance, in feet, the coin travels as it drops? is $90,000 and occurs when 4,000 units are sold,
A) H(t) = –4.9t which of the following equations could represent
P(x) ?
B) H(t) = 28 – 2.55t
A) P(x) = –2x2 + 30x + 90
C) H(t) = 2.55t – 14
B) P(x) = –5x2 + 40x + 10
D) H(t) = 2.4t2

MATH
C) P(x) = –10x2 + 4,000x
D) P(x) = –15x2 + 90x + 4,000

18
During the month of July, the daily number of
visitors, V(x), to a library can be modeled by the
function V(x) = ax + k, where x is the average daily
temperature, in degrees Fahrenheit, and a and k are
constants. If the daily number of visitors decreases
as the average daily temperature rises, which of the
following must be true?
A) a < 0
B) –1 < a < 1
C) a = 0
D) a > 0

CONTINUE
© TPR Education IP Holdings, LLC  |  605
SAT MANUAL

FUNCTIONS AND GRAPHS


READING

7 10
y

4 y
3
WRITING AND

3
LANGUAGE

2
2
1
x
–5 –4 –3 –2 –1 0 0 1 2 3 4 5 1
–1
x
–2 –3 –2 –1 O 1 2 3
MATH

–3 –1

–4
–2

Which of the following equations is shown in the –3


graph above?
A) y = x2 – 3 The graph of f(x) is shown above. What must be
B) y = x2 + 3 true about f(x) ?
C) y = –x2 – 3 A) f(x) has three solutions for which f(x) = 0.
D) y = e3x B) f(0) has three values.
C) The minimum value of f(x) is −2.
D) The maximum value of f(x) is 2.

CONTINUE
606  |  © TPR Education IP Holdings, LLC
FUNCTIONS PRACTICE

20

READING
y
6
6

44

22

–4 –3 –2 –1 x
O0 11 22 33 44

WRITING AND
–5 –4 –3 –2 –1 5

LANGUAGE
–2
–2

–4
–4

–6
–6

Which of the following equations best describes the


figure above?

MATH
x2
A) y = − −5
x3
x3
B) y =
5

C) y = x2 – 5x + 5

D) y = (x – 5)(x – 1)

CONTINUE
© TPR Education IP Holdings, LLC  |  607
SAT MANUAL

COMPOUND FUNCTIONS
READING

17 33
For two functions, where f(x) = 4 – 2x and where If f(x) = 2x + 3 and g(x) = 3x, then what is f(g(2)) ?
2
g(x) =  x , which of the following represents g(f(x)) ?
WRITING AND
LANGUAGE

4
A) 4 -
x

2
B) x+2

 x - x2 
C) 4  
MATH

 x 
 
1
D) 2 - x

23

x f(x) x g(x)
–4 6 –6 2
–2 2 –2 –4
0 0 0 4
2 –2 4 4
4 –6 6 2

Two functions, f and g, are defined according to the


tables above. What is the value of g(f(2)) ?
A) –4
B) –2
C) 2
D) 6

Answers can be found on page 741.

608 | © TPR Education IP Holdings, LLC


COORDINATE
GEOMETRY 101

Even though a graphing calculator can help with


many questions in the xy-plane, you won’t always be
able to use it. Gain a solid understanding of how to
answer these questions by completing this section
with only your pencil as a tool.

609
SAT MANUAL
READING

DRAWING LINES

Slope
WRITING AND

rise y 2 − y 1
LANGUAGE

=
run x 2 − x 1
MATH

Line #1
Plot these ordered pairs and draw a line to connect them.

(–4, –5) y
(–2, –1)
(1, 5)
(3, 9)

1. Use (1, 5) and (3, 9) to calculate the slope. (Which one is y1, and
which one is y2? It doesn’t matter! Just be consistent: Match the
x-coordinates in the same order.)

_____________________________________________________________

2. At what y-value does the line cross the y-axis? _____________________

3. What does x equal when the line crosses the y-axis? ________________

610 | © TPR Education IP Holdings, LLC


COORDINATE GEOMETRY 101

READING
Equation of a Line
There are many forms for the equation of a line, but the one you’ll likely see most
often on the SAT is the slope-intercept form.

Slope-intercept form

WRITING AND
LANGUAGE
y = mx + b

4. What does m represent? ______________________________________________

5. What does b represent? _______________________________________________

MATH
6. What do x and y represent? ___________________________________________

7. What is the slope-intercept form of Line 1? ____________________________

Line #2
Plot these ordered pairs and draw a line to connect them.
y
(–6, 8)
(–2, 5)
(2, 2)
(6, –1)

1. Use two of the ordered pairs to calculate the slope of this line:

___________________________________________________________

2. Use the slope and one of the ordered pairs to solve for the
y-intercept: _________________________________________________

3. Put them together to write the slope-intercept form of the line:

___________________________________________________________

© TPR Education IP Holdings, LLC | 611


SAT MANUAL
READING

Parallel and Perpendicular Lines


Plot the ordered pairs for each of these three lines, draw the three separate lines
on the facing page, label them, then calculate the slope for each. (Note that you
already have the y-intercept for each line—look at the third ordered pair in each
line.)
WRITING AND
LANGUAGE

Line #3
(–4, 2)
(–2, 3)
(0, 4)
(2, 5)
(4, 6)
MATH

Slope-intercept form:_______________________________________________

Line #4
(–4, –8)
(–2, –7)
(0, –6)
(2, –5)
(4, –4)

Slope-intercept form:_______________________________________________

Line #5
(–4, 10)
(–2, 6)
(0, 2)
(2, –2)
(4, –6)

Slope-intercept form:_______________________________________________

612  |  © TPR Education IP Holdings, LLC


COORDINATE GEOMETRY 101

READING
y

WRITING AND
LANGUAGE
x

MATH
1. What is the relationship between Lines #3 and #4?________________

2. What is the relationship between their slopes?____________________

3. What is the relationship between Lines #3 and #5?________________

4. What is the relationship between their slopes?____________________

© TPR Education IP Holdings, LLC  |  613


SAT MANUAL
READING

Reflections
One topic that is less likely but still possible to be tested is reflections of lines.
Sketch these three lines and label them 6, 7, and 8.

Line #6 y = 3x + 5
WRITING AND
LANGUAGE

Line #7 y = −3 x + 5

Line #8 y = −3 x − 5
MATH

1. Line #7 is the reflection of Line #6 across which axis?______________

2. What happened to the slope?_________________________________

3. What happened to the y-intercept?_____________________________

4. Line #8 is the reflection of Line #6 across which axis?______________

5. What happened to the slope?_________________________________

6. What happened to the y-intercept?_____________________________

614  |  © TPR Education IP Holdings, LLC


COORDINATE GEOMETRY 101

WE ♥ OUR CALCULATORS

READING
Now that your drawing skills have reminded you why all these neat facts hold up, On Section 4, you can
you can use your calculator again. use your calculator to
graph lines in the xy-
plane, so practice those
skills now. No calculator?
No problem! PITA is also
Tables

WRITING AND
LANGUAGE
a good option for these
Most graphing calculators are able to not only graph an equation in the xy-plane, questions.
but also create a table of values. Look up how to make tables using your specific
calculator.

11. In the standard coordinate plane, which of the following ordered pairs
falls on the line described by the equation 3y – x = 2 ?

MATH
A) (0, 1)

B) (–1, 1)
 1 
C)  , 1 
 3 
D) (–5, –1)

Intersections
Graphing calculators can also find the intersection of two lines. As above, look
up how to find the intersection between two lines on your specific calculator. You
might have to solve the equation for y before putting it into your calculator.

10. The lines y = 3x + 4 and y = 5x + 6 have one point in common. What


are the coordinates of that point?
A) (3, 4)
B) (2, –2)
C) (2, 2)
D) (–1, 1)

Answers can be found on pages 741–742.

© TPR Education IP Holdings, LLC | 615


SAT MANUAL
READING

FUN FACTS ABOUT COORDINATE GEOMETRY

Transformations
Translation
• A translation moves a figure without rotation or reflection; basically,
WRITING AND

it just slides around. This is the type of transformation most likely to


LANGUAGE

show up on the SAT. Just in case, though, below are some other types
you may see.

Rotation
• Rotation means turning an object about a single point, which is
called the center of rotation. For example, a second hand on a clock is
rotating every second, and its center of rotation is the middle of the
MATH

clock.

Reflection
• To reflect an object means to create its mirror image across a line of
reflection.
• Lines reflected across the x-axis have slopes that are negatives of each
other and also y-intercepts that are negatives of each other.
• Lines reflected across the y-axis have the same y-intercept, but their
slopes are negatives of each other.

Symmetry
• A figure has reflective symmetry if it looks the same after a reflection.
Such a figure reflects across a line of symmetry.
• A figure has rotational symmetry if it can be rotated and still look the
same. Such a figure is rotated around a point of symmetry.

616  |  © TPR Education IP Holdings, LLC


COORDINATE GEOMETRY 101

READING
COORDINATE GEOMETRY FORMULAS

Distances
• Distance: d = ( x1 − x2 )2 + ( y1 − y2 )2

WRITING AND
LANGUAGE
x + x 2 y1 + y2 
• Midpoint:  1 ,
 2 2 

Lines
y2 − y1
• Slope:

MATH
x 2 − x1

• Slope-intercept form: y = mx + b

• Standard form: Ax + By = C

Circles
• Circle centered at (0, 0): x 2 + y 2 = r 2

Circle centered at (h, k): ( x − h ) + ( y − k ) = r 2


2 2

© TPR Education IP Holdings, LLC  |  617


COORDINATE
GEOMETRY

11:15 Restate my assumptions: 


1. Mathematics is the language of nature. 
2. Everything around us can be represented and
understood through numbers. 
3. If you graph these numbers, patterns emerge.
Therefore: There are patterns everywhere in nature.
—Max Cohen in the film Pi

619
SAT MANUAL
READING

GOALS REVIEW
At the conclusion of this chapter, you will be able to accomplish the following:

• Apply skills of graphing functions on the xy-plane to questions about


lines
• Interpret the meanings of graphs and their values and roots in the
WRITING AND

xy-plane
LANGUAGE

• Apply skills of graphing functions on the xy-plane to questions about


parabolas
• Understand how to use the different forms of linear and quadratic
equations
MATH

COORDINATE GEOMETRY
Now that you know all about functions in the xy-plane, let’s cover a few related
topics about coordinate geometry, which deals with graphing lines, functions, and
shapes in the xy-plane.

LINES IN THE COORDINATE PLANE


We’ll start with finding and using the various properties of linear functions in the
xy-plane, including slope and intercepts, as well as how to interpret their real-life
applications.

SLOPE-INTERCEPT FORM
The equation of a line in the xy-plane can be written in slope-intercept form, which
is very useful for seeing how the line angles through the xy-plane and where it
crosses the y-axis.

Slope-Intercept Form of a Linear Equation: y = mx + b


• The slope of the line is m.
• The y-intercept of the line is b.
• The x and y are the coordinates of any point on the line.

620 | © TPR Education IP Holdings, LLC


COORDINATE GEOMETRY

Sometimes, it is necessary to calculate the slope of a line.

READING
y

8
Line m
6  36 

WRITING AND
 4, 5 

LANGUAGE
 5   
 - 4 , 3 4
 
2

–8 –6 –4 –2 2 4 6 8
x

–2
 12 
 -8, - 5 

MATH
  –4

–6

–8

33. What is the slope of line m, shown in the xy-plane above?

y2 - y1
Slope =
x 2 - x1

© TPR Education IP Holdings, LLC | 621


SAT MANUAL

18. In the xy-plane, line l passes through the points (2, 9) and (–4, 0).
READING

If line l crosses the y-axis at the point (0, k), what is the value of k ?
WRITING AND
LANGUAGE
MATH

Some questions ask about features of a graph that could be made from a table of
data.

16. Oliver is looking to rent an apartment for an indeterminate amount


of time and is choosing among three different apartments. The table
below shows the real estate fee, rent, and utilities costs for three
apartments below.

Apartment Real Estate Fee, F Rent, R Utilities, U


(dollars) (dollars per month) (dollars per month)
I 500 750 70
II 650 800 85
III 900 625 55

The total cost, C, of the real estate fee, rent, and utilities in terms of the
number of months, m, is given by C = F + (R + U)m. If the relationship
for total cost, C, and the number of months, m, for renting apartment
II is graphed in the xy-plane, what does the slope of the line represent?
A) The total monthly cost of utilities
B) The total monthly cost of the rent and utilities
C) The real estate fee
D) The total cost of the apartment

622  |  © TPR Education IP Holdings, LLC


COORDINATE GEOMETRY

READING
PARALLEL AND PERPENDICULAR LINES
Knowing the slope of a line is essential to determine whether that line is parallel or
perpendicular to a second line. Parallel lines have the same slope and no solutions,
while perpendicular lines have one solution and slopes that are negative reciprocals.

WRITING AND
LANGUAGE
(0, 4)
Line m
(–14, 0)
x
O (7, 0)
Line n

MATH
(0, k)

7. In the xy-plane above, line m is parallel to line n. What is the value


of k ?

A) –14
B) –8
C) –7
D) –4

Parallel lines have the same slope and no solutions.

© TPR Education IP Holdings, LLC | 623


SAT MANUAL
READING

College Board often makes the slope of a line harder to find by giving answer
choices that are not in slope-intercept form. Knowing how to calculate the slope
and y-intercept from an equation in standard form will save you from having to
convert it into slope-intercept form.
WRITING AND
LANGUAGE

Standard Form of a Linear Equation: Ax + By = C

A
• The slope of the line is - .
B
C
• The y-intercept of the line is .
MATH

3x – 5y = 15

12. In the xy-plane, the graph of which of the following equations is


perpendicular to the graph of the equation shown above?

A) 3x + 15y = 5
B) 3x + 9y = 15
C) 5x + 9y = 15
D) 5x + 3y = 15

Perpendicular lines have negative reciprocal slopes


and one solution.

624 | © TPR Education IP Holdings, LLC


COORDINATE GEOMETRY

READING
SIMULTANEOUS EQUATIONS—
INFINITE/NO SOLUTIONS
It is possible that two equations represent the same line and thus have infinitely
many solutions. Another possibility is that the two equations represent parallel
lines and thus have no solution.

WRITING AND
2x – cy = 9

LANGUAGE
5x – 9y = 11

8. In the system of equations above, c is a constant and x and y are variables. If


the system of equations has no solution, what is the value of c ?

81

MATH
A) −
11
18
B) −
5
18
C)
5
81
D)
11

© TPR Education IP Holdings, LLC  |  625


SAT MANUAL
READING

Questions on systems of equations with infinitely many solutions might sound


very different from questions on systems of equations with no solutions. However,
the two types of questions are actually very similar.

3x + 5y = 80
mx + ny = 20
WRITING AND
LANGUAGE

19. The system of equations above has infinitely many solutions. If m and n are
m
constants, what is the value of ?
n
MATH

Whether a system of linear equations has infinitely many solu-


tions or no solution, the slopes of the equations are equal.

626 | © TPR Education IP Holdings, LLC


COORDINATE GEOMETRY

READING
CURVES IN THE COORDINATE PLANE
Sometimes, you will be dealing with non-linear graphs in the coordinate plane.
Often, simply plugging in a given point will help you find the answer.

33. In the xy-plane, function g is defined as g(x) = 2x2 + c, and the point (–3, 19)
lies on the graph of g. What is the value of constant c ?

WRITING AND
LANGUAGE
MATH
The graph of the previous function would form a parabola, a symmetrical,
U-shaped curve that represents a quadratic equation.

The standard form of a quadratic equation is y = ax 2 + bx + c.

© TPR Education IP Holdings, LLC | 627


SAT MANUAL
READING

Though you may be most familiar with the standard form of a quadratic, factored
and vertex forms are also important on the SAT.

10. Which of the following equations is an equivalent form of y = x2 – x – 20 for


which the solutions to the equation are shown as constants or coefficients?
WRITING AND
LANGUAGE

A) y = (x + 5)(x – 4)

B) y = (x – 5)(x + 4)

2
1 81
C) y   x   
 2 4
2
MATH

 1  81
D) y   x   
 2 4

The factored form of a quadratic equation is


y = a(x – m)(x – n), where m and n are
the x-intercepts of the parabola.

If a question asks for you to switch between different forms of an equation, use
POE. If you know the factored form of the equation, you know the roots. If you
know the roots of the equation, you know the factored form.

628 | © TPR Education IP Holdings, LLC


COORDINATE GEOMETRY

g(x) = (x – 7)(x + 3)

READING
27. Which of the following is an equivalent form of the function g above in
which the minimum value of g appears as a constant or coefficient?
A) g(x) = (x – 2)2 – 25
B) g(x) = (x + 2)2 – 9

WRITING AND
LANGUAGE
C) g(x) = x2 – 4x – 21
D) g(x) = x2 – 21

f(x) = x2 – 12x + 9

8. What are the coordinates of the minimum value of function f ?

MATH
A) (6, –27)
B) (–6, –27)
C) (6, 27)
D) (–6, 27)

The vertex form of a quadratic equation is y = a(x – h)2 + k,


where (h, k) is the vertex of the parabola.
• If a is positive, the parabola opens upward and
k represents the minimum value of the equation.
• If a is negative, the parabola opens downward and
k represents the maximum value of the equation.
• If the equation of a parabola is given in standard form,
b
then h = - .
2a
• If the equation of a parabola is given in factored form, then
h is equal to the average of the roots of the equation.
• Once the value of h is determined by any method, the
value of k can be determined using k = f(h).

© TPR Education IP Holdings, LLC | 629


SAT MANUAL
READING

COORDINATE GEOMETRY DRILL


Time: 8 minutes

Unless you are aiming for a top score, don’t try all the questions! Use your POOD to choose the best ones for you.
WRITING AND
LANGUAGE

9 11
Line l can be modeled by the equation y = 5x – 9. y
If line m is parallel to line l, which of the following
8
could represent line m ?
MATH

A) x + 5y = –2 6

B) –3x + 5y = 15 4
C) –5x + y = 11
2
D) 10x + 2y = 9

O x
–4 –2 2 4
–2

–4

–6

–8

If the graph above describes the equation


y  x  k  y  x  , what is the value of the constant
k?

A) −3

1
B) −
2
1
C)
2

D) 3

630  |  © TPR Education IP Holdings, LLC


COORDINATE GEOMETRY

23 27

READING
y
y = k(x – 9)(x + 3)

In the quadratic equation above, k is a nonzero


a
constant. The graph of the equation in the xy-
plane is a parabola with vertex (a, b). Which of the 1

WRITING AND
following is equal to b ?

LANGUAGE
x
1
A) –48k
B) –36k
C) –27k
D) –3k The graph of the linear function a is shown in the
xy-plane above. The slope of the graph of linear
function b is –3 times the slope of the graph of a. If

MATH
b passes through the point (0, 3), what is the value
of b(5) ?

A) –15
B) –8
C) –5
D) –2

© TPR Education IP Holdings, LLC  |  631


SAT MANUAL

29 32
READING

Depth of Topsoil 10x − 5y = p


d
2x − y = 3

In the xy-plane, the equations above represent the


same line for some constant p. What is the value
5.0
WRITING AND

of p ?
LANGUAGE

Topsoil depth (cm)

4.5
MATH

4.0 t
0 2 4 6 8 10 12 14
Time (years)

Every year that a certain field is planted and


harvested, the land loses 0.1 cm of topsoil. Because
topsoil contains vital nutrients and microorganisms,
the depletion of topsoil is of great ecological
concern. The graph above displays the actual and
projected soil erosion in terms of the depth of
topsoil d over a period of t years, starting in 2010.
Which of the following represents the depth of the
topsoil in 2010?
A) The t-intercept
B) The d-intercept
C) The point (10, 4.2)
D) The slope of the line

632  |  © TPR Education IP Holdings, LLC


COORDINATE GEOMETRY

Summary
• For questions about the graphs of
functions, remember that f(x) = __________.
What can you do if the graph contains
a labeled point?
___________________________________

• The y-intercept can be found using the


________________________ form.

• What are three other names for the roots of


a function?

______________________________________

______________________________________
______________________________________
At the root, y = ____.
To find the roots, you can
___________________________________,
___________________________________,
or ________________________________.

• What is the formula for the slope of a line?

______________________________________
Parallel slopes are
___________________________________.
Perpendicular slopes are
___________________________________
___________________________________.

• What are the three forms of a quadratic


equation?

___________________________________,
___________________________________,
and __________________________________.

• Roots can be found using the


________________ form.

• Maximum or minimum values can be


found using the ________________ form.

• I have accomplished _________ of the 4


goals stated at the beginning of this chapter.

© TPR Education IP Holdings, LLC | 633


COORDINATE
GEOMETRY
PRACTICE

635
SAT MANUAL

ROOTS AND SOLUTIONS


READING

4 10
y y
WRITING AND

y = f(x)
LANGUAGE

(s, 0) (t, 0) (c, 0)


x x
O (–a, 0) O
MATH

(0, –b)

The figure above shows the graph in the xy-plane of The graph in the xy-plane of the function f is shown
f (x )  x 2  4 x  3 . What is the value of s ? above. Which of the following functions is f ?
A) 1 A) f(x) = (x – a)(x – b)
B) 2 B) f(x) = (x – a)(x + c)
C) 3 C) f(x) = (x + a)(x – c)
D) 4 D) f(x) = (x + b)(x – c)

CONTINUE
636  |  © TPR Education IP Holdings, LLC
COORDINATE GEOMETRY PRACTICE

SLOPE AND INTERCEPTS

READING
12 20
If b is a negative constant, which of the following Line l has a y-intercept of 2 and contains the
could be the graph of 3(y + 1) + 4x = – b + 3 in the points (a, a) and (2a, 12). If a is a positive
xy-plane? constant, what is the slope of line l ?

WRITING AND
LANGUAGE
y
A) y B)

x
x

MATH
y y
C) D)

x x

16
y

(6, 9)

(2, 3)
x

What is the slope of the line in the graph above?

CONTINUE
© TPR Education IP Holdings, LLC  |  637
SAT MANUAL

PARALLEL AND PERPENDICULAR LINES


READING

3 8
Line p can be described by the equation y

–8x + 4y =  12. If line r is perpendicular to line p,


which of the following could be the graph of line r
WRITING AND
LANGUAGE

in the xy-plane?

A) y B) y
x

x x
MATH

Line s is shown in the graph above. If line t is parallel


to line s, which of the following could be the equation
C) D) of line t ?
y y
A) 2x – 3y = 6
B) 3x – 2y = 2
x x C) 3x + 2y = 2
D) 2x + 3y = 12

CONTINUE
638  |  © TPR Education IP Holdings, LLC
COORDINATE GEOMETRY PRACTICE

FORMS OF EQUATIONS

READING
9 18
The equation y = (3x – 3)(x – 3) represents a The profit that a smoothie bar makes can be
parabola in the xy-plane. The x- and y-coordinates expressed by the equation P = –3(x – 4)2 + 1,500,
of the vertex of this parabola appear as constants where x is the price per smoothie sold (in dollars).

WRITING AND
LANGUAGE
or coefficients in which of the following equivalent What price, in dollars, should the smoothie bar
equations? charge its customers in order to maximize its
A) y = 3x2 – 12x + 9 profits?
B) y = 3(x – 2)2 – 3
C) y = 3x(x – 4) + 9
D) y = 3(x2 – 4x) + 9

MATH

CONTINUE
© TPR Education IP Holdings, LLC  |  639
SAT MANUAL

ROOTS AND SOLUTIONS


READING

4 24
2
The function g is defined by g (x )  x  px  39 . If y
–13 is a root of g, what is the value of p ? 7
A) 3
WRITING AND

6
LANGUAGE

B) 9 5
C) 13 4
D) 16 3
2
1
0 x
MATH

1 2 3 4 5 6 7 8
9
10
Which of the following equations has x-intercepts The graph above has roots at approximately x =
7
at (2, 0) and (5, 0) ? 20
and x = . Which of the following equations most
A) y = x2 – 2x + 5 7
B) y = x2 – 7x + 10 closely represents the graph?
C) y = x2 + 2x – 5
A) y  x 2  4.3x  4.1
D) y = x + 7x – 10
2

B) y  x 2  1.7 x  2.9

C) y  2 x 2  1.7 x  2.8

D) y  2 x 2  4.1x  4.3

33
The function f in the xy-plane is defined by
f(x) = (x – 8)(x + 17). What is the straight-line
distance between the x-intercepts of f ?

CONTINUE
640  |  © TPR Education IP Holdings, LLC
COORDINATE GEOMETRY PRACTICE

SLOPE AND INTERCEPTS

READING
5 27
Which of the following could be the graph of the y
3
equation y  6 x  3 in the xy-plane?
2

WRITING AND
LANGUAGE
A) B)
y y
 22 
0,
 7 
8 
 , 0 
7
x
x x

MATH
C) y D) y
Which of the following could be the equation of
line q, shown above in the xy-plane?
A) 2(y – 1) = –1.75x
x x B) 3(0.9y – 2.5) = –8.1x
C) 4(y – 4) = –11x
D) 4(0.7y – 2.2) = –7.7x

33
Line l contains the points (0, –2) and (5, 2). Line m
has the same slope as line l and a y-intercept of 2. If
point P on line m has the coordinates (5, y), what is
the value of y ?

CONTINUE
© TPR Education IP Holdings, LLC  |  641
SAT MANUAL

PARALLEL AND PERPENDICULAR LINES


READING

13 26
y y
WRITING AND
LANGUAGE

(0, 2)
x
x 9 
, - 2
(–6, 0)  2 
MATH

The graph above shows two perpendicular lines


Line f is shown in the xy-plane above. If line g is
perpendicular to line f, which of the following could in the xy-plane. If the lines intersect at the point
be the equation of line g ? 9 
 2 , 2  , which of the following could be the
A) 6x + 2y = 2  
B) x + 3y = 12 equations of the two lines?
C) –2x + 6y = –12 A) y = –2x + 7
D) –6x + 3y = –3 4y = 2x – 17
B) y = 3x + 5
2y = –x + 20
C) y = 4x – 20
4y = –x +20
D) y = –4x + 2
6y = 2x – 2

CONTINUE
642  |  © TPR Education IP Holdings, LLC
COORDINATE GEOMETRY PRACTICE

FORMS OF EQUATIONS

READING
7 27
y = (x – 4)2 + 2
3
y = –x + 6 h(y )  -16 y 2  18 y 
16

WRITING AND
LANGUAGE
One of the solutions to the system of equations The height of a cannonball, h, y seconds after its
above is the vertex of the parabola. Which of the launch, is described by the function above. Which
following represents the coordinates of that point of of the following equivalent forms of the function
intersection? has the maximum height reached by the cannonball
A) (6, 0) during its flight as a coefficient or constant?
B) (4, 2)
3
C) (3, 3) A) h(y) = -16 y 2 + 18 y +

MATH
16
D) (–4, 10)
 1  19 
B) h(y) = -16  y    y - 
 16  16 

1
C) h(y) =
16
16 y  119 -16 y 
2
84  9 
D) h(y) = - 16  y - 
16  16 

Answers can be found on page 742.

© TPR Education IP Holdings, LLC | 643


ADVANCED
COORDINATE
GEOMETRY

The power of mathematics is often to change one


thing into another, to change geometry
into language.
—Marcus du Sautoy

645
SAT MANUAL
READING

GOALS REVIEW
At the conclusion of this chapter, you will be able to accomplish the following:

• Apply skills of graphing in the xy-plane to more unusual concepts


• Identify which quadrants will contain parts of a graph in the xy-plane
• Identify the graphs of functions of different degrees in the xy-plane
WRITING AND

• Know how to work with the equation of a circle in the xy-plane


LANGUAGE

• Understand how changes to an equation will change the graph in the


xy-plane
MATH

646  |  © TPR Education IP Holdings, LLC


ADVANCED COORDINATE GEOMETRY

READING
MORE COORDINATE GEOMETRY
In the last chapter, you learned about the most common ways that the SAT tests
concepts in the xy-plane. If you are aiming for a top score, you will also want to be
familiar with these advanced ideas that come up once in a while on the test.

WRITING AND
LANGUAGE
MORE LINES IN THE COORDINATE PLANE
In Coordinate Geometry 101, you were given the formulas for distance and mid-
point in the xy-plane. Of course, College Board will find unique ways to tests
those concepts.

MATH
y = |x − 2|
y=4

12. In the xy-plane, the equations above intersect at points C and D. What is
the distance between C and D ?

A) 4
B) 6
C) 8
D) 12

8. In the xy-plane, the parabola with equation y = (x − 4)2 − 3 contains points A How can you use the
and B, which have the same y-coordinate. If point A has an x-coordinate of 1, vertex form to find the
what is the x-coordinate of point B ? midpoint of AB ?

A) 1
B) 4
C) 6
D) 7

© TPR Education IP Holdings, LLC  |  647


SAT MANUAL
READING

Occasionally, a question on the SAT will refer to the quadrants of the xy-plane.
The x-axis and y-axis divide the coordinate plane into 4 parts. The one in the
upper right, where all x- and y-values are positive, is Quadrant I, then the remain-
ing quadrants are numbered going counter-clockwise from there, as seen below.

y
WRITING AND
LANGUAGE

II I

III IV
MATH

9. Line m has a positive y-intercept and a negative slope in the xy-plane.


Which of the following must be true?
A) The line passes through Quadrants I and II only.
B) The line passes through Quadrants I, II, and IV only.
C) The line passes through Quadrants II and III only.
D) The line passes through Quadrants I, III, and IV only.

The next question takes the quadrant concept and adds the idea of the graph of
a system of inequalities to it. Sketch the system of inequalities and shade in the
graph to determine where the solutions or overlap will lie.

1
26. If the system of inequalities y ≤ –3x + 2 and y < – x + 2 is graphed in
3
the xy-plane, which quadrants contain the solutions to the system?
A) All four quadrants contain solutions to the system.
B) Quadrants I and II
C) Quadrants III and IV
D) No quadrants contain any of the solutions to the system.

648  |  © TPR Education IP Holdings, LLC


ADVANCED COORDINATE GEOMETRY

READING
MORE CURVES IN THE COORDINATE PLANE
On the SAT, most of the non-linear graphs in the xy-plane are parabolas, but there
will be the occasional third or fourth degree function. It helps to know what those
look like.

WRITING AND
LANGUAGE
Third-degree polynomial with Fourth-degree polynomial with three
one real root distinct real roots
f (x) = x3 + 2 f (x) = x4 – 2x2
y y

4 4

MATH
2 2

x
x –2 O 2
–2 O 2

–2
–2

The function on the right above also has a “double root,” which means that the
graph touches the x-axis at just one point—(0, 0)—before bending away again in
the opposite direction. This means that the equation of the line will have the factor
that yields that value of x raised to an even exponent (in this case, the x is squared).

© TPR Education IP Holdings, LLC  |  649


SAT MANUAL
READING

2
WRITING AND
LANGUAGE

x
–1 –0.5 O 0.5 1
MATH

–1

13. Which of the following could be the equation of the graph above?
A) y = x2 − 1
B) y = x3 − x
C) y = x3 − x2
D) y = x4 − x2

650  |  © TPR Education IP Holdings, LLC


ADVANCED COORDINATE GEOMETRY

READING
CIRCLES IN THE COORDINATE PLANE
Like parabolas, circle equations in the xy-plane also have a standard form, and
College Board likes to ask questions about circles in non-standard form. Just as
with parabolas, knowing the parts of the standard form and how to manipulate
circle equations into different forms can be useful.

WRITING AND
LANGUAGE
The standard form of a circle is (x – h)2 + (y – k)2 = r 2,
where (h, k) is the center and r is the radius. If the circle is
centered at the origin, the equation is x 2 + y 2 = r 2.

MATH
7. In the coordinate plane, if the point (1, 3) is on circle M, and circle M has a
radius of 4, which of the following equations could describe circle M ?

 x  2    y  3
2 2
A)  16

 x  1   y  1
2 2
B)  16

 x  1   y  3
2 2
C) 4

 x  5   y  3
2 2
D) 4

© TPR Education IP Holdings, LLC | 651


SAT MANUAL
READING

To convert a circle equation into the standard form,


complete the square.
1. Move any constants over to the right side of the
equation.
WRITING AND
LANGUAGE

2. If there is a coefficient on the squared terms,


factor it out of both sides of the equation.
3. Take half of the coefficient on the x-term (not
the x 2-term), square it, and add it to both sides
of the equation.
4. Convert the x 2-term, the x-term, and the
MATH

constant on the left to square form: (x – h)2.


5. Repeat steps 2 and 3 with the y-terms.

20. Which of the following represents the center and the radius,
respectively, of the circle described by the equation
x2 + y2 – 8x + 6y = –21 graphed in the coordinate plane?
A) Center: (–4, 3), Radius: 2
B) Center: (–4, 3), Radius: 4
C) Center: (4, –3), Radius: 2
D) Center: (4, –3), Radius: 4

(x − 2)2 + (y − 5)2 = 36

27. Circle C lies in the xy-plane and has the equation above. Which of the
following points lies outside the circumference of Circle C ?
A) (7, 1)
B) (2, 5)
C) (0, 0)
D) (−1, 5)

652 | © TPR Education IP Holdings, LLC


ADVANCED COORDINATE GEOMETRY

READING
TRANSFORMATION OF GRAPHS
Occasionally, a question will ask about a transformation of a graph. Graph trans-
formation means the function has been changed in some way, either moved up,
down, or side-to-side, or sometimes even flipped over.

32. In the xy-plane, a parabola with the equation y = (x − 1)2 − 6 is

WRITING AND
LANGUAGE
translated 9 units to the left, resulting in a parabola with the equation
y = (x + c)2 − 6. What is the value of c ?

MATH
In relation to f(x):
• f(x) + k is shifted upward k units in the xy-plane.
• f(x) – k is shifted downward k units in the xy-plane.
• f(x + k) is shifted to the left k units in the xy-plane.
• f(x – k) is shifted to the right k units in the xy-plane.
• –f(x) is flipped upside down over the x-axis.
• f(–x) is flipped left-right over the y-axis.

© TPR Education IP Holdings, LLC | 653


SAT MANUAL

h(x) = 3 – 3x
READING

15. The function h is defined by the equation above. Which of the following is the
graph of y = –h(x) in the xy-plane?

A) y B) y
WRITING AND
LANGUAGE

x x
MATH

C) y D) y

x x

654  |  © TPR Education IP Holdings, LLC


ADVANCED COORDINATE GEOMETRY

Summary
• In the xy-plane, Quadrant I is on the
__________________ corner, and the
remaining quadrants are numbered
__________________________ from there.

• What is the standard form of a circle?

_____________________________________

• How do you get a circle equation into


standard form?

_____________________________________

• When a function undergoes a


transformation, remember to try
____________________ and to use _________.

• When dealing with descriptions of graphs


with no picture, try ____________________
______________________________.

• I have accomplished _________ of the 5


goals stated at the beginning of this chapter.

© TPR Education IP Holdings, LLC | 655


ADVANCED
COORDINATE
GEOMETRY
PRACTICE

657
SAT MANUAL

CIRCLES
READING

5 14

y
A circle graphed in the xy-plane has the equation
x2 + y2 –  4x + 2y – 4 = 0. Which of the following
8
equivalent forms of the equation of the circle has
WRITING AND

7
LANGUAGE

the square of the radius as a coefficient or constant?


6
A) x2 + y2 – 4x + 2y – 1 = 3
5

4
B) y2 + 2y – 2 = 2 + 4x – x2
3 C) (x – 2)2 + (y + 1)2 – 4 = 5
2 D) (x – 2)2 + (y + 1)2 = 9
1
0 x
MATH

–1 0 1 2 3 4 5 6 7 8
–1

Which equation best represents the circle in the


figure above?
A) (x – 3) – (y – 3) = 3
B) (x – 3) + (y – 3) = 9
C) (x – 3)2 – (y – 3)2 = 3
D) (x – 3)2 + (y – 3)2 = 9

CONTINUE
658  |  © TPR Education IP Holdings, LLC
ADVANCED COORDINATE GEOMETRY PRACTICE

TRANSFORMATION OF GRAPHS

READING
3 12
y
6

WRITING AND
LANGUAGE
5

MATH
0 x
–8 –7 –6 –5 –4 –3 –2 –1 0 1 2
–1

–2
The complete graphs of the functions m and n
are shown in the xy-plane above. Which of the
following could be equal to n(x) ? The graph above has roots at x = –5 and x = –2. If
A) m(x + 5) the function is shifted two units to the right on the
x-axis, what is the new equation of the parabola?
B) m(x – 5)
A) y = x2 + 7x + 12
C) m(x) + 5
B) y = x2 + 11x + 28
D) m(x) – 5
C) y = x2 + 3x – 2
D) y = x2 + 3x

CONTINUE
© TPR Education IP Holdings, LLC  |  659
SAT MANUAL

OTHER TOPICS
READING

17 19
y The parabola y = (x – 6)2 – 4 is graphed in the
xy-plane. If a triangle is formed by connecting
the vertex of the parabola with the 2 roots on the
WRITING AND
LANGUAGE

x-axis, what is the area of the triangle?

x
MATH

The graph of f(x) is shown above in the xy-plane.


What is the y-intercept of the graph of f(x – 1) + 2 ?

CONTINUE
660  |  © TPR Education IP Holdings, LLC
ADVANCED COORDINATE GEOMETRY PRACTICE

CIRCLES

READING
25 28
The equation of a circle in the xy-plane is x + 4x +
2 A circle (not shown) has its center at the point
y2 – 10y = 20. Which of the following is an equation (–3, –2) and is tangent to the y-axis. Which of the
of the same circle? following points lies outside the circle?

WRITING AND
LANGUAGE
A) (–5, 1)
A) (x  2)2  ( y  5)2  10
B) (–4, –2)
2 2
B) (x  2)  ( y  5)  49 C) (–2, 0)
D) (0, –2)
C) (x  4)2  ( y  10)2  20

D) (x  4)2  ( y  10)2  400

MATH
26
3x2 + 18x + 3y2 – 6y = 18
What is the radius of the circle in the xy-plane
described by the equation above?

A) 12
B) 108
C) 6
D) 4

CONTINUE
© TPR Education IP Holdings, LLC  |  661
SAT MANUAL

TRANSFORMATION OF GRAPHS
READING

9 15
y y
5
4
WRITING AND
LANGUAGE

3
2
1
x
–5 –4 –3–2 –1 0 1 2 3 4
–1
1
–2
x
If the graph of y = f(x) is shown above, which of the 1
MATH

following could be the graph of y = f(x + 1) – 2 ?


A) y
5
4
3
2
1
x
–5 –4 –3–2 –1 0 1 2 3 4
–1
y
B)
5
The parabolic graph shown above is of a function of
4
the form f(x) = a(x + 1)2 + b.
3
2
1 Which of the following are the values of a and b,
x respectively?
–5 –4 –3–2 –1 0 1 2 3 4
–1 A) –2 and –6
–2
B) –1 and 6
y
C) C) 1 and 6
5
4 D) 2 and –6
3
2
1
x
–5 –4 –3–2 –1 0 1 2 3 4
–1
–2

D) y
5
4
3
2
1
x
–5 –4 –3–2 –1 0 1 2 3 4
–1
–2 CONTINUE
662  |  © TPR Education IP Holdings, LLC
ADVANCED COORDINATE GEOMETRY PRACTICE

OTHER TOPICS

READING
17 20
The maximum value for the polynomial function
t is 3. If the solutions of t are 4 and –2, which of y
the following could be the graph of y = t(x) in the

WRITING AND
LANGUAGE
xy-plane?
A) y
10
y = h(x)

5
1
x x
–20 –15 –10 –5 0 5 10 15 20 O1
–5

MATH
–10

The path of a particle relative to a sensor placed at


the origin is plotted in the xy-plane above. What is
B) the positive difference between the minimum value
y of h(x) and the maximum value of h(x) ?
10 A) 2
5 B) 6
x C) 7
–20 –15 –10 –5 0 5 10 15 20
D) 8
–5

–10 35
y ≥ 4x
y ≤ 12x – 2400
y In the xy-plane, if a point with coordinates (m, n)
C)
10 lies in the solution set of the system of inequalities
above, what is the minimum possible value of n ?
5

x
–20 –15 –10 –5 0 5 10 15 20
–5

–10

D) y
10

x
–20 –15 –10 –5 0 5 10 15 20 Answers can be found on pages 742–743.
–5

–10
© TPR Education IP Holdings, LLC | 663
PLANE
GEOMETRY 101

There are not many geometry questions on the


SAT, but the few you’ll see cover a wide range of
topics. Make sure you know the basic geometry
rules and formulas.

REFERENCE INFORMATION

l c 2x 60° x s 45° s 2

r h b
w 30° 45°
b a x 3 s
A = πr 2 A = lw 2
c = a2 + b 2
C = 2 πr A = 1 bh Special Right Triangles
Reference Information

•r h
h r h
h
w r w
l 
4 3
V = lwh V = π r 2h V= 3 πr 1 1
V = 3 π r2h V = 3 wh

The number of degrees of arc in a circle is 360.


The number of radians of arc in a circle is 2p.
The sum of the measures in degrees of the angles of a triangle is 180.

665
SAT MANUAL
READING

FORMULAS OF BASIC SHAPES


College Board is kind enough to provide a reference box with the basic geometry
facts and formulas at the start of each Math section. Although you can always flip
back to it as needed, you can save time if you already know these facts by heart.
Try to work the following questions without using the information provided on
the previous page.
WRITING AND
LANGUAGE

Area

Figure Formula

Square
MATH

Rectangle

Parallelogram

Triangle

B C

A 5 D

1. What is the area of the shaded region of rectangle ABCD ?

_______________________________________________________________

L M
5 cm

4 cm

O 9 cm N

2. The dimensions of parallelogram LMNO, shown above, are given in


centimeters. What is the area, in square centimeters, of parallelogram
LMNO ?

_______________________________________________________________

666  |  © TPR Education IP Holdings, LLC


PLANE GEOMETRY 101

READING
Perimeter
How do you get the perimeter of any figure with sides?

_______________________________________________________________

WRITING AND
LANGUAGE
Circles
It’s easier to think of the formulas for circles together.

Area Circumference

MATH
3. What is the area of a circle with a circumference of 14p ?

_______________________________________________________________

Volume

Figure Formula
Cube

Rectangular solid

Cylinder

4. The volume of a cube is 64 cubic inches. What is the length, in inches,


of one edge of the cube?

_______________________________________________________________

© TPR Education IP Holdings, LLC  |  667


SAT MANUAL
READING

Geometry IRL
There are only so many real-world concepts the SAT can apply a geometric for-
mula to. So when you’re stuck on a word problem, it’s very helpful to associate the
geometric formula to the real-life concept. Even the units can provide a clue.

Match perimeter/circumference, area, or volume to the following terms and


WRITING AND

concepts.
LANGUAGE

Real Life Geometry

Tile

Tire revolution
MATH

Fences

Cubic feet

m2

Carpet

Ribbon

Water

Square centimeters

Brick

in3

Snow

5. A rectangular dance floor has a tightly stretched rope running around its
edge. If the dance floor measures 20 feet by 30 feet, what is the length, in feet,
of the rope?

__________________________________________________________________________

668  |  © TPR Education IP Holdings, LLC


PLANE GEOMETRY 101

READING
ANGLES
The SAT will test the measure of degrees in given shapes. Fill in the chart below
with the measure, in degrees, of each angle and shape.

Figure Degrees

WRITING AND
LANGUAGE
Right Angle

Straight Line

Triangle

Quadrilateral

MATH
Circle

A B

xº yº
C

6. In the figure above, if AC is perpendicular to BC, which of the following


represents the value of x, in terms of y ?

A) 180 + y
B) 180 – y
C) 90 – y
D) 45 + y

100º


7. In the figure above, what is the value of x + y ?

__________________________________________________________________________

© TPR Education IP Holdings, LLC  |  669


SAT MANUAL
READING

Parallel Lines and Angles

When a line intersects two parallel lines


• Two kinds of angles are formed: big angles and small
angles.
WRITING AND
LANGUAGE

• Each big angle is equal to the other big angles.


• Each small angle is equal to the other small angles.
• Any big angle plus any small angle is 180°.

C
MATH

45°
A

B
d

8. In the figure above, two parallel lines, line A and line B, intersect line C. What
is the measure of angle d ?

__________________________________________________________________________

a 160°
p

q
b
r

9. In the figure above, lines p and q are parallel. Line r intersects both p and q, as
shown. What is the sum of the measures of angle a and angle b ?

__________________________________________________________________________

670 | © TPR Education IP Holdings, LLC


PLANE GEOMETRY 101

READING
RIGHT TRIANGLES
If you know two sides of a right triangle, you can find the third side using the
Pythagorean Theorem.

WRITING AND
Pythagorean Theorem: a 2 + b 2 = c 2

LANGUAGE
(where c is the hypotenuse)

Fill in the third side of each triangle listed below. Pay close attention to the famous
Pythagorean Triples—the ones that are all integers. College Board loves to use

MATH
those in questions, so recognizing them immediately can save you time. For the
others, either leave your answer unsimplified under the root, or use your calculator
and leave it as a decimal.

a b c

1 10

3 4

5 13

8 10

7 25

2 2 3

5 5 2

6 7

3 6

4 2 8

© TPR Education IP Holdings, LLC | 671


SAT MANUAL
READING

Special Right Triangles


Special right triangles are unique in that we know the measurement of every angle
and every side without using trigonometry. You may have spotted them in some
of the triangles in the previous chart. If you can spot them and memorize the
relationships, you’ll save yourself some time. But if you can’t, vocabulary and the
Pythagorean Theorem will come to the rescue.
WRITING AND
LANGUAGE

45 o
o
30
x x 2 x 3 2x

o
45 o 60
MATH

x x

a. The measure of each angle in an isosceles right triangle:_________________

b. The measure of each angle in an equilateral triangle:___________________

c. How would you find the height of an equilateral triangle?_______________

___________________________________________________________

672  |  © TPR Education IP Holdings, LLC


PLANE GEOMETRY 101

READING
B

A C

WRITING AND
LANGUAGE
10. In the right isosceles triangle above, what is the length of AC ?

__________________________________________________________________________
B

10

MATH
A C
D

11. What is the length of BD in equilateral triangle ABC, shown above?

__________________________________________________________________________

© TPR Education IP Holdings, LLC  |  673


SAT MANUAL
READING

CIRCLES
If you know just one thing about circles, you know everything. Just organize the
information in a CArd chart.

Circumference Area radius diameter


WRITING AND
LANGUAGE

d
pd or 2pr pr 2 2r
2

C A r d
MATH

10

36p

4p

18p

144p

14

9p

20p

674  |  © TPR Education IP Holdings, LLC


PLANE GEOMETRY 101

READING
VOCABULARY
Sometimes, the key to getting a question right is knowing the geometry vocabu-
lary. Know your basic terms and the ones below, and you’ll be in good shape.

Term Definition
Arc

WRITING AND
LANGUAGE
Area
Bisect
Chord
Circumference
Circumscribed
Collinear

MATH
Congruent
Diagonal (of a polygon)
Diameter
Edge (of a 3-D figure)
Equilateral triangle
Face (of a 3-D figure)
Inscribed
Isosceles triangle
Parallel
Perimeter
Perpendicular
Plane
Polygon
Quadrilateral
Radius
Regular polygon
Sector
Similar
Surface area
Tangent
Vertex/Vertices

© TPR Education IP Holdings, LLC  |  675


SAT MANUAL
READING

FUN FACTS ABOUT FIGURES


Read and review the following facts you need to know about plane geometry. Put
an “x” next to any rule(s) that you don’t know, and ask your teacher about them.

Angle Facts
WRITING AND
LANGUAGE

• There are __________ in a right angle.


• When two straight lines intersect, angles __________ each other are
equal.
• There are __________ in a straight line.
• Two lines are ___________________ when they meet at a 90° angle.
• The sign for perpendicular is __________ .
• __________ means to cut exactly in half.
MATH

• There are __________ in a triangle.


• There are __________ in any four-sided figure.

Triangle Facts
In any triangle
• The __________ side is opposite the largest angle.
• The __________ side is opposite the smallest angle.
• All angles add up to __________.
• Area = ____________________
• The __________ is the perpendicular distance from the base to the
opposite vertex.
• __________________ is the sum of the sides.
• The third side of any triangle is always less than the __________ and
greater than the ____________________ of the other two sides.

In an isosceles triangle
• Two __________ are equal.
• The two __________ opposite the equal __________ are also equal.

In an equilateral triangle
• All three __________ are equal.
• All angles are each equal to __________.

676  |  © TPR Education IP Holdings, LLC


PLANE GEOMETRY 101

READING
Four-Sided Figure Facts
In a quadrilateral
• All four angles add up to __________.

In a parallelogram
• Opposite sides are __________ and equal.

WRITING AND
• Opposite angles are __________.

LANGUAGE
• Adjacent angles are _______________ (add up to 180°).
• Area = ____________________
• The height is the _______________ distance from the base to the
__________ side.

In a rhombus
• Opposite sides are __________.

MATH
• Opposite angles are __________.
• Adjacent angles are _______________ (add up to 180°).
• All 4 sides are __________.
• Area = ____________________
• The height is the _______________ distance from the base to the
__________ side.
• The diagonals are _______________.

In a rectangle
• Rectangles are special parallelograms; thus, any fact about parallelo-
grams also applies to rectangles.
• All 4 angles are each equal to __________.
• Area = ____________________
• Perimeter = ______________________________
• The diagonals are __________.

In a square
• Squares are special rectangles; thus, any fact about rectangles also
applies to squares.
• All 4 sides are __________.
• Area = __________
• Perimeter = __________
• The diagonals are _______________.

Circle Facts
Circle
• There are __________ in a circle.

Radius (r)
• The distance from the __________ to any point on the edge of the
circle
• All __________ in a circle are equal.

© TPR Education IP Holdings, LLC  |  677


SAT MANUAL
READING

Diameter (d)
• The distance of a line that connects two points on the edge of the circle,
passing through the __________
• The longest __________ in a circle.
• Equals twice the __________.
WRITING AND

Chord
LANGUAGE

• Any _______________ connecting two points on the edge of a circle


• The longest chord is called the _______________.

Circumference (C)
• The __________ around the outside of the circle
• C = __________
MATH

Arc
• Any part of the _______________
• The length of an arc is _______________ to the size of the interior angle.

Area
• The amount of space within the _______________ of the circle
• A = __________

Sector
• Any part of the __________ formed by two __________ and the out-
side of the circle
• The area of a sector is _______________ to the size of the interior angle.

Line Facts
Lines
• A line has no width and extends infinitely in both directions.
• Any line measures __________. 
• A line that contains points A and B is called AB (line AB).
• If a figure on the SAT looks like a straight line, and that line looks
like it contains a point, it does.

Line segments
• A line segment is a part of a line and has two _______________.
• The degree measure of a line segment is __________.
• A line segment, which has endpoints of A and B, is written as AB .

Perpendicular
• Two lines that intersect in a __________ angle are perpendicular and
their __________ are negative reciprocals.

Answers can be found on pages 743–745.

678 | © TPR Education IP Holdings, LLC


PLANE GEOMETRY 101

READING
PLANE GEOMETRY FORMULAS
Here’s a list of all the plane geometry formulas that could show up on the SAT.
Memorize the formulas for perimeter/circumference, area, and volume for basic
shapes. Remember, though the SAT provides many of these formulas, knowing
them well can save time on the test.

WRITING AND
LANGUAGE
Circles
• Area: A = pr 2
• Circumference: C = 2pr = pd
• Arc length: r q, where (q) is the central angle in radians
• An inscribed angle in a circle has half the measure of the central
angle with the same endpoints on the circumference of the circle.

MATH
Triangles
1
• Area: A = bh
2
• Perimeter: P = sum of the sides

• Pythagorean Theorem: a 2 + b 2 = c 2

SOHCAHTOA
opposite
• sin(q) =
hypotenuse
adjacent
• cos(q) =
hypotenuse
opposite
• tan(q) =
adjacent

Quadrilaterals
Parallelograms
• Area: A = bh
• Perimeter: P = sum of the sides

Rhombus
• Area: A = bh
• Perimeter: P = sum of the sides

© TPR Education IP Holdings, LLC  |  679


SAT MANUAL
READING

Trapezoids
1
• Area: A = h (b1 + b2 )
2
• Perimeter: P = sum of the sides

Rectangles
WRITING AND
LANGUAGE

• Area: A = lw
• Perimeter: P = 2(l + w )

Squares
• Area: A = s
2

• Perimeter: P = 4 s
MATH

Polygons
• Sum of angles in an n-sided polygon: (n − 2 )180

• Angle measure of each angle in a regular n-sided polygon:


(n − 2)180
n

3-D Figures
• Surface area of a rectangular solid: S = 2 (lw + lh + wh )
• Surface area of a cube: S = 6 s 2
• Surface area of a right circular cylinder: S = 2π r 2 + 2π rh
• Surface area of a sphere: S = 4pr 2
• Volume of a cube: V = s 3
• Volume of a rectangular solid: V = lwh
• Volume of a right circular cylinder: V = pr 2h
4π r 3
• Volume of a sphere: V =
3

680  |  © TPR Education IP Holdings, LLC


PLANE GEOMETRY 101

READING
GLOSSARY
Arc:  Any part of the circumference

Area: The size of a surface, or the amount of space inside the boundary of a 2-D
shape
Bisect:  To cut in half

WRITING AND
LANGUAGE
Chord:  Any line segment connecting two points on the edge of a circle

Circumference: The distance around the outside of a circle, or the circle’s perimeter
Circumscribed:  Surrounded by a circle as small as possible
Collinear:  Lying on the same line

MATH
Congruent:  Equal in size
Diagonal (of a polygon):  A line segment connecting non-adjacent vertices

Diameter: The distance of a line that connects two points on the edge of a circle,
passing through the center

Edge (of a 3-D figure): A line segment that joins two vertices on the boundary
or where faces meet
Equilateral triangle:  All sides are equal and each angle measures 60°

Face (of a 3-D figure): Any of the individual surfaces of a solid object
Inscribed:  An angle in a circle with its vertex on the circumference or more
generally, one shape within another so that their boundaries touch but do
not intersect
Isosceles triangle:  A triangle with two equal sides
Parallel:  Two distinct lines that do not intersect

Perimeter: The total distance around the edge of a 2-D figure


Perpendicular:  At a 90° angle
Plane:  A flat surface extending in all directions
Polygon:  A closed figure with three or more sides
Quadrilateral  A four-sided figure

Radius: The distance from the center to any point on the edge of a circle
Regular polygon  A figure with all equal sides and angles
Sector:  Any part of the area formed by two radii and the outside of the circle

© TPR Education IP Holdings, LLC  |  681


SAT MANUAL

Similar:  Equal angles and proportional sides


READING

Surface area:  The sum of areas of each face of a figure


Tangent:  A line that intersects a circle at one point on the circumference, form-
ing a right angle with the radius that extends from the center to that point
Vertex/Vertices:  A corner point. For angles, it’s where two lines meet. For
WRITING AND
LANGUAGE

figures, it’s where two adjacent sides meet.


MATH

682  |  © TPR Education IP Holdings, LLC


PLANE
GEOMETRY

I guess a sock is also a geometric shape—


technically—but I don’t know what you’d call it.
A socktagon?
—Stephen King

REFERENCE INFORMATION

l c 2x 60° x s 45° s 2

r h b
w 30° 45°
b a x 3 s
A = πr 2 A = lw 2
c = a2 + b 2
C = 2 πr A = 1 bh Special Right Triangles
Reference Information

•r h
h r h
h
w r w
l 
4
V = lwh V = π r 2h V = 3 πr3 1 1
V = 3 π r2h V = 3 wh

The number of degrees of arc in a circle is 360.


The number of radians of arc in a circle is 2p.
The sum of the measures in degrees of the angles of a triangle is 180.

683
SAT MANUAL
READING

GOALS REVIEW
At the conclusion of this chapter, you will be able to accomplish the following:

• Use the Geometry Basic Approach on all geometry problems


• Solve questions related to lines and angles within shapes
• Solve questions related to triangles and circles
WRITING AND

• Solve questions related to overlapping shapes


LANGUAGE

• Solve questions related to volume


MATH

684  |  © TPR Education IP Holdings, LLC


PLANE GEOMETRY

READING
GEOMETRY
As with many other questions in the Math sections of the SAT, keeping the
information organized and having a plan of attack will help with the geometry
questions.

GEOMETRY BASIC APPROACH

WRITING AND
LANGUAGE
1. Draw a figure if one is not provided.
2. Label all information from the question on the figure.
3. Write the complete formula on the paper.

MATH
Let’s start by practicing the first step—draw your own figure based on the
following descriptions.

1. An equilateral triangle 2. An isosceles right triangle

3. A rectangular solid 4. A cylinder

5. A sector of a circle 6. A parallelogram

© TPR Education IP Holdings, LLC | 685


SAT MANUAL
READING

Now practice applying the Basic Approach to the following geometry ques-
tion about an isosceles trapezoid.

19. In a given quadrilateral, opposite sides are parallel and equal. If the
height of the quadrilateral were multiplied by 2 and the width of the
quadrilateral were halved, how would the area of the quadrilateral be
WRITING AND
LANGUAGE

affected?
A) The area would be doubled.
B) The area would remain the same.
C) The area would be divided by 2.
D) The area would be divided by 4.
MATH

Apply the first three steps of the Basic Approach to all geometry problems.
However, if you get STUCK…

686  |  © TPR Education IP Holdings, LLC


PLANE GEOMETRY

READING
GEOMETRY BALLPARKING
As long as the diagram doesn’t say, “Note: Figure not drawn to scale.” below it,
you can Ballpark to eliminate incorrect answers. Answers that don’t agree with the
figure cannot be correct.

WRITING AND
LANGUAGE

M a° a° O

L P

MATH
16. What is the value of a in the figure above?
A) 120
B) 110
C) 90
D) 60

© TPR Education IP Holdings, LLC  |  687


SAT MANUAL
READING

LINES AND ANGLES


Many geometry questions about lines will be about parallel or perpendicular lines.
Questions about angles are often about congruent angles, right angles, or two
angles that add up to 90º.
WRITING AND

When parallel lines are intersected by the same line, two kinds
LANGUAGE

of angles are created—BIG and small.


• BIG angles = BIG angles
• small angles = small angles
• BIG + small = 180 degrees
MATH

1

2

Label the information on 7. In the figure above, line 1 is parallel to line 2, x = 120° and y = 110°. What is
the figure. the measure of z, in degrees?

A) 10
B) 30
C) 50
D) 70

688 | © TPR Education IP Holdings, LLC


PLANE GEOMETRY

READING
TRIANGLES
Triangle questions on the SAT will cover some of these same ideas about lines and
angles, and other topics such as trigonometry, similarity, and more.

WRITING AND
LANGUAGE
145° 110°
A B E D

16. In the figure above, points A, B, D, and E lie on the same line and

MATH
BE = CE = CD. What is the measure, in degrees, of ∠DCE ? (Disregard the
degree symbol when gridding your answer.)

© TPR Education IP Holdings, LLC  |  689


SAT MANUAL
READING

SPECIAL RIGHT TRIANGLES


Although there are all sorts of triangles, College Board does like to test particular
ones often, such as 30:60:90 and 45:45:90 triangles. Although the relationships of
the sides in these triangles are in the reference box, you can save time by knowing
them well before the test.
WRITING AND
LANGUAGE

SPECIAL RIGHT TRIANGLES

45°
60° s 2
2x s
x
MATH

30° 45°
s
x 3

W Z 5 Y
60° 60°

17. In ∆WXY above, what is the length of WX ?

A) 2.5

B) 5 2

C) 5 3

D) 10

690 | © TPR Education IP Holdings, LLC


PLANE GEOMETRY

READING
SIMILAR TRIANGLES
Two triangles are similar if the three angles in the first triangle are the same as the
three angles in the second triangle. Similar means “same shape, different size.”

WRITING AND
LANGUAGE
The corresponding sides of two similar triangles are
proportional in length.

MATH
37°

53°
Q S

18. In the figure above, triangle QRS is divided into two smaller
triangles, as shown. Which of the following pairs of sides has the same
ratio as QT : ST ?

ST
A)
RT
ST
B)
RS
RT
C)
ST
RS
D)
RT

© TPR Education IP Holdings, LLC | 691


SAT MANUAL
READING

PYTHAGOREAN THEOREM
You are likely quite familiar with this method of finding the third side of a right
triangle given two other sides. College Board loves to use the Pythagorean triples
such as the 3:4:5, the 6:8:10, and the 5:12:13 triangle, often along with other con-
cepts, so look out for them.
WRITING AND
LANGUAGE

Pythagorean Theorem: a 2 + b 2 = c 2, where c is the hypotenuse

H
MATH

13

G I
5

F J

Look for Pythagorean 35. In the figure above, GI is parallel to FJ and IJ = 13. What is the length
triples!
of FH ?

692 | © TPR Education IP Holdings, LLC


PLANE GEOMETRY

READING
TRIGONOMETRY
The SAT might test you on Trig! Knowing the basic definitions of sine, cosine, and
tangent will be helpful in solving some questions about right triangles.

SOHCAHTOA

WRITING AND
LANGUAGE
sin = cos = tan =

MATH
B

X

A x°
C


W Y

1
8. In the figure above, if sin a = and b = x, what is the value of sin x ?
2
1
A)
2

3
B)
2
3
C)
3

D) 3

© TPR Education IP Holdings, LLC | 693


SAT MANUAL
READING

On the No-Calculator section, College Board may ask about the relationship
between sine and cosine of the complementary angles of a right triangle. Use the
formulas below to solve these questions on the test.
WRITING AND
LANGUAGE

sin x = cos(90° – x) and cos x = sin(90° – x)

E
MATH

m° n°
D F

Note: Figure not drawn to scale.

What strategy can you 21. In triangle DEF shown above, the cosine of ∠D is equal to the sine of
use when the question ∠F, m = 3a + 18, and n = 8a – 5 for some constant a. What is the value
asks for a specific value? of a ?
A) 15.2
B) 7.0
C) 4.6
D) 3.0

694 | © TPR Education IP Holdings, LLC


PLANE GEOMETRY

READING
RADIANS AND DEGREES
Radians and degrees are different ways of measuring angles. Your calculator has
both modes, so make sure to use the right mode when calculating anything.
Sometimes, College Board will ask you to convert degrees to radians or vice versa.

WRITING AND
LANGUAGE
To convert radians to degrees, set up a proportion:
 radians
=
180 degrees

5p
11. If an angle measures radians, what is the measure of the angle, in
8

MATH
degrees?

A) 112.5
B) 125
C) 247.5
D) 288

© TPR Education IP Holdings, LLC | 695


SAT MANUAL
READING

CIRCLES
Central angles of circles may also be measured in radians. Aside from basic circle
questions about circumference and area, College Board may ask about a slice of a
circle (a sector) or a part of the circumference (an arc) formed by the central angle.

W
WRITING AND

X
LANGUAGE

O
MATH

Note: Figure not drawn to scale.

To find an arc length of a 10. The figure above shows a circle with center O. The central angle described by
circle in radians, use
s = rθ, where s is the arc  measures π radians. If the circle has a radius of 5, what is the length of
WXY
length, r is the 4
radius, and θ is the 
WXY ?
central angle.

A) 3π
4
B) p

C) 5π
4
D) 3π
2

Arcs and sectors are proportional to the central angle of a circle.

part central angle arc length sector area


= = =
whole 360° 2r r 2

696 | © TPR Education IP Holdings, LLC


PLANE GEOMETRY

READING
OVERLAPPING SHAPES
When given one shape inside another, use the information given about the first
shape to determine the necessary information about the second shape.

WRITING AND
LANGUAGE
R T

6
O

MATH
35. In the circle above with center O, the area of the sector formed by
RO, TO, and minor arc RT  is 10p. Line segments RS and ST intersect

at point S as shown and are tangent to the circle at points R and T,


respectively. If the length of RO = 6, what is the measurement of angle
RST ?

© TPR Education IP Holdings, LLC  |  697


SAT MANUAL

P
READING

Q U

R T
S
WRITING AND
LANGUAGE

A three-sided figure has 30. The figure above shows a regular hexagon inscribed within a larger
180° and a four-sided regular hexagon, such that each vertex of the smaller inscribed hexagon
figure has 360°. For is the midpoint of a side of the large hexagon. Each side of the larger
each additional side, add external hexagon is 8 inches. What is the combined area of the 6 triangles
another 180°.
formed in the figure above, rounded to the nearest tenth?
A) 6.9
B) 41.6
MATH

C) 83.1
D) 166.3

VOLUME
Volume measures shapes in three dimensions rather than two dimensions, but the
Geometry Basic Approach still applies. Try using it on these volume questions.

10. A solid cube made of gold with a volume of 64 cm3 was melted and
recast into a solid cylinder with a radius of 4 cm. If there was no loss or
addition of material during the process, which of the following is closest
to the height of the resulting cylinder?
A) 1.27 cm
B) 2.0 cm
C) 2.54 cm
D) 4.0 cm

Plug in when given 34. A child owns a set of building blocks shaped as rectangular pyramids,
a range of possible each with a base width of 5 inches and a base length of 5 inches. If the
values instead of a height of the pyramids varies between 5.5 inches and 5.75 inches, what
specific one. is one possible volume, rounded to the nearest cubic inch, of a building
block in this set?

698  |  © TPR Education IP Holdings, LLC


PLANE GEOMETRY

READING
MEANING IN CONTEXT FOR GEOMETRY
Some geometry questions will ask for what some part of a formula means in con-
text. Use a modified form of the meaning-in-context approach for these questions.

MEANING IN CONTEXT GEOMETRY APPROACH

WRITING AND
LANGUAGE
1. Read the Final Question
2. Label the Formula or Figure
3. Use POE
4. Plug-and-Play
5. Guess and Go

MATH
1
w
2
w
2w

1 
SA  2  w 2  2w 2  w 2 
 2 
25. For the rectangular box shown above, where w is the width of the base of the
box, the given formula can be used to determine the total surface area of the
box. What must the expression 4w2 represent?

A) The area of the base of the box


B) The sum of the areas of the top and base of the box
C) The sum of the areas of the front and back of the box
D) The sum of the areas of all four vertical faces of the box

© TPR Education IP Holdings, LLC | 699


SAT MANUAL
READING

PLANE GEOMETRY DRILL


Time: 8 minutes

Unless you are aiming for a top score, don’t try all the questions! Use your POOD to choose the best ones for you.
WRITING AND
LANGUAGE

8 10
The volume of a cylinder is 54π cm3. If the height of
A E H the cylinder is twice the radius, what is the height?
B
A) 3 cm
B) 6 cm
MATH

C) 9 cm

I D) 12 cm

D
G F
C

Note: Figure not drawn to scale.

In the figure above, AB || CD. Which of the


following must be true?
A) ÐAEI = ÐBHI
B) DEIH ~ DFIG
C) EF ^ GH
D) ÐEIG = ÐEIH

700  |  © TPR Education IP Holdings, LLC


PLANE GEOMETRY

13 27

READING
In a circle with center P (not shown), points Q, R,
and S lie on the circumference. If PR = 7 and
ÐQPS = 45°, then what is the area of the minor A B
sector PQS ? D

49 π

WRITING AND
LANGUAGE
A)
8

B) 49 π C
4
Note: Figure not drawn to scale.
C) 14p
In the circle with center D shown above, DABC is
D) 49p equilateral and is inscribed in the circle. If AD = x,

MATH
then which of the following represents the length of
 , in terms of x ?
BC
πx
A)
6
2 πx
B)
20 3
While flying a kite with a string of length 90 feet,
πx2
Lauren found that the wind kept her kite in the air C)
6
at an average angle measure of 60° relative to the
ground, but she was dissatisfied with how high the 2π x 2
kite flew. She tried a different kite with a string of D)
3
length 120 feet, and the wind continued to keep
her new kite in the air at an average angle measure
of 60° relative to the ground. How much higher (in
vertical feet) did Lauren’s second kite fly than her
first kite flew?
A) 15.00
B) 25.98
C) 77.94
D) 103.92

© TPR Education IP Holdings, LLC  |  701


SAT MANUAL

32
READING

B
C
y° 100°


WRITING AND
LANGUAGE

A D

Note: Figure not drawn to scale.

In parallelogram ABCD above, x = 40° and


ÐDBC = 100°. What is the value of 2y ?
MATH

702  |  © TPR Education IP Holdings, LLC


PLANE GEOMETRY

Summary
• What three steps should you follow for all
geometry questions?

______________________________________

______________________________________

______________________________________

• What can you do if you get stuck on a


geometry question?

______________________________________

• What two kinds of angles are created when


two parallel lines are both intersected
by another line, and how are those angles
related?

______________________________________

______________________________________

• What two special right triangles appear


often on the SAT?

______________________________________

• What is relationship of the sides of those


special right triangles?

______________________________________

______________________________________

• In what situation can two triangles be


described as similar?

______________________________________

• What is true of similar triangles?

______________________________________

• What are the three Pythagorean triples


that are commonly tested on the SAT?

______________________________________

______________________________________

______________________________________

© TPR Education IP Holdings, LLC | 703


SAT MANUAL

• What are the three key definitions in the


mnemonic SOHCAHTOA?

______________________________________

______________________________________

______________________________________

• What is the formula that describes the


relationship between the sine and cosine of
complementary angles in a right triangle?

______________________________________

• What is the formula you need to con-


vert back and forth between radians and
degrees?

______________________________________

• What formulas can you use to express the


proportional relationship between the
various aspects of a circle?

______________________________________

• What can you do when an SAT question


gives you overlapping or 3-D shapes?

______________________________________

• What are the five steps to determining the


meaning of expressions in the context of
geometry questions?

______________________________________

______________________________________

______________________________________

______________________________________

______________________________________

• I have accomplished _________ of the 5


goals stated at the beginning of this chapter.

704 | © TPR Education IP Holdings, LLC


PLANE
GEOMETRY
PRACTICE

REFERENCE INFORMATION

l c 2x 60° x s 45° s 2

r h b
w 30° 45°
b a x 3 s
A = πr 2 A = lw c 2= a 2 + b 2
C = 2 πr A = 1 bh Special Right Triangles
Reference Information

•r h
h r h
h
w r w
l 
4
V = lwh V = π r 2h V = 3 πr3 1 1
V = 3 π r2h V = 3 wh

The number of degrees of arc in a circle is 360.


The number of radians of arc in a circle is 2π.
The sum of the measures in degrees of the angles of a triangle is 180.

705
SAT MANUAL

LINES AND ANGLES


READING

6 12
G F
C
A I J B
WRITING AND
LANGUAGE

B D
K

C L M D
75°
A E
F
E H
MATH

In the figure above, ÐCAE measures 75°, and AE is In the figure above, if AB || CD, then which of the
parallel to BD. What is the measure of ÐCDF ? following must be true?
A) 15° A) ÐAIG @ ÐBJF
B) 75° B) ÐCLK + ÐELM = 180°
C) 90° C) DIJK ~ DMLK
D) 105° D) EF ^ GH

14


45° 40°
In the figure above, what is x + y + z ?
A) 95°
B) 265°
C) 290°
D) 360°

CONTINUE
706  |  © TPR Education IP Holdings, LLC
PLANE GEOMETRY PRACTICE

SIMILAR TRIANGLES

READING
9 19
9 C
D E
H

WRITING AND
LANGUAGE

3
8
C

2 A 4 B G I

A B Note: Figure not drawn to scale.


3

MATH
In the figure above, AB || DE. What is the length of In ∆ABC above, AB = 4, BC = 3, and ∠BAC = m.
AE ? In ∆GHI above, HI = 8 and ∠GHI = m. What is the
length of GH ?
A) 4
B) 6
C) 8
D) 10

15
H


B
e° 14 21
2 3

d° f° d° f°
A C G I

If the length of AC is an integer, then what is one


possible value of GI ?
A) 7
B) 15
C) 28
D) 35

CONTINUE
© TPR Education IP Holdings, LLC  |  707
SAT MANUAL

PYTHAGOREAN THEOREM TRIGONOMETRY


READING

2 5
A right triangle (not shown) has legs of lengths 7
and 8. Which of the following is the length of the C
hypotenuse?
WRITING AND
LANGUAGE

A) 15

B) 9

C) 113

D) 15 26 cm
MATH

4
A B
DABC (not shown) is a right triangle with side 10 cm
lengths that are consecutive integers. What are the
lengths of the two legs? In the figure above, what is the measure of sin A ?
A) 2, 3
B) 3, 4 5
A)
C) 3, 5 13

D) 4, 5 12
B)
13
5
C)
12
13
D)
12

CONTINUE
708  |  © TPR Education IP Holdings, LLC
PLANE GEOMETRY PRACTICE

12 17

READING

L E

WRITING AND
LANGUAGE
M N
Note: Figure not drawn to scale.

MATH
In the figure above, if MN = 6, and tan ÐMLN =
12 D F
, what is the length of LN ? 12 cm
5
3
A) 6.5 In DDEF above, the measure of sin E is . What is
5
the length of side EF ?
B) 12
C) 13
D) 26

CONTINUE
© TPR Education IP Holdings, LLC  |  709
SAT MANUAL

RADIANS AND DEGREES


READING

9 16
If cos (60°) = x, which of the following is also
π
equivalent to x ? An angle measures radians. What is the measure
2
A) sin (30°)
WRITING AND
LANGUAGE

of the angle in degrees?


B) sin (60°)
C) sin (120°)
D) sin (180°)
MATH

15


Which of the following is equal to sin   ?
7
 5 
A) sin  
 14 

 5 
B) cos  
 14 


C)  sin  
7


D)  cos  
7

CONTINUE
710  |  © TPR Education IP Holdings, LLC
PLANE GEOMETRY PRACTICE

CIRCLES

READING
9 15

A
A

WRITING AND
LANGUAGE
B

B
O O

MATH
In the circle above, O is the center and the
circumference is x. What is the length of minor arc If the circle above has area x, ÐAOB is 60°, and O is
 , in terms of x ?
AB the center of the circle, what is the length of minor
 , in terms of x ?
arc AB
x
A)
3
A) πx
x 3
B)
4
B) πx
x 2
C)
5
C) πx
x
D)
6
D) 2 πx

CONTINUE
© TPR Education IP Holdings, LLC  |  711
SAT MANUAL

OVERLAPPING SHAPES
READING

8 10
If a square has an area of x, then, in terms of x, what
B is the circumference of the largest circle that can be
inscribed in the square?
WRITING AND
LANGUAGE

A) p x

O B) π x
2
C) px2
C
D)
πx
MATH

4
In the figure above, ÐBOC is 120° and the radius of
the circle with center O is 4. What is the length of
BC ?

A) 2 3

B) 3 3

C) 4 3

D) 8

CONTINUE
712  |  © TPR Education IP Holdings, LLC
PLANE GEOMETRY PRACTICE

VOLUME

READING
11 14

A rectangular solid has a width of x inches, a length
of 2x inches, and a height of x2 inches. If a piece
with a volume of 4 cubic inches is removed from the

WRITING AND
LANGUAGE
solid, what is the resulting volume of the solid, in
10 terms of x ?
A) x(3 + x) – 4
B) 3x2 – 4
6 C) 4(x2 – 1)
D) 2(x4 – 2)

MATH
In the cone shown above, the radius of the base is 6
and the slant height is 10. What is the volume of the
15
cone?
A) 20π Three spherical balls with radius r are contained
in a rectangular box. Two of the balls are each
B) 96π touching 5 sides of the rectangular box and the
C) 120π middle ball. The middle ball also touches four sides
D) 200π of the rectangular box. What is the volume of the
space between the balls and the rectangular box?
A) r3(3 – 4p)
B) 4r2(14 – p)
C) 4r3(6 – p)
D) 12r2(r – p)

CONTINUE
© TPR Education IP Holdings, LLC  |  713
SAT MANUAL

LINES AND ANGLES


READING

4 33

C D

115° m
WRITING AND

40°
LANGUAGE


l 70°
A B
Note: Figure not drawn to scale.
In the figure above, if line l is parallel to line m,
what is the value of x ?
In the figure above, AB @ AC and AB || CD. What is
MATH

A) 115°
the value of x ? (Disregard the degree symbol when
B) 75° gridding your answer.)
C) 65°
D) 40°

24

G I F

K
A x x B
y

C D

E J
H

In the figure above, AB || CD and IJ bisects ÐEKH.


Which of the following is equal to 4y, in terms of
x?
A) 360 – 4x
B) 180 – 3x
C) 360 + 4x
D) 180 + 2x

CONTINUE
714  |  © TPR Education IP Holdings, LLC
PLANE GEOMETRY PRACTICE

SIMILAR TRIANGLES

READING
9 36
Y A

B

WRITING AND
LANGUAGE
b° C 13

a° b°
X Z
A
5
D E
Based on the two triangles shown above, which of

MATH
the following statements MUST be true?
A) BC = XZ
AB XY
B) =
BC YZ
B C
C) DABC @ DXZY
AB YZ In the figure above, DE || BC, the length of DE = 5,
D) = and the length of AE = 13. If DE bisects AB, what is
AC XY
the perimeter of DABC ?

18

20

5 5 5

Samuel is building a slide as shown above. If the


supports are 5 meters apart and the slide is 20
meters long, approximately how tall, in meters, is
the first support?
A) 4.4
B) 5.0
C) 6.7
D) 13.2

CONTINUE
© TPR Education IP Holdings, LLC  |  715
SAT MANUAL

PYTHAGOREAN THEOREM
READING

1 14
In a right triangle, the two shorter sides are 7 cm B
and 11 cm long. What is the length of the longest
side, in cm?
WRITING AND

26 C
LANGUAGE

A) 18 10

B) 72

C) 77 A
D
D) 170
1
In the figure above, CD = AD. What is the
MATH

3
perimeter of quadrilateral ABCD ?
A) 32
B) 64
C) 74
D) 144

CONTINUE
716  |  © TPR Education IP Holdings, LLC
PLANE GEOMETRY PRACTICE

TRIGONOMETRY

READING
7 20

WRITING AND
LANGUAGE
C B

1
In DABC above, the measure of cos A is . What is
2

MATH
the measure of sin A ?
1
A)
2
3
B)
2
3
C)
3
60°
D) 2
6m

17
A surveyor wants to calculate the shortest distance
across a river. She begins directly across the river
from a tree on the opposite riverbank, then walks
6 meters along the riverbank on her side of the
river. From there, she measures a 60° angle between
43 feet her starting point and the tree. What is the distance
from the tree to the point at which the surveyor
started, to the nearest tenth of a meter?
A) 5.2
60° B) 8.5
C) 10.4
The figure above shows a ladder leaned against a D) 12
wall so that the top of the ladder touches the top of
the wall. The ladder is placed so that the base is at
a 60° angle to the ground. If the wall is 43 feet tall,
what is the length of the ladder, to the nearest foot?
A) 25
B) 50
C) 75
D) 85

CONTINUE
© TPR Education IP Holdings, LLC | 717
SAT MANUAL

RADIANS AND DEGREES


READING

6 21

y
WRITING AND
LANGUAGE

a° b°
X Y Z

x
MATH

In the figure above, a + b = 180. What is the


measure, in radians, of ÐXYZ ?
In the figure above, the measure of angle y is equal
A) 2π
to
π radians. Which of the following is equal to
3π 6
B)
2 sin x ?

C) π 1
A)
π 2
D)
2 B) 1

C) 2
3
D)
2
17
An angle measures 1 radian. What is the measure
of the angle in degrees?

A) 180
π
360
B)
π

C) 180

D) 360

CONTINUE
718  |  © TPR Education IP Holdings, LLC
PLANE GEOMETRY PRACTICE

CIRCLES

READING
9 36
A
B D
A

WRITING AND
LANGUAGE
y
O O

C
B
Note: Figure not drawn to scale.
In the figure above, AB and CD pass through the

MATH
5π center of the circle. If the total area is 144π and the
In the circle above, y = and the length of AO is 3. shaded area is 66π, what is the measure of ÐBOC,
6
If y is the radian measure of a central angle, what is in degrees? (Disregard the degree symbol when
? gridding your answer.)
the length of minor arc AB

A) 5π
18

B)
2

C) 8π
3
15π
D)
4

21

D
In the figure above, O is the center of the circle, the
 is 3π, and ÐCOD is 45°.
length of minor arc CD
What is the area of the shaded region?
A) 9π
B) 18π
C) 27π
D) 36π
CONTINUE
© TPR Education IP Holdings, LLC  |  719
SAT MANUAL

OVERLAPPING SHAPES
READING

22 24
A cylinder (not shown) has circular faces with
circumferences of 8π. The height of the cylinder is
equal to the radius of its circular faces. What is the P
WRITING AND
LANGUAGE

volume of the cylinder?


A) 8π O 60°
B) 16π
C) 32π Q

D) 64π

If the area of the circle above is 9π and OP and OQ


MATH

are both radii, what is the area of the shaded region?

A) 9π − 6 3

B) 6π − 9 3

6π − 9 3
C)
4
9π − 6 3
D)
4

CONTINUE
720  |  © TPR Education IP Holdings, LLC
PLANE GEOMETRY PRACTICE

VOLUME

READING
27 28

WRITING AND
LANGUAGE
The pyramid above has a regular hexagonal base. If

MATH
the lengths of the edges of the hexagonal base are In the figure above, a cube is inscribed in a sphere
equal to 3, and the height of the pyramid is 4, what with a volume of 36π. What is the length of one
is the volume of the pyramid? edge of the cube?

(Note: The volume of a pyramid is given by the 2


1 A)
equation V = Bh, where B is the area of the base 2
3
and h is the height as measured from the center of B) 2 3
the base to the apex.)
C) 4 3

45 3
A) D) 12 3
2
B) 18 3

27 3
C)
2
D) 5

CONTINUE
© TPR Education IP Holdings, LLC  |  721
SAT MANUAL

29
READING
WRITING AND
LANGUAGE

1 2

The figure above shows a semi-cylindrical plastic


cover used to hide an electrical cord run along
MATH

a wall. The plastic cover has a groove cut out of


its base in the shape of a rectangular solid. The
diameter of the semi-cylindrical plastic cover is 8
cm, while the groove is 1 cm high and 2 cm wide.
The cover and the groove cut out of it are both 10
cm in length. The plastic has a density of 1.25 grams
per cubic centimeter. What is the mass of the plastic
cover, with the groove cut out, to the nearest gram?
(Density is mass divided by volume.)
A) 289
B) 315
C) 482
D) 603

Answers can be found on page 746.

722 | © TPR Education IP Holdings, LLC


ANSWER
KEY

723
SAT MANUAL

READING

Passage 4: Nonviolence
Basic Approach Practice 32. B
33. A
34. B
Passage 1: Ethan Frome 35. C
1. C 36. D
2. B 37. B
3. A 38. D
4. D 39. C
5. B 40. A
6. A 41. C
7. B
8. C
9. D Passage 5: Ocean Plastic
10. C 42. A
43. C
44. B
Passage 2: Monuments Men 45. D
11. A 46. A
12. C 47. B
13. B 48. C
14. A 49. D
15. D 50. D
16. D 51. B
17. A 52. B
18. C
19. B
20. B
21. C Mastering POE Practice

Passage 3: Songbirds Passage 1: Ms. Ice Sandwich


22. B 1. B
23. B 2. A
24. A 3. D
25. C 4. A
26. D 5. C
27. D 6. B
28. A 7. D
29. B 8. C
30. C 9. A
31. B 10. D

724  |  © TPR Education IP Holdings, LLC


ANSWER KEY

Passage 2: Susan B. Anthony 48. B


11. C 49. A
12. A 50. C
13. D 51. C
14. B 52. B
15. A
16. C
17. B
18. D Identifying Question Types
19. A
20. A
Practice
21. D
VIC Questions
1. B
Passage 3: Secret Lives of the Brain 2. D
22. C 3. A
23. C 4. B
24. D 5. C
25. D 6. C
26. A
27. A
28. C Paired Sets
29. C 42. A
30. B 43. B
31. D 44. C
45. D
46. C
Passage 4: Wilson/Lodge Dual 47. B
Passage 48. A
49. B
32. A 50. C
33. B 51. A
34. A 52. D
35. B
36. A
37. B Passage 2: The Outsiders
38. C 1. A
39. B 2. A
40. D 3. C
41. C 4. B
5. B
6. D
Passage 5: What Makes a Pun Funny 7. B
43. B 8. C
44. D 9. D
45. D 10. C
46. B
47. A

© TPR Education IP Holdings, LLC  |  725


SAT MANUAL

Passage 3: Tsunamis 11. D


22. C 12. B
23. A 13. A
24. C 14. C
25. B 15. B
26. D 16. C
27. B 17. D
28. A 18. A
29. B 19. D
30. D 20. D
31. D 21. A
22. C
23. B
Passage 4: Spanish Flu 24. C
12. A
13. A
14. D Verbs
15. D 1. A
16. A 2. C
17. B 3. B
18. C 4. B
19. C 5. A
20. D 6. B
21. C 7. C
22. A 8. D
9. B
10. B
11. D
12. C
WRITING AND LANGUAGE 13. D
14. B
15. C
16. A
Words Part I Practice 17. B
18. D
19. D
Transitions 20. B
1. D 21. C
2. A 22. D
3. B 23. D
4. D 24. D
5. C
6. A
7. C
8. C
9. B
10. A

726  |  © TPR Education IP Holdings, LLC


ANSWER KEY

Pronouns 21. D
1. C 22. B
2. D 23. A
3. C 24. C
4. B
5. A
6. C Concision
7. D 1. D
8. D 2. B
9. C 3. D
10. C 4. D
11. B 5. B
12. A 6. C
13. D 7. B
14. A 8. A
15. C 9. D
16. D 10. A
17. B 11. B
18. C 12. D
19. A 13. A
20. B 14. D
21. A 15. B
22. C 16. B
23. B 17. D
24. C 18. B
19. C
20. D
Precision 21. C
1. A 22. D
2. B 23. D
3. B 24. D
4. C
5. C
6. D
7. D Questions Practice
8. B
9. C
10. A Purpose
11. B 1. D
12. B 2. B
13. C 3. D
14. A 4. B
15. C 5. C
16. D 6. A
17. A 7. A
18. A 8. D
19. C 9. C
20. B 10. A

© TPR Education IP Holdings, LLC  |  727


SAT MANUAL

11. C Order
12. A 1. C
13. D 2. D
14. D 3. D
15. B 4. C
16. A 5. D
17. C 6. C
18. B 7. D
19. A 8. C
20. C 9. D
21. B 10. B
22. B 11. A
23. A 12. B
24. D

Combining Sentences
Words in Disguise 1. C
1. C 2. C
2. A 3. A
3. A 4. D
4. B 5. B
5. C 6. A
6. A 7. A
7. B 8. B
8. D 9. D
9. A 10. C
10. B 11. B
11. B 12. D
12. A

Charts and Graphs


Adding and Deleting 1. A
1. A 2. D
2. C 3. C
3. D 4. B
4. A 5. D
5. A 6. C
6. B 7. C
7. C 8. A
8. D 9. A
9. C 10. C
10. D 11. D
11. B 12. B
12. C

728  |  © TPR Education IP Holdings, LLC


ANSWER KEY

Punctuation Practice Apostrophes


1. D
2. D
Stop, Go, and Half-Stop 3. C
1. A 4. B
2. D 5. A
3. D 6. D
4. A 7. C
5. C 8. B
6. B 9. A
7. C 10. C
8. D 11. D
9. A 12. B
10. B 13. B
11. A 14. C
12. C 15. A
13. A 16. D
14. C 17. B
15. A 18. A
16. A 19. B
17. D 20. D
18. C 21. C
19. D 22. B
20. A 23. C
21. D 24. A
22. C
23. B
24. D
Words Part II Practice
Commas
1. A, Rule #3 Complete Sentences
2. C, Rule #4 1. D
3. D 2. C
4. A, Rule #4 3. A
5. C 4. D
6. B, Rule #3 5. C
7. C 6. B
8. B, Rule #1 7. C
9. D, Rule #4 8. A
10. D 9. B
11. B, Rule #1 10. D
12. D, Rule #3 11. B
13. A, Rule #4 12. D
14. D, Rule #4 13. B
15. B 14. A
16. A, Rule #4 15. C
16. D

© TPR Education IP Holdings, LLC  |  729


SAT MANUAL

17. A Parallelism
18. A 1. C
19. B 2. C
20. D 3. B
21. C 4. C
22. B 5. A
23. C 6. B
24. A 7. D
8. B
9. D
Idioms, Homophones, and Slang 10. D
1. C 11. D
2. C 12. A
3. D 13. D
4. D 14. C
5. B 15. B
6. C 16. D
7. A 17. C
8. B 18. D
9. B 19. B
10. C 20. D
11. D 21. D
12. C 22. A
13. A 23. C
14. B 24. A
15. B
16. B
17. C Modifiers
18. B 1. C
19. A 2. A
20. C 3. D
21. D 4. B
22. D 5. B
23. A 6. C
24. C 7. B
8. C
9. C
10. D
11. A
12. A

730  |  © TPR Education IP Holdings, LLC


ANSWER KEY

Grammar 101 9. building could save

10. people are


Transitions
Which one is correct: 11. jury must decide
page 284: #2
page 286: #1
Pronouns
1. takes
What comes next? 2. take
1. contradiction 3. thinks
2. example
3. conclusion
4. additional point Pronoun Agreement
5. additional point 1. they: writers, plural
6. additional point 2. it: College Board, singular
7. additional point or example
8. contradiction
9. conclusion Relative Pronouns
10. contradiction Which one, page 292: #1
11. contradiction, additional point,
or conclusion
12. conclusion Verb Tense
13. additional point or example 1. am
14. contradiction 2. will take
15. conclusion 3. have read
4. had taken
5. sent
Prepositions
1. is: one, singular
2. have: students, plural

Subject/Verb Drill
1. name is

2. dude abides

3. you build, he will come

4. I thought, I was, they pull

5. stuff is, dreams are

6. liver pairs

7. children make

8. they are storming

© TPR Education IP Holdings, LLC  |  731


SAT MANUAL

Full-Length Practice Passages


Passage 1: What Actually is an Actuary? Passage 4: The Science of Self-Talk
1. C 34. B
2. B 35. C
3. D 36. D
4. D 37. B
5. B 38. D
6. D 39. C
7. A 40. D
8. C 41. C
9. D 42. B
10. A 43. A
11. D 44. C

Passage 2: Bag Ban Debate Passage 5: Forensic Anthropology


12. B 1. A
13. D 2. D
14. A 3. D
15. C 4. D
16. A 5. A
17. B 6. C
18. D 7. B
19. C 8. C
20. A 9. B
21. D 10. B
22. C 11. C

Passage 3: The Rise of Urban Art Passage 6: A Whale of a Tale


23. B 12. C
24. C 13. B
25. A 14. B
26. B 15. D
27. A 16. B
28. C 17. A
29. D 18. C
30. D 19. D
31. D 20. A
32. C 21. C
33. A 22. D

732  |  © TPR Education IP Holdings, LLC


ANSWER KEY

Passage 7: The Compassionism Model Passage 10: The Work of Upton Sinclair
23. D 12. D
24. B 13. A
25. C 14. B
26. D 15. C
27. C 16. D
28. A 17. B
29. B 18. C
30. B 19. B
31. A 20. B
32. D 21. B
33. B 22. D

Passage 8: A Plethora of Senses Passage 11: The Pros and Cons of Urban Farming
34. B 23. C
35. C 24. D
36. D 25. B
37. B 26. D
38. C 27. B
39. B 28. A
40. C 29. C
41. A 30. B
42. D 31. D
43. B 32. C
44. C 33. A

Passage 9: The Ethics of Photojournalism Passage 12: The Final Mosquito Solution
1. B 34. B
2. C 35. C
3. D 36. C
4. C 37. D
5. D 38. D
6. C 39. A
7. B 40. C
8. B 41. A
9. A 42. C
10. A 43. A
11. C 44. D

© TPR Education IP Holdings, LLC  |  733


SAT MANUAL

Passage 13: Transportation Engineers Passage 16: There May Not Be Life on Mars,
1. D But at Least There’s Gas
2. C 34. D
3. B 35. C
4. D 36. B
5. A 37. D
6. C 38. B
7. A 39. C
8. D 40. A
9. B 41. A
10. D 42. B
11. B 43. C
44. C
Passage 14: Behind Every Good Movement…
12. A Passage 17: Physical Therapy
13. D 1. C
14. B 2. D
15. D 3. C
16. D 4. C
17. C 5. D
18. C 6. D
19. B 7. B
20. C 8. B
21. B 9. C
22. D 10. B
11. A
Passage 15: America’s Love Affair with Cars
23. A Passage 18: The Legacy of Watergate
24. D 12. B
25. B 13. C
26. B 14. D
27. C 15. C
28. D 16. D
29. C 17. B
30. A 18. C
31. C 19. A
32. D 20. B
33. B 21. B
22. D

734  |  © TPR Education IP Holdings, LLC


ANSWER KEY

Passage 19: The Disappearing Bee Calculator


23. D 1. C
24. D 4. A
25. B 6. D
26. C 7. D
27. C 12. B
28. B 14. C
29. D 17. B
30. D 19. A
31. C 20. C
32. C 22. C
33. D 24. A
26. B
Passage 20: Do You See What I See? 27. C
34. C 28. B
35. D 29. A
36. B 32. 6
37. A
38. C
39. A
40. C Plugging In the Answers
41. C
42. A
Practice
43. B
44. A No Calculator
2. A
4. C
8. C
10. C
MATH 13. D
15. C

Plugging In Practice Calculator


3. B
6. C
No Calculator 8. C
3. C 13. B
5. A 15. C
6. C 18. A
9. D 21. B
10. A 27. C
11. C
13. B
1
19. or 0.25
4

© TPR Education IP Holdings, LLC  |  735


SAT MANUAL

Translating and Meaning in Absolute Value: The distance from zero on the
number line
Context Practice
Rational Numbers (And Their Irrational
No Calculator Friends)
Translating
1
8. A 1.
10. A 2
3
17. 12 2.
1
Meaning in Context 3. 3
8. D 4. 57
5. No
Calculator 6. No
Translating 3
7.
7. C 8
21. B 1
8.
31. 5 6
34. 19 9. 3.14159…
22. C 10. .1575721
27. D
Manipulating Equations
Linear Equations
1. 2
Solving 101 2. 32
Order of Operations
8
Sum: The result of addition 3.
3
Difference: The result of subtraction 4. 7
Product: The result of multiplication 5. −6
6. 28
Dividend: The result of division 7. 9

The Order of Operations: Parenthesis; Exponents; Inequalities


Multiplication and Division; Addition and 1. x > −10
Subtraction 2. x < 6
Famous Saying: Please Excuse My Dear Aunt Sally 3. x > 11

System of Equations
Absolute Value
5
Absolute Value makes a number positive if it is 1. x = 10, y =
negative 3
2. x = −3, y = 5
1. 8 3. x = 6, y = 5
2. −3
3. 1
4. 3

736  |  © TPR Education IP Holdings, LLC


ANSWER KEY

Solving (Linear) Practice 5. −1, 6


6. −12, 2
7. −7, 2
No Calculator 8. −3
Solving Equations 9. −2, 2
3. D 10. 4
9. A

Solving Inequalities Exponents


20. 0, 1, 2, 3, or 4 Basic Rules
1. x5
Solving Rational Expressions 2. x4
2. B 3. x8
15. D 4. 12x 2y3
17. 1 5. 8x6
6. 2x
7. 8x 2
Calculator 8. 2x 2 + 4y3
Solving Equations
3. C Special Rules
13. A 1. 1
34. 2 2. 1
1
Solving Inequalities 3. x−2 or
6. B x2
1
28. D 4.
33. 1 x2
5. x
Solving Rational Expressions 6. 1
10. A 7. 0
8. 4
3
35. or .6 9. −8
5
1
Simultaneous Equations 10.
5. D 4
21. B 11. 1
34. 5.5 12. take the reciprocal
13. that number
14. 1
Solving 201 15. 0
Fractional Exponents 16. positive
17. negative
i. 9 3 18. smaller

Exponents Practice
Quadratics 1. 53 = 125
1. −2, −1
2. 1, 5 2. x5
3. −4, 2 3. 81x12y24z20
4. −2, 5
© TPR Education IP Holdings, LLC  |  737
SAT MANUAL

4. 125 – 27 = 98 x
5.
5. 3x y 2 2 y
6. 3x6 + 2x5 6. 4
7. 5 × 6 = 5 × 36 = 180
2
11
7.
8. 73 = 343 13
5x 2 8. c
9. 5x 2y−2 or
y2 9. 3xy
10. abcdefg 6
10. −
Fractional Exponents Practice 7
1. 8d2
2. 9
3. 4 Solving (Nonlinear) Practice
4. 1
5. 4xy3
6. 2 No Calculator
1 Solving Quadratics
7.
x5 16. 2
19. 10
8. 125 or 5 5
Exponents and Roots
x6
9. 10 5. C
9y
3
10. 36x6y4 18. or 1.5
2

Roots Calculator
Rationalizing the Denominator Solving Quadratics
ii. 2 3 32. 4
9
Combining Roots 35. or .9
10
1. 6 x
Solving Rational Expressions
2. 6 x 23. D
3. xy
Exponents and Roots
x
4. 15. B
y
5. 48 Growth and Decay
6. 2 6. B
7. 147 15. B
8. 2 23. B
26. A
Roots Practice
1. 10b
2. 5
3. 4
4. 31

738  |  © TPR Education IP Holdings, LLC


ANSWER KEY

Advanced Solving Practice 6. 6


7. 6
8. 60
No Calculator 9. 3
Simultaneous Equations
14. A Line Graphs
1. Time (days)
Complex Numbers 2. Average (square inches)
14. D 3. 21
4. 11
5. 30
Calculator
Complex Numbers
10. C
21. D Charts and Data Practice
Scatterplot Graphs
18. C
Charts and Data 101 19. C
Scatterplots 20. D
1. Protein (grams) 20. C
2. Fiber (grams) 21. D
3. 5 22. A
4. 9
5. 6.5 Other Line Graphs
6. 4 7. D
22. C
Bar Graphs and Histograms
1. Year Bar Graphs
2. % Unemployment 6. D
3. 3.9% 7. A
4. 2010 8. B
5. 9.2%
Two-Way Tables
Two-Way Tables
11. D
1. Preferred beverage
12. C
2. Gender
13. B
3. 2,476
5. A
4. 616
6. D
5. 1,210
7. D
6. 89
7. 655
Translating Figures
Frequency Tables 8. B
1. 20 16. C
2. 0, 0, 2, 2, 2, 2, 2, 3, 3, 3, 3, 3, 3, 3, 4, 24. C
4, 5, 5, 5, 6
3. 3 Justifying Conclusions
4. 15 3. A
5. 3 17. D
21. D

© TPR Education IP Holdings, LLC  |  739


SAT MANUAL

Data Collection Methods Retail Therapy


7. B 1. i. $75
23. B ii. $25
iii. multiply by 25%, then subtract; or
multiply by 75%
Discounts: multiply, then subtract
Word Problems 101 Delivery fees: multiply, then add
Parts & Wholes Sales taxes: multiply, then add
1. C Income taxes: multiply, then subtract
2. C Interest: multiply, then add
3. C 11. D
4. A
Ratios
Mixed Numbers 1. i. 3:6 or 1:2
16 ii. 7:2
1.
3 iii. 9
6 2
iv. or
31 9 3
2.
7 2. i. 30
191
ii. 18
3. 18 3
40 iii. or
48 8
136 3
4. iv.
55 8

13. D
263
5.
72

115 Word Problems Practice


6. Mean, Median, Mode, and Range
24
13. A
7. B 14. B
15. B
Percents 23. D
of: multiply 27. D
is: equals 29. C
what percent: divide by 100 31. 1800

2. B Probability
15. D 19. B
28. C

Rates and Unit Conversion


7. B
18. C
21. B
30. D
5
32. , .138, or .139
36

740  |  © TPR Education IP Holdings, LLC


ANSWER KEY

Proportions and Variation Functions IRL


6. D 8. D
14. B 18. A
29. B
Percents
2. D Functions and Graphs
5. A 7. A
22. A 10. A
28. C 20. B

Extended Thinking Compound Functions


37. 34 17. D
23. A
5
38. or .128 33. 15
39
37. 32.5
38. 9
Coordinate Geometry 101

Functions Practice Slope


Line #1
1. 2
No Calculator 2. 3
Function Basics 3. 0
7. A 4. slope
11. C 5. y-intercept
16. 2 6. any ordered pair on the line
7. y = 2x + 3
Functions IRL
1. B Line #2
9. B 3
1 1. −
19. or .030 4
33
7
2.
Functions and Graphs 2
5. D 3 7
3. y = − x +
16. 2 4 2

Line #3
Calculator
Function Basics 1
y= x+4
3. A 2
23. A
34. 6 Line #4
35. 10
1
y= x −6
2

© TPR Education IP Holdings, LLC  |  741


SAT MANUAL

Line #5 Calculator
y = −2x + 2 Roots and Solutions
4. D
1. parallel 9. B
2. same 24. A
3. perpendicular 33. 25
4. negative reciprocals
Slope and Intercepts
Lines #6, 7, and 8 5. A
1. y 27. D
2. opposite sign 33. 6
3. same
4. x Parallel and Perpendicular Lines
5. opposite sign 13. A
6. opposite sign 26. A

Forms of Equations
We ¤ Our Calculators 7. B
Tables 27. D
11. D

Intersections
10. D Advanced Coordinate Geometry
Practice

Coordinate Geometry Practice No Calculator


Circles
5. D
No Calculator 14. D
Roots and Solutions
4. A Transformation of Graphs
10. C 3. D
12. D
Slope and Intercepts 17. 2
12. C
Other Topics
3
16. or 1.5 19. 8
2
5
20. or .714
7 Calculator
Parallel and Perpendicular Lines Circles
3. C 25. B
8. C 26. D
28. A
Forms of Equations
9. B
18. 4

742  |  © TPR Education IP Holdings, LLC


ANSWER KEY

Transformation of Graphs Snow: Volume


9. B 5. 100
15. D
Angles
Other Topics Right Angle: 90°
17. A Straight Line: 180°
20. D Triangle: 180°
35. 1200 Quadrilateral: 360°
Circle: 360°
6. C
7. 80
Plane Geometry 101
Parallel Lines and Angles
8. 135°
Formulas of Basic Shapes 9. 40°
Area
Square: A = s2 Right Triangles
Rectangle: A = lw
Parallelogram: A = bh
a b c
1
Triangle: A = bh
2 1 3 10
1. 20
2. 36 3 4 5
Perimeter: Add up all the sides 5 12 13

6 8 10
Circles
Area: A = πr 2 7 24 25
Circumference: C = πd or 2πr
3. 49π 2 2 3 4

Volume 5 5 5 2
Cube: V = s3
Rectangular Solid: V = lwh 6 7 85
Cylinder: V = πr 2h
4. 4 3 3 3 6

Geometry IRL 4 2 4 2 8
Tile: Area
Tire revolution: Circumference
Fences: Perimeter
Cubic feet: Volume Special Right Triangles
m2: Area a. 45°-45°-90°
Carpet: Area b. 60°
Ribbon: Perimeter c. Turn it into a 30°-60°-90° triangle
Water: Volume 10. 4 2
Square centimeters: Area
Brick: Area 11. 5 3
in.3: Volume

© TPR Education IP Holdings, LLC  |  743


SAT MANUAL

Circles
• The height is the perpendicular distance
from the base to the opposite vertex.
C A r d • Perimeter is the sum of the sides.
• The third side of any triangle is always
8p 16p 4 8
less than the sum and greater than the
10p 25p 5 10 difference of the other two sides.

12p 36p 6 12 In an isosceles triangle


• Two sides are equal.
4p 4p 2 4
• The two angles opposite the equal sides
18p 81p 9 18 are also equal.

24p 144p 12 24 In an equilateral triangle


• All three sides are equal.
14p 49p 7 14
• All angles are each equal to 60°.
6p 9p 3 6
Four-Sided Figure Facts
2p p 1 2
In a quadrilateral
16p 64p 8 16
• All four angles add up to 360°.
20p 100p 10 20
In a parallelogram
• Opposite sides are parallel and equal.
• Opposite angles are equal.
Fun Facts About Figures • Adjacent angles are supplementary (add
Angle Facts up to 180°).
• Area = base × height = bh
• There are 90° in a right angle. • The height is the perpendicular distance
• When two straight lines intersect, from the base to the opposite side.
angles opposite each other are equal.
• There are 180° in a straight line. In a rhombus
• Two lines are perpendicular when they • Opposite sides are parallel.
meet at a 90° angle. • Opposite angles are equal.
• The sign for perpendicular is ⊥ . • Adjacent angles are supplementary (add
• Bisect means to cut exactly in half. up to 180°).
• There are 180° in a triangle. • All 4 sides are equal.
• There are 360° in any four-sided figure. • Area = base × height = bh
• The height is the perpendicular distance
Triangle Facts from the base to the opposite side.
• The diagonals are perpendicular.
In any triangle
• The longest side is opposite the largest In a rectangle
angle. • Rectangles are special parallelograms;
• The shortest side is opposite the smallest thus, any fact about parallelograms also
angle. applies to rectangles.
• All angles add up to 180°. • All 4 angles are each equal to 90°.
• Area = length × width = lw
1 1
• Area = (base × height) = bh
2 2

744  |  © TPR Education IP Holdings, LLC


ANSWER KEY


Perimeter = 2(length) + 2(width) = Area
2l + 2w • The amount of space within the bound-
• The diagonals are equal. aries of the circle
• A = pr 2
In a square
• Squares are special rectangles; thus, any Sector
fact about rectangles also applies to • Any part of the area formed by two
squares. radii and the outside of the circle
• All 4 sides are equal. • The area of a sector is proportional to
• Area = (side)2 = s2 the size of the interior angle.
• Perimeter = 4(side) = 4s
• The diagonals are perpendicular. Line Facts

Circle Facts Lines


• A line has no width and extends infi-
Circle nitely in both directions.
• There are 360° in a circle. • Any line measures 180°.
• A line that
 contains points A and B is
Radius (r) called AB (line AB).
• The distance from the center to any • If a figure on the SAT looks like a
point on the edge of the circle straight line, and that line looks like it
• All radii in a circle are equal. contains a point, it does.

Diameter (d) Line segments


• The distance of a line that connects two • A line segment is a part of a line and
points on the edge of the circle, passing has two endpoints.
through the center • The degree measure of a line segment is
• The longest chord in a circle 180°.
• Equals twice the radius • A line segment, which has endpoints of
A and B, is written as AB .
Chord
• Any line segment connecting two
points on the edge of a circle Perpendicular
• The longest chord is called the diameter. • Two lines that intersect in a 90° angle
are perpendicular and their slopes are
Circumference (C) negative reciprocals.
• The distance around the outside of the
circle
• C = 2pr = pd

Arc
• Any part of the circumference
• The length of an arc is proportional to
the size of the interior angle.

© TPR Education IP Holdings, LLC  |  745


SAT MANUAL

Plane Geometry Practice Calculator


Lines and Angles
4. B
No Calculator 24. A
Lines and Angles 33. 55
6. D
12. C Similar Triangles
14. B 9. D
18. A
Similar Triangles 36. 60
9. C
15. C Pythagorean Theorem
19. 10 1. D
14. B
Pythagorean Theorem
2. C Trigonometry
4. B 7. B
17. B
Trigonometry 20. C
5. B
12. A Radians and Degrees
17. 20 6. C
17. A
Radians and Degrees 21. D
9. A
15. B Circles
16. 90 9. B
21. B
Circles 36. 15
9. B
15. A Overlapping Shapes
22. D
Overlapping Shapes 24. C
8. C
10. A Volume
27. B
Volume 28. B
11. B 29. A
14. D
15. C

746  |  © TPR Education IP Holdings, LLC

You might also like